TrueLearn Questions

¡Supera tus tareas y exámenes ahora con Quizwiz!

A carpenter presents with a medical diagnosis of hip osteoarthritis. His chief complaint is an inability to squat down to reach the floor due to hip stiffness. Which of the following femoral glides at the hip would BEST address the dysfunction? a) posterior glide b) anterior glide c) anteromedial glide d) medial glide

Correct answer: A Squatting requires closed chain hip flexion. Based on the convex concave rule, to increase hip flexion a posterior glide of the femur is required. Answer B:An anterior glide would improve hip extension and not flexion. Answer C:An anteromedial glide does not increase hip flexion. Answer D:A medial glide would improve abduction of the hip and not flexion. Bottom Line:Squatting requires closed chain hip and knee flexion, and closed chain ankle dorsiflexion. Hip flexion can be increased by applying a posterior glide to the proximal femur.

A 67-year-old patient falls and suffers a right humeral shaft fracture and subsequent radial nerve injury. Which of the following muscles will be unaffected? a) triceps b) wrist extensors c) biceps d) finger extensors

Correct answer: C The radial nerve is sometimes injured as it winds around the posterior aspect of the shaft of the humerus. The injury frequently occurs when the humeral shaft is fractured. Nerve damage in this location would cause weakness of the elbow, wrist, and finger extensors. Answers A & B & D:The radial nerve stems from nerve roots C5-C8 and T1. It provides motor innervation for the dorsal arm muscles including the triceps, wrist extensors, and finger extensors.

A physical therapist is examining a patient with shoulder pain. After completing a full examination with all special tests completed, the following special tests were positive: Hawkins-Kennedy test, infraspinatus resisted test, and painful arc sign. Which of the following disorders is MOST likely present? a) glenohumeral instability b) labral tear c) rotator cuff tear d) subacromial impingement

Pain in the subacromial space can be caused by a number of disorders. Clinical prediction rules (CPR) combine certain findings to improve the likelihood of predicting the source of the dysfunction. Positive findings for subacromial impingement include a painful arc sign and positive Hawkins-Kennedy and infraspinatous resisted tests.The pain generator stems from a structure being compressed within the subacromial space. The Hawkins-Kennedy test creates a gradual compression to the subacromial space and is compression provocative. The infraspinatus resisted test emphasizes the external rotators as a source of tissue not stabilizing correctly, thus allowing the humeral head to move more freely, creating compression within the subacromial space. Lastly, a painful arc occurs in the range in which the structures in subacromial space are most compressed. Answer A: Glenohumeral instability can lead to a subacromial impingement, but there are other causes as well. This answer is not the best choice because the tests listed do not directly test for instability; they test for subacromial compression of the structures within the subacromial space. Answer B: These special tests do not stress the labrum. Answer C: The CPR for a rotator cuff tear has some similarity to the CPR for subacromial impingement since the location of the supraspinatous in the subacromial space makes it susceptible to injury and tearing. However, the CPR for a rotator cuff tear includes a positive drop arm test. The CPR for an incomplete rotator cuff tear is painful arc sign, drop arm sign, and infraspinatus resisted test. In this scenario, rotator cuff tear is not the best option.

A physical therapist performs an examination of a patient with low back pain, radicular symptoms to the lower extremity, and lower extremity weakness. The therapist decides to perform lower extremity myotome testing. Which key muscle must be tested to check for L4? a) tibialis anterior b) iliopsoas c) quadriceps d) extensory hallucis longus

Correct answer: A The correct answer is tibialis anterior. Tibialis anterior is the key muscle for L4. Answer B: Iliopsoas is at L2 and L3 level. Answer C: Quadriceps is at L2, L3, and L4 level. Answer D: Extensor hallucis longus is at L5 level.

A patient presents with a 2-month history of worsening pain in the buttocks region. Examination reveals muscle spasm of the piriformis, and the therapist decides to include ultrasound in the plan of care. Which of the following is the most appropriate setting to use in this case? a) 1MHz continuous at 1.0 W/cm2 b) 3 MHz continuous at 1.0 W/cm2 c) 1 MHz pulsed at 1.0 W/cm2 d) 3 MHz pulsed at 1.0 W/cm2

Correct answer: A Ultrasound at 1 MHz continuous frequency is considered best for heating tissues up to 5 cm deep. This is the desired frequency for the piriformis muscle as it is located deep to the gluteus maximus muscle. By performing the ultrasound at a continuous versus pulsed duty cycle, thermal effects will be produced to help decrease the muscle spasm. Answer B:3 MHz frequency is considered best for heating tissues only 1 to 2 cm deep. Answers C & D:Pulsed ultrasound is used when nonthermal effects are desired to promote healing at a cellular level. Bottom Line: 1 MHz frequency is used for deep tissues (up to 5 cm deep), while 3 MHz is used for superficial tissues (1-2 cm deep). Continuous ultrasound is used to produce thermal effects, while pulsed ultrasound produces a number of nonthermal effects at the cellular level. Ultrasound has a variety of biophysical effects. It can increase the temperature of deep and superficial tissues and has a range of nonthermal effects. Establishing the location of the tissue being treated and the desired effect will assist the therapist in determining the proper settings.

A physical therapist is treating a patient with a slow-healing pressure ulcer. Which of these is an electrical modality that can be used to ASSIST WOUND HEALING? a) burst mode alternating current b) neuromuscular electrical nerve stimulation c) high-voltage pulsed current d) interferential current

Correct answer: C High-voltage pulsed current (HVPC) is a twin peaked direct (monophasic) current that can be used to assist in wound healing. Answer A:Burst mode alternating current is used primarily to assist in muscle strengthening. Answer B:Neuromuscular electrical nerve stimulation is a biphasic current with a duty cycle that can be used for muscle strengthening. Answer D:Interferential current is used primarily for pain control.

Which of the following locations is correct for palpation of the pedal pulse? A. The medial aspect of the foot inferior to the medial malleolus B. The mid line of the posterior knee crease C. The volar forearm medial to the stylus process D. The mid line or slightly medial of the dorsum of the foot

Correct answer: D The pedal pulse is found on the dorsum of the foot at midline or slightly medial. Answer A:The medial aspect of the foot inferior to the malleolus is the location of the posterior tibial pulse. Answer B:The midline of the posterior knee crease is the location of the popliteal pulse. Answer C:The volar forearm medial to the stylus process is the location of the radial pulse.

During gait analysis, a patient walks with an upright posture with symmetrical but short steps. There is a noticeable early heel rise on each lower extremity when the patient transitions from heel-first initial contact to terminal stance, resulting in little time spent in single limb support on either side during the gait cycle. No deviations in the frontal plane were noted. Based on these observations, which of the following tests and measures is the MOST appropriate to perform in order to help explain this presentation? a) ankle dorsiflexion passive range of motion b) hamstring length c) hip flexor length d) knee flexion passive range of motion

Correct answer: A A thorough understanding of gait requires the study of joint kinematics during the gait cycle. Knowing the normal ranges of motion required at each phase of gait will help when trying to identify the cause of a deviation. A lack of dorsiflexion range of motion, whether due to a tight gastrocsoleus or limited joint mobility, will result in an early heel rise. Becoming familiar with these foundational concepts in movement analysis will help guide musculoskeletal examination in order to identify the cause.During the gait cycle, after the loading response phase, the tibia should begin to move anteriorly over the fixed foot, producing closed chain dorsiflexion. In this case, the gait deviation is decreased tibial progression during stance, causing an early heel rise. Because of the lack of dorsiflexion range of motion, as the tibia moves in an anterior direction, the heel cannot stay on the ground and rises earlier than it should.During the stance phase of gait, the ankle normally begins in a neutral position (0 degrees of dorsiflexion and plantarflexion) at initial contact. The ankle then progresses into approximately 5 degrees of plantarflexion with loading response. With the foot on the ground, the tibia is then advanced forward over the foot with approximately 5 degrees of dorsiflexion into mid-stance. The tibia continues forward advancement into about 15 degrees of dorsiflexion to terminal stance in preparation for push-off. In this case, the problem is seen between mid-stance and terminal stance. Ankle dorsiflexion PROM should be tested because normal forward progression of the tibia over the foot between these two phases requires approximately 10 degrees of dorsiflexion range of motion, and this is not occurring. Answer B:Shortened hamstring length can cause a variety of gait deviations, depending on the phase of gait as well as the joint that is most affected. However, in this case, the gait deviation of interest is decreased tibial progression during stance with an early heel rise. A shortened hamstring length would not directly contribute to this problem. Answer C:In this case, the gait deviation of interest is decreased tibial progression during stance with early heel rise. Although shortened hip flexors can cause various gait deviations throughout the gait cycle, they would more likely cause postural deviations such as anterior pelvic tilt and lumbar lordosis, or reduced hip extension. In addition, even though it is possible for shortened hip flexors to be present in someone with this gait presentation, ankle ROM would more directly explain the observed deviation. Answer D:Limitations in passive knee flexion would likely create gait deviations most apparent during the swing phase of gait with possible compensations seen in the frontal plane. In normal gait, the largest amounts of knee flexion ROM are mainly required during swing, up to approximately 60 degrees. In this case, the swing phase of gait is not impaired. In addition, no deviations are seen in the frontal plane. Therefore knee flexion PROM is unlikely to be involved. Bottom Line:A lack of dorsiflexion range of motion, whether due to a tight gastrocsoleus or limited joint mobility, will result in an early heel rise.

The physical therapist is working with a 17-year-old patient with scoliosis. The therapist decides to focus on improving trunk stability, including exercises for the oblique muscles of the trunk. If the target is right external oblique muscle, which of the following is the BEST exercise? a) trunk rotation to the right b) trunk rotation to the left c) side bending towards the left d) trunk extension

Correct answer: B Action of unilateral external oblique muscle is trunk rotation toward the opposite side. Answer A:This exercise targets the left external oblique muscle. Answer C:The external obliques could also assist in trunk bending. Action of the right external oblique assists in side bending toward the right. Answer D:Bilateral action of the external obliques will assist in trunk flexion, not trunk extension.

A 45-year-old patient presents with severe right hand pain, hypersensitivity, and decreased range of motion following a crush injury at work 6 months ago that resulted in multiple contusions, now healed. Which modality is BEST for pain control? A. Neutral warmth B. Hot water immersion C. Ice D. Aggressive fluidotherapy

Correct answer: A Complex regional pain syndrome (CRPS) is pain believed to involve overactivation of the sympathetic nervous system. Physical agents can be used to control pain with these patients. In general, low-level sensory stimulation of the involved area, such as neutral warmth, mild cold, water immersion, or gentle agitation of fluidotherapy, may be effective. Answers B & C & D:More aggressive stimulation such as hot water, ice, or aggressive agitation of water or fluidotherapy probably will not be tolerated and may aggravate this type of pain.

During analysis of static standing posture in a 16-year-old female, the physical therapist notices unilateral pronation in the right ankle as well as a slight genu valgus in the right knee. Which of the following SHOULD BE MEASURED in order to help identify the underlying cause of this postural deviation? a) bilateral leg length to determine whether there is a leg length discrepancy b) dorsiflexion range of motion on the right c) hip flexor tightness on the left d) medial collateral ligament laxity on the right

Correct answer: A Leg length discrepancy can cause various postural abnormalities as well as gait deviations. In static standing, when one leg is longer than the other, one side would have to compensate in order to maintain stability. If the shorter limb compensates, deviations typically appear in the hip and pelvis. If the longer limb compensates, which is what is happening in this case, deviations typically occur in the foot and ankle in the form of excessive pronation. Answer B:This answer is incorrect because decreased dorsiflexion range of motion alone would not cause unilateral pronation in static standing. It is possible to see increased pronation during the stance phase of gait as a compensation for limited dorsiflexion range of motion. However, in this patient case, with pronation seen in static standing, a different cause is much more likely. Answer C:This answer is incorrect because the deviations seen in this case are found in the frontal plane in the foot and ankle. Neither unilateral nor bilateral hip flexor tightness would cause these deviations. Postural deviations resulting from hip flexor tightness would likely be seen in the spine or pelvis. Although it is possible to see manifestations of proximal deviations appear distally in the foot and ankle (e.g., hip flexor tightness causing extreme lumbar lordosis, leading to knee hyperextension and then apparent increase in plantarflexion in standing), these deviations nonetheless all occur in the sagittal plane. Answer D:In this case, the presence of genu valgus could very well suggest ligamentous laxity in the medial structures. However, it is not the correct answer because ligamentous laxity is unlikely to be the cause of excessive foot pronation. Conversely, excessive foot pronation is often the cause of stress to the medial structures of the knee. Therefore, if there were laxity in the medial collateral ligament, it would most likely be a consequence of the underlying abnormality and not the cause of it. Bottom Line:Typically,when there is a leg length discrepancy, the body compensates to level out, and one compensation seen is pronation of the longer leg to effectively shorten it so knees and hips and iliac crests are leveled. Also, when performing postural assessments and looking at abnormalities, the PT must be aware of the plane wherein deviations are found. Paying attention to the plane of potential motion not only allows for better differentiation of the types of postural deviations that are possible in a certain presentation, but also allows for better appreciation of the biomechanics of postural stability and movement.

Which of the following statistical tests is the MOST appropriate for demonstrating the reliability of a patient questionnaire? a) coefficient alpha b) factor analysis c) multiple regression d) paired t test

Correct answer: A Reliability of a questionnaire would be established through the coefficient alpha, which demonstrates whether the scores on the survey are internally consistent and stable over time. Answer B:Factor analysis is used to determine the validity of survey items and determine whether meaningful and useful inferences can be made from the scores of the instrument. Answer C:Multiple regression is a statistical test done to determine the relationship between the variables and is not used to determine validity or reliability. Answer D:Paired t test is a statistical test that is used to look at the relationship between two groups or two sets of variable and is not used for validity or reliability. Bottom Line:The best test to determine the reliability of a survey instrument is the coefficient alpha, which shows if the scores are internally consistent (the instrument is going to measure the same thing every time).

An individual presents to the clinic with a report of distal numbness along the dorsum of their foot toward the big toe. Which KEY MUSCLES should the physical therapist examine during myotomal testing? a) extensor hallicus longus, gluteus medius b) extensor hallicus brevis, gluteus maximus c) flexor hallicus longus, gluteus medius d) extensor digitorum longus, gluteus maximus

Correct answer: A The area described has sensation provided by the L5 dermatome. The L5 myotome key muscles include gluteus medius, extensor hallicus longus, and fibularis longus and brevis. Answers B & C & D:These muscles are not key muscles within the L5 myotome.

A physical therapist observes that a patient who is performing moderate exercise is beginning to show signs of hypoglycemia. The therapist most likely observes all of the following signs EXCEPT which sign? a) bradycardia b) confusion c) complaint of headache d) perspiration

Correct answer: A Acute hypoglycemia causes a number of physiological responses in order to maintain the supply of glucose to the brain and help the liver add more glucose to the circulation. The autonomic nervous system is activated, and epinephrine is released to help this activation, and that creates tachycardia, not bradycardia. People report a rapid heart rate, dizziness and shakiness, perspiration, and headache. They also begin to act confused as they become more hypoglycemic. Answers B & C & D:This is a common sign of hypoglycemia.

The physical therapist performs manual muscle testing on a patient with low back pain. The therapist instructs the patient to rotate his trunk to the right. Which of the muscles below is BEING TESTED with the motion? a) rectus abdominis b) left internal oblique c) left external oblique d) transverse abdominis

Correct answer: C The correct answer is left external oblique. The external oblique is activated when rotating the trunk toward the opposite side. Bilateral action of the external obliques helps to produce flexion of the trunk without rotation. Answer A:Rectus abdominis performs trunk flexion. Answer B:Left internal oblique performs trunk rotation toward the left. Answer D:The transverse abdominis muscle provides pelvic stability by helping to compress the viscera and ribs.

A patient is being seen in an outpatient physical therapy clinic following a lateral ankle sprain. The patient is able to perform 20 unilateral calf raises off a step, evert the foot 15 times pain-free with a blue TheraBand, and perform single leg balance on an Airex pad for 10 seconds. The patient is a lifeguard, and their goal is to get back to running on the beach pain-free. Which of the following interventions would best help this patient achieve their rehabilitation goal? A. Single limb stance on an Airex pad with eyes closed B. Single limb stance on an Airex pad performing mock running motions with arms and non-stance leg C. Squats on a Bosu ball D. Progress to beginning a running program on a treadmill

Correct answer: B After an ankle sprain, joint proprioception needs to be restored. Even if strength is 5/5 for all ankle musculature, if proprioception is affected, the muscles will not react in a timely manner to prevent future ankle sprains because the motion is not sensed until it is too late. Running on the beach further increases the demand on the proprioceptors as the surface is unstable and does not provide reliable feedback to the joint as to position sense. Therefore, the best activity to reach this patient's goals is to replicate the unstable surface and incorporate the activity they will be doing, through single limb stance on an Airex pad performing mock running motions with arms and non-leg stance. This is the best activity of the options provided in this question to get the patient back to running on the beach.Moving one or more extremities while stabilizing another area of the body on unstable surfaces is a great way to improve proprioception and can mimic an activity identified as a long-term goal. After an ankle sprain, besides restoring strength, restoring proprioception is key to preventing future sprains. The proprioceptors need to detect the ankle motion in order to signal the body to activate the musculature needed to prevent an ankle sprain. Answer A:Single limb stance on an Airex pad with eyes closed is excellent for a patient at this level status post ankle sprain because the unstable surface challenges the proprioceptors, and closing the eyes removes the cues from the visual system. However, in this specific case, the goal is to return to running on the sand, and thus the option with the most specific activity to recreating beach running is adding running motions to the unilateral balancing on an unstable surface. Answer C:Squats on a Bosu ball are good for strengthening and improving proprioception but is a bilateral exercise. At this point in their rehabilitation, a unilateral exercise on an unstable surface will allow the affected ankle to do all the work. Answer D:Running on a treadmill will improve running ability on a flat surface but will not improve ability to maintain the position of the ankle while running, or while running on an unstable surface like sand. Bottom Line:Moving one or more extremities while stabilizing another area of the body on unstable surfaces improves proprioception. After an ankle sprain, besides restoring strength, restoring proprioception is key to preventing future sprains.

Breath that has a sweet and fruity odor is a sign of WHAT condition? a) bronchiectasis b) GERD c) hyperglycemia d) ketoacidosis

Correct answer: D When there is an insulin deficiency with type I diabetes, insulin does not move glucose to the cells. Thus, the cells may metabolize fat instead, which produces ketones as a byproduct. The ketones accumulate in the blood and cause the pH to fall, causing metabolic acidosis. The ketones are excreted through the kidneys and the lungs. The ketones cause acetone breath, which smells fruity. Diabetic ketoacidosis is a medical emergency, and physical therapists should recognize this sign. Answer A:Chronic inflammation after a pulmonary infection can cause irreversible destruction and dilation of the bronchi and bronchioles, called bronchiectasis. Patients have persistent coughing and sputum production. There is either no smell or a foul smell to breath from the sputum as the disease progresses. Answer B:This is caused by reflux of the gastric contents into the esophagus. Heartburn is the most common symptom. The breath is not affected. Answer C:Acetone breath or fruity breath is not a symptom of hyperglycemia. It may develop later on if cells that lack glucose metabolize fat instead and produce ketones as a byproduct, but hyperglycemia alone does not produce acetone breath.

A physical therapist is planning a research study investigating the effect of a lower extremity strengthening program on gait speed in community-dwelling older adults. Subjects will be assessed at three different time points: baseline (assessment 1), after 6 weeks of training (assessment 2), and after 12 weeks of training (assessment 3). Gait speed will be calculated for each participant using distance in meters and time in seconds. What type of statistical test would be used to determine if the intervention was able to significantly improve gait speed? a) chi square b) independent t test c) pearson r correlation d) repeated measures analysis of variance

A repeated measures analysis of variance (ANOVA) is used to compare three or more measures of interval or ratio level data collected from the same research subject over time. Since the design of this study measured the same group of individuals over a period of three assessments, repeated measures analysis of variance would be an appropriate statistic for this data. Answer A: A Chi square is used to analyze nominal level data to determine if the proportion of categories of data differ from what you would expect to find by chance. Since the design of this study measured the same group of individuals over a period of three assessments and gait speed is ratio level data, a Chi square would be an incorrect statistic for this data. Answer B: An independent t test is used to compare the means of interval or ratio data from two independent groups of individuals. Since the design of this study measured the same group of individuals over a period of three assessments, an independent t test would be the incorrect statistic for this data. Answer C: A Pearson r correlation is used to determine the relationship between two dependent variables, which are interval or ratio level data. The design of this study was not correlational since it was attempting to determine the effect of an intervention. Bottom Line: Analysis of variance is used to compare three or more measures of interval or ratio level data. A physical therapist needs to understand why a particular statistical analysis was completed so that they can determine the validity of study findings.

A physical therapist is treating a patient who was recently diagnosed with a stage III brainstem glioma. Which of the following symptoms would be LEAST likely to be found in this patient? a) dysdiadochokinesia b) diplopia c) dysphagia d) dysarthria

Correct answer: A A glioma is a common type of brain tumor that can occur in multiple locations in the brain. A stage III tumor indicates that the tumor is malignant with poor differentiation. Since this tumor is located in the brain stem, dysdiadochokinesia would not be expected since that is a function associated with the cerebellum. Dysdiadochokinesia is the ability to perform rapid alternating movements. Answer B:Diplopia is double vision and is often found in patients with brainstem injuries. Cranial nerves III, IV, and VI originate in the brainstem and control extraocular eye movements. Answer C:Dysphagia is a deficit in swallowing function due to weakness or decreased coordination of the oropharyngeal musculature or due to cognitive deficits.Dysphagia may lead to choking or aspiration pneumonia. Dysphagia is often found in patients with brainstem injuries. Cranial nerves V, VII, IX, X, XI, and XII originate in the brainstem and control the ability to swallow. Answer D:Dysarthria is a speech disorder that affects the motor aspect of speech and causes impairments in articulation, resonance, and phonation. Language comprehension and expression are not affected by dysarthria, but patients are often difficult to understand. Dysarthria is often found in patients with brainstem injuries. Cranial nerves V, VII, IX, X, and XII originate in the brainstem and control the motor component of speech. Bottom Line:Brainstem injuries, including tumors, can give symptoms of diplopia, dysphagia, and/or dysarthria depending on the cranial nerves affected. Physical therapists must be able to predict motor, sensory, and cognitive deficits based on the location of a brain tumor in order to plan efficient examinations andeffective interventions.

A patient is being seen in an outpatient clinic with a report of stiffness that limits their ability to make a fist and grip items. The physical therapist notes the limitation is from hypomobility at the third metacarpophalangeal (MCP) joint. Which of the following mobilization techniques applied to the proximal phalange would be the BEST intervention to improve this patient's functional deficit? a) grade III volar glide b) grade III dorsal glide c) grade II volar glide d) grade IV dorsal glide

Correct answer: A A grade 3 volar glide would improve finger flexion since the concave phalange is moving on a convex metacarpal, and that is the correct functional deficit in this question. In addition, the use of grade 3 is appropriate to help restore range of motion as it uses large amplitude movements and takes the joint to the end point of the available range of movement. Answer B:A dorsal glide would improve finger extension, which is not needed to make a fist or grip. Answer C:This answer is incorrect as the direction is appropriate, but the grade would focus primarily on pain and not stiffness. Grade 2 does not go to the end range of the available movement. Answer D:A dorsal glide would improve finger extension, which is not needed to make a fist or grip, although the grade is appropriate. Bottom Line:A volar directed glide to the proximal phalange improves finger flexion since the concave phalange is moving on a convex metacarpal. Finger flexion is what is required to make a fist or grip. A grade of 3-4 would be appropriate for stiffness/ROM, whereas a grade of 1-2 is appropriate for pain

A physical therapist is completing an initial examination with a patient who had a stroke and needs a wheelchair for mobility. The patient is 5 feet 6 inches tall and has a normal BMI. The physical therapist plans to teach the patient to use their less-affected leg to assist with mobility. Which of the following is the APPROPRIATE wheelchair seat height that should be trialed with this patient? a) 2 inches lower than standard height b) 4 inches lower than standard height c) 2 inches higher than standard height d) 4 inches higher than standard height

Correct answer: A A hemi-height wheelchair typically has a floor-to-seat height of 17 inches, which is 2 inches lower than the standard seat height of 19 inches and will allow for the use of the less-affected lower extremity to assist with propelling the wheelchair. Answer B:A floor-to-seat height of 15 inches is considered a "super-hemi" height since it is so low to the ground. A seat-to-floor height of 15 inches is typically used with pediatric patients or short patients. Answer C:A floor-to-seat height of 21 inches is 2 inches higher than the height of a standard wheelchair and would make propulsion with the lower extremity difficult. Answer D:A floor-to-seat height of 23 inches would make propulsion with the lower extremity difficult because the leg would be much higher off the ground. The patient is not unusually tall but of more average height.

When taking the history and physical of a 60-year-old female patient, the therapist should consider which of the following findings to be a major risk factor for osteoporosis and related bone fractures a) history of glucocorticoid use b) distant history of smoking c) history of osteoarthritis d) high body mass index

Correct answer: A A history of glucocorticoid use is a primary risk factor for osteoporosis and subsequent bone fracture. Answer B:Current smokers are at greater risk of developing osteoporosis than those with a distant history. Answer C:A history of rheumatoid arthritis is a greater predictor of osteoporosis than a history of osteoarthritis. Answer D:Low body mass index is a greater predictor of osteoporosis than high body mass. Bottom Line:Women who are current smokers, drink more than three alcoholic drinks per day, have a history of rheumatoid arthritis, have a history of glucocorticoid use, and have a low body weight present with the highest risk factors for osteoporosis. These high-risk factors and other issues would need to be considered, such as height loss, posture, and a prior fracture, to fully determine risks.

A physical therapist is treating a 21-year-old patient who had an ASIA A T6 spinal cord injury 2 weeks ago. He has no precautions on his activity. He has limited flexibility in his hamstrings and mild spasticity in his legs. Which of the following is the MOST appropriate transfer method for the patient to learn to use initially? a) lateral transfer with transfer board b) anteroposterior transfer c) lateral transfer d) squat pivot transfer

Correct answer: A A lateral transfer using a transfer board allows the patient to transfer using their upper extremities without requiring the use of the lower extremities. A transfer board is often used initially in patients with thoracic spinal cord injuries to allow them to transfer as safely as possible by using the board to bridge the gap between surfaces as the patient learns how use the head-hips relationship to transfer independently without using a board. It is also a safer way to start since he has some lower extremity (LE) spasticity. Answer B:An anteroposterior transfer, also known as a front on transfer, would not be appropriate in this scenario since the patient has tight hamstrings, which would not allow them to get in the proper position. A patient with tight hamstrings should not be placed in a long sitting position since that would cause their lower back to become loose and prevent the transfer of momentum from the upper body to the lower body during functional activities. Answer C:Eventually the patient will be able to transfer from the bed to the chair without the transfer board as the patient learns how use the head-hips relationship to transfer independently without the board and with the upper extremity strength. It is safer to start with the board. Answer D:A squat transfer would not be appropriate in this scenario since the patient has a complete spinal cord injury and has no lower extremity strength to use their legs to bear weight during the transfer.

A physical therapist is treating a patient with a C2 ASIA A spinal cord injury who is in an inpatient rehabilitation facility and is using mechanical ventilation with 40% supplemental oxygen via the tracheal collar. The patient is positioned supine in bed, and they have their compression stockings and abdominal binder on. Blood pressure is 90/60, while heart rate is 90, and the pulse oximetry is 95%. While completing a dependent roll with the assistance of a certified nursing assistant to place a sling for an overhead lift transfer, the low pressure alarm on the ventilator goes off. What is the MOST appropriate response that the physical therapist can make? a) check to see if the ventilator tubing has been disconnected b) initiate suctioning of the patient to increase the pressure c) increase the rate of supplemental oxygen during the transfer d) call for help and start manual ventilation with a resuscitation bag

Correct answer: A A low pressure alarm is used to indicate that the pressure of the ventilator circuit has decreased. Typically, it is caused by the ventilator being disconnected due to the patient being moved during functional activities. If the ventilator tube is disconnected, the physical therapist will simply reconnect it to the patient. Answer B:A high pressure alarm is used to indicate that the pressure of the ventilator circuit has increased. Typically, it is caused by secretion buildup, which can be managed by suctioning the patient. Answer C:Increasing the supplemental oxygen for the patient would not be effective since the low pressure alarm indicates that the ventilator circuit has been compromised. Answer D:The decision to manually ventilate the patient with a resuscitation bag would occur after the ventilator circuit was assessed and it was determined to be intact. At that time, manual ventilation should be given to maintain adequate oxygen levels while help was being called to determine the cause of the low pressure alarm. Bottom Line:Patients with a high cervical spinal cord injury often are placed on ventilators to assist with breathing. Physical therapists must be able to identify the cause and management strategy when there is a problem with the ventilator. In this scenario, the therapist first checks to see whether the ventilator tubing has been disconnected. Patient and family education regarding ventilator management is a critical area to address during rehabilitation of patients with spinal cord injury in preparation for discharge to home since they will need to be able to independently and safely utilize the ventilator if they require it at discharge.

The wheelchair being ordered for a patient with T10 paraplegia needs to have which of the following parts? A. Regular height back, removable arm rests, swing-away foot rests B. High back, removable arm rests, swing-away foot rests C. High back, removable arm rests D. Regular height back, removable arm rests, elevating foot rests

Correct answer: A A patient with T10 paraplegia should have good trunk control and does not need a high-back wheelchair. That height my even limit UE ROM. They do need removable arm rests and swing-away leg rests in order to transfer in and out of the wheelchair independently. Answers B & C:A high back is unnecessary because trunk control is good. Answer D:The leg rests should swing away to make transfers safer and easier but do not need to be able to elevate as there is no issue with edema.

A physical therapist is examining a patient with anterior cord syndrome due to spinal stroke that occurred at C6. When the therapist assesses light touch of the middle finger, which spinal tract is the therapist assessing? A. dorsal column-medial lemniscus tract B. lateral corticospinal tract C. reticulospinal tract D. spinothalamic tract

Correct answer: A A patient with anterior cord syndrome typically presents with damage to the anterior two-thirds of the spinal cord due to a spinal stroke or hyperflexion injury that affects the anterior spinal artery. The patient will present clinically with a loss of motor function, pain, and temperature sensation below the level of injury since those tracts are in the anterior spinal cord. However, the dorsal column-medial lemniscus tract that contains ascending information regarding proprioception, vibrations, and light touch would not be affected by this injury.Physical therapists must be able to predict motor and sensory deficits based on the blood vessels affected by spinal cord injury or spinal stroke to plan efficient examinations and effective interventions. Answer B:The lateral corticospinal tract, which contains descending motor commands for the distal extremities, would be damaged in a patient with anterior cord syndrome since it is with the distribution of the anterior spinal artery. Answer C:The reticulospinal tract, which contains descending motor commands for the trunk and proximal muscles, would be damaged in a patient with anterior cord syndrome since it is with the distribution of the anterior spinal artery. Answer D:The spinothalamic tract, which contains information regarding pain and temperature, would be damaged in a patient with anterior cord syndrome since it is with the distribution of the anterior spinal artery. Bottom Line:A patient with anterior cord syndrome typically presents with damage to the anterior two-thirds of the spinal cord due to a spinal stroke or hyperflexion injury that affects the anterior spinal artery. The patient will present clinically with a loss of motor function, pain, and temperature sensation below the level of injury since those tracts are in the anterior spinal cord. However, the dorsal column-medial lemniscus tract that contains ascending information regarding proprioception, vibrations, and light touch would not be affected by this injury.

A posterior leaf spring orthotic, when compared with a solid molded ankle-foot orthosis set in neutral dorsiflexion for a patient with foot drop, has the advantage of: a) allowing more activity of the ankle plantarflexors during push-off b) being easily modified to change the amount of dorsiflexion available c) providing better control of inversion and eversion of the ankle during stance d) providing better control of knee flexion during stance

Correct answer: A A posterior leaf spring (PLS) is a thin plastic ankle foot orthosis that is placed into a shoe. A PLS brace supports the bottom of the foot and then extends up behind the calf muscle and attaches to the proximal calf with a velcro strap to hold it in place. It is somewhat firm so that it allows slight plantarflexion as well as dorsiflexion in stance because of its posterior trimline. The PLS allows the foot to clear the floor in patients who cannot dorsiflex on their own. Because it is thinner and a little flexible, it bends a little during gait, so there is potential energy to help with dorsiflexion. It is not the same as a solid ankle-foot orthosis (AFO), which molds over the entire ankle and gives support in all planes. The PLS brace is functioning only in the sagittal plane. Answer B:A PLS is made of thin plastic without articulating joints so that the amount of dorsiflexion available is not easily modified. Answer C:A PLS is made of thin plastic that does not enclose any of the ankle joint so that it does not control ankle inversion and eversion during stance. A molded solid AFO would be more effective for that purpose. Answer D:A PLS is made of thin plastic that bends during stance so that it will not assist with control of the knee during midstance.

Eight weeks ago, a patient sustained a fracture of the distal radius that is now healed. The patient presents at the outpatient clinic after getting out of the cast with generalized stiffness and a major limitation in wrist extension ROM. When assessing joint play, the therapist can expect which of the following glides at the radiocarpal joint to be MOST limited based on the loss of ROM? a) ventral glide b) dorsal glide c) ulnar glide d) radial glide

Correct answer: A A ventral/palmar glide is correct as the proximal row of carpals glides ventrally on the radius during wrist extension and, if limited, that motion needs to be restored. Answer B:A dorsal glide is incorrect as it is a component of wrist flexion. Answer C:An ulnar glide is incorrect as it is a component of ulnar deviation. Answer D:A radial glide is incorrect as it is a component of radial deviation. Bottom Line:At the radiocarpal joint, the proximal row of carpals moves ventrally with wrist extension and dorsally with wrist flexion.

A patient would be a candidate for an abdominal binder with a complete injury at which of the following spinal cord levels? a) T6 b) T10 c) T12 d) L2

Correct answer: A Abdominal binders are used for patients with loss of abdominal wall strength, generally lesions above T6. It is suggested that the increase in abdominal pressure prevents postural hypotension positions and situates the diaphragm in a more favorable position to improve inspiration. The available evidence is not yet sufficient to support or discourage the use of an abdominal binder. Studies have generated findings that support its use and some that say it has no effect. Answers B & C & D:The abdominal muscles function at this level, and there is no need for an abdominal binder.

Which of the following patients is the MOST appropriate to participate in cardiac rehabilitation in an outpatient setting? A. 62-year-old female who had a myocardial infarction 10 days ago and has been home for a week B. 42-year-old female with a history of hypertension, diabetes mellitus (type II), and morbid obesity presenting with bilateral knee pain C. 50-year-old male with a history of hypertension controlled with medication; blood pressure consistently under 140/90 mmHg D. 90-year-old male with a history of congestive heart failure who has been admitted to the hospital for heart failure symptoms three times this month

Correct answer: A After an acute myocardial infarction, it is considered standard of care to participate in cardiac rehab as an outpatient when stable, which is typically 1 week after the cardiac event for uncomplicated cases. This answer is correct because it fits the typical description of a patient that is appropriate for cardiac rehab. Answer B:In this case, the patient has modifiable cardiac risk factors. She would likely benefit from a wellness and fitness program for primary prevention. Although it is possible for this type of patient to be enrolled in a cardiac rehabilitation program, it is not the most appropriate answer because another scenario describes a much more pressing indication (i.e., an individual with an acute MI). Answer C:In this case, the patient has hypertension, which is a modifiable cardiac risk factor. Since he has not yet had a cardiac event, he would likely benefit from a wellness and fitness program for primary prevention. However, it is not the most appropriate answer because another scenario describes a much more pressing indication (i.e., an individual with an acute MI). Answer D:Heart failure is an indication for outpatient cardiac rehab. However, this diagnosis is appropriate only if heart failure is stable. In the case of someone who has had multiple admissions to the hospital for heart failure symptoms, their condition is not considered stable. Therefore, it is not the most appropriate answer. Bottom Line:Many cardiac diagnoses are eligible for outpatient cardiac rehab. Some common ones include myocardial infarction (MI), percutaneous coronary intervention (PCI), coronary artery bypass graft (CABG), and valve replacements. In the case of a new MI, it is considered standard of care to enroll in cardiac rehab once stable, which is typically about a week after the cardiac event.

Which of the following is the MOST appropriate adaptation to a wheelchair that will be used by a patient with bilateral transfemoral amputations? a) move wheel axis more posterior b) make the wheel base wider c) lower the height of the chair d) increase the depth of the seat

Correct answer: A After bilateral transfemoral amputation, a patient's center of gravity when sitting is more posterior, thus the wheel axis needs to be moved posterior as well to keep the chair balanced and prevent tipping. Answers B & C & D:This adaptation does not address the shift of the center of gravity to a more posterior location when the patient is sitting in the wheelchair.

When the therapist assesses capillary refill, what is the expected amount of TIME for circulation to return? a) 2 seconds b) 10 seconds c) 30 seconds d) 1 minute

Correct answer: A Although sources differ on actual technique and interpretation, a capillary refill time of 2-3 seconds has generally been accepted as the cutoff time for normal. Answer B:A capillary refill time of 10 seconds is too long and may suggest pathology. However, environmental factors (such as cold temperature) can lead to increased time for capillary refill. Nonetheless, in optimal testing conditions, capillary refill times more than 2-3 seconds often suggest vascular insufficiency. Answer C:A capillary refill time of 30 seconds is too long and likely suggests pathology. However, environmental and circumstantial factors (such as temperature or dehydration) can lead to increased capillary refill time. Answer D:A capillary refill time of 1 minute is abnormal and might be seen in cases of shock or severe vascular dysfunction. Therefore, this is not the correct answer. Bottom Line:The capillary refill test is a quick way of grossly assessing for peripheral circulation and perfusion. It is commonly assessed in children and neonates. It is typically performed by pressing on the nail bed of a finger that is raised above the level of the heart (tested on the sternum in neonates) and observing for the time required for the blanched area to refill with color. Generally, a refill time of 2-3 seconds is considered to be normal.

A physical therapist is treating a patient who has difficulty ambulating due to consequences of diabetic neuropathy. The patient has bilateral foot drop, absent bilateral sensation below the ankle, and fluctuating edema. Which of the following orthoses is MOST appropriate for this patient to improve ability to ambulate? a) double upright AFO b) KAFO c) molded AFO d) PLS

Correct answer: A An orthosis would be indicated in this scenario since the patient's diagnosis of diabetic neuropathy is not expected to resolve and because the patient exhibits functional deficits in ambulation. A double upright ankle-foot orthosis (AFO) is preferred to a molded ankle-foot orthosis (MAFO) since the double uprights would allow room for fluctuating edema and prevent contact of the skin against the brace, which the patient would be unable to feel. A MAFO would place the patient at risk for skin breakdown, which could lead to a potential amputation for a patient with diabetic neuropathy. Answer B:A knee-ankle-foot orthosis (KAFO) is unlikely to be recommended for this patient since the patient has intact function above the ankle, so that compensation for weakness of the knee extensors with an orthotic would not be necessary. Answer C:A molded ankle-foot orthosis (MAFO) would place the patient at risk for skin breakdown since the patient has no sensation in the ankles and fluctuating edema. Skin breakdown could lead to a potential amputation for a patient with diabetic neuropathy. Answer D:A posterior leaf spring (PLS) is a thin plastic ankle-foot orthosis that is placed into a shoe. During early stance, the PLS bends slightly, and during late stance it recoils to assist with foot clearance and to allow activity of the dorsiflexors during swing. A PLS does not enclose any of the ankle joint, so it does not control ankle inversion and eversion, and it is not likely to be recommended for a patient with absent ankle sensation and motor function. Bottom Line:The decision to recommend the use of an orthotic device is based on a number of factors, including patient's impairments, functional deficits, and prognosis. If there is fluctuating edema, a brace cannot constrict that area of the body. In addition, if sensation is diminished, the risk of skin breakdown from a compressive brace is increased. In this scenario, a double upright ankle-foot orthosis (AFO) is recommended.

An individual presents to physical therapy with a report of stiffness in their right temporomandibular joint (TMJ). Examination of range of motion (ROM) reveals a limitation both actively and passively with mouth opening. What JOINT MOBILITY ASSESSMENT should the physical therapist perform? a) anterior glide b) posterior glide c) medial deviation d) lateral deviation

Correct answer: A Anterior glide is the motion required for jaw opening. This motion is combined with a distraction force and can also help with protrusion. Answer B:Posterior glide would be used to improve jaw retraction. Answer C:Medial deviation would assess a side glide of the contralateral TMJ. Answer D:Lateral deviation would assess a side glide of the ipsilateral TMJ. Bottom Line:When assessing mouth opening, which is a common problem area in those with temporomandibular dysfunction, the mandible needs to roll and glide anteriorly. When the physical therapist assesses this, there is typically a distraction force and an anterior glide that enable mouth opening bilaterally.

Which of the following is the MOST appropriate position to splint a patient status post burn to the wrist and back of the hand? A. Slight wrist extension, metacarpophalangeals (MCPs) in 90 degrees of flexion, interphalangeals (IPs) in extension, and thumb abducted away from palm B. Slight wrist extension, MCPs in zero degrees of extension, IPs in zero degrees of extension, and thumb adducted next to 2nd MCP C. Slight wrist extension, MCPs in 90 degrees of flexion, IPs in extension, and thumb adducted next to 2nd MCP D. Neutral wrist, MCPs in zero degrees of flexion, IPs in 90 degrees of flexion, and thumb abducted away from palm

Correct answer: A Anti-contracture positioning after a burn involves avoiding a position of flexion, such as keeping knees, hips, or elbows extended or feet in zero degrees of plantarflexion. People tend to rest joints and limbs in a position of comfort after a burn. This is generally a position of flexion and also the position of contracture. Allow the wrist to extend slightly Place the MCPs in the position of function of 90 flexion, which is also the most lengthened position for the collateral ligaments, and Keep the thumb in abduction away from the palm Proper positioning preserves function and prevents contractures. Answer B: Slight wrist extension, MCPs in zero degrees of extension, IPs in zero degrees of extension, and thumb adducted next to 2nd MCP is not the position of function, and some contractures may occur, limiting hand function. Answer C: Slight wrist extension, MCPs in 90 degrees of flexion, IPs in extension, and thumb adducted next to 2nd MCP is not the position of function, and some contractures may occur, limiting hand function. Answer D: Neutral wrist, MCPs in zero degrees of flexion, IPs in 90 degrees of flexion, and thumb abducted away from palm is not the position of function, and some contractures may occur, limiting hand function. Bottom Line: Anti-contracture positioning after a burn involves avoiding a position of flexion, such as keeping knees, hips, or elbows extended or feet in zero degrees of plantarflexion. Proper positioning preserves function and prevents contractures.

A 45-year-old patient with multiple sclerosis (MS) is referred to physical therapy to improve functional status and endurance. One afternoon while on the recumbent bike, the patient gets extremely fatigued, more than would be expected for that level of exercise. What is the BEST course of action? a) discontinue treatment for that day and consider switching to morning appointments b) refer the patient back to the physician c) allow the patient an adequate rest break and resume therapy if symptoms resolve d) continue therapy, but have them bike at a slower pace

Correct answer: A Approximately 75% of individuals with MS describe fatigue as one of the most debilitating symptoms. Fatigue is characterized by overwhelming sleepiness, excessive tiredness, and a sense of weakness that comes on suddenly and severely. It is imperative for a skilled therapist to recognize the difference between MS-related fatigue and expected exercise fatigue. MS-related fatigue during exercise is often associated which thermal stress. Exercise should be scheduled at optimal times, usually the morning, when body core temperatures tend to be lowest and before fatigue sets in. Sessions should involve discontinuous work with adequate rest periods. Progress is usually slower than with a healthy individual. Exercising to the point of fatigue is contraindicated and can result in worsening of symptoms. Exercise should be discontinued for the day in this situation. Answer B:The therapist should recognize that the patient is experiencing MS-related fatigue. The level activity is too strenuous for the patient and should be discontinued; however, this is not unexpected in patients with MS, so an immediate referral back to the physician is not warranted. Answer C:The therapist should always allow and plan for adequate rest breaks when treating MS patients. However, during this case, the patient has overexerted and exercised to the point of fatigue. Continuing to exercise that day could result in worsening of symptoms. Answer D:Continuing therapy at this point is contraindicated. The therapist should take note of the level of exercise that induced MS-related fatigue and take care to provide better rest breaks and to decrease the intensity of work for the patient in the future. Bottom Line:Patients with MS benefit from strength training and endurance training. Sessions should involve discontinuous work with adequate rest periods. The beginning of the day is optimal for scheduling, as fatigue has not yet set in and body core temperatures may be lower. Exercising to the point of fatigue is contraindicated and can result in worsening of symptoms. Thermal stress should be avoided with adequate temperature control in the exercise environment.

A physical therapist is examining a patient in outpatient rehabilitation with a history of left middle cerebral artery cardiovascular accident (CVA). The patient is unable to put her shoes on upon command but was observed by her husband putting them on independently earlier that same day. The patient's verbal comprehension appears intact, and a motor screen found at least 4/5 through her upper and lower extremities. What is the name of this clinical condition? a) apraxia b) ataxia c) dysmetria d) receptive aphasia

Correct answer: A Apraxia is the inability to perform motor activities upon command while the sensory and motor are intact. Specifically, ideomotor apraxia refers to patients who are unable to perform activities upon command while these same motions can be performed spontaneously. Answer B:Ataxia is characterized by incoordination of movement in which errors of force, speed, or trajectory occur. Ataxia is typically found in patients with damage to the cerebellum. Answer C:Dysmetria is characterized by incoordination of movement in which overshooting or undershooting occurs while reaching. Dysmetria is typically found in patients with damage to the cerebellum. Answer D:Receptive aphasia is a deficit in language comprehension that is typically found in patients with damage to the left superior temporal lobe. Since verbal comprehension is intact, the patient does not have receptive aphasia. Bottom Line: Apraxia is the inability to perform motor activities upon command while the sensory and motor are intact. Specifically, ideomotor apraxia refers to patients who are unable to perform activities upon command while these same motions can be performed spontaneously.

Which of the following BEST describes the indication for an arterial blood gas (ABG) analysis? a) determine acidosis vs alkalosis b) obtain the arterial pH only c) obtain mixed venous oxygen saturation (SVO2) d) accurately measure oxygen saturation

Correct answer: A Arterial blood gas (ABG) analysis is used to assess for problems in acid-base balance, ventilation, and oxygenation. The arterial pH, partial pressure of carbon dioxide (PaCO2), partial pressure of oxygen (PaO2), and bicarbonate (H3CO-) can all be obtained from ABG analysis. These results are used to help determine if a patient is presenting with acidosis versus alkalosis and whether the cause is respiratory or metabolic.In the acute care and critical setting, it is important to be aware of the reason an ABG analysis is performed. Although interpretation of the results is considered a more advanced skill, having a working understanding of this area allows the physical therapist to have an overall clinical picture of a patient's medical stability. Answer B:An ABG analysis provides much more than just the arterial pH. The PaCO2, PaO2, and H3CO- can also be obtained from an ABG analysis. Answer C:Mixed venous oxygen saturation (SVO2) cannot be obtained from an ABG analysis. Rather, SVO2is obtained from venous blood returning to the right side of the heart, which can be accessed only via a central venous line. This reading is used to help estimate the amount of oxygen being extracted from the tissues. Answer D:ABG analysis provides much more than just oxygen saturation. In addition, the arterial pH, partial pressure of carbon dioxide (PaCO2), partial pressure of oxygen (PaO2), and bicarbonate (H3CO-) can all be obtained from ABG analysis. The main purpose of an ABG analysis in most cases is to determine the medical stability of a patient from a respiratory and ventilatory standpoint. Bottom Line:Arterial blood gas analysis is used to assess for problems in acid-base balance, ventilation, and oxygenation. Correlating with other clinical tests and measures, the results from this analysis can be used to determine acidosis versus alkalosis as well as respiratory causes versus metabolic causes.

A 65-year-old male was referred to physical therapy. The patient has been staying at a skilled nursing facility following a hip fracture. The physical therapist noted a stage 2 pressure ulcer on the sacral area, and weakness of the upper and lower extremities. As part of the treatment plan, what is the most appropriate instruction to give the patient and his family? a) a turning schedule every 30 minutes b) use of a donut shaped cushion for seating c) use of a pad or brief to prevent infection d) use of a firm mattress for better support

Correct answer: A As pressure ulcers/sores are caused by prolonged and unrelieved pressure on bony prominences, an essential part of pressure management is having positioning/turning schedules. Standard time interval for repositioning is about every 2 hours. Patients who are fragile or who have existing wounds may benefit from a more frequent turning schedule. Answer B:Donut-shaped cushions can increase your risk of getting a pressure ulcer by reducing blood flow and causing tissue to swell. Answer C:This may help prevent infection but will not assist in wound healing. Answer D:Using a firm mattress will provide pressure on the wound, which may worsen the condition. Bottom Line:As pressure ulcers/sores are caused by prolonged and unrelieved pressure on bony prominences, an essential part of pressure management is having positioning/turning schedules. The standard time interval for repositioning is about every 2 hours. Patients who are fragile or who have existing wounds may benefit from a more frequent turning schedule.

The physical therapist is performing a physical examination of a patient with a suspected visceral disorder. The therapist screens the abdomen for bowel sounds. Which component of the physical exam of the abdomen is being completed? A. Auscultation B. Inspection C. Palpation D. Percussion

Correct answer: A Auscultation is performed to listen for bowel sounds, which are normally made by the movement of food, liquid, and air through the intestines. Answer B:Inspection is the process of visually looking at the body for symmetry, alignment, skin color, and scars. Answer C:Palpation in the abdominal region is usually deep palpation (firm pressure) and is used to assess for tenderness and the presence of visceral organs. Answer D:Percussion uses sound to determine size, shape, and density of tissue. Bottom Line:Auscultation is performed to listen for bowel sounds, which are normally made by the movement of food, liquid, and air through the intestines. TrueLearn Insight :Auscultation is performed to listen for bowel sounds, which are normally made by the movement of food, liquid, and air through the intestines. Bowel sounds occur frequently, about every 2 to 5 seconds, although there is a lot of variability.

A physical therapist is completing an initial examination with a patient with a history of relapsing remitting multiple sclerosis. Which of the following classes of medication would the therapist expect the patient to take in order to decrease the frequency of exacerbations? a) Beta interferon b) Corticosteroids c) Immunoglobulin d) Muscle relaxants

Correct answer: A Beta interferon medications such as Avonex and Betaseron are FDA-approved medications used as disease-modifying therapies in the treatment of multiple sclerosis. Answer B:Corticosteriods are used for the acute treatment sclerosis exacerbations and not to reduce the frequency of exacerbations. Answer C:Immunoglobulins are used for the treatment of Guillain-Barré syndrome, which causes demyelination of the peripheral nervous system while multiple sclerosis causes deymyelination of the central nervous system. Answer D:Muscle relaxants are used to treat spasticity, which is an underlying symptom found in some patients with multiple sclerosis.

A physical therapist is treating a patient with a T6 ASIA A spinal cord injury who is in an inpatient rehabilitation facility. The patient reports having difficulty in making quick turns while propelling their wheelchair. Which of the following will improve wheelchair maneuverability for this patient? a) increase the camber of the back wheel b) increase the dump of the chair c) increase the size of the front wheels d) increase the floor-to-seat height of the wheelchair

Correct answer: A Camber is the angle of the back wheel in relation to the ground. Increasing the camber of a wheelchair makes it more maneuverable and increases lateral stability of the wheelchair. Answer B:Increasing the distance between the front of the seat compared to the back of the seat so that the back of the seat is lower (also known as "dump") does not affect the maneuverability of the wheelchair. Increasing the dump is used to improve a patient's sitting position in the wheelchair by making them more stable with respect to balance. Answer C:Increasing the diameter of the front wheels will decrease the maneuverability of the wheelchair since larger casters are more difficult to turn. Answer D:Increasing the floor-to-seat height does not affect the maneuverability of the wheelchair. Bottom Line:Camber is the angle of the back wheel in relation to the ground. Increasing the camber of a wheelchair makes it more maneuverable and increases lateral stability of the wheelchair.

A patient with congestive heart failure (CHF) regularly attends pulmonary rehabilitation. When arriving for today's session, the patient reports shortness of breath (SOB) at rest. Resting HR is 82, BP is 110/70, and respiratory rate is 23. Which of the following is the BEST course of action for the physical therapist? a) cancel today's session. Advise the patient to rest, and call the physician if symptoms persist b) have the patient go to the ER or urgent care c) have the patient participate in the pulmonary rehab session and monitor vital signs closely d) modify the session so the patient does not increase their HR or RR during the treatment

Correct answer: A Cancelling today's session and advising the patient to rest, and call the physician if symptoms persist, is the safest and most practical course of action. SOB with CHF is indicative of fluid buildup. The biggest concern would be the symptoms not resolving, so the patient should check with their physician if this occurs. Answer B:Though SOB at rest is a concern, it is not unexpected with CHF. The patient's other vital signs of HR and BP are WNL and for all of those reasons, this is not an emergency. Answer C:Though SOB at rest is not normal, it can occur with CHF and may be indicative that the condition is worsening. Participation in the session would increase the dyspnea and is not advised. The patient should go home and rest and call the physician if the SOB does not resolve. Answer D:With no change in HR or RR, there would be no therapeutic purpose to the session. In addition, it would be nearly impossible to avoid an increase in either HR or RR if the patient begins any activity in the rehabilitation session. Bottom Line:A patient with a cardiovascular or pulmonary condition should not engage in a rehabilitation session if they are symptomatic at rest. Shortness of breath with congestive heart failure is indicative of increased fluid buildup. It is not an emergency, but if it persists the physician should be contacted to determine if medications need to be adjusted or other measures need to be taken.

A physical therapist is treating a patient with a history of ruptured cerebral aneurysm. Which of the following procedures would be indicated to detect if the patient also has an unruptured aneurysm? a) cerebral angiography b) computerized tomography c) diffuse tensor imaging d) magnetic resonance imaging

Correct answer: A Cerebral angiography is a neurodiagnostic procedure that uses a contrast medium along with x-ray technology to detect abnormalities of cerebral blood flow, such as unruptured aneurysms and arteriovenous malformations. Cerebral angiography is also used to image the brain in conjunction with interventions to close an aneurysm, such as endovascular coiling. Answer B:Computerized tomography would be used to detect a burst aneurysm but would not be able to diagnose an unruptured aneurysm. Answer C:Diffuse tensor imaging is a research tool used to image the white matter tracts of the brain and but would not be able to diagnose an unruptured aneurysm. Answer D:Magnetic resonance imaging would be used to detect a burst aneurysm but would not be able to diagnose an unruptured aneurysm. Bottom Line: Cerebral angiography is a neurodiagnostic procedure that uses a contrast medium along with x-ray technology to detect abnormalities of cerebral blood flow such as unruptured aneurysms and arteriovenous malformations. Cerebral angiography is also used to image the brain in conjunction with interventions to close an aneurysm, such as endovascular coiling. Physical therapists must be knowledgable about the diagnostic procedures used to detect neuroanatomical abnormalities in order to understand the differential diagnosis process so that they can provide necessary information to their patients.

A physical therapist is examining a 10-year-old patient diagnosed with Duchenne's muscular dystrophy. The best anatomical location for measurement to determine lower chest wall excursion is which location? a) midpoint between the xiphoid process and umbilicus b) sternal angle c) xiphoid process d) midpoint between the sternal angle and the xiphoid process

Correct answer: A Chest wall excursion measurements made at the midpoint between the xiphoid process site and the umbilicus represent lower chest wall motion. Answer B:Chest wall excursion measurements made at the sternal angle may best represent upper chest wall motion. Answer C:Chest wall excursion measurements made at the xiphoid process site represent middle chest wall motion. Answer D:Chest wall excursion measurement made at midpoint between the sternal angle and the xiphoid process is not a standard measurement of chest wall excursion.

A physical therapist is treating a patient with a chronic T6 ASIA CA spinal cord injury who has a neurogenic bladder. The patient has been taking cholinergic agonists in order to assist with their bladder program. Which of the following adverse effects would need to be MONITORED in this patient scenario during treatment? A. Bradycardia B. Depression C. Muscle cramping D. Tremor

Correct answer: A Cholinergic agonists are commonly used to treat patient with a neurogenic bladder to prevent involuntary bladder contractions. Bradycardia is common adverse effect associated with cholinergic agonists and should be monitored in patients with complete spinal cord injury since they will have autonomic dysfunction due to the injury. Answer B:Depression is not a common adverse effect associated with cholinergic agonists but is commonly found in patients with spinal cord injury. Answer C:Muscle cramping is not a common adverse effect associated with cholinergic agonists but may be related to spasticity in patients with spinal cord injury. Answer D:Tremor is not a common adverse effect associated with cholinergic agonists but may be related to muscle fatigue in patients with spinal cord injury.

Which of the following infection transmission routes might accompany a syndrome that requires a therapist to use a mask with face shield, gown, and two pairs of gloves in order to work with the patient? a) contact b) droplet c) airborne d) airborne plus contact

Correct answer: A Contact syndromes such as Zika virus have the most dangerous mode of transmission and would require a face shield and double gloves for any medical professional working with the client. Other contact syndromes also require gown and gloves, although ones such as methicillin-resistant Staphylococcus aureus (MRSA) do not require a mask. Answer B:Droplet infection includes mumps and streptococcus A and requires a mask when within 3 feet of the patient,,and neither gloves or gown unless lesions are present. Answer C:Airborne infections include measles and tuberculosis and do require a mask but do not require gown or gloves. Answer D:Airborne plus contact includes chickenpox and smallpox and requires a mask, gown, and gloves, but not a face shield or double gloves. Bottom Line:The most dangerous syndromes are those that are contact transmissible and include the Zika virus. These require any medical professional to take extreme precautions, including a face mask, double gloves, and gown at all times. Droplet requires additional precautions when within 3 feet of the client, and airborne does not require gloves or gown. Knowing the mode of transmission of the infection can inform any healthcare professional the appropriate level of personal protective equipment needed at a minimum.

Which of the following interventions is the MOST appropriate for airway clearance for a 32-year-old male with a history of cystic fibrosis? a) autogenic drainage b) bi-level positive airway pressure (BIPAP) c) incentive spirometry d) lymphatic drainage

Correct answer: A Cystic fibrosis is a genetic condition that affects mainly the lungs, endocrine, and digestive systems characterized by excessive secretions. Airway clearance techniques for cystic fibrosis patients are necessary to maintain pulmonary hygiene and should be performed daily. Some of the many techniques are postural drainage, chest percussion, positive expiratory pressure, and high frequency oscillatory devices. Effectiveness of the different techniques depends on the individual's needs. Autogenic drainage is a technique that involves a series of controlled breathing periods to mobilize secretions. It uses breathing at different lung volumes to loosen, mobilize, and move secretions in three stages toward the larger central airways. It requires a great deal of effort and practice to perform correctly. It is an effective method of airway clearance, especially if the patient is able to actively engage in treatment. Answer B:BiPAP is a form of mechanical ventilation that provides positive pressure support during inspiration and expiration. BiPAP is not used for airway clearance. It is typically used for sleep apnea as well as in cases where a patient requires pressure support during respiratory failure. Answer C: Incentive spirometry is not primarily used for airway clearance. Although it can promote deep breathing and good pulmonary hygiene, its use is primarily for increasing tidal volume. Answer D: Lymphatic drainage is not directly related to airway clearance.

A physical therapist is working with a patient who has a diagnosis of atelectasis. The patient exhibits tachypnea, has excess pulmonary secretions, and is very anxious. Which of the following breathing strategies is MOST appropriate to use with this patient? A. Diaphragmatic breathing B. Pursed lip breathing C. Stacked breathing D. Paced breathing

Correct answer: A Diaphragmatic breathing is most appropriate for individuals with atelectasis, tachypnea, anxiety, and excess pulmonary secretions with a goal of achieving eupnea, improving oxygen levels, decreasing anxiety, and mobilizing the secretions. Answer B:Pursed lip breathing is most appropriate for individuals with dyspnea at rest or with exertion and wheezing. The goal is to improve activity tolerance and reduce wheezing. Answer C:Stacked breathing is used for hypoventilation, atelectasis, and ineffective cough. The goal of stacked breathing is to improve ventilation and perfusion, resolve the atelectasis, and improve the effectiveness of the cough. Answer D:Paced breathing is specifically for individuals with poor endurance, especially with activities of daily living and ambulation. It is also used for individuals with dyspnea on exertion, fatigue, and anxiety but is not used for atelectasis or excess secretions. The goal is to increase activity tolerance and reduce fatigue and anxiety. Bottom Line:Different breathing strategies can be used for different indications. Although anxiety and dyspnea can be managed with paced breathing, the presence of atelectasis would warrant diaphragmatic or stacked breathing. The presence of secretions makes diaphragmatic breathing the best option for this patient.

A physical therapist carefully applies a downward pressure to a patient's head while the patient is sitting with the head positioned in neutral. The patient reports a reproduction of their symptoms of radiating pain into the right arm. What test should the physical therapist perform NEXT? a) distraction in neutral b) foraminal compression in right extension quadrant c) distraction in right extension quadrant d) foraminal compression in right flexion quadrant

Correct answer: A Distraction in neutral is the correct answer since the reproduction of radicular symptoms has already occurred with compression with the head in neutral and is indicative of a cervical radiculopathy. There is no need to cause further compression to the area since symptoms have already been reproduced. Since compression causes symptoms, the best follow-up is to see if distraction relieves the symptoms. Answer B:This answer is incorrect as this is appropriate only if neutral position was not provocative. Answer C:This answer is incorrect since the test was positive in neutral. There is no reason to perform distraction in this quadrant. Answer D:This answer is incorrect as this is appropriate only if neutral was not provocative. Bottom Line:A cervical compression test looks at a possible diagnosis of cervical radiculopathy. If radicular pain is reproduced with the head in neutral, it is helpful to see if distraction helps the symptoms. If no pain was reproduced in neutral, the head can be positioned to compress the foramina further to see if symptoms are reproduced with cervical rotation, sidebend, and extension.

A patient is attending his second session of physical therapy for acute back pain. The pain shoots down the back of the leg when bending forward. Active range of motion revealed pain down the leg with forward bending and less intense pain when bending backward. Passive flexion range of motion also caused pain down the back of the leg, while passive extension caused mild pain in the lower back. Which of the following interventions is MOST appropriate for this patient based on the information provided?

Correct answer: A Dura inflammation stemming from injuries such as a herniated nucleus pulposus are commonly irritated with flexion and improved with extension. The acuity of the injury can make active extension aggravating, and the individual typically improves initially with passive extension with a progression toward active movements. Prone on elbows for 5 minutes is the correct answer as this is a static (passive) position that can be modified to become easier or harder but will not activate the muscular system. Answer B:A prone press-up will activate the muscles as it is active assisted to improve repetitive extension. Answer C:Standing repeated extensions is an active movement and will aggravate the patient with acute symptoms. Answer D:Prone lying with a pillow under the hips will cause flexion, which is painful, rather than extension.

During observational gait analysis of a 69-year-old male, the physical therapist notes that the patient consistently leans posteriorly during initial contact (heel strike). This finding is present bilaterally. Which of the following muscles is MOST likely to be weak and should be tested manually? a) gluteus maximus b) gluteus medius c) iliopsoas d) quadriceps

Correct answer: A During initial contact (heel strike), the hip extensors (primarily gluteus maximus) are active and generate a large amount of force in order to maintain stability during the stance phase. A common compensation for a weak gluteus maximus is to lean backward at that phase to prevent the trunk from falling/flexing forward due to the ground reaction force (GRF) anterior to the hip. Leaning backward moves the GRF posterior to the hip joint and creates an extension moment which prevents the trunk from falling forward unsupported. Therefore, in this scenario, the gluteus maximus is likely to be weak and should be manually tested. Answer B:The gluteus medius is most active during single limb support of the stance phase. It is critical in maintaining stability in the frontal plane by allowing for a stable pelvis and preventing hip drop. However, weakness of this muscle would not cause leaning back during weight acceptance. Therefore, this answer is incorrect. Answer C:The iliopsoas muscle, which is a hip flexor, is critical during the swing phase of gait. Adequate foot clearance requires the ability to initiate hip flexion during initial swing. However, this muscle would not be involved in deviations seen during the stance phase of gait. Answer D:Weakness of this muscle would cause an anterior lean during the stance phase to move the GRF in front of the knee joint and create an extension moment, to prevent knee buckling. Bottom Line:During gait, leaning back at initial contact is a compensation for a weak gluteus maximus. Leaning forward is a compensation for weak knee extensors.

A patient presents to the outpatient clinic with osteoarthritis of the right knee. The physical therapist notices the patient lacks 10 degrees of knee extension PROM. Which of the following motions should the therapist expect to be LIMITED upon further examination? a) anterior glide of proximal tibia and tibial ER b) anterior glide of proximal tibia and tibial IR c) posterior glide of proximal tibia and tibial ER d) posterior glide of proximal tibia and tibial ER

Correct answer: A During knee extension the proximal tibia glides anteriorly and also spins into external rotation in the last few degrees of knee extension, called the screw home mechanism. Answer B:This is incorrect as the direction of the glide is appropriate, but the direction of rotation is incorrect. The tibia externally rotates at end range of extension. Answer C:This is incorrect as this direction of glide would promote knee flexion ROM. Answer D:This is incorrect as this direction of rotation would promote knee flexion

Which of the following interventions is MOST appropriate for a patient who does not advance the tibia over the foot in stance phase and instead shortens the stance phase and moves that limb earlier into swing? A. Posterior glide of talus B. Bridging exercises C. Calf raises D. Hamstring stretches

Correct answer: A During stance phase, the ankle dorsiflexes in closed chain as the tibia advances over the foot from the start of midswing to the end of terminal swing. In order to perform that action, the ankle must have the range of motion available. If it does not, the issue could either be shortness/tightness of plantar flexors or restriction in talocrural joint mobility.Of the options here, only the posterior mobilization of the talus increases dorsiflexion range of motion. Answer B:Bridging will strengthen the hip extensors, which does not alter dorsiflexion range of motion. Weak hip extensors would not affect closed chain dorsiflexion. Answer C:A weak gastrocsoleus would not cause this gate deviation. Calf raises strengthen the gastrocnemius and soleus but do not increase dorsiflexion range of motion. Answer D:A tight hamstring will not limit ankle dorsiflexion range of motion, and so stretching the hamstring will have no effect. Bottom Line:During stance phase, the ankle dorsiflexes in closed chain as the tibia advances over the foot from the start of midswing to the end of terminal swing. In order to perform that action, the ankle must have the ROM available. If it does not, the issue could either be shortness/tightness of plantar flexors or restriction in talocrural joint mobility.

When educating a patient about the benefits of diet and exercise for patients with diabetes mellitus, the therapist keeps in mind that which of the following is NOT a benefit? a) increases the body's ability to produce insulin b) decreases blood glucose levels c) reduces insulin resistance d) increases insulin sensitivity

Correct answer: A Exercise has many positive effects, such as increasing insulin sensitivity and reducing insulin resistance. However, it does not actually enable the body to produce more insulin. Answer B: Exercise increases the effectiveness of moving glucose from the blood into the cells, thus reducing the blood glucose levels. Muscle contractile activity and exercise improve skeletal muscle glucose transport, which contributes to glucose homeostasis in the blood. Answer C: Exercise reduces the resistance of the cells to insulin, allowing glucose to be transported into the cells from the bloodstream. Answer D: Increased insulin sensitivity occurs after exercise and can last 12-72 hours. It allows the body to use the available blood glucose, thus lowering blood glucose levels. Bottom Line:Exercise has many benefits in the diabetic population, including increasing insulin sensitivity, decreasing insulin resistance, and mobilizing blood glucose to the cells, resulting in glucose homeostasis in the blood. Exercise does not enable the body to produce more insulin.

A patient with a whiplash injury initially scored a 15/50 on the Neck Disability Index (NDI). Research indicates that the minimal detectable change (MDC) for the NDI is 5 points. After 4 weeks of physical therapy, the patient completed another NDI and scored 4/50. How should this data be interpreted? a) statistically significant improvement greater than the measurement error b) statistically significant worsening of symptoms greater than the measurement error c) patient did not score at least five points so change cannot be detected d) patient score indicates almost complete disability

Correct answer: A For many outcome measures that are self reports on functional status, like the Neck Disability Index and the Oswestry low back pain disability questionnaire, higher scores mean more pain and disability. This question also requires the tester to understand that minimal detectable change (MDC) detects a true change and not one due to measurement error.The neck disability index (NDI) is a common self-report functional status questionnaire. It is reliable and valid. Each of the 10 questions is scored 0 to 5, where zero indicates "no pain." The score is reported out of 50 points.The areas of assessment are: Pain intensity Personal care Lifting Reading Headaches Concentration Work Driving Sleeping Recreation Since the minimal detectable change (MDC) for the NDI is five, then a change in at least 5 points is not attributed to measurement error or chance and is significant. In this example, the score decreased from 15 to 4, which is a change of 9 points and shows the patient started with mild disability and is now almost pain-free. Answer B:An NDI score that goes from a higher number to a lower number shows a reduction in pain, and an improvement in the patient's symptoms. Answer C:The MDC refers to a change in the score, not the actual score. Therefore, if the score itself is smaller than the number 5, that is not addressing the issue of the MDC. To determine if there was a change (for better or for worse), one must compare two scores. In this case the score went down from 15 to 4, a change of 9 points and an improvement. Answer D:The NDI is an adaptation of the Oswestry and for each of the ten categories, as the pain increases, the number increases. Therefore, this patient's NDI score of 4/50 is nowhere near the score of complete disability, which would be considered anything higher than 35/50. Bottom Line:The neck disability index (NDI) is a common self-report functional status questionnaire. It is reliable and valid. Each of the 10 questions is scored 0 to 5, where zero indicates "no pain." The score is reported out of 50 points.

A physical therapist is following a 20-year-old female with a diagnosis of low back pain. During a scheduled physical therapy visit, she reports she has nausea and diarrhea that just began yesterday and that it kept her up in the middle of the night. She has taken antacids for relief. She now complains of right and left lower quadrant abdominal pain. The therapist decides to examine her abdomen through auscultation and palpation. The therapist confirms the patient has hyperactive bowel sounds and guarding of the abdomen. The therapist is correct in determining the client needs to be referred for follow-up secondary to suspicion of which of the following DIAGNOSES? a) viral gastroenteritis b) pneumoperitoneum c) appendicitis d) pelvic inflammatory disease

Correct answer: A Gastroenteritis is an inflammation of the large intestine caused most commonly by a norovirus infection. The virus is typically spread in contaminated food and water but may also be airborne. Viral gastroenteritis has a sudden onset and generally lasts 2-4 days. The illness is usually characterized by diarrhea and abdominal cramps/pain, nausea, and vomiting. The stools are typically loose, watery, and without blood or mucus. This patient reported diarrhea as a symptom, so viral gastroenteritis is likely to be the cause of her abdominal pain. Answer B:The patient presentation, history, and physical examination do not support a diagnosis of pneumoperitoneum. Pneumoperitoneum is pneumatosis (abnormal presence of air or other gas) in the peritoneal cavity, a potential space within the abdominal cavity. The illness is usually characterized by hypoactive bowel sounds, guarding of the superior and anterolateral abdominal wall, and tenderness of the epigastric and left hypochondrial regions during deep palpation. This patient did not report epigastric pain as a symptom, so pneumoperitoneum is not likely to be the cause of her abdominal pain. Answer C:The patient presentation, history, and physical examination do not support a diagnosis of appendicitis. A patient with appendicitis typically presents with diffuse paraumbilical discomfort that is consistent with the distribution of referred pain from the appendix. The pain would be severe and precisely located in the RLQ, indicating that the inflamed appendix has irritated the adjacent parietal peritoneum. Guarding and rebound tenderness would potentially be present as well as positive McBurney and obturator signs, further indicating involvement of the peritoneum. Answer D:The patient presentation, history, and physical examination does not support a diagnosis of pelvic inflammatory disease (PID). Pelvic inflammatory disease is initiated by infection, usually sexually transmitted, that moves superior from the vagina and cervix into the uterus and uterine tubes. Women with PID present with a wide variety of symptoms, but the most common presenting complaint is lower abdominal pain. Most women (75%) also exhibit abnormal vaginal discharge. The diagnosis of acute PID is based primarily on patient history of previous PID and clinical findings and is not consistent with this patient's information.

A physical therapist is working with a patient 2 days post total knee replacement. The therapist is performing goniometry measurements and states that the patient is unable to achieve full extension by 10 degrees and is able to flex the knee to 90 degrees. Which of the following notations of these measurements would be correct? A. 10 to 90 degrees of flexion B. -10 to 90 degrees of flexion C. 90 degrees of flexion D. 10 to 0 to 90 degrees of flexion

Correct answer: A Goniometer measurements are never given in negatives. A patient who is unable to achieve full extension by 10 degrees but can perform 90 degrees of flexion would be documented as 10-90 degrees of flexion. Answer B:Negatives are not used to document goniometry, so using -10-90 degrees would be an incorrect documentation and would not tell the reader the range of motion available to the patient. Answer C:90 degrees of flexion is incorrect because the patient started the flexion movement at 10 degrees, so technically has only 80 degrees of flexion in the knee at this time. Answer D:10-0-90 would be the correct way to document a joint that has 10 degrees of hypermobility and then passes through 0 to achieve 90 degrees. In this case, the patient would have 100 degrees of flexion since they started at 10 degrees hyperextended. Bottom Line: Goniometry measurements do not include negatives. The measurement is written based on the movement, in this case, flexion. The starting point is where the patient commences the movement, in this case, 10 degrees less than full extension. So the patient is already in 10 degrees of flexion. The flexion would be documented as 10-90 degrees.

Which of the following describes the most appropriate location for auscultation of the mitral valve? a) halfway along the length of the clavicle at the level of the fifth intercostal space b) left sternal border at the level of the second intercostal space c) left inferior sternal border near the level of the fourth intercostal space d) right sternal border at the level of the second intercostal space

Correct answer: A Halfway along the length of the clavicle at the level of the fifth intercostal space is the correct answer because it describes the location of the mitral area, where blood flow would arrive from the mitral valve. Abnormalities of the mitral valve can be best appreciated from this location.Answer B:This answer describes the location of the pulmonic area, where sounds generated by the pulmonic valve are best heard. Answer C:This answer describes the location of the tricuspid area, where sounds generated by the tricuspid valve are best heard. Answer D:This answer describes the location of theaortic area, where sounds generated by theaortic valveare best heard. Bottom Line:Auscultation of heart sounds produced by the heart valves is performed over areas of the chest that correspond to where blood is heading, not the anatomical position of the valves. Therefore, it is helpful to note the path of blood flow throughout the cardiac cycle in order to learn the appropriate locations where valve sounds can be best heard. The mitral valve is best appreciated halfway along the length of the clavicle at the level of the fifth intercostal space.

A patient reports a 2-week history of right shoulder pain after lifting several heavy boxes when moving. He has full active and passive range of motion, tenderness to palpation over his anterior shoulder, and normal strength with manual muscle testing. The physical therapist suspects bicipital tendinitis and wants to perform Speed's test to confirm. Which of the following is the proper procedure for Speed's test? A. Patient positioned with elbow extension, forearm supination; resist shoulder flexion B. Patient positioned with elbow flexion at 90 degrees; resist further elbow flexion C. Patient positioned with 90 degrees abduction, slight medial rotation; resist abduction D. Patient positioned with elbow flexion at 90 degrees, forearm pronated; resist supination and external rotation of shoulder

Correct answer: A Having the patient positioned with elbow extension and forearm supination, then resist shoulder flexion, is the proper technique for performing Speed's test. A positive test elicits pain in the bicipital groove. Answer B:Patient positioned with elbow flexion at 90 degrees; resist further elbow flexion is used for testing biceps strength. Answer C:Patient positioned with 90 degrees abduction, slight medial rotation; resist abduction is the positioning for the empty can test to detect pathology of the supraspinatus muscle. Answer D:Patient positioned with elbow flexion at 90 degrees, forearm pronated; resist supination and external rotation of shoulder is the position for Yergason's test for bicipital tendonitis. The test is designed primarily to check the ability of the transverse humeral ligament to hold the biceps tendon in the bicipital groove. The examiner palpates the bicipital groove during the test, and it will feel like the tendon "pops out" of the groove, indicating the ligament is torn. Tenderness in the groove alone without the popping out may indicate tendonitis. However, Speed's test is more effective than Yergason's for testing for tendonitis specifically. Bottom Line:Many special tests may be used during examination of the shoulder to either confirm or rule out a certain diagnosis. Speed's and Yergason's are two tests used when suspecting biceps pathology. Speed's has a higher sensitivity when testing for bicipital tendonitis.

A team physical therapist observes a player getting injured during a football game. The player prepared to throw a football overhead, and he was tackled while the arm was ready to throw, with his arm being pulled back behind him by the opposing player. The patient heard a loud "pop" and was then unable to use his arm and had severe pain. Based on this mechanism of injury, which of the following injuries does the physical therapist suspect? a) anterior glenohumeral dislocation b) posterior glenohumeral dislocation c) anterior glenohumeral subluxation d) SLAP lesion (superior labral tear)

Correct answer: A Having the shoulder in a position of abduction and external rotation to throw a football, and then adding a violent force into external rotation. forces the head of the humerus in an anterior direction. This is a typical mechanism of injury for an anterior shoulder dislocation. The loud pop and inability to use the arm are signs of anterior glenohumeral dislocation. The individual would be unable to use his arm until the humeral head was relocated back into the glenoid, which is urgent. Answer B:The position for a posterior dislocation is flexion, adduction, and internal rotation with a posteriorly directed force. Answer C:The humeral head would self-reduce if it was only a subluxation, and the patient would be able to use his arm (although guarded/painful) depending on the severity. Answer D:A SLAP lesion could produce a popping sound, but the patient would still be able to use his arm after the injury. In addition, the mechanism of injury here is more likely to result in an anteriorly directed force to the humeral head, causing a dislocation. Bottom Line:An anterior glenohumeral dislocation occurs when the arm is placed in a position of excessive abduction and external rotation and there is a violent force pulling the arm back into more external rotation. The individual may report a "pop" sound and will be unable to use their arm until the dislocation is reduced.

A patient who experienced a left cerebrovascular accident with right hemiplegia is being evaluated by the physical therapist. During the course of the evaluation, the therapist tells the patient to close their eyes and then places a key into their right hand and asks the patient to identify the item. Which SENSORY FUNCTION is being tested by the therapist? A. Stereognosis perception B. Proprioceptive awareness C. Tactile localization D. Barognosis recognition

Correct answer: A Identifying an item by feel is a test for stereognosis perception. The patient is asked to name a familiar item through feel alone by identifying the size and shape. Answer B:Proprioceptive awareness is awareness of joint position sense and awareness of joints at rest. To test this, a joint is moved, and the patient describes the movement to the therapist. Answer C:Tactile localization is the ability to localize sensation on the skin. For example, the patient is asked to identify where on their body, the tip of a finger or the elbow, an item, such as a cotton swab, is being placed. Answer D:Barognosis recognition is the ability to discriminate weight. To test this, items of the same size and shape but different weights are given to the patient, and the patient is asked to compare the weights. Bottom Line:Identifying an item by feel is a test for stereognosis perception. The patient is asked to name a familiar item through feel alone by identifying the size and shape. Stereognosis perception is one of many sensory concepts that can be lost in clients due to neurological deficits. TrueLearn Insight :Stereognosis perception is one of many sensory concepts that can be lost in clients due to neurological deficits. It is a critical concept allowing the client to find items in their pocket or purse, and also identify items that may be dangerous, just by feel.

A physical therapist in the intensive care unit (ICU) briefly reviews the telemetry monitor before seeing a patient status post coronary artery bypass graft surgery. The therapist notices that the monitor shows inconsistent P waves. What does a P wave represent on an electrocardiogram? a) atrial depolarization b) atrial repolarization c) ventricular depolarization d) ventricular repolarization

Correct answer: A In a normal electrocardiogram, a P wave, representing atrial depolarization, marks the beginning of the cardiac cycle. It is then followed by the QRS complex, representing ventricular depolarization. Answer B:The P wave is the first electrical activity of the heart and is the depolarization of the atria. When the atria repolarize, this is not really seen on the ECG because it is hidden in the QRS complex, which records ventricular depolarization. Answer C:Ventricular depolarization is represented by the QRS complex, not P wave. Rather, P waves indicate atrial depolarization. Answer D:The T wave records the electrical activity of ventricular repolarization, not the P wave. Bottom Line:The electrical activity of the heart can be recorded and represented by electrocardiography. In a normal electrocardiogram, P waves represent atrial depolarization, which is essential for the start of the cardiac cycle. The QRS complex represents ventricular depolarization, and the T wave represents ventricular repolarization. Atrial repolarization also occurs during the QRS but is hidden by the electrical activity of the ventricles depolarizing. Any absence or irregular intervals of any waves should be considered abnormal and are likely indicative of a conduction problem.

A 37-year-old male with a history of a C7 complete spinal cord injury was admitted to the hospital due to community-acquired pneumonia. Physical therapy was consulted to help the patient maximize his functional capacity and to prevent deconditioning in the hospital. During the physical therapy examination, the patient presents with a slumped and kyphotic posture in sitting, rhonchi heard in the middle lung fields, decreased breath sounds in bilateral lung bases, and a weak cough. Which of the following interventions is the MOST appropriate at this time and should be performed FIRST? a) assisted cough techniques b) incentive spirometry c) pursed-lip breathing d) therapy band exercises for upper extremities

Correct answer: A In this patient presentation, a slumped and kyphotic posture would lead to very poor chest wall mechanics, making inspiration and expiration more difficult. Effective airway clearance would also be limited. In this case, the patient is presenting with rhonchi, which is suggestive of an airway obstruction or secretions, as well as a weak cough. Both of these findings should alert the therapist of an airway clearance problem. Helping the patient achieve airway clearance through assisted cough techniques is essential in order to allow for optimal breathing patterns as well as subsequent ability to participate in functional mobility. Answer B:The patient's postural presentation (slumped and kyphotic) is not uncommon for patients with this level of spinal cord injury. This tendency prevents optimal chest wall movements and consequently decreases lung capacity. Encouraging chest wall expansion with incentive spirometry is indicated; however, it is not the correct answer because airway clearance in this scenario should take precedence. After that is achieved, incentive spirometry should then be encouraged. Answer C:Pursed-lip breathing is a breathing strategy that is most helpful in someone with an obstructive breathing pattern, that is, with difficulty in expiration such as in chronic obstructive pulmonary disease (COPD). Pursed-lip breathing is also useful in slowing and normalizing breathing patterns when a patient is hyperventilating. However, it is not effective in cases where airway clearance is needed, which is the case in this scenario. Answer D: Strengthening upper extremity muscles will help build stability in the upper trunk and chest wall, which would ultimately help improve this patient's postural stability as well as chest wall mechanics. However, this is not the correct answer because strengthening activities do not immediately contribute to airway clearance, which is what is needed FIRST in this patient. Bottom Line:The presence of rhonchi and a weak cough should prompt a therapist to perform assisted cough techniques to ensure airway clearance.

The concept that "excitation from stronger motor units will flow to recruit weaker or inhibited motor units" is the principle behind which of the following proprioceptive neuromuscular facilitation (PNF) principles or techniques? a) irradiation b) approximation c) quick stretch d) dynamic reversals

Correct answer: A Irradiation is a PNF technique that occurs when the overflow of neuronal excitation from strong motor units flows to weaker or inhibited motor units. Irradiation can occur in any direction and across any segment of the body. Answer B:An approximation is a compression of the joint of the extremity or spine to facilitate muscle responses in the extensor patterns or during stabilization. Answer C:Quick stretch is the use of an elongated position of a muscle and the stretch reflex to initiate dynamic movements and facilitate contraction through increased motor unit recruitment. Answer D:Dynamic reversals use isotonic concentric contractions of the agonists and then the antagonists and improve intramuscular and intermuscular coordination. Bottom Line:The concept of irradiation is that the excitation of a muscle and its motor units can flow throughout the body to increase motor unit recruitment in other areas.

Which of the following tests or procedures is the MOST appropriate for the accurate taking of skinfold caliper measurements? a) continue to pinch the skinfold site while reading the caliper number b) read the skinfold measurement within one second of setting the caliper c) take all measurements on the left side of the body d) take one measurement at each site

Correct answer: A It is important to continue pinching the skinfold site while reading the caliper, as otherwise the thickness of the skinfold pinch can change, which will lead to an inaccurate reading. Answer B:Waiting 1-2 seconds before reading the caliper to obtain the most accurate reading. Answer C:Proper skinfold measurement protocol is for all measurements to be taken on the right side of the body. Answer D:Multiple measurements should be taken at each site to make sure that the reading obtained was accurate. Bottom Line:Proper caliper skinfold measurement procedure requires maintaining the pinching of the skinfold site while reading the caliper in order to ensure an accurate reading.

A patient is referred to physical therapy with chief complaint of low back pain as well as left hip pain. Which of the following examination findings during gait analysis are MOST consistent with weakness of the left gluteus medius? a) LEFT lateral trunk lean during mid-stance on the LEFT lower extremity b) RIGHT lateral trunk lean during mid-stance on the RIGHT lower extremity c) LEFT hip drop during mid-stance on the RIGHT lower extremity d) RIGHT hip drop during mid-stance on the RIGHT lower extremity

Correct answer: A It is important to have a good understanding of the biomechanics of lower extremity muscles and joints throughout the gait cycle. This knowledge is required in order to correctly rule in or rule out involvement of relevant components during gait analysis and musculoskeletal examination. Weakness of the gluteus medius can result in various gait deviations, depending on the presence of functional compensations. Of the choices presented here, left lateral trunk lean during mid-stance on the left lower extremity is the correct answer because it describes a compensated Trendelenburg gait due to weakness of the left gluteus medius.As a compensation, a common learned behavior is to lean the trunk laterally past midline toward the weak side. This motion decreases the moment arm during mid-stance of the body weight trying to drop that weak side of the pelvis by moving the line of gravity closer to the hip joint. Therefore, the force requirement for the gluteus medius on that weak side to keep the pelvis level is reduced.Alternatively, weakness of the left gluteus medius could lead to a right hip drop (pelvis falling on the right) during mid-stance on the left lower extremity due to an inability to overcome the large moment created by the pelvis and the rest of the body. This would be described as an uncompensated Trendelenburg gait. However, that was not one of the options here. Answer B:RIGHT lateral trunk lean during mid-stance on the RIGHT lower extremity describes a compensated Trendelenburg gait that is more consistent with right gluteus medius weakness. Answer C:LEFT hip drop during mid-stance on the RIGHT lower extremity describes an uncompensated Trendelenburg gait owing to right gluteus medius weakness. Answer D:A right hip drop would occur with left gluteus medius weakness. However, it would be seen when the patient was in left mid-stance, not right mid-stance. When in unilateral stance, the line of gravity tries to drop the hip on the unsupported side of the pelvis. It is up to the gluteus medius on the stance leg to hold up that contralateral side of the pelvis. Thus, a weak left gluteus medius would have difficulty keeping the pelvis level during left midstance. Bottom Line:Weakness of the gluteus medius on one side may result in either a contralateral hip drop or a trunk lean toward the weak side. The compensation would be seen when the weak lower extremity is in the mid-stance phase of gait.

A patient has a resting BP of 130/70. Which of the following blood pressure recordings taken after a brisk 5-minute walk on the treadmill is considered an ABNORMAL response to exercise? a) 140/60 b) 140/70 c) 145/80 d) 150/80

Correct answer: A It is normal to see a systolic BP increase of 10-20 mmHg after 5 minutes of moderate exercise. Diastolic BP may remain the same or increase slightly with exercise. A diastolic BP that drops during exercise, such as 140/60, is considered abnormal. Answer B:A 10 mmHG increase in systolic and a diastolic that is unchanged is a normal response to moderate exercise. Answer C:A 15 mmHG increase in systolic and a 10 mmHg increase in diastolic is a normal response to moderate exercise. Answer D:A 20 mmHG increase in systolic is possibly bordering abnormal, and a 10 mmHg increase in diastolic is a normal response to moderate exercise. Bottom Line:Systolic BP increases linearly with the intensity of exercise because blood gets directed out to the muscles, and the pressure within the vessels delivering that blood increases. Diastolic BP may remain the same or increase slightly with exercise. A diastolic that decreases, such as the 140/60 reading, is considered abnormal.

A patient is having dyspnea. Which of the following is the MOST appropriate suggestion to decrease the dyspnea? a) lean forward with upper extremity support on your lap or a table b) breathe with a wide-open mouth to increase airflow c) take shorter, more frequent breaths d) hug a pillow against your abdomen to give some counter pressure to the diaphragm

Correct answer: A Leaning forward with UE support positions the thorax so it can hold the maximal amount of air and also may improve the function of a flattened diaphragm. Answer B:Breathing through the mouth loses the warmth, humidification, and filtering properties of the nasal passages and is counterproductive. It is better to do pursed lip breathing with a long exhalation phase, which keeps the airway passage open longer to get rid of CO2. Answer C:CO2buildup can cause dyspnea. It is important to take deep breaths and fill the lungs, rather than shallow, more frequent breaths because one can expel more CO2with deep breaths. Answer D:Hugging a pillow against the abdomen is a technique to reduce the pain of coughing after a sternal incision. It does not help with dyspnea. Bottom Line:Dyspnea is shortness of breath (SOB) and can occur at rest or on exertion with various cardiopulmonary conditions. The strategies to manage SOB include leaning forward with a straight back with the arms supporting the trunk and pursed lip breathing.

A physical therapist is treating a client with a history of Parkinson disease. The therapist notes the client is taking levodopa. The therapist would be correct in identifying levodopa as belonging to which of the following drug categories? A. dopamine precursors B. dopamine agonists C. anticholinergic drugs D. COMT inhibitors

Correct answer: A Levodopa, one of the most common drugs used for the treatment of Parkinson disease, is a dopamine precursor. Answer B:Dopamine agonists are used for treating Parkinson disease, but are drugs such as pergolide. Levodopa does not fall into this category. Answer C:Anticholinergic drugs such as benztropine are used for treating Parkinson disease, but levodopa does not fall into this category. Answer D:COMT inhibitors are used for the treatment of Parkinson disease, but these include entacapone. Levodopa does not fall into this category.

A physical therapist is working with an 80-year-old patient who is extremely fearful of falling. The patient demonstrates a loss of balance with only a very small perturbation by the therapist. Which of the following strategies should the therapist address FIRST in this patient? a) ankle b) hip c) stepping d) reaching

Correct answer: A Loss of balance with a minor perturbation is a sign of limited ankle strategy. Since this strategy is the first to be used to prevent loss of balance, working on ankle strategies first is critical with this patient. In addition, their fear of falling means that small challenges should be introduced and once mastered, progress to greater ones. Addressing the other strategies first would require greater perturbations, which would only increase this patient's fear. Answer B:The hip strategy is the response to a forceful disturbance, usually when standing on a narrow surface. Older adults often use hip strategies instead of the ankle strategies, but in someone fearful of falling, training the ankle strategies should be considered first. The larger perturbations needed to activate hip strategies would not be good for a patient with fear of falling. Answer C:Stepping strategies are a compensatory reaction to large perturbations, and older adults often need several steps to recover their balance. This would not be a good initial training for someone fearful of falling. Answer D:Reaching strategy should be used as a compensatory reaction to large perturbations so should not be the first strategy to be addressed in a patient fearful of falling. Bottom Line: Ankle strategies are crucial for fall prevention and should be addressed early in training for fall prevention, particularly in an individual fearful of falling.

A patient had a right lower lobe segmentectomy 2 days ago, and the chest tube has been removed. Which position should the therapist tell the patient to avoid? a) left sidelying, propped on pillows b) supine head of bed at 45 degrees c) right side lying, propped on pillows d) sitting with feet dangling

Correct answer: A Lying on the left will allow fluid from the right to drip to the left. If a surgical procedure was performed on the right lung and a portion was removed, one should not let that fluid travel to the other lung and become trapped in the body. Answer B:Supine with head elevated is an acceptable position in this scenario. Answer C:Lying on the right may be uncomfortable but is perfectly acceptable. The chest tube was removed, and this position would keep the serosanguinous fluid away from the healthy lung. Answer D:Sitting with feet dangling is an acceptable position in this scenario. Bottom Line:Gravity will cause fluid to move to whatever part of the body is in the most dependent position. Postural drainage techniques utilize this concept, but it can also be applied to a patient post lobectomy to prevent fluid from traveling to other parts of the lung. For example, lying on the left will allow fluid from the right to drip to the left. If a surgical procedure was performed on the right lung and a lobe was removed, one should not let that fluid travel to the other lung and become trapped in the body.

A patient status posts mastectomy with lymph node removal complains of heaviness, tightness, and sensory changes in the right upper extremity. The therapist inspects and performs circumferential measurements of both upper extremities, and determines there is no swelling present. The therapist is correct in documenting the patient is in WHICH of the following stages of lymphedema? a) Stage 0 b) Stage 1 c) Stage 2 d) Stage 3

Correct answer: A Lymphedema stage 0 is also known as the subclinical stage. Lymph transport is impaired, but swelling is not evident. The patient may report heaviness, tightness, or sensory changes in the affected limbs. Answer B:In lymphedema stage 1, edema is present and may be variable (i.e., decrease with rest or sleep and increase with activity). The edema is very soft and easily pits with little to no fibrosis. Elevation of the limb results in dissipation of the edema. Answer C:In lymphedema stage 2, edema is present, but the edema no longer dissipates with elevation of the limb. Palpation is positive for fibrosis andresults in indentation of the skin. Fibrosis ranges from soft and pitting (early stage 2) to hard and non-pitting (late stage 2). Answer D:In lymphedema stage 3, fibrotic changes of the skin and fat deposits in the subcutaneous tissues continue to increase in severity. When the PT palpates the skin, the limb is hard with little mobility and increased skin folds. Pitting is absent or minimally present with significant pressure and, if present, dissipates rapidly. Bottom Line:Lymphedemastage 0 is also known as the subclinical stage. Lymph transport is impaired, but swelling is not evident. The patient may report heaviness, tightness, or sensory changes in the affected limbs. TrueLearn Insight :Lymphedema stage 0 is also known as the subclinical stage. Lymph transport is impaired; however, lymphatic load has not yet exceeded the transport capacity. Swelling is not evident, and the patient may report heaviness, tightness, or sensory changes in the affected limbs. This stage may persist for months or years before edema is detectable.

A cardiopulmonary certified specialist is conducting stress tests in a hospital outpatient clinic. Which of the following patients is the MOST appropriate candidate for a stress test? a) active metabolic syndrome b) endocarditis c) MI 3 days ago d) unstable angina

Correct answer: A Metabolic syndrome is a cluster of metabolic conditions that occur together and increase the risk of cardiovascular disease or type 2 diabetes mellitus (DM) greater than the risk of any one of the conditions occurring by itself.Included are: insulin resistance visceral adiposity dyslipidemia hypertension elevated blood pressure A patient with active metabolic syndrome would benefit from knowing the results of a stress test so that an appropriate plan of care could be designed. Answer B: Endocarditis is inflammation of the endocardium, and increasing the stress on the heart could potentially cause a myocardial infarction (MI). Answer C: It is contraindicated to stress the heart of a patient status post recent MI. The stress could cause another MI. Answer D: Unstable angina does not respond to rest or medications such as nitroglycerin. It is indicative of an inability to stabilize the functions of the heart, and the increased demand during a stress test could lead to an MI. Bottom Line: Stress tests are used for the diagnosis and management of patients with cardiac artery disease. They are contraindicated in patients with MI less than 6 weeks before, resting or unstable angina, pericarditis, endocarditis, and/or serious arrhythmias.

A patient with multiple sclerosis reports that activities of daily living are often limited by severe fatigue and muscle stiffness. Which of the following intervention strategies is LEAST likely to be effective with this patient's condition? a) thermal heat modalities for stiffness b) cardiovascular endurance training c) energy conservation techniques d) therapeutic exercise

Correct answer: A Multiple sclerosis is an autoimmune disease caused by demyelination of the central nervous system. While the symptoms of multiple sclerosis are variable, patients with multiple sclerosis often exhibit difficulty with balance, decreased vision, fatigue, and heat sensitivity. Due to a high likelihood of heat sensitivity, the use of heat modalities to decrease muscle stiffness is least likely to be effective and may increase the patient's fatigue levels. Answer B:Cardiovascular endurance training is commonly used in patients with multiple sclerosis to decrease their levels of fatigue by improving their activity tolerance. Answer C:Energy conservation techniques are commonly taught to patients with multiple sclerosis in order to compensate for their high levels of fatigue. Answer D:Therapeutic exercise is commonly used in patients with multiple sclerosis to decrease their levels of fatigue by improving their activity tolerance. Bottom Line:Multiple sclerosis is an autoimmune disease caused by demyelination of the central nervous system. Patients with multiple sclerosis often exhibit difficulty with balance, decreased vision, fatigue, and heat sensitivity. Heat modalities to reduce stiffness are unlikely to be effective.

A patient presents 1 day status post right total hip arthroplasty. With the patient resting in bed, the patient's vitals are: HR 89 bpm, BP 146/72, O2 98%. During the evaluation, the patient is assisted to a sitting position. She begins to complain of severe dizziness. HR is 102 bpm, BP 100/61, O2 95%. What does the clinician think may be happening with this patient? a) orthostatic hypotension b) vasovagal response c) pulmonary embolism d) sympathetic nervous system reaction to pain

Correct answer: A Orthostatic hypotension is a sudden drop in blood pressure that occurs with movement into an upright position after a period of immobility or bedrest. The positional changes cause gravitational blood pooling in the lower extremities. Venous return and cardiac outputs are reduced, resulting in cerebral hypoperfusion. Lightheadedness, dizziness, or even loss of consciousness can occur. Answer B:A vasovagal response is a syncopal episode caused when a patient's body overreacts to certain triggers. Triggers can include extreme emotional distress, sight of blood, straining, or fear. This causes a sudden drop in heart rate and blood pressure, allowing blood to pool in the lower extremities, which reduces blood flow to the brain, causing brief loss of consciousness. Answer C:A pulmonary embolism is a risk after hip arthroplasty, where a clot can go to the lung, but symptoms would include chest pain, shortness of breath, and an increase in HR. Answer D:Responses to a variety of stressors, including pain, activate the sympathetic nervous system, which will result in an increased heart rate and an increase in blood pressure. The patient in this scenario had a slight increase in heart rate, which would be expected. The concerning change in vitals was the significant drop in blood pressure, which is more likely the cause of the patient's dizziness. Bottom Line:Orthostatic hypotensionis a sudden drop in blood pressure that occurs with movement into an upright position after a period of immobility or bedrest. The positional change causes gravitational blood pooling in the lower extremities. Venous return and cardiac outputs are reduced, resulting in cerebral hypoperfusion. Lightheadedness, dizziness, or even loss of consciousness can occur. As a precautions, any patient restricted to the recumbent position for even a short time should be considered at risk for orthostatic hypotension. The patient should be reclined from the upright position with legs elevated if orthostatic hypotension occurs.

A 24-year-old patient has been using axillary crutches for the past 6 weeks due to non-weight-bearing status following an ankle fracture. The patient has now been advised to ambulate with partial weight bearing for an additional 2 weeks before progressing to weight bearing as tolerated. Which of the following assistive devices and gait patterns should the therapist currently use with the patient? A. Axillary crutches with modified three-point gait pattern B. Axillary crutches with three-point gait pattern C. Single-point cane with modified four-point gait pattern D. Single-point cane with modified two-point gait pattern

Correct answer: A Partial weight bearing orders require that the patient has a bilateral device such as axillary crutches or a walker, depending on their age and balance. The gait pattern for partial weight bearing is a modified three-point gait as they progress from the three-point, which was for non-weight-bearing. The age of this patient makes the crutches most appropriate for the modified three-point gait pattern. Answer B:The three-point gait pattern is used for non-weight-bearing - this is what the patient has been using with their crutches. The patient now needs to progress to a modified weight-bearing pattern. Answer C:Modified four-point gait pattern requires that the patient be full weight bearing, and partial weight bearing requires a bilateral device, so a cane is not appropriate at this time. Answer D:Modified two-point pattern requires that the patient be full weight bearing, so this answer is not appropriate. Partial weight bearing requires a bilateral device. Bottom Line:Partial weight bearing orders require that the patient has a bilateral device such as axillary crutches or a walker, depending on their age and balance. The gait pattern for partial weight bearing is a modified three-point gait as they progress from the three-point, which was for non-weight-bearing.

A physical therapist is treating a patient with a traumatic brain injury in an acute rehabilitation facility to prevent contractures. The patient can open their eyes but does not display any voluntary movements. Which of the following scales is the MOST appropriate for assessing the patient's risk for skin breakdown while in this state? a) Braden b) Glasgow c) Morse d) Modified Ashworth

Correct answer: A Patients on bedrest or who are immobile are at increased risk for skin breakdown and need to be assessed using valid and reliable tools. The Braden Scale is a reliable tool for the assessment of the risk for skin breakdown, which includes the assessment of mobility, activity, sensation, moisture, nutrition, friction, and shear. Answer B:The Glasgow Outcome Scale is used for the assessment of disability in patients with traumatic brain injury, not skin breakdown risk. Answer C:The Morse Falls Scale is used for the assessment of falls risk and would not be appropriate in this scenario. Answer D:The Modified Ashworth Scale is used for the assessment of spasticity. While spasticity may be related to an increased risk of pressure ulcers, the Modified Ashworth Scale would not be appropriate as a risk assessment tool for skin breakdown in this scenario. Bottom Line:The Braden Scaleis a reliable tool for the assessment of the risk for skin breakdown and includes the assessment of mobility, activity, sensation, moisture, nutrition, friction, and shear.

A 25-year-old patient with traumatic brain injury, Rancho Level V, was referred for physical therapy. During the examination, the physical therapist observes lower extremity weakness causing difficulty in ambulation. Which of the following is MOST appropriate to include in the patient's treatment plan? a) ambulation using 1 or 2 word commands b) community ambulation c) ambulation using crutches d) wheelchair ambulation

Correct answer: A Patients who are Rancho Level V (Confused-Inappropriate) are able to respond to simple commands, but they have a tendency to be easily distracted and inattentive. They might perform previously relearned tasks given that the activity is structured. Answer B:Activities for patients with Rancho Level V should be structured. Also, since they are easily distracted, ambulation in the community could present a lot of stimulus and be too distracting for the patient. Answer C:Ambulation using crutches might be used but may not be the most appropriate. Patients with Rancho Level V usually can perform previously relearned tasks. Given that ambulation with crutches requires learning a new skill, the patient might have difficulty with the activity. Answer D:Wheelchair ambulation can be used but might not be the most appropriate for the patient because patients rated at Rancho Level V usually can perform previously relearned tasks. Given that wheelchair ambulation requires learning a new skill, the patient might have difficulty with the activity. Bottom Line:Patients who are Rancho Level V (Confused-Inappropriate) are able to respond to simple commands, but have a tendency to be easily distracted and inattentive. They might perform previously relearned tasks given that the activity is structured.

A physical therapist is working with a patient who experienced an acute myocardial infarction 5 days ago. The patient has been hospitalized since the event and working with physical therapy daily. The patient currently requires moderate assistance from the therapist to sit at the edge of bed and can tolerate unsupported sitting for only 1 minute before needing to lie down, although vital sign response is within normal limits. Which of the following treatments should the therapist provide AT THIS TIME? a) continue with edge of bed sitting until patient can tolerate 3-5 minutes unsupported b) progress to sit-to-stand since it is 5 days post event c) initiate ambulation with a front-wheeled walker to improve cardiac status d) discontinue physical therapy until patient is more medially stable

Correct answer: A Patients with cardiopulmonary conditions should be able to sit on the edge of the bed for at least 3-5 minutes and perform some full knee extensions before attempting to stand and then initiate ambulation. At this time, this patient should continue with edge-of-bed sitting and working on building tolerance in this position. Answer B:Although vital signs are staying stable, the patient needs to be able to sit for 3-5 minutes edge of bed and work on knee extensions before progressing to sit-stand. This patient needs to continue working on sitting tolerance before progressing to standing. Answer C:Cardiopulmonary patients need to be able to sit edge of bed for at least 3-5 minutes before attempting to stand and then initiate ambulation. At this time, this patient needs to continue to build sitting tolerance before progressing further. Answer D:This patient needs ongoing skilled PT to work on sitting tolerance and developing unsupported sitting, working on knee extensions to prepare for standing, and then to begin ambulation. Bottom Line:Cardiopulmonary patients need to build their abilities in sequence. A patient needs to be able to tolerate sitting for 3-5 minutes before working on standing and ambulation. TrueLearn Insight: This patient needs to continue with skilled PT to specifically work on sitting tolerance to build up to more functional activities. The therapist also can work on bridging to promote strengthening and added functional ability for tasks such as dressing and bed pans.

A physical therapist is assessing blood pressure. The therapist places the patient in a quiet environment, inflates the cuff until the brachial artery is obliterated, and then releases the cuff pressure. After 30 seconds the therapist raises the pressure 30 mmHg above obliteration pressure and begins to releasethe air in the cuff so that the pressure falls at a rate of 2 to 3 mmHg/s. When the therapist hears the "last sound" they correctly identify the Korotkoffphase as WHICH of the following? a) Phase V b) Phase I c) Phase III d) Phase IV

Correct answer: A Phase VKorotkoff sounds are associated with the pressure level when the last sound is heard; this is diastolic blood pressure. Answer B:Phase I Korotkoff sounds are associated with the pressure level at which the first faint, clear tapping sounds are heard. This is systolic blood pressure. Answer C:Phase III Korotkoff sounds are associated with crisper and more intense tapping and/or swishing sounds that are heard as the cuff continues to be deflated. Answer D:Phase IV Korotkoff sounds are associated with a distinct, abrupt, muffling of sound (with a blowing quality) that is heard as the cuff continues to be deflated. Bottom Line:Phase VKorotkoff sounds are associated with the pressure level when the last sound is heard; this is diastolic blood pressure.

Which of the following scenarios is an example of an intervention for MODIFYING CARDIOVASCULAR RISK FACTORS for primary prevention? A. 42-year-old morbidly obese male who is participating in a wellness program that consists of nutrition counseling as well as a walking routine B. 59-year-old female with a history of diabetic foot ulcers who is getting fitted for custom orthotics C. 72-year-old male with a recent hospitalization for heart failure exacerbation receiving training for energy conservation D. 75-year-old female with a a history of congestive heart failure starting a program for inspiratory muscle training

Correct answer: A Primary prevention is an important component of any wellness and fitness program for individuals with risk factors for cardiovascular disease such as hypertension, hyperlipidemia, obesity, diabetes, and many others. It is important to note that primary prevention refers to treatment to reduce or modify risk factors for individuals without documented cardiovascular disease. A wellness program that consists of aerobic exercise, nutrition counseling, and walking for weight loss is an example of such a program. Treatment for individuals who already have cardiovascular disease is known as secondary prevention. Interventions for both types of programs may look similar, but would follow different guidelines and parameters. Answer B:Custom orthotics do not directly affect risk for cardiovascular disease. Nonetheless, it is still very important for individuals who have had diabetic foot ulcers to get the necessary interventions to prevent further complications. Answer C: Education for energy conservation is an appropriate intervention for individuals with heart failure. However, this intervention would not be considered risk factor modification; it is treatment for an impairment that resulted from cardiovascular disease. Answer D: Inspiratory muscle training for individuals with heart failure is typically performed with the goal of improving pulmonary function and overall exercise tolerance. This intervention would not be considered risk factor modification since it is for individuals who already have cardiovascular disease. Bottom Line:Primary prevention refers to treatment to reduce or modify risk factors for individuals without documented cardiovascular disease. A wellness program that consists of aerobic exercise, nutrition counseling, and walking for weight loss is an example of such a program. Treatment for individuals who already have cardiovascular disease is known as secondary prevention.

Which of the following techniques is a neuromuscular facilitation technique specifically designed to inhibit muscle contraction and tone that is due to peripheral reflex effects? a) prolonged stretch b) quick stretch c) joint approximation d) joint traction

Correct answer: A Prolonged stretch is a slow maintained stretch applied at the maximum available lengthened range and will inhibit muscle contraction and tone due to the stretch protection reflex. This technique is often achieved through splinting and casting or traction with low load weights. Answer B:Quick stretch facilitates, not inhibits, both intrafusal and extrafusal agonist muscle contraction. Answer C:Joint approximation is compression of joint surfaces with manual pressure or a weighted vest or belt and facilitates postural extensors and stabilizing responses. It also enhances joint awareness. Answer D:Joint traction is the manual distraction of the joint or use of weighted cuffs at the wrist or ankle, and facilitates joint motion and enhances joint awareness. Bottom Line:The only technique listed that inhibits muscle contraction or tone is the prolonged stretch; all the other techniques are used to facilitate muscle response and contraction.

A physical therapist has been asked to determine whether a patient with known cardiac symptoms demonstrates a pulse deficit. Which of the following techniques would be the CORRECT METHOD for the therapist to use? A. Two therapists simultaneously measuring and comparing the apical and radial pulses B. Two therapists simultaneously measuring and comparing right and left radial pulses C. Two therapists simultaneously measuring and comparing unilateral radial and carotid pulses D. Two therapists simultaneously measuring and comparing unilateral radial and femoral pulses

Correct answer: A Pulse deficit is a difference between the rate of the radial and apical pulses and indicates some cardiovascular deficits.To determine whether a patient has a pulse deficit, two therapists would simultaneously measure and compare the apical and radial pulses.It is important to report pulse deficit because it provides information about the ability of the cardiovascular system to perfuse the body. The deficit may be caused by blood being pumped from the heart and not reaching the distal sites, either from heart contraction weakness or a blockage. Answer B:The radial pulse measures the contractions of the ventricle and indicates the rate of the heartbeat. Pulse deficit is a disconnect between the beating occurring at the heart in comparison to the peripheral locations. Pulse deficit can be correctly measured only by comparison of the apical and radial pulses. Answer C:Pulse deficit can be accurately measured only through a direct comparison of apical and radial pulses. It is not accurately measured through the carotid pulse. Answer D:Pulse deficit is measured specifically as a comparison of the apical pulse, which is the pulse directly over the apex of the heart, with the radial pulse. All other locations will not produce the requested information. Bottom Line:Pulse deficit is a difference between the heartbeat as heard through a stethoscope at the apex of the heart,the apical pulse, and the radial pulse. It indicates some deficit, such as a blockage or diminished stroke volume, that does not allow a strong enough radial pulse to be detected.

What is the benefit of randomly assigning subjects into either a control group or experimental group in a randomized controlled trial? a) increases the likelihood that the two groups are the same at the outset b) minimizes the number of subjects needed in the study c) blinds the researchers to group assignment d) gives the study better power

Correct answer: A Randomly assigning participants to a group strengthens an experimental study because subject differences that could potentially affect the outcomes are equally distributed between groups. It increases the likelihood that the two groups are the same at the outset. That way, any changes that result from the application of the independent variable can be assumed to be the result of the treatment of interest. Answer B:Random assignment of participants does not affect the number of subjects needed for a study. Answer C:Though subjects are randomly assigned, the researcher may or may not know the group assignment. Blinding of group assignment is determined by the design of the study. Answer D:Power is related to the likelihood that a study will detect an effect when there truly is an effect. Power relates to sample size, not whether subjects are assigned randomly.

Which of the following statements most accurately defines reactive motor control? A. Movements are adapted in response to ongoing feedback B. Movements are adapted in advance of ongoing movements C. An idea for purposeful movement made up of component programs D. An abstract representation that results in coordinated movement

Correct answer: A Reactive motor control requires that movements are adapted in response to ongoing feedback, such as muscle stretch as a result of forward weight shifting. Answer B:Proactive motor control, or anticipatory control, is the adaptation of movements in advance or in anticipation of ongoing movements, such as shifting position to catch a heavy item. Proactive motor control is described in this answer. Answer C:This is a definition of motor planning, which is the idea or plan for a purposeful movement that is made up of component motor programs. Answer D:This describes motor programming, which is abstract but can be initiated to cause a coordinated movement sequence. Bottom Line:Reactive motor control requires that movements be adapted in response to ongoing feedback, such as muscle stretch as a result of forward weight shifting. TrueLearn Insight :Motor control has a variety of terminology, and understanding the difference between these concepts is critical for appropriate patient care. Reactive motor control occurs when the body responds to feedback and adapts accordingly to prevent a fall or respond to the situation. Anticipatory motor control occurs when the body moves in preparation for an activity, and motor planning occurs when the body plans specific movements, and programming is abstract but initiates the production of a movement sequence.

A patient comes into the clinic with a medical diagnosis of left thoracic scoliosis. During the postural examination, the physical therapist asks the patient to bend forward from the waist, and the therapist observes from a posterior view. The therapist observes a hump along the left ribcage. Describe the position of the thoracic vertebral bodies that contributed to the presence of the rib hump. a) right sidebend, left rotation b) right sidebend, right rotation c) left sidebend, left rotation d) left sidebend, right rotation

Correct answer: A Right sidebend, Left rotation is the correct answer since an individual that has a rib hump demonstrates ipsilateral rotation of the hump with contralateral (coupled) sidebending. When a thoracic vertebrae that articulates with a rib rotates, the rib follows along. And so, if the vertebrae rotates to the left, the ribs will move more posteriorly on the left side causing a rib hump. Answers B & C:This answer is incorrect as it does not address the coupled motions of rotation and sidebending. Answer D:This answer is incorrect as it would be appropriate for a right-sided rib hump. Bottom Line:The procedure describes an Adam's forward bend test, which is a screen for scoliosis. The individual bends forward and the physical therapist looks to see if the ribs are level or if one side of the rib cage is higher. When a thoracic vertebra that articulates with a rib rotates, the rib follows along. And so, if the vertebra rotates to the left, the ribs will move more posteriorly on the left side, causing a rib hump. In the lower thoracic region there is coupled motion of contralateral sidebending with ipsilateral rotation.

An individual presents to physical therapy with right shoulder pain that limits their ability to throw a baseball. The therapist notices that the patient's humeral head translates anteriorly during shoulder internal rotation, flexion, and abduction motions with mild instability. The patient also has a positive Speed's test, positive painful arc, and weakness of the external rotators. Which strengthening exercise is MOST appropriate for this patient? a) side-lying external rotation with a 1 pound weight b) standing external rotation with a blue resistance band c) standing external rotation with arm abduction to 90 degrees and yellow resistance band d) prone external rotation with arm abduction to 90 degrees and a 5 pound weight

Correct answer: A Side-lying external rotation with a 1-pound weight would emphasize the beginning phase of strengthening the external rotators. The scapula is in a stable position to avoid compensations, the shoulder is in a stable position, and there is less chance of anterior translation in this sidelying position. Answer B:Because the patient already demonstrates anterior translation with shoulder motions in standing, it may be more difficult to stabilize the humeral head while standing and also overcoming the resistance of the elastic band. This would eventually be a good progression from the sidelying exercise as strength and control improve. Answer C:This is a relatively unstable position of the shoulder in 90 degrees of abduction. Asking the arm to move into external rotation when the patient is already unstable and translating anteriorly is inappropriate. This exercise would be an eventual progression as strength and control improves. Answer D:In this position, gravity is actually contributing to the anterior force at the head of the humerus. It is best not to start in this position. In addition, the patient would end up in 90 degrees of abduction and moving into full external rotation with this activity, which is an unstable position and too advanced at present. Bottom Line:An imbalanced force of the external rotators and internal rotators where the internal rotators overpower the external rotators may cause the humeral head to translate anteriorly. Instability can also contribute to the problem. Eventually, with poor mechanics, impingement can occur. Interventions should be directed to the strengthening the external rotators, and controlling humeral head translation. External rotation in sidelying is a good position to begin strengthening.

A patient presents with a 3-month history of plantar fasciitis. Upon examination, the plantar fascia is extremely tight and inflexible. As part of the PT interventions to warm the tissue and improve extensibility, ultrasound treatment parameters should consist of which setting? a) continuous ultrasound at 3 MHz b) pulsed ultrasound at 3 MHz c) continuous ultrasound at 1 MHz d) pulsed ultrasound at 1 MHz

Correct answer: A Since the at-fault tissue is tight, the goal is to increase tissue extensibility with heat. Because the plantar fascia is very superficial, a continuous setting of 3 MHz should be used. The setting of 3 MHz is used when the goal is to heat tissues only 1 to 2 cm deep. Continuous setting is used to produce the thermal effects. Answer B:Pulsed ultrasound is used when the goal of ultrasound is to produce nonthermal effects. Examples of nonthermal effects achieved by ultrasound include: increasing intracellular calcium, increasing skin and cell membrane permeability, increasing mast cell degranulation, increasing chemotactic factor and histamine release, increasing macrophage responsiveness, and increasing the rate of protein synthesis by fibroblasts. All of these serve to improve healing. Pulsed ultrasound at 3 MHz is used at times for dermal ulcers. Answer C: Ultrasound of 1 MHz frequency is used to heat tissues up to 5 cm deep. Deep heating is more appropriate, for example, when trying to improve range of motion in the hip joint. Answer D:Pulsed ultrasound of 1 MHz has been shown in some studies to assist with tendon healing after surgical incision and repair.

A physical therapist is examining a patient with a history of traumatic brain injury who just started taking oral baclofen to manage theirspasticity. What is a common side effect of oral baclofen? a) drowsiness b) dry mouth c) liver toxicity d) respiratory depression

Correct answer: A Spasticity is a common impairment in patients after a central nervous system (CNS) injury. Baclofen is an agonist of the GABA neurotransmitter, which inhibits alpha motor activity in the spinal cord. Since oral baclofen is CNS inhibitor, a common side effect of the medication is drowsiness. Answer B:Dry mouth is a common side effect of tizanidine, which is another oral medication used to treat spasticity. Tizanidine is an adrenergic agonist, which is a class of medications that often cause dry mouth. Answer C:Liver toxicity is a common side effect of dantrolene sodium, which is another oral medication used to treat spasticity. Dantrolene sodium acts directly on the muscle membrane and may cause severe damage to the liver, so regular blood work is needed to monitor liver function if this medication is prescribed. Answer D:Respiratory depression is a side effect of sudden intrathecal baclofen withdrawal that may be due to pump failure. Baclofen may be delivered to the intrathecal space by an implanted pump for patients with severe spasticity who do not respond to oral medications. Bottom Line:Physical therapists who treat patients with spasticity must be able to recognize the common side effects of anti-spasticity medications. A common side effect of Baclofen is drowsiness. Dry mouth is a common side effect of tizanidine, and liver toxicity is a common side effect of dantrolene sodium.

A patient is walking on the treadmill at 2 miles per hour with a 1% grade as part of his physical therapy session. After 4 minutes at this intensity, he begins feeling chest pain that resolves with a slower speed and the grade reduced to 0%. The patient has a history of coronary heart disease and was cleared for exercise by his physician. He has similar symptoms during other activities at the same level of intensity. Which of the following is the MOST appropriate way to document this finding in his file? A. The patient was showing signs of stable angina during exercise B. The patient was presenting with early signs of myocardial infarction C. The patient was presenting with an acute coronary syndrome that resolved D. The patient was showing signs of unstable angina during exercise

Correct answer: A Stable angina is chest pain or discomfort that is characterized by a predictable pattern, such as with the same intensity or amount of physical activity or emotional stress. In this scenario, the patient's symptoms are predictable and are relieved by reducing the level of activity. Therefore, it is most likely stable angina and should be documented accordingly. Answer B:Chest pain is a common symptom of a myocardial infarction. However, in this scenario, since symptoms are reduced by reducing the intensity of the activity, it is more appropriate to describe it as stable angina. Nonetheless, a myocardial infarction may need to be ruled out in the setting of worsening symptoms in someone with stable angina, but there is not enough information to claim he is showing early signs of having an MI. Answer C:Acute coronary syndrome (ACS) describes various cardiac conditions that limit blood flow to the heart, one of which is unstable angina. In this scenario, the patient is likely presenting with stable angina because his symptoms were predictable and were relieved with reduction in the level of activity. Stable angina is not typically described as ACS unless it becomes unstable angina. Answer D:Unstable angina is characterized by unexpected chest pain with symptoms possible even at rest. In this scenario, it is more likely stable angina because symptoms were predictable and were relieved with reduction in the level of activity. Therefore, it is not the most appropriate answer. Bottom Line:Angina pectoris is defined as chest pain or discomfort resulting from poor blood flow to the heart. It can be classified as either stable or unstable. In stable angina, the symptoms are predictable and can be controlled. Usually the same amount or intensity of an activity brings on symptoms that can be resolved with rest or if needed, with sublingual nitroglycerin. In unstable angina, symptoms are unpredictable and may be uncontrolled, which would require medical attention.

A physical therapist is working with a wellness program for diabetic patients and reviewing their medical records. Besides a fasting glucose level, WHICH of the following lab values also measures the blood glucose levels, looking at their average over the past 2 to 3 months? a) A1c b) BUN c) HCT d) LDL

Correct answer: A Tests for diabetes include fasting glucose and A1c levels. A1c estimates the average blood glucose levels over a 2-3 month period. The lab value for a normal A1c is below 5.7%. A1c values above 6.5% are diagnosed as diabetes. Answer B:Blood Urea Nitrogen measures kidney function. Answer C:Hematocrit measures the proportion of red blood cells in the blood. Answer D:Low density lipoproteins are the "bad" cholesterol in your blood that can leave plaques in the arteries causing atherosclerosis.

A cardiac patient is performing the 6-minute walk test. They have been walking for 2 minutes when they indicate that they need to stop and rest. Which of the following procedures should be followed in these circumstances? a) allow the patient to rest, but the stopwatch continues b) stop the stopwatch until the patient resumes walking c) terminate the test, it is now invalid d) terminate the test and document patient only able to complete 2 minutes

Correct answer: A The 6-minute walk test allows for the patient to stop and rest, but the stopwatch continues throughout their rest period. See references for 6-minute walk. Answer B:The stopwatch is not stopped during the 6-minute walk test even if the patient takes a rest. Answer C:The patient is allowed to rest during the 6-minute walk test, so the test does not need to be terminated and is not invalid. Answer D:The 6-minute walk test allows for a rest period while the stopwatch continues. The test does not need to be terminated. Bottom Line:The 6-minute walk test allows the patient to stop and rest, but the stopwatch continues throughout their rest. TrueLearn Insight :The 6-minute walk test appears to provide information about the functional status, exercise tolerance, oxygen consumption, and survival of persons with cardiac pump failure. Although the exercise performed during the 6-minute walk test is considered submaximal, it nonetheless closely approximates the maximal exercise of persons with cardiac pump failure and is correlated to peak oxygen consumption.

A patient is being seen for shoulder rehabilitation and on their initial examination scored a 70 on the Disabilities of the Arm, Shoulder, and Hand (DASH) outcome measure. Four weeks later the DASH score was 50. How should the physical therapist interpret these results? a) there is a statistically significant reduction in disability, and the patient is doing well b) there is a statistically significant increase in disability, and the patient needs to go back to the physician c) there is no significant change based on the outcome measure scores d) there is improvement but no significant reduction in disability

Correct answer: A The Disabilities of the Arm, Shoulder, and Hand (DASH) questionnaire is a 30-item questionnaire that examine's the patient's ability to perform certain upper extremity activities. The score ranges from 0 (no disability) to 100 (most severe disability). The minimal detectable change (MDC) of the DASH, which is the smallest amount of change that has to occur before the change can be considered a true change and not an error, is a 10.2. The MDC in this scenario was a 20-point positive change in the patient's outcome measure. The PT can interpret this change as significant. Answer B:The DASH ranks higher scores with more disability, and this patient's score decreased. Answer C:The scenario describes the minimal clinically important difference (MCID), which is similar to the MDC but is the improvement perceived by the patient. Answer D:The MDC of the DASH is a 10.2, so this answer would be true if the change in scores were less than 10.2.

A 36-year-old, active woman presents with a 1-month history of positional vertigo with rolling to the right side. She is having trouble with bed mobility, bending over to dry her hair, and looking up to get things out of high cabinets. The onset was insidious with no prior issues. She has to sit on the bed for a minute or so until the symptoms subside before getting up in the morning. What outcome measure would be BEST to use for this patient? a) Dizziness Handicap Inventory b) Dynamic Gait Index c) Berg Balance Test d) The SF-36 Health Survey

Correct answer: A The Dizziness Handicap Inventory is a popular tool used to measure a patient's self-perceived handicap as a result of a vestibular disorder. It has excellent test-retest reliability and good internal consistency reliability. It is useful to establish subjective improvement. It has very specific vestibular questions pertaining to symptoms with looking up and down, and rolling over in bed. Answer B:The Dynamic Gait Index is a very useful tool for evaluating high-level gait. It examines the patient's ability to perform variations in walking on command. It can be used for individuals with vestibular dysfunction (examining gait with head turns is particularly useful with vestibular patients). However, it would not be best for this particular patient since she is not complaining of balance issues. Answer C:The Berg Balance Test is an objective measure of static and dynamic balance abilities. Berg Balance scores have been shown to be useful in predicting falls in the elderly and evaluating changes in patients undergoing physical therapy. The Berg Balance Test would not be particularly useful with this patient, as she is an active 36-year-old woman without a history of balance deficits. It may be useful for an older patient with vestibular dysfunction. Answer D:The SF-36 Health Survey is a more generalized questionnaire with 36 health-related questions pertaining to health status and general functioning. It can be used with a variety of impairments but is not specific to dizziness or vestibular patients. Bottom Line:The Dizziness Handicap Inventory (DHI) is a very useful outcome measure to use with vestibular patients. The questions are aimed at specific vestibular symptoms, and the test has excellent test-retest reliability.

The physical therapist is assessing a full-term infant for primitive reflex activity. The therapist places the infant in prone and strokes his paravertebral skin. The therapist notes that the spine curves laterally on the stimulated side. The reflex demonstrated is: a) Galant reflex b) Asymmetric tonic neck reflex c) Crossed extension reflex d) Moro reflex

Correct answer: A The Galant reflex appears at 28 weeks' gestation and is integrated at approximately 3 months of age. The stimulus to elicit the response in prone is stroking of paravertebral skin. The response consists of lateral curvature of the trunk on the stimulated side. Answer B:The ATNR appears at 20 weeks' gestation and is integrated at 4-5 months of age. The stimulus to elicit the response is turning of the head. The response consists of facial arm extensions with occipital arm flexion and abduction. Answer C:The crossed extension reflex appears at 28 weeks' gestation and is integrated at 1-2 months of age. The stimulus to elicit the response is a noxious stimulus to the foot. The response consists of flexion in the stimulated leg and then extension of the opposite leg with adduction. Answer D:The Moro reflex appears at 28 weeks' gestation and is integrated at 3-5 months of age. The stimulus to elicit the response is dropping the head backward. The response consists of abduction and extension of arms and splaying of fingers, and may be followed by arm flexion and adduction.

A physical therapist is performing developmental screenings at a day care. Which of the following children is MOST likely to be referred to early intervention for further evaluation? a) 8-month old who is unable to roll independently from prone to sidelying b) 10-month old who is unable to creep independently c) 10-month old who is unable to cruise independently d) 14-month old who is unable to walk independently

Correct answer: A The ability to roll independently from prone to sidelying is typically achieved at the age of 5-7 months, so this child is most likely to be referred to early intervention for further evaluation. Early intervention services are provided under the Individuals with Disabilities Education Act to children under the age of 3 years. Variability in achieving motor milestones in infant development is large. Answer B:The ability to crawl is typically achieved at the age of 8-10 months, so this child is unlikely to be referred to early intervention for further evaluation. Answer C:The ability to cruise independently is typically achieved at the age of 8-10 months, so this child is unlikely to be referred to early intervention for further evaluation. Answer D:The ability to walk independently is typically achieved at the age of 9-17 months, so this child is unlikely to be referred to early intervention for further evaluation.

A physical therapist is working with an 80-year-old patient following a fall. The patient sustained no major injuries, but the family is questioning her future fall risk. Which of the following outcome measure results would be most indicative that the patient is a fall risk? a) Activities specific balance confidence scale at 55% b) The four square step test in 13 seconds c) The timed up and go test in 12 seconds d) The short falls efficacy scale international at 10

Correct answer: A The acute balance scale (ABC) is a questionnaire designed to determine balance confidence during normal daily activities. A cutoff score of 69 has been identified as increased fall risk, so a score of 55 would put this individual at high risk for falls. Additionally, a score close to 50 suggests low levels of physical function. Review the ABC instrumental details on SRALab.org: Activities-Specific Balance Confidence Scale in the references. Answer B:The four square step test performed in greater than 15 seconds suggests the need for further testing to eliminate fall potential. In this case, 14 seconds is within limits to suggest a lower risk for falls. Check out the four square step test on SRALab.org: four square step test in the references. Answer C:The timed up and go has a cut off of 13.5 for older adults, and this score of 12 seconds again is not highly predictive of falls. Review the timed up and go test on SRALab.org: Timed Up and Go in the references. Answer D:The falls efficacy scale short form is scored from 7-28, with higher scores indicating greater fear of falling, so a score of 10 is not indicative of high fall risk.Check out the short falls efficacy scale international on SRALab.org: short falls efficacy scale international in the references. Bottom Line:While this patient may fall, because no test can absolutely predict falls, the ABC score is particularly worrying in this patient as it suggests a high fear of falling that may limit activity and also cause tension and anxiety during mobility, which can increase fall risk.

A patient reports spraining his ankle when stepping off a curb 2 weeks ago. Upon examination, the patient has some mild swelling, pain on the lateral side of the ankle with passive and active inversion near end range, and a positive anterior drawer and talar tilt test. Which ligaments are most likely injured? a) anterior talofibular, calcaneofibular b) anterior talofibular, posterior talofibular c) anterior tibiofibular, anterior talofibular d) anterior tibiofibular, calcaneofibular

Correct answer: A The anterior talofibular ligament is usually the first to tear with an ankle sprain and if torn will show excessive motion on an anterior drawer test. The calcaneofibular ligament is also frequently sprained with excessive ankle inversion, especially from a plantarflexed posture, and is tested with the talar tilt test. Answer B:The posterior talofibular ligament can be sprained with an ankle inversion sprain; however, it is tested with a posterior drawer test because of its alignment from the posterior portion of the fibula to a more posterior position on the talus. Answer C:The anterior tibiofibular ligament is not tested by the anterior drawer or the talar tilt. It supports the distal tibiofibular joint and has no attachment on the talus. This is stressed with a Kleiger's test for a high ankle sprain. Answer D:The calcaneofibular ligament is tested with the talar tilt. However, the anterior tibiofibular ligament is not tested by the anterior drawer or the talar tilt. It supports the distal tibiofibular joint and has no attachment on the talus. This is stressed with a Kleiger's test for a high ankle sprain. Bottom Line:Special tests of the ankle stress different ligaments depending on how the foot is positioned. Talar tilt is best for the calcaneofibular ligament. Anterior drawer is best for the anterior talofibular ligament.

As part of the cardiovascular and pulmonary examination, a physical therapist decides to auscultate for heart sounds. When the stethoscope is placed over the right lateral sternal border in the 2nd intercostal space, which valve is the physical therapist trying to auscultate?

Correct answer: A The aortic valve is best auscultated when the stethoscope is placed over the right lateral sternal border in the 2nd intercostal space.Auscultation of the heart allows the physical therapist to screen for and identify abnormalities. Common problems include murmurs and gallops. Although interpretation of heart sounds is considered to be an advanced skill, it is still possible for novice clinicians to be able to at least distinguish between normal and abnormal heart sounds. The key basic skill here is to know the locations associated with each heart valve as well as the point of maximal impaction (the apex).The location described in this question (right lateral sternal border, 2nd intercostal space) indicates that the aortic area is being auscultated. In this area, sounds generated by the aortic valve are best heard. When auscultating heart sounds, it is important to note that the location for auscultation is not the anatomical location of the valves. Rather, it is where blood is going after it has left the valve. Answer B:The mitral valve is best auscultated over the mitral area, located in the 5th intercostal space on the left, medial to midclavicular line. This area is also where the apex of the heart can be auscultated. Answer C:The pulmonic valve is best auscultated over the pulmonic area, located in the 2nd intercostal space on the left, lateral to the sternal border. Answer D:The tricuspid valve is best auscultated over the tricuspid area, located along the lower left sternal border, on either the 4th or 5th intercostal space. Bottom Line:The aortic valve is best auscultated when the stethoscope is placed over the right lateral sternal border in the 2nd intercostal space.

During a scheduled physical therapy visit, a 30-year-old in the 20th week of her first pregnancy complains of shortness of breath and fatigue. She reports a couple of recent exercise sessions in which she noticed that her heart was racing. The physical therapist decides to perform auscultation of the heart. The BEST auscultation point for assessing the aortic valve would be which of the following? A. Right sternal border between the second and third rib. B. Just left of the midline at the level of the 3rd costal cartilage C. Just left of midline, deep to the 4th costal cartilage D. Slightly right of midline, deep to the 4th and 5th intercostal spaces

Correct answer: A The aortic valve lies near the midline at the level of the 3rd intercostal space. It is best auscultated at the right sternal border between the 2nd and 3rd rib. Answer B:The pulmonary valve lies just left of the midline at the level of the 3rd costal cartilage. Answer C:The left atrioventricular (mitral) valve lies left of midline, between the 5th and 6th rib. Answer D:This is not a site of auscultation for valves Bottom Line:The aortic valve lies near the midline at the level of the 3rd intercostal space. It is best auscultated at the right sternal border between the 2nd and 3rd rib

A 75-year-old ex-smoker presents with complaints of significant leg cramping and numbness with walking. He states that when the symptoms occur, he sits to rest for about 5 minutes, and the symptoms dissipate. Walking more than 5 minutes usually brings the symptoms back. What test would be MOST appropriate to differentiate intermittent claudication versus spinal stenosis? a) Bicycle test of van Gelderen b) Brudzinski-Kernig test c) Lasegue's straight leg raise test d) Slump test

Correct answer: A The bicycle test of van Gelderen is used to diagnose neurogenic claudication. The patient is seated on an exercise bicycle and pedals against resistance. The patient initially starts pedaling while leaning backward to accentuate lumbar lordosis. If symptoms occur (pain into buttocks or thigh, tingling), then the first part of the test is positive. The patient then begins to pedal while leaning forward. If the pain subsides while in a flexed position, the second part of the test is positive. If the patient sits upright again and the pain returns, the test is positive for neurogenic intermittent claudication. Answer B:The Brudzinski-Kernig test is a type of dural tension test, similar to the straight leg raise test, except the patient performs the movement actively. A positive test indicates meningeal irritation, nerve root involvement, or dural irritation. This test is performed when meningitis is suspected. Answer C:Lasegue's SLR test is a neurological test of the lower limb. The lower leg is raised in supine with the knee straight, and the ankle is dorsiflexed to put tension on the nerve. A positive test is pain in the back and/or along the posterior leg and confirms irritation along the L5 S1 nerve root. Answer D:The slump test is a neural tension test that identifies neural tissue sensitivity. There are different stages that increase the tension on the dura, and a positive test is pain at any stage. The patient is seated and first slumps their lumbar and thoracic spine. Then they flex the neck. Then they slowly extend the knee. Then they dorsiflex the foot. The test can be stopped at any of these steps and is not always fully executed. Bottom Line:Intermittent claudication is a condition that implies vascular arterial insufficiency to the tissues. It is most evident with physical activity and exercise (usually walking) due to increased vascular demand. There are two types, vascular and neurogenic. Vascular intermittent claudication is more common. is usually related to arteriosclerosis or peripheral vascular disease, and commonly manifests in the legs. Neurogenic claudication is associated with spinal stenosis and its effect on circulation to the spinal cord and cauda equina. The bicycle test of Van Gelderen is used to diagnose neurogenic claudication.

An individual has been attending outpatient physical therapy for 2 weeks after a meniscus tear 4 weeks ago. The patient now has minimal swelling, significant muscle atrophy, pain in weight-bearing, and decreased balance. The patient would like to return to playing tennis as soon as it is advisable. Which of the following would be the MOST appropriate intervention to perform at this time? a) concentric/eccentric manually resisted short-arc-quads b) home exercise program with long-arc quads with an ankle weight c) single-limb balance with eyes open for proprioception d) cutting drills for return to sport

Correct answer: A The clinical practice guidelines promote supervised, progressive, range-of-motion exercises, progressive strength training of the knee and hip muscles, and neuromuscular training in patients with knee meniscus tears and articular cartilage lesions, and after meniscus or articular cartilage surgery. Concentric/eccentric manually resisted short-arc quads is the correct answer as the clinical practice guidelines promote a progressive strengthening program, and this would enable the physical therapist to provide strengthening in a safe, structured way. Answer B:Though a home exercise program is appropriate and a nice adjunct to the clinic, the range of motion may be too large and irritate the meniscus. In addition, the clinical practice guidelines promote a supervised, progressive strength program, and therefore this is not the best answer. Answer C:The patient has pain with weight-bearing and therefore unilateral balancing at this time is not the most appropriate choice. Answer D:Cutting drills for return to sport is too advanced for a patient with pain in weight-bearing, atrophy, and decreased balance.

A physical therapist examines a patient who complains of neck pain. The therapist suspects that poor body alignment may be contributing to the problem and decides to use a plumb line to assess posture from the lateral view. In a person with normal posture, the plumb line should pass posterior to WHICH STRUCTURE? a) hip joint b) earlobe c) lateral malleolus d) shoulder joint

Correct answer: A The correct answer is hip joint. A plumb line is typically used for reference and represents the relationship of the body parts with the line of gravity. Specific location of the plumb line in relation to surface landmarks include the following: slightly anterior to the lateral malleolus; slightly anterior to the axis of the knee joint; through the greater trochanter; slightly posterior to the axis of the hip joint; through the bodies of the lumbar and cervical vertebrae; through the shoulder joint; and through the lobe of the ear. Answer B:The plumb line passes through the earlobe, not posterior to it. Answer C:The plumb line passes slightly anterior to the lateral malleolus, not posterior to the structure. Answer D:The plumb line passes through the shoulder joint, not posterior to it. Bottom Line:The correct answer is hip joint. A plumb line is typically used for reference and represents the relationship of the body parts with the line of gravity. Specific location of the plumb line in relation to surface landmarks include the following: slightly anterior to the lateral malleolus; slightly anterior to the axis of the knee joint; through the greater trochanter; slightly posterior to the axis of the hip joint; through the bodies of the lumbar and cervical vertebrae; through the shoulder joint; and through the lobe of the ear.

A 25-year-old long-distance runner came to the physical therapy clinic complaining of right heel pain. He reported that the pain often occurs at the start and toward the end of training. He reported that the pain is worse in the morning and after training. On examination, the physical therapist noticed that the patient has a low arch on the right foot compared to the left. Which of the following structures is most likely affected? a) plantar fascia b) spring ligament c) flexor retinaculum d) anterior talofibular ligament

Correct answer: A The correct answer is plantar fascia. The patient described in the scenario may have plantar fasciitis. The plantar fascia supports the arch of the foot and acts as a shock-absorber. Plantar fasciitis is an overuse injury caused by repetitive stretching of the plantar fascia. Patients who regularly engage in activities that place increased load on the foot, such as running and jumping, are at higher risk for developing this condition. Answer B:The spring ligament or plantarcalcaneonavicular ligament attaches from the sustentaculum tali of the calcaneus to the inferior surface of the navicular bone. It supports the head of the talus. Although this ligament will be affected with repetitive loading of the foot, pathology of the spring ligament will not cause heel pain. Answer C:The flexor retinaculum is located on the medial aspect of the ankle. It is attached to the medial malleolus of the tibia and the medial aspect of the calcaneus. It acts as a pulley to prevent bowstringing of the muscles that insert into the plantar aspect of the foot. Pathology of this ligament will not cause heel pain. Answer D:The anterior talofibular ligament (ATFL) supports the lateral aspect of the ankle. It is the most commonly sprained ligament of the ankle, followed by thecalcaneofibular ligament. Injury to the ATFL is usually caused by inversion ankle sprain, not an overuse injury. Bottom Line:Plantar fasciitis is an overuse injury caused by repetitive stretching of the plantar fascia. It usually manifests as heel pain that is worse in the morning or at onset and end of activity.

A physical therapist is assessing a patient with complaints of low back pain. The patient has a history of recurrent bladder infections. The therapist decides to perform medical screening for urogenital disease and begins with percussion over the costovertebral angle. Costovertebral angle testing is performed in which of these locations? a) the acute angel formed on either side of the human back between the 12th rib and the vertebral column b) the acute angel formed on either side of the human back between the 11th rib and the vertebral column c) the acute angel formed on either side of the human back between the 10th rib and the vertebral column d) the acute angel formed on either side of the human back between the 9th rib and the vertebral column

Correct answer: A The costovertebral angle is the acute angle formed on either side of the human back between the 12th rib and the vertebral column. The kidney lies directly below this area, so pain can be elicited here with percussion when the person has kidney inflammation. The presence of pain is marked as a positive Murphy's punch sign or as costovertebral angle tenderness. Answers B & C & D:The costovertebral angle is the acute angle formed on either side of the human back between the 12th rib and the vertebral column. Bottom Line:The costovertebral angle is the acute angle formed on either side of the human back between the 12th rib and the vertebral column.

A physical therapist is designing a study to determine the influence of different interventions on the discharge range of motion in patients post total knee arthroplasty (TKA). The therapist decides to compare ice and soft-tissue interventions, use of a continuous passive movement machine, and knee mobilizations. Which of the following listed variables would be considered the dependent variable in this study? A. Knee range of motion. B. Ice and soft tissue intervention. C. Continuous passive movement machine. D. Knee mobilizations.

Correct answer: A The dependent variable in a study is the one that "depends on" or is the outcome of the independent variable. The dependent variable results from the influence of the independent variable or treatment. In this case, the dependent variable being measured is the knee range of motion of the patient post-TKA. Answer B:Ice and soft tissue in this example are types of interventions and are therefore independent variables and not the dependent variable, which is the outcome or effect on the range of motion results. Answer C:A continuous passive movement machine is an intervention or treatment and therefore is considered an independent variable and not a dependent variable, which is the outcome or effect, so therefore the knee range of motion. Answer D:Knee mobilizations are an intervention so, therefore, an independent variable. The dependent variable is the outcome or effect of the interventions and is, therefore, the knee range of motion.

Which of the following ACCURATELY DESCRIBES the proper procedure for assessing a patient's peripheral pulse? a) two fingers on the lateral palmar aspect of the wrist just proximal to the base of the carpometacarpal joint b) thumb on the dorsal surface of the foot near head of the 2nd metatarsal c) thumb placed posterior and silghtly distal to the medial malleolus c) two fingers on the lateral forearm just distal and lateral to the antecubital fossa

Correct answer: A The evaluation of circulation is a vital part of the physical examination. It not only provides information on heart rate and rhythm but also aids in the assessment of arterial disease. The proper procedure for assessing a patient's pulse is two fingers on the lateral palmar aspect of the wrist, just proximal to the base of the carpometacarpal joint. The placement of two fingers is appropriate because it allows the examiner to accurately palpate a patient's pulse without mistaking it for their own pulse.Knowledge of surface anatomy and where peripheral pulses can be palpated are important skills that physical therapists should be able to demonstrate. Answer B:Using the thumb to palpate a peripheral pulse is inappropriate because the examiner can confuse their own pulse with the patient's pulse due to the presence of a strong peripheral pulse in the thumb (princeps pollicis artery). In addition, the location referenced in this answer does not contain a palpable pulse. The next closest palpable pulse is the dorsal pedis pulse, which is often hard to find and detect. Answer C:The examiner's thumb should not be used to palpate a patient's peripheral pulse due to the presence of a strong peripheral pulse in the thumb (princeps pollicis artery). Nonetheless, the described location correctly indicates the posterior tibial pulse. Answer D:The described method, using two fingers, is appropriate. However, the described location does not contain a palpable pulse. The next closest palpable peripheral pulse would be the brachial artery pulse, which can be found in the antecubital fossa just medial to the biceps tendon. Bottom Line:The proper procedure for assessing a patient's pulse is two fingers on the lateral palmar aspect of the wrist, just proximal to the base of the carpometacarpal joint.

Which of the following tests is MOST appropriate for an elderly patient who said she fell while trying to get clothes out of her closet? A. functional reach test B. Romberg test C. tandem Romberg test D. four-square step test

Correct answer: A The functional reach test is an indicator of the ability to reach within the base of support and is most appropriate for testing the balance strategies for an individual reaching into a closet.Balance testing should be aimed at detecting the problems that caused the fall. If the patient fell during a reaching action, then the functional reach test to assess their reaching and noting their limited strategies is the most appropriate test. Answer B:The Romberg test is a static test of balance with feet together, and although it will test balance, it will not duplicate the actions being performed by the patient in this scenario. Answer C:The tandem Romberg test is a more advanced static balance test and again does not duplicate the actions being performed by the patient reaching into her closet. Answer D:The four-square step test is a test of multi-directional movements that assesses the ability to change direction. This is not the movement being described in this patient. Bottom Line:Balance testing should be aimed at detecting the problems that caused the fall. If the patient fell during a reaching action, then the functional reach test to assess their reaching and noting their limited strategies is the most appropriate test.

A patient with a right transtibial amputation received a new prosthetic device with a patellar tendon-bearing socket and a foam insert, and uses 5-ply socks. The physical therapist observes that during swing, the toe occasionally catches, and after doffing the prosthetic there are skin abrasions on the residual limb. Which of the following is the MOST appropriate recommendation to correct this gait deviation? a) add prosthetic socks b) change the foam insert c) shorten the length of the prosthesis d) modify the socket

Correct answer: A The gait deviation that is described here is due to the patient's limb pistoning in the socket. There is a loss of suspension of the prosthesis because it is too loose, which may result in poor clearance of the floor and abrasions on the limb. To improve suspension, adding prosthetic socks will decrease the extra space in the socket and improve suspension without permanently changing the socket. This patient is using 5-ply socks, and it is appropriate to use up to 10-ply as necessary. Answer B:A change in the foam insert when the patient is not using an excessive amount of sock ply (over 10-ply) would not be recommended to improve socket fit because fluctuations in sock ply are expected. It is more appropriate to alter the sock ply than to fabricate a new socket. Answer C:A permanent change in the prosthetic length would not be recommended to improve foot clearance if the problem is due to poor socket fit. Answer D:A modification of the socket when the patient is not using an excessive amount of sock ply (over 10-ply) would not be recommended to improve socket fit because fluctuations in sock ply are expected. Bottom Line: To insure good socket fit, besides having the appropriate socket shape, the appropriate ply sock should be utilized. If the socket pistons due to poor suspension, options include using more sock ply (if there is not already more than 10-ply being used). Signs of pistoning include reports of feeling loose, tripping, or catching the prosthetic foot on the floor during gait, and abrasions on the limb from sliding up and down in the socket.

A patient is receiving biofeedback training to the deep neck flexors. The goal is to improve the control and the strength. The patient lies supine with a blood pressure cuff under the cervical lordotic curve inflated to 20 mm Hg. The patient is then told to flex the head on the spine and press on the cuff to reach 24 mm Hg for 10 seconds and then release. When the patient gets to 24 mmHg, the needle on the read-out starts to bounce around, and the patient cannot hold smoothly. What is the BEST course of action for this training? a) regress to 22 mmHg as the goal b) continue at 24 mmHg but do not go higher c) set the goal to 26 mmHg d) discontinue the exercise as the patient is too weak for this challenge

Correct answer: A The goal is to improve control, timing, and coordination, so the patient needs to back off from this level, which is too challenging for now, and try a slightly easier goal of 22 mmHg, where the lack of strength and control will not negatively impact their training. This activity is also used to assess neck strength and is called the craniocervical flexion test. Answer B:The bouncing of the needle demonstrates that the patient is unable to perform the task, possibly due to weakness, and the activity needs to be regressed to a level where they can perform with the control required. Answer C:This would be a progression if the activity had been performed well, which was not the case. Answer D:The patient can still benefit from this activity; the biofeedback goal can be altered and still be greatly beneficial. There is no need to discontinue. Bottom Line:The craniocervical flexion test is used for coordination, strength, and endurance impairments of the deep neck flexors. When the patient is unable to keep fluidcontrol at a certain level the parameters are reduced to an easier level, such as 22 mmHg, and the exercise is continued there.

A physical therapist is treating a patient who has a tumor of the hippocampus. Which of the following functions should the physical therapist expect to be affected based on the location of the tumor? a) declarative memory b) receptive language c) motor planning d) spatial attention

Correct answer: A The hippocampus is found in the medial temporal lobe, which is responsible for declarative memory. The physical therapist would need to modify how he or she provides feedback and obtains past medical and social history information as a result of these memory deficits. The hippocampus is also considered part of the limbic system, which controls our emotional responses. Answer B:The posterior portion of the temporal lobe (Wernicke's area) is primarily responsible for receptive language. Answer C:The system of motor planning is complex and involves the frontal cortex, cerebellum, and basal ganglia. Answer D:The system for spatial attention is complex and involves the frontal, parietal, and occipital lobes of the cerebral cortex as well as subcortical structures of the right hemisphere of the brain. Bottom Line:Physical therapists need knowledge of neuroanatomy as they plan their examinations and treatment plans for patients afer a neurological injury. A patient with damage to the hippocampus would have poor declarative memory, and the physical therapist would need to modify their method of providing feedback and obtaining past medical and social history information as a result of the patient's memory deficits.

A patient with right shoulder pain has a prominent inferior angle of the scapula upon observation. During active range of motion, that same scapula also demonstrates winging on return from elevation. Which of the following are the BEST interventions for the physical therapist to perform to help this patient's restrictions? A. Manual pectoralis minor stretching; supine scapular punch/protraction with a 5 lb weight in the hand B. Manual pectoralis minor stretching; seated rows with black therapy band C. Manual pectoralis major stretching; prone horizontal rows with a 5 lb weight in the hand D. Pectoralis major stretching with arms at 90/90 in a corner; prone push-up with a "plus" adding scapular protraction at end range SHOW EXPLANATION

Correct answer: A The inferior angle of the scapula protrudes when the scapula tilts anteriorly. The reason for the anterior tilt is a tight pectoralis minor pulling on the coracoid. Thus manual stretching of the pectoralis minor with pressure applied to the coracoid is the most effective method to elongate it and correct the anterior tilt. The scapula is winging due to serratus weakness. A supine scapular punch with the protraction motion against resistance will help to strengthen the serratus anterior. Answer B:It is appropriate to stretch pectoralis minor, but a rowing motion strengthens scapular retractors, not protractors, and is not appropriate for this patient. Answer C:Stretching the pectoralis major would not improve anterior scapular tilting. The pectoralis attaches to the humerus and to the thorax, but not to the scapula.In addition, a rowing motion strengthens scapular retractors, not protractors. Answer D:Stretching the pectoralis major would not improve anterior scapular tilting because it does not attach on the scapula. The prone push-up exercise with the extra motion to protraction in a closed chain is appropriate for serratus strengthening. Bottom Line:Anterior tilting compensations of the scapula are due to a short pectoral minor pulling the scapula anteriorly by the coracoid process. Medial winging occurs with weakness or lack of control of the serratus anterior.

A 50-year-old man in the acute care hospital was seen by the physical therapist for ambulation. While walking he presented with exertional dyspnea and syncope. He has a history of a left atrial myxoma. The physical therapist decides to perform auscultation of the heart. The BEST auscultation point to assess the mitral valve would be where? a) just left of mid-line deep to the 4th costal cartilage b) near the mid line at the level of the 3rd intercostal space c) just left of the mid-line at the level of the 3rd costal cartilage d) slightly right of mid line deep to the 4th and 5th intercostal spaces

Correct answer: A The left atrioventricular (mitral) valve lies just left of midline, deep to the 4th costal cartilage. Answer B:The aortic valve lies near the midline at the level of the 3rd intercostal space. Answer C:The pulmonary valve lies just left of the midline at the level of the 3rd costal cartilage. Answer D:The right atrioventricular (tricuspid) valve lies slightly right of midline, deep to the 4th and 5th intercostal spaces.

A physical therapist is examining a patient who had a left middle cerebral artery stroke and a diagnosis of Broca's aphasia. In which area of language function will the patient have the MOST severe deficits? a) ability to express language verbally b) ability to repeat language verbally c) comprehension of verbal language d) comprehension of written language

Correct answer: A The left hemisphere of the brain is responsible for most of language function in the majority of people. A patient with Broca's aphasia would have damage to the left frontal lobe, which is responsible for language output, and the patient would have severe deficits in the ability to express language verbally. Answer B:A patient who is unable to repeat verbal language would be diagnosed with conduction aphasia. Conduction aphasia is caused by a lesion to the arcuate fasciculus, which connects Broca's area to Wernicke's area. Answer C:A patient with a primary deficit with the comprehension of verbal language would be diagnosed with Wernicke's aphasia, which is also known as receptive or fluent aphasia. Answer D:A patient who has severe deficits in the comprehension of written information would be diagnosed with alexia. Alexia is a symptom that may be found in all types of aphasia but is unlikely to be the most severe deficit in a patient who is diagnosed with Broca's aphasia. Bottom Line:Broca's aphasia causes deficits in verbal language expression, which makes communication difficult during physical therapy. Physical therapists must be able to predict language and cognitive deficits based on the patient's diagnosis in order to adapt their examination and interventions to the patient's abilities. Therapists should simplify their requests for verbal information to simple yes/no questions when examining and treating a patient with Broca's aphasia.

A physical therapist is assessing a patient with complaints of right flank and low back pain over the last year. The patient has a history of recurrent indigestion and bladder infections. The therapist decides to perform a physical examination of the abdominal region and begins with the spleen. Palpation of the spleen occurs in WHICH ABDOMINAL QUADRANT? a) left upper quadrant b) left lower quadrant c) right upper quadrant d) right lower quadrant

Correct answer: A The left upper quadrant contains the spleen and stomach. Answer B:The left lower quadrant contains the small and large intestine. Answer C:The right upper quadrant contains the gallbladder, liver, pancreas, and right kidney. Answer D:The right lower quadrant contains the small intestine, appendix, and large intestine. Bottom Line:The left upper quadrant contains the spleen and stomach. TrueLearn Insight :Performing a medical screen is an important step in making a physical therapy diagnosis. Medical screening helps the physical therapist determine whether a patient referral to another practitioner is needed. When the therapist is screening, if findings are suspicious or require diagnostic skills or equipment outside of the physical therapy scope of practice, a medical referral is warranted.

An individual presents to physical therapy after being in a cast for 8 weeks status post right tibial fracture. The patient reports difficulty going down the stairs step over step. When the left foot goes forward to touch the next step, the right ankle feels so stiff that it makes the motion very difficult. What joint mobility assessment should the physical therapist perform at the talocrural joint to see if joint hypomobility is the source of the limitation in motion? a) posterior glide of the talus b) anterior glide of the talus c) posterior glide of the tibia d) anterior glide of the calcaneus

Correct answer: A The limited motion in this scenario is right ankle dorsiflexion. During dorsiflexion, the talus needs to glide posteriorly. If posterior talar glide is limited, dorsiflexion will be limited. Answer B:An anterior glide of the talus is a component motion of ankle plantarflexion. Answer C:A posterior glide of the tibia at the talocrural joint would occur during ankle plantarflexion. Answer D:The calcaneus is not part of the talocrural joint and does not have an accessory motion during ankle plantarflexion and dorsiflexion. Bottom Line:A functional activity such as a step down requires dorsiflexion range of motion (ROM). To improve dorsiflexion ROM, a posterior glide of the talus is needed.

A patient presents with damage to the long thoracic nerve after vigorously shoveling snow 2 weeks ago. The therapist asks the patient to actively flex the shoulder. What should the PT expect to see upon examination of shoulder flexion active range of motion? a) winging scapula and limitation in shoulder flexion b) winging and anterior tipping of the scapula during flexion c) winging scapula and excessive scapular protraction during flexion d) winging and excessive upward rotation of the scapula during flexion

Correct answer: A The long thoracic nerve innervates the serratus anterior muscle. The serratus anterior acts to abduct the scapula (protract), rotate the inferior angle of the scapula laterally (upward rotation), and hold the medial border of the scapula firmly against the rib cage. Winging of the scapula is a classic sign of serratus anterior damage. Because the serratus is not stabilizing the scapula against the thorax, muscles that act from the scapula and attach to the humerus do not have a stable base to from which to work. In addition, if the serratus is not helping the scapula travel along the chest wall in protraction, the rotator cuff muscles will get stretched out and become ineffective as the humerus keeps moving up. Therefore, flexion of the humerus with the deltoid/rotator cuff force couple becomes difficult and we see an inability to flex the shoulder through the full range of motion.Possible mechanisms of injury to the long thoracic nerve include heavy lifting overhead, pressure to the nerve from backpacking, or vigorous upper extremity (UE) activities such as shoveling or chopping. The serratus anterior muscle is innervated by the long thoracic nerve and is an extremely important scapular stabilizer. In addition, when the arm elevates, the scapula is supposed to elevate, upwardly rotate and protract/abduct. If the serratus is not working, the scapula does not abduct along the chest wall, and therefore, the shoulder mechanics are thrown off. The patient may experience weakness with flexion of the extended arm, difficulty with abduction greater than 90 degrees, and even shortening of the rhomboids in some cases as their adduction pull is unchallenged. Answer B:Winging of the scapula would be present; however, there would not be tipping. Anterior tipping is due to a tight pectoralis minor pulling on its attachment at the coracoid process, causing the scapula to tip forward in the sagittal plane, not wing in the transverse plane. Answer C:Winging of the scapula would be present. However, scapular protraction would beinsufficient, not excessive. The serratus helps the scapula abduct/protract so if it is not functioning, that motion does not occur. Answer D:Winging of the scapula would be present. However, scapular upward rotation would not be excessive. Bottom Line:Winging of the scapula is a classic sign of serratus anterior damage. Because the serratus is not stabilizing the scapula against the thorax, muscles that act from the scapula and attach to the humerus do not have a stable base to from which to work.

A physical therapist is treating a patient who presents with a resting tremor in his hands, a shuffling gait pattern, and freezing when walking into a new room. Based on the signs and symptoms, in which area of the brain is a lesion located? A. basal ganglia B. cerebellum C. medulla D. occipital cortex

Correct answer: A The patient described in the scenario is displaying symptoms of Parkinson disease. Parkinson disease is caused by damage to the dopaminergic neurons of the basal ganglia and typically is treated with dopamine replacement medications, physical therapy, and occupational therapy.Physical therapists must be able to predict lesion location so that they can perform effective and efficient examinations in patients with neurological injuries. Answer B:Patients with damage to the cerebellum typically present with ataxia, an intention tremor (the tremor gets worse as it approaches its target), and decreased balance. Answer C:Patients with damage to the medulla typically present with inconsistent patterns of sensory loss, quadriplegia, diplopia (double vision), dysarthria (difficulty with motor aspects of speech), and dysphagia (difficulty with swallowing). Answer D:Patients with damage to the occipital cortex present with deficits in their visual fields. Bottom Line:Patients with basal ganglia dysfunction, such as with Parkinson disease, display resting tremors in the hands, a shuffling gait pattern, and difficulty initiating movement.

An individual is being treated in an outpatient physical therapy clinic following a grade 2 ankle sprain. Four weeks have passed, and the patient has been gradually improving their strength and balance. The patient was able to perform single limb stance on the ground with their eyes closed for 30 seconds without pain or loss of balance (LOB). Which exercise would be the NEXT progression in difficulty? a) single limb stance on a balance board b) modified tandem on an airex pad c) tandem stance on an airex pad d) single limb stance with eyes closed on a balance board

Correct answer: A The patient is able to stand on one leg on level ground and can do it with his eyes closed, removing visual cues. The next level of difficulty is to stand on a surface that is moveable, which will challenge the proprioceptors since the feedback from stable ground is not available. Once this is mastered, then the visual system cues can be removed again. Answers B & C:This answer is incorrect as this is not a progression in comparison to what the patient was already able to accomplish. Answer D:This exercise is too difficult of a progression at this time. It modifies the proprioceptive and visual input. It is better to stage each one and increase the level of difficulty more gradually. Bottom Line:Balance can be separated from easiest to hardest which is bilateral stance to tandem and then single limb stance. Once a stable surface is successfully achieved,eyes closed is used next, followed by unstable surfaces, then the addition of eyes closed again, and lastly perturbations. There can also be options to challenge the vestibular system with a dome around the head or moveable walls to prevent environmental cues.

The physical therapist is following a 25-year-old female with a diagnosis of low back pain due to a fall at work. During a scheduled physical therapy visit, she reports she has persistent indigestion that woke her up in the middle of the night. She has taken antacids for relief. She now complains of right lower quadrant (RLQ) abdominal pain that is relieved by lying still in fetal position. The therapist decides to examine her abdomen through auscultation and palpation. The therapist confirms that the patient has hypoactive bowel sounds, guarding of the inferior anterolateral abdominal wall, positive McBurney's and obturator signs, and right lower quadrant rebound tenderness. The therapist is correct in determining the client needs to be referred for follow-up secondary to suspicion of which of the following diagnoses? a) appendicitis b) viral gastroenteritis c) pelvic inflammatory disease d) pneumoperitoneum

Correct answer: A The patient presentation, history, and physical examination support a diagnosis of acute appendicitis. The patient's initial, diffuse paraumbilical discomfort is consistent with the distribution of referred pain from the appendix. The pain progressed, becoming more severe and precisely located in the RLQ, indicating that the inflamed appendix has irritated the adjacent parietal peritoneum. This accounts for the RLQ somatic pain, guarding, and rebound tenderness. The positive McBurney and obturator signs are a further indication of the involvement of the peritoneum. Answer B: Gastroenteritis is an inflammation of the large intestine caused most often by a norovirus infection. Viral gastroenteritis has a sudden onset and generally lasts for 2-4 days. The illness is usually characterized by diarrhea and abdominal cramps/pain, nausea, and vomiting. The stools are typically loose, watery, and without blood or mucus. This patient did not report diarrhea as a symptom, so viral gastroenteritis is not likely to be the cause of her abdominal pain. Answer C:Pelvic inflammatory disease (PID) is initiated by infection, usually sexually transmitted, that moves superior from the vagina and cervix into the uterus and fallopian tubes. Women with PID present with a wide variety of symptoms, but the most common presenting complaint is lower abdominal pain. Most women (75%) also exhibit abnormal vaginal discharge. The diagnosis of acute PID is based primarily on patient history of previous PID and clinical findings, and is not consistent with this patient's information. Answer D: Pneumoperitoneum is pneumatosis (abnormal presence of air or other gas) in the peritoneal cavity, a potential space within the abdominal cavity. The illness is usually characterized by hypoactive bowel sounds, guarding of the superior and anterolateral abdominal wall, and tenderness of epigastric and left hypochondrial regions during deep palpation. This patient did not report epigastric pain as a symptom, so pneumoperitoneum is not likely to be the cause of her abdominal pain. Bottom Line:RLQ somatic pain, guarding, rebound tenderness and a positive McBurney and obturator signs are indicators of appendicitis.

A 38-year-old female is being examined by a physical therapist at an urgent care outpatient center after a fall from a 3-foot height at work at an agricultural production plant. The patient reports that she fell "hard" on her left side but did not have any pain after the fall. She was told to go to the urgent care center by her employer. On physical therapy examination, the patient reports left shoulder pain and stiffness. Blood pressure is hypotensive, and heart rate is 100 beats per minute. She also has shortness of breath (SOB) and abdominal pain that improves when standing. Sitting and lying exacerbate both the SOB and the abdominal pain symptoms. The therapist decides to examine her chest and abdomen through auscultation and palpation. The therapist confirms the patient has bilateral wheezing, diffuse abdominal discomfort, left upper quadrant abdominal pain with rebound tenderness, and a positive Kehr sign. The therapist is correct in determining the client needs to be referred for follow-up secondary to suspicion of which of the following diagnoses? a) ruptured spleen b) rib fracture c) tubo-ovarian abscess d) pelvic inflammatory disease

Correct answer: A The patient presentation, medical history, and physical examination support a diagnosis of a ruptured spleen due to abdominal trauma. Splenic rupture potentially allows a large volume of blood to leak into the peritoneal cavity. With a small rupture, the bleeding may stop without surgical intervention.Rupture of a normal spleen is most likely the result of trauma. Signs and symptoms include tachycardia, hypotension, tenderness of the left abdominal upper quadrant, and splenomegaly. Hemoperitoneum is a cardinal sign. Hemoperitoneum can cause diffuse abdominal pain but is more likely to irritate parietal peritoneum adjacent to the spleen. This results in significant left upper quadrant (LUQ) pain and rebound tenderness. Left shoulder tenderness (Kehr sign) may also be present due to irritation of diaphragmatic parietal peritoneum and referred pain to C3-5 dermatomes. Blood filling the subphrenic recess of the peritoneal cavity can also put pressure on the diaphragm, causing dyspnea. Answer B:This patient presents with signs and symptoms related to blunt abdominal trauma. Simple rib fractures are the most common form of chest injury. Signs and symptoms of rib fracture include pain on inspiration, parasternal pain, dyspnea, and local pain at site of injury. The patient did not report these symptoms and had symptoms related to hemorrhaging (tachycardia with low blood pressure), so rib fracture is not likely to be the cause of her symptoms. Answer C:The patient presentation, history, and physical examination do not support a diagnosis of tubo-ovarian abscess. Tubo-ovarian abscess (TOA) is an inflammatory mass involving the uterine tube, ovary, and occasionally, adjacent pelvic viscera. These abscesses typically result from recurrent infections of the upper reproductive tract. The exudate produced by the abscess may lead to rupture, which results in peritonitis. Signs and symptoms include lower abdominal and pelvic pain, rebound tenderness in both lower quadrants, nausea and vomiting, fever, tachycardia, and a feeling of fullness. The patient did not report these symptoms, so TOA is not likely to be the cause of her abdominal pain. Answer D:The patient presentation, history, and physical examination does not support a diagnosis of pelvic inflammatory disease (PID). Pelvic inflammatory disease is initiated by infection, usually sexually transmitted, that moves superior from the vagina and cervix into the uterus and uterine tubes. Women with PID present with a wide variety of symptoms, but the most common presenting complaint is lower abdominal pain. Most women (75%) also exhibit abnormal vaginal discharge. The diagnosis of acute PID is primarily based on patient history of previous PID and clinical findings, and is not consistent with this patient's information. Bottom Line:Signs and symptoms of splenic rupture include tachycardia, hypotension, tenderness of the left abdominal upper quadrant (LUQ), and splenomegaly. Hemoperitoneum is a cardinal sign that can result in significant LUQ pain, rebound tenderness and left shoulder tenderness (Kehr sign). TrueLearn Insight :The Kehr sign results when blood from an injured spleen irritates the diaphragm and creates referred pain in the region of the left shoulder.

A patient reports pain and tenderness on the radial side of the wrist near the base of the thumb that has gradually been increasing in severity. The patient is a baker and reports that the action of kneading dough, especially when stretching and spreading out the dough by stretching out his fingers and thumb, causes the thumb/radial wrist pain to increase. Based on this patient's history, which of the following special tests would be MOST appropriate to identify the structure(s) involved? a) have the patient flex the thumb into the palm, and then the therapist passively ulnarly deviates the patient's wrist b) the patient is asked to flex both wrists and keep the backs of the hands pressed against each other for 60 seconds to see if the symptoms are reproduced c) the patient is asked to fully extend both wrists and press the palms together for 60 seconds to see if the symptoms are reproduced d) while maintaining the thumb in extension, the therapist applies a valgus force to the thumb's metacarpophalangeal (MCP) joint

Correct answer: A The patient's symptoms of pain and tenderness are localized to the radial side of the wrist at the base of the thumb, and worsen with the motions of thumb extension and abduction, such as when spreading out the fingers and thumb. The abductor pollicus longus (APL) and the extensor pollicus brevis (EPB) make up the first extensor compartment found on the dorsum of the wrist and share a tendon sheath. Tenosynovitis of the APL and EPB would cause tenderness in the area of the radial side of the wrist near the snuff-box and is called de Quervain's tenosynovitis. To test for de Quervain's tenosynovitis, the Finkelstein's test as described in this answer choice of ulnar wrist deviation while the thumb is flexed and adducted against the hand will stress the area and recreate the pain and tenderness. Answer B:Maintaining both wrists in full flexion for 60 seconds describes the Phalen's test for carpal tunnel syndrome. A positive test results in the patient experiencing numbness and tingling along the distribution of the median nerve. In this patient scenario, the symptoms were not indicative of a problem with a nerve such as numbness or tingling, and were not located in the median nerve distribution areas of the palmar side of the thumb and second and third digits. Thus the median nerve is unlikely to be the affected structure. Answer C:Maintaining both wrists in full extension for 60 seconds describes the Reverse Phalen's test for carpal tunnel syndrome. It is similar to the original Phalen's test where wrists are maintained in flexion for 60 seconds but may create even more intracarpal tunnel pressure to elicit symptoms. A positive test results in the patient experiencing numbness and tingling along the distribution of the median nerve. In this patient scenario, the symptoms were not indicative of a problem with a nerve, such as numbness or tingling, and were not located in the median nerve distribution areas of the palmar side of the thumb and second and third digits. Thus the median nerve is unlikely to be the affected structure. Answer D:Applying a valgus stress to the MCP joint of the thumb tests the integrity of that joint's ulnar collateral ligament (UCL). A complete tear would result in excessive valgus motion at the joint. The patient would usually provide some sort of traumatic history to tear the UCL, and the pain would be localized to the MCP area. In addition, the patient might report feelings of instability in that joint. In this patient scenario, there was no history of trauma, and the symptoms are not located in the area of the MCP of the thumb. Bottom Line:DeQuervain's tenosynovitis is usually caused by excessive and repetitive thumb abduction and extension and inflames the sheath of the EPB and APL tendons, causing pain along the radial border of the snuff-box on the radial side of the wrist where the tendons are located.

A patient presents to the clinic with a report of numbness and tingling into the right thumb and index finger on the palmar side. The therapist suspects neurodynamic dysfunction. In which position should the therapist place the upper extremity to perform an upper limb tension test (ULTT) to determine if it is contributing to this patient's symptoms? A. Shoulder abduction and external rotation, elbow extension, forearm supination, and finger extension B. Shoulder adduction and internal rotation, elbow extension, forearm supination, and finger extension C. Shoulder abduction and external rotation, elbow flexion, forearm pronation, and finger extension D. Shoulder abduction and internal rotation, elbow extension, forearm pronation, and finger flexion

Correct answer: A The position of shoulder abduction and external rotation, elbow extension, forearm supination, and finger extension correctly describes the ULTT for the median nerve. Answer B:This position does not directly tension any of the upper extremity nerves. Answer C:This position would bias the ulnar nerve. Answer D:This position would bias the radial nerve. Bottom Line:The median nerve is placed under tension or "biased" when the upper extremity (UE) is placed in a position of shoulder abduction and external rotation, elbow extension, forearm supination, and finger extension.

A patient with a delayed healing wound is referred to a therapist. As the therapist decides which modality would be best, he wants to the current stage of healing of the wound. WHICH STAGE OF HEALING is marked by epithelialization, collagen production, wound contraction, and neovascularization to cover the wound and impart strength to the injury site? a) proliferation phase b) inflammation phase c) maturation phase d) epithelialization phase

Correct answer: A The proliferation phase typically lasts days 3-20. Its purpose is to cover the wound and impart strength to the injury site. It does this through four processes: epithelialization, collagen production, wound contraction, and neovascularization. Answer B:The inflammation phase typically lasts day 1-6. It is the immediate protective response that attempts to destroy, dilute, or isolate cells or agents that might be at fault. It is a normal and necessary prerequisite to healing. If there is no inflammation, healing cannot occur. Answer C:The maturation phase is the longest phase of the healing process. It last from day 9 until the tissue is completely healed, which can last over a year from the initial insult. The ultimate goal of this phase is to restore prior level of function to the injured tissue. Answer D:Epithelialization is one part of the proliferation phase. Epithelialization is the reestablishment of the epidermis. It provides a protective barrier to prevent fluid and electrolyte loss and to decrease the risk of infection. Bottom Line:The ultimate goal of inflammation and repair is to restore function by eliminating the pathological or physical insult, replacing the damaged tissue, and promoting regeneration of normal tissue structure. The healing process consists of three phases: inflammation, proliferation, and maturation.

A researcher is trying to document the experiences of individuals who have suffered a spinal cord injury. The researcher is analyzing data from numerous sources, including observation of the subjects in the therapy department, interviews with the subjects, and collection of personal journal entries by the subjects. Which type of research methodology is being used in this study? A. Qualitative B. Quantitative C. Concurrent mixed method D. Sequential mixed method

Correct answer: A The research being described utilizes various data sources, all of which are qualitative. Qualitative research includes data such as observation, open-ended interviews, analysis of documents including journals, and audio and visual materials. Any combination of these types of data can be used in qualitative work, which often includes more than one data source to strengthen the work. Answer B:Quantitative research is experimental and tests theories by comparing the relationship between variables. The type of data collection described in the question is used in qualitative work and is not experimental. Answer C:Mixed-method research includes both quantitative and qualitative data, and concurrent mixed method collects both types of data at the same time. None of the data-collection methods described in the question are quantitative, and so this is not a mixed-method design. Answer D:Mixed method research includes both quantitative and qualitative data, and sequential mixed method work collects one type of data to establish the other. For example, qualitative data is collected to develop a new type of measurement tool which is then quantitatively tested. None of the data in the question is quantitative, and therefore this is not a mixed-method study. Bottom Line: Qualitative research often involves the study of lived experiences and utilizes several forms of data collection, such as open-ended interviews, observation, and documents. This data can be collected concurrently or sequentially, but for the study to be considered quantitative or mixed method, some experimental work must be included in the study. The study described in the question is purely qualitative.

The therapist at an outpatient clinic is responsible for maintaining the proper pool temperature for therapy patients. The caseload of the therapists using the pool consists mostly of elderly patients using the pool for light exercise due to deconditioning and arthritis. What is the BEST temperature to try to maintain in the pool? A. 93 degrees Fahrenheit B. 90 degrees Fahrenheit C. 82 degrees Fahrenheit D. 64 degrees Fahrenheit

Correct answer: A The temperature of the water in an exercise pool should be kept between 79 and 97 degrees Fahrenheit. The amount of movement expected to be performed by the patient should be used to determine optimal temperature.The warmer end of the range (93 to 97) should be used for therapeutic exercise when low-intensity activities, such as light exercise by elderly patients, will be performed. Answer B:Due to the low intensity of activity expected by the patient population, the higher end of the range of temperature is more appropriate to use. Answer C:The cooler range of 79 to 82 degrees Fahrenheit is recommended for recreational pools or when more intense exercise will be performed. Because the cooler temperature dissipates heat produced by the patient, patients are allowed to perform more vigorous exercise with less fatigue. Answer D:The water temperature should never be allowed to be below 65 degrees because such low temperatures can impair the ability of muscles to contract. Bottom Line:To optimize the cardiovascular, respiratory, renal, and psychological benefits of hydrotherapy, the use of an exercise pool that allows full-body immersion and exercise is recommended. Maintaining proper temperature in the pool is imperative and based on the amount of movement expected to be performed by the patient.A range of 79 to 97 degrees Fahrenheit is acceptable but should be adjusted based on patient population.

Which of the following tests and measures is the MOST useful in supporting a diagnosis of a denervation injury in a lower extremity peripheral nerve? a) nerve conduction study b) reflex testing c) lumbar puncture d) sensation testing with monofilaments

Correct answer: A This is the best answer because nerve conduction studies measure action potential conduction in peripheral nerve fibers. If a denervation injury is present, there would be definitive and objective measurements to support the presence of lesions along a peripheral nerve. Answer B:Reflex testing tests specific nerve root levels, such as C5 or C6, not peripheral nerves. Answer C:Lumbar punctures do not provide specific information about the integrity of peripheral nerves. They are performed to obtain a sample of cerebral spinal fluid for analysis. Answer D:Monofilament testing involves using thin fibers to test for sensation. It is often used to evaluate for the presence of protective sensation in the distal extremities. However, it is not the best answer in this case because it can provide only limited information on the function of sensory nerves. Nonetheless, it is a valuable screening tool.

A patient with low back pain (LBP) and back-related buttock pain is being treated by a physical therapist. His pain is moderate, and he has limited mobility in the low back. Positional changes such as flexion or extension do not alter the symptoms. The pain initially radiated down to his ankle but is now localized to the buttock. His score on the Fear Avoidance Belief Questionnaire (FABQ) indicates he is not fearful or avoidant of activity. Which of the following interventions should the physical therapist perform based on the above information? a) manipulation b) traction c) active rest d) prone press ups

Correct answer: A This patient has symptoms that are centralizing, areas of stiffness, and no red flags. He would benefit from manipulation to improve range of motion and decrease pain. As per the clinical practice guideline by the Orthopedic section of the American Physical Therapy Association (APTA), thrust manipulative procedures can be used as a component of a comprehensive treatment plan to reduce pain and disability in patients with mobility deficits, acute LBP, and/or back-related buttock or thigh pain. Answer B:This answer is incorrect as there are no symptoms distal to the knee that are not responding to a directional preference. In addition, evidence for the benefit of traction is weak. Answer C:This answer is incorrect as active rest would be best used in a patient where the low back pain was highly irritable and with high symptom provocation. Answer D:This answer is incorrect as the symptoms are no longer distal to the knee and have plateaued, and the symptoms are not known to respond to directional preferences.

A patient presents with complaints of neck pain and right upper extremity numbness, tingling, and pain that began about 1 month ago. He describes it as intermittent and notices it much more when he is working on his computer. During the evaluation, he states the symptoms are not present. What test would be BEST to elicit the patient's symptoms? A. Spurling's compression test B. Distraction test C. Vertebral artery test D. Transverse ligament of the atlas stress test

Correct answer: A This patient is describing a nerve-type pain.Spurling's compression test is performed if the patient complains of any nerve root symptoms during the history. The test is designed to provoke the symptoms. The patient side bends toward the unaffected side followed by the affected side. The examiner presses carefully straight down on the head. The test is positive if the patient has pain that radiates toward the arm in which the head is side bent. Answer B:The distraction test is also used for patients who have complained of radicular symptoms in the history or show radicular symptoms during the evaluation. During this test the examiner slowly distracts the patient's head by placing one hand on the chin and the other on the occiput. The test is positive if the patient reports pain is decreased, indicating pressure on the nerve roots has been relieved. This test would also be used on the patient in the scenario above, except it would RELIEVE the symptoms and not ELICIT them. Answer C:The vertebral artery test is used to assess the integrity of the vertebrobasilar vascular system. This test should be performed on all patients before performing cervical mobilizations or manipulations. This test would likely be performed on the patient; however, it would be used only to rule out vascular symptoms, NOT to elicit neurological symptoms. Answer D:The transverse ligament test is used to test the integrity of the transverse ligament. The patient is supine, and C1 is glided anteriorly. The examiner should have a firm end feel. This test is not necessarily indicated from the symptoms described in the scenario. Bottom Line:Spurling's foraminal compression test is commonly used to diagnose cervical radiculopathy symptoms. It is designed to provoke the symptoms by narrowing the foraminal space and thereby applying pressure to the nerve. It is commonly followed by the distraction test to see if cervical traction relieves the symptoms.

A physical therapist is treating a 13-year-old patient during a scheduled physical therapy visit. The patient was prescribed crutches for a lower extremity fracture and has non-weight bearing status. The patient is now complaining of right upper extremity weakness, numbness, and tingling. The therapist assesses the upper extremity and finds sensory impairment on the radial side of the dorsum of the hand over first dorsal interosseous muscle, and motor weakness of elbow flexion and extension, wrist extension and supination, and hand extension of metacarpophalangeal joints. The sensory and motor deficits described are MOST consistent with which neuropathy? a) compressive radial neuropathy b) compressive axillary neuropathy c) compressive distal radial neuropathy d) compressive ulnar neuropathy

Correct answer: A This patient presents with signs and symptoms related to a compressive radial neuropathy. Injury to the radial nerve in the axilla can result from compression against the humeral shaft. Sensory deficit in this patient included radial side of dorsum of the hand, especially between metacarpals 1 and 2. Motor deficit included elbow extension, wrist extension, extension of metacarpophalangeal joints, extension of interphalangeal joints, elbow flexion (brachioradialis), and supination. Answer B:A patient with axillary neuropathy would present with deltoid weakness and sensory loss in the shoulder region. The diagnosis of axillary neuropathy is not consistent with this patient's information. Answer C:Distal radial neuropathy is diagnosed by weakness in wrist extension ("wrist drop") and sensory deficit from the distribution of the superficial radial nerve. Elbow extension would be normal in distal radial neuropathy because motor innervation to the triceps muscle branched from the radial nerve proximal to the fracture. The diagnosis of distal radial neuropathy is not consistent with this patient's information. Answer D:A patient with injury of the ulnar nerve would present with sensory loss on the anteromedial aspect of the hand as well as loss of motor innervation of the intrinsic hand muscles. The diagnosis of ulnar neuropathy is not consistent with this patient's information.

A physical therapist is assessing a 65-year-old female patient status postsurgical repair of a displaced right humeral neck fracture. The patient has a history of osteoporosis. The following findings are present on evaluation: swelling and pain in the right arm and proximal forearm, inability to abduct the arm beyond 30 degrees, and diminished sensation in skin on the lateral deltoid. The sensory and motor deficits described are MOST consistent with which fracture? a) fracture of the humeral neck with axillary neuropathy b) fracture of the humeral neck with radial neuropathy c) fracture of the humeral neck with ulnar neuropathy d) fracture of the humeral neck with radial neuropathy in the axilla

Correct answer: A This patient presents with signs and symptoms related to axillary neuropathy. A patient with axillary neuropathy would present with deltoid weakness and sensory loss in the shoulder region. With a displaced surgical neck fracture and axillary nerve injury, the abduction of the arm may be initiated (supraspinatus) but not carried beyond 30 degrees due to the compromise of the nerve supply to the deltoid. Sensation from skin on the lateral aspect of the deltoid would be diminished because it is also supplied by the axillary nerve. Sensation from skin over the proximal deltoid would not be impaired because it is supplied by supraclavicular nerves (C3-C4).Abduction of the arm (0-90 degrees) occurs at the glenohumeral joint. Beyond horizontal, movement of the arm depends on scapular rotation.• Abduction is initiated by supraspinatus (0-30 degrees).• Deltoid is the prime mover for arm abduction beyond 30 degrees.• Scapular rotation involves serratus anterior and trapezius. Answer B:Radial neuropathy is diagnosed by weakness in wrist extension ("wrist drop") and sensory deficit from the distribution of the superficial radial nerve. Elbow extension was normal because the motor innervation to the triceps muscle branched from the radial nerve proximal to the fracture. The patient presentation is not consistent with radial neuropathy. Answer C:Injury of the ulnar nerve affects dermatomes on the anteromedial aspect of the hand as well as the motor innervation of intrinsic hand muscles. The patient presentation is not consistent with ulnar neuropathy. Answer D:Injury to the radial nerve in the axilla can result from compression against the humeral shaft. Sensory deficit would include radial side of dorsum of the hand, especially between metacarpals 1 and 2. Motor deficit would include elbow extension, wrist extension, extension of metacarpophalangeal joints, extensionof interphalangeal joints, elbow flexion (brachioradialis), and supination. The patient presentation is not consistent with radial neuropathy in the axilla.

The physical therapist is assessing a 23-year-old patient status postsurgical repair of a compound fracture of the distal third of the left humerus secondary to a motor vehicle accident. The following findings are present: swelling and pain in the left arm and proximal forearm, inability to extend the wrist, and diminished sensation in skin on the dorsum of the hand, especially over the first dorsal interosseous muscle. The sensory and motor deficits described are MOST consistent with which condition? A. Fracture of the humerus with radial neuropathy B. Fracture of the humerus with compressive radial neuropathy in the axilla C. Fracture of the humerus with axillary neuropathy D. Fracture of the humerus with ulnar neuropathy

Correct answer: A This patient presents with signs and symptoms related to radial neuropathy. Radial neuropathy was diagnosed by weakness in wrist extension ("wrist drop") and sensory deficit from the distribution of the superficial radial nerve. Elbow extension was normal because the motor innervation to the triceps muscle branched from the radial nerve proximal to the fracture. Answer B:Injury to the radial nerve in the axilla can result from compression against the humeral shaft. A patient with radial nerve injury in the axilla would have sensory deficit, including radial side of dorsum of the hand, especially between metacarpals 1 and 2. Motor deficit would include elbow extension, wrist extension, extension of metacarpophalangeal joints, extension of interphalangeal joints, elbow flexion (brachioradialis), and supination. Answer C:A patient with axillary neuropathy would present with deltoid weakness and sensory loss in the shoulder region. Answer D:A patient with ulnar neuropathy would have impaired sensation on the anteromedial aspect of the hand as well as the motor innervation of intrinsic hand muscles. Bottom Line:This patient presents with signs and symptoms related to radial neuropathy. Radial neuropathy was diagnosed by weakness in wrist extension ("wrist drop") and sensory deficit from the distribution of the superficial radial nerve. Elbow extension was normal because the motor innervation to the triceps muscle branched from the radial nerve proximal to the fracture.

A physical therapist performs a lateral flexion alar ligament stress test to rule in ligamentous damage in a patient status post motor vehicle accident (MVA) with reported neck pain. When palpating C2, the physical therapist laterally flexes the head right and at 2 degrees feels the spinous process move left. The physical therapist then laterally flexes the head left and at 4 degrees feels the spinous process move right. What should the physical therapist do NEXT? A. Continue the cervical exam as these findings are normal B. Refer the patient out due to alar ligament instability C. Provide a hard collar due to partial ligamentous damage and refer out D. Provide a soft collar and finish examining the rest of the body except the C spine

Correct answer: A This test is about feeling the C2 spinous process immediately move into the therapist's fingers to make sure the ligament is intact. The alar ligaments attach the skull to C2. They start on the odontoid of C2 and end on either the right or left occipital condyles. As the head sidebends left, the right occipital condyle will be moving superiorly, and if the alar ligament is intact and following along, the distal end must then also move. In other words, sidebending to the left tightens the right alar ligament. If the ligament is intact, when the head is laterally flexed to one side, the therapist should feel the spinous process or transverse process immediately move to the other side. If not, laxity is suspected. The PT felt the spinous process move within the first 5 degrees of motion on each side, which is immediate. Since the ligament stress testing is negative and there are no other signs and symptoms of red flags noted it is safe to continue the cervical examination with no collar needed and no concerns of laxity.Continuing the cervical exam is the correct answer since the alar ligament stress test was negative and there are no other red flag signs and symptoms noted. Sidebending to the left tightens the right alar ligament. If the ligament is intact, when the head is laterally flexed to one side, the therapist should feel the spinous process or transverse process immediately move to the other side. If not, laxity is suspected. The PT felt the spinous process move within the first 5 degrees of motion on each side, which is immediate. Answer B:The alar ligament stress test was negative so there is no need to refer the patient out due to alar ligament instability. Answer C:The alar ligament stress test was negative, so there is no need to provide a hard collar. The exam of the cervical spine can continue. Answer D:The alar ligament stress test was negative, so there is no need to provide a soft collar. The exam of the cervical spine can continue. Bottom Line:Sidebending to the left tightens the right alar ligament. If the ligament is intact, when the head is laterally flexed to one side, the therapist should feel the spinous process or transverse process immediately move to the other side. If not, laxity is suspected

A patient with a mild brain injury is seen in the acute care hospital. The patient scores a 14 on the Glasgow Coma Scale (GCS). Based on these findings, the patient is MOST likely to have demonstrated which of the following reactions during testing? a) localized motor response to pain b) eye opening to pain only c) inappropriate words d) extensor motor response to pain

Correct answer: A To be categorized as a mild brain injury, the patient has to score 13 or more on the scale (out of a maximum of 15). To achieve this, they have to be in the higher categories of the scale, and localized motor response is the highest scoring category of the answers given. The others all score only 2 or 3 in their category, which would not allow the patient to score a 14 as noted in the question stem. See the video in the references for a description of how the motor response in the GCS is determined. Answer B:The "eye opening to pain only" response scores only a 2 and is the second lowest in the category. Scoring a 2 in this category would be too low for the patient to score high enough to be a mild injury with a GCS of 14. Answer C:The "inappropriate words" response scores only a 3 in the verbal category and is therefore too low for the patient to score high enough to be a mild injury with a GCS of 14. Answer D:Extensor motor response to pain is a 2 in the motor category. A score of 2 would not allow the patient to score high enough to get a mild rating. Bottom Line:The Glasgow Coma Scale is scored such that a higher score is best, with 15 as a maximum score. To be categorized as mild brain injury, the score has to be 13-15. Therefore, to achieve a mild rating, the patient must score well in each of the 3 categories.

A physical therapist is conducting an evaluation on a patient who is 1 day post cerebrovascular accident. The therapist is moving the patient's joints through a relatively small range of motion and asking the patient to identify the movement as it is happening. Which of the following sensory concepts is the therapist testing? A. Kinesthesia awareness B. Stereognosis perception C. Touch awareness D. Tactile localization

Correct answer: A To test kinesthesia awareness, the therapist moves the patient's joint in small movements and asks the patient to identify the movement, using pre-agreed terminology, during the movement. Kinesthesia is the awareness of movement and is a critical test for individuals post CVA to identify the sensory areas that are damaged. Answer B:Stereognosis perception is the ability to recognize familiar objects through touch. This would involve the patient picking up an item such as a paperclip, and without looking at it, be able to identify the item. Answer C:Touch awareness is determining if the patient can feel tactile touch in specific areas of the body. It is tested through the application of a cotton swab or ball or tissue brushed lightly on the patient, and the patient reporting if they have been touched. Answer D:Tactile localization tests whether a patient can localize touch on the skin, that is, identifying where on their body they are being touched by the therapist. It does not involve movement. Bottom Line:All sensory function should be tested in a patient post CVA, with different tests for each function. The test described in the question involves movement and is, therefore, testing kinesthesia awareness. None of the other answers listing sensory functions involve movement, although they are all critical within a neurological examination.

A patient is unable to flex the distal interphalangeal joints (DIP) of digits 4 and 5 of their right hand following a fall onto their right arm. A completeneuropathy is diagnosed. Cervical spine range of motion was cleared. What nerve is injured, and where is the site of the injury based on these findings? a) ulnar nerve at the elbow b) median nerve at the wrist c) ulnar nerve at the wrist d) radial nerve at the axilla

Correct answer: A Ulnar nerve at the elbow is the correct answer as the ulnar nerve innervates the flexor digitorum profundus (FDP), which flexes the DIPs of digits 4 and 5. The FDP is in the forearm, therefore the injury has to be proximal to the muscle belly. Answer B:This answer is incorrect as the median nerve does not innervate the FDP. Answer C:This answer is incorrect as the FDP is in the forearm, therefore the injury has to be proximal to the muscle belly. Answer D:This answer is incorrect as the radial nerve does not innervate the FDP. Bottom Line:The FDP has a dual innervation by the median nerve and ulnar nerve. If the ulnar nerve is injured, the patient will be unable to flex the DIPs of digits 4 and 5 of the hand. When nerve injuries occur, they will affect the muscles distal to the injury. Since the FDP muscle belly is in the forearm, the injury would have to be at the elbow or higher.

A patient with low back pain demonstrates a lordotic posture with an anterior pelvic tilt. Which of the following stretches would be MOST appropriate to help improve their posture? A. Kneeling lunge with pelvis held in neutral position, repeated on both sides B. Lying prone over a therapy ball C. Prone press up onto extended arms D. Quadruped, sitting back onto heels with hands stretched forward

Correct answer: A Understanding the anatomy of the hip flexors and how to best position the body to stretch them is a key characteristic of this question. A shortened iliopsoas and rectus femoris cause the pelvis to tilt anteriorly and the spine to become more lordotic in upright standing. To correct this posture the hip flexors must be stretched in the direction of hip extension. In this case, the patient would perform a kneeling lunge with pelvis held in neutral position, repeated on both sides. Answer B:A posture of lordosis with anterior pelvic tilt indicates shortening of the hip flexors. Lying prone over a ball would reduce the lordosis temporarily. However, it does not address shortening of the hip flexors. Answer C:A prone press up would increase the amount of lordosis. Answer D:A posture of lordosis with anterior pelvic tilt indicates shortening of the hip flexors. Sitting back onto the heels would reduce the lordosis temporarily. However, it does not address shortening of the hip flexors because the hips are flexed during this stretch. Bottom Line:A shortened iliopsoas and rectus femoris cause the pelvis to tilt anteriorly and the spine to become more lordotic in upright standing. To correct this posture the hip flexors must be stretched in the direction of hip extension.

An accountant reports neck pain with symptoms radiating into his right arm. During a postural examination, the physical therapist observes a forward head posture that worsens when the patient is sitting. In what position is the patient's neck during this posture? a) upper cervical extension with lower cervical flexion b) upper and lower cervical flexion c) upper and lower cervical extension d) upper cervical flexion with lower cervical extension

Correct answer: A Upper cervical extension with lower cervical flexion is the correct answer for forward head posture in which the lower cervical spine is flexed, moving the head in front of the body, and then the upper cervical spine moves into extension to keep the eyes level with the horizon. Answer B:This answer is incorrect and describes a reversal of the lordosis. Answer C:The upper cervical spine is in extension, but the lower cervical spine is flexed with forward head posture. Answer D:This is the reverse of what forward head posture would look like and as long as it is not excessive could describe a normal cervical posture.

Which of the following tests is the BEST for supporting a diagnosis of axonotmesis? A. Electromyography B. Monofilament testing C. Hand grip strength D. MRI with contrast

Correct answer: A Used in conjunction with other tests and measures, especially nerve conduction studies,electromyography(EMG) can be used to help confirm the diagnosis of an axonotmesis, which is a nerve injury where there is disruption in axon continuity. Answer B:Monofilament testing involves the use of thin fibers to test for the integrity of sensory nerves. It is often used to test for protective sensation in the fingers, toes, and the bottom of the feet. It does not provide useful information for nerve injuries such as axonotmesis, but it is nonetheless an important screening tool. Answer C:Hand grip strength can be tested with a variety of hand-held dynamometers. It is an important part of the physical examination of suspected muscle or nerve disease. However, it does not confirm the diagnosis of axonotmesis, which has to be done via nerve conduction studies as well as electromyography. Therefore, it is not the best answer in this case and is incorrect. Answer D:MRI with contrast cannot provide meaningful information on the integrity of nerve fibers or the changes in the muscle fibers seen in this condition. Bottom Line:Axonotmesis is a nerve injury in which there is a partial or complete disruption in axon continuity. Clinical electrophysiologic testing provides valuable information on the functional integrity of nerve and muscle. In the case of axonotmesis, changes in the muscle fiber occur in about 14 to 21 days after nerve injury.Electromyography(EMG) can help determine the extent and nature of muscle fiber denervation.

A physical therapist is listening to the lung sounds of a patient who has been diagnosed with an obstructive disorder. Which of the following adventitious lung sounds would the therapist MOST expect to hear on this patient, during exhalation? a) wheezes b) crackles c) tubular sounds d) vesicular

Correct answer: A Wheezes are continuous adventitious lung sounds most frequently heard during exhalation and are associated with airway obstructions. Wheezes on inspiration are less common and indicate a severe obstruction. Answer B:Crackles are discontinuous adventitious lung sounds that are more commonly heard during inspiration and are associated with restrictive or obstructive disorders. Answer C:Tubular sound is the term used for bronchial breath sounds that are normal sounds, not adventitious, heard over areas of the tracheobronchial tree. They are loud high-pitched sound with almost equal inspiratory and expiratory duration. Answer D:Vesicular sounds are considered normal, not adventitious, and are heard during inspiration and the first third of exhalation only. Bottom Line:Different lung sounds can be heard in patients both with and without lung pathology. Adventitious lung sounds are abnormal and include crackles, wheezes, and pleural rub. Wheezes are mostly heard during exhalation, while crackles are more common during inhalation. The other two sounds in the answer, tubular and vesicular, are both normal breath sounds, not adventitious ones.

Which of the following injuries would result in wrist drop? a) humeral shaft fracture b) laceration of the posterior hand c) medial epicondyle fracture d) radial head fracture

Correct answer: A When a nerve is injured, the resultant motor and/or sensory dysfunction will occur distal to the level of the injury. The radial nerve is commonly injured during humeral shaft fractures because the nerve is tightly wrapped around the humerus in the radial groove. If the humerus is fractured, the radial nerve can also be injured during the trauma.When the radial nerve is injured, the wrist extensors are paralyzed and there is unopposed wrist flexion, leading to wrist drop. Of the options provided, humeral shaft fracture is the most proximal level of injury to the radial nerve and would result in denervation of the most radially innervated muscles.The triceps may be unaffected, depending on the level of the injury. Answer B:A laceration at this distal level would not affect the motor function of the wrist extensors, which are innervated more proximally than at the level of the hand. The sensory branch of the radial nerve provides sensation to the lateral posterior surface of the hand. Answer C:Wrist drop occurs due to an injury of the radial nerve paralyzing the wrist extensors. The radial nerve does not course by the medial epicondyle of the humerus and would not be affected by a fracture at that location. Answer D:The extensor carpi radialis longus is innervated by the radial nerve before the level of the radial head and would still function after radial head fracture, even though all other muscles in the posterior forearm would be affected.

An individual is being seen for physical therapy following a grade 2 sprain to their anterior cruciate ligament (ACL) and partial tear to the medial meniscus 5 days ago. They are currently weight-bearing as tolerated (WBAT) with antalgic gait, but minimal discomfort. The patient has clear quadriceps atrophy with mild to moderate swelling at the joint lines. Which of the following exercises would BEST target this individual's impairments? a) mini-squat at the edge of the plinth b) full squat at the edge of the plinth c) straight leg raise d) long arc quad at the edge of the plinth

Correct answer: A When the ACL is damaged and in the acute phase, open kinetic chain exercises at end range of knee extension provide an anterior shear force to the tibia and are contraindicated. Closed chain exercises also can strengthen the knee and avoid the anterior shear. For meniscal damage, closed chain weight-bearing exercises in a great deal of knee flexion are contraindicated as the meniscus is at risk after 60 degrees of knee flexion.A closed chain quadriceps activity does not stress the ACL. In addition, doing the activity toward end range does not stress the meniscus. The weak quadriceps can be strengthened safely in this range, and the mini-squat at the edge of the plinth is an appropriate and functional exercise for 5 days post injury. Answer B:A full squat flexes the knee greatly and would stress the meniscus, which becomes engaged after 60 degrees of knee flexion. Answer C:A straight leg raise may be difficult to perform while maintaining appropriate terminal knee extension stressing the ACL as the quadriceps pull on their attachment on the tibia. This activity is not functional, and therefore this is not the best choice. Answer D:This open chain end range quadriceps activity would strain the ACL and apply an anterior shear to the tibia. Bottom Line:When the ACL is damaged and in the acute phase, closed chain exercises strengthen the knee and avoid anterior shear.

A physical therapist is evaluating the gait of a patient with recurrent ankle sprains on the right lower extremity. The therapist notices excessive supination on the right during the push-off phase. The therapist then decides to assess the foot in subtalar neutral for any abnormalities. Which of the following foot deformities would be MOST likely to contribute to the gait deviation? A. Forefoot valgus B. Forefoot varus C. Rearfoot valgus D. Rearfoot varus

Correct answer: A With forefoot valgus, as compared to a normally aligned foot, the medial side of the forefoot is more inferior or closer to the floor, and the lateral side is more superior or further from the floor. Therefore, in order to get the lateral side of the forefoot down, the foot needs to supinate. The supination is seen especially during push off because the deformity is in the forefoot, and during the end of gait the heel is off the floor, and the patient is weight bearing only on the metatarsal heads. Answer B:With forefoot varus, as compared to a normally aligned foot, the lateral side of the forefoot is more inferior or closer to the floor and the medial side is more superior or further from the floor. Therefore in order to get the medial side of the forefoot down the foot would pronate more than normal. Answer C:Rearfoot valgus is rarely seen in the subtalar neutral position. However, even if it was present, the gait deviation in this scenario is observed when the foot is in push off, when the rearfoot is no longer on the ground. Therefore the source of the problem must be in the forefoot. Answer D:Rearfoot varus would place the heel in an inverted position in subtalar neutral and the deviation to compensate would be to evert the calcaneus upon weight bearing to get the medial side of the calcaneus on the ground. That would increase pronation. In addition, the gait deviation was observed when the foot was in push off. At that phase the rearfoot is no longer on the ground therefore the source of the problem must be in the forefoot. Bottom Line:Forefoot valgus causes a valgus angulation of the forefoot where the medial side of the forefoot is more inferior or closer to the floor than the lateral side. The compensation to get the lateral side down is to supinate. It is most noticeable when the calcaneous is off the floor and weight-bearing is on the forefoot.

A patient with a C4 complete spinal cord injury has been treated in an inpatient rehabilitation facility for 2 months and is ready to be discharged. The patient's spouse is supportive and eager for the patient to be discharged home. Which of the following discharge goals is the MOST appropriate for this patient? a) Patient's wife will be independent in assisting with coughing by performing manual pressure to the diaphragm b) Patient will be independent in the use of a power wheelchair adapted with joystick control c) Patient will be independent in feeding while donning strap on hand with utensil attached d) Patient's wife will be independent in monitoring mechanical ventilator settings

Correct answer: A At the C4 spinal cord injury level, the diaphragm is still innervated, and the patient can breathe independently. However, ability to cough and clear secretions is severely compromised. The patient will be dependent on someone else to support clearance of any fluids or phlegm. Having the spouse be independent in the cough assist will prevent respiratory complications, which are common in patients with spinal cord injury. Working toward independence of the spouse in caring for this complex patient is an important priority and is the best goal from the options presented. Answer B:At the level of C4, most of the upper extremity muscles do not function. The patient would not be unable to control a joystick with the wrist and hand. A head or shoulder controlled drive, or a sip-and-puff wheelchair would be more appropriate. Answer C:Although the utensil is strapped to the hand, the patient lacks the ability to utilize the upper extremity musculature, such as the biceps, to bring the hand to the mouth. This goal as written is not achievable. Answer D:The diaphragm is innervated in a patient with a C4 spinal cord injury, and so the patient does not need a mechanical ventilator. The patient may have been on a ventilator during the acute stage, but it is now 2 months later, and the patient would now be weaned off. Bottom Line:A patient with a complete C4 injury is able to breathe independently (because the diaphragm is innervated) and is able to use a power wheelchair with head and neck controls. The upper extremity and lower extremity muscles do not function except for some trapezius activity, and the patient will be dependent for most ADLs. Patients will need assistance clearing airway secretions.

A factory worker jammed his finger about 2 months ago at work. He iced the swelling and ignored it, but now he is coming to physical therapy because he noticed his finger does not work as well as normal. While examining the patient, the physical therapist notices that the third finger rests in flexion at the proximal interphalangeal (PIP) joint, and extension at the distal interphalangeal (DIP) joint. What structure should the PT suspect is MOST likely injured? a) central slip of extensor tendon b) flexor digitorum profundus c) flexor digitorum superficialis d) terminal slip of extensor tendon

Correct answer: A The extensor mechanism in the fingers has a complicated method for allowing us to extend each joint of the fingers. One portion inserts into the proximal phalanx called the central tendon, and then two lateral bands branch off from there and head distally to rejoin together as the terminal tendon and insert into the distal phalanx.If the central tendon is torn from an injury or longstanding arthritis, the lateral bands drift to the palmar side of the finger, as do the lumbricals, which attach to those lateral bands and have more of a flexion line of pull at the PIP joint but still can extend the DIP joint because that attachment is intact. Thus, after a period of time, the finger assumes a posture of PIP flexion and DIP extension, called a Boutonniere deformity. Answers B & C:An injury to the flexor muscles would not cause the PIP joint to rest in a flexed posture and DIP to be hyperextended. Answer D:The terminal tendon of the extensor mechanism inserts into the distal phalanx. If torn, it could no longer extend the DIP joint, and it would rest in flexion, called a mallet finger. In this example, the DIP is in extension, indicating that the terminal tendon is intact.

A physical therapist examines a patient's subtalar position at heel strike/initial contact on video and measures 3 degrees of calcaneal varus. How should the therapist interpret this finding? a) normal b) excessive calcaneal varus c) excessive calcaneal valgus d) insufficient calcaneal varus

Correct answer: A There is a lot of variability regarding non-weight-bearing versus weight-bearing subtalar motion.It is accepted that during initial contact the calcaneus is in 0-4 degrees of varus. Once weight bearing begins during transition from heel strike/initial contact through loading response, the calcaneus then starts to evert from that initially varus position at the subtalar joint to allow the foot to pronate and get the medial side down. Answer B:A calcaneal position of greater than 4 degrees of varus would be excessive and may be due to lateral ligamentous laxity or a bony malalignment. Answer C:Hitting at heel strike in a position of varus is not considered to be excessive calcaneal valgus. Answer D: 0-4 degrees of varus is considered the normal position for the calcaneus during heel strike/initial contact.

A patient is being seen in an outpatient physical therapy clinic with limited knee flexion PROM 8 weeks status post anterior cruciate ligament (ACL) reconstruction. Which of the following techniques are the BEST ones for the physical therapist to apply to improve this patient's limitation? a) posterior tibial glide, inferior patellar glide, quadriceps stretch b) posterior tibial glide, superior patellar glide, hamstring stretch c) anterior tibial glide, inferior patellar glide, quadriceps stretch d) anterior tibial glide, superior patellar glide, hamstring stretch

Correct answer: A This is the correct answer as the appropriate arthrokinematic glides to improve knee flexion include posterior tibial glide and inferior patellar glide. In addition, the quadriceps cross the anterior knee, and if they are tight they may limit knee flexion and should be stretched/lengthened. Answer B:The posterior glide does increase knee flexion, but the superior patellar glide and hamstring stretch increase extension. Answer C:An anterior tibial glide increases knee extension. The patellar glide and stretch listed here increase flexion. Answer D:This is incorrect as these techniques help improve end range knee extension, and not knee flexion.

A 77-year-old patient is admitted to the hospital with a diagnosis of COPD exacerbation. He smokes and was diagnosed with emphysema 10 years ago. He reports a lot of sputum production in the AM, and a chronic cough. His main complaint is being limited with his activities due to shortness of breath. What is the BEST physical therapy intervention for this patient? A. Smoking cessation education and controlled breathing techniques B. Secretion clearance techniques and postural drainage C. Teach him energy conservation techniques D. Instruction in the use of recovery from shortness of breath positions

Correct answer: A Emphysema is defined as a condition of the lung characterized by destruction of alveolar walls and enlargement of the air spaces distal to the terminal bronchioles. Cigarette smoking is a major cause of emphysema. Clinical manifestations include use of accessory muscles to breathe, and learning forward to breathe.Smoking cessation is the single most effective intervention in most people to reduce the risk of developing COPD and to stop its progression. Instruction in controlled breathing techniques will allow the patient to maximize the amount of oxygen he takes in and the amount of CO2he expels in order to reduce his shortness of breath. Answer B:Secretion clearance techniques and drainage would be more appropriate for a patient with a chronic pulmonary condition where they produce a lot of mucus throughout the day and night, such as cystic fibrosis or a chronic severe case of bronchitis. This patient will reduce his sputum production by stopping smoking. He also only has mucus in the morning, so this is not the best answer. Answer C:Energy conservation techniques help conserve energy but do not directly improve breathing techniques. They also do not address the issue of his continued smoking. This is not the best answer. Answer D:Instruction in the use of recovery from shortness of breath positions would be appropriate for this patient; however, it is not the best answer as smoking cessation education combined with breathing technique instruction is a better answer.

Which of the following MOST accurately describes the purpose of a Swan-Ganz catheter? a) gold standard method for measuring systolic BP b) hemodynamic monitoring of the heart c) measurement of mean arterial pressure d) obtaining arterial blood gas measurements

Correct answer: B A Swan-Ganz catheter, or pulmonary artery catheter, is an invasive but important monitoring device that allows for hemodynamic monitoring of the heart. The catheter is inserted via central venous access (typically internal jugular vein or subclavian vein) and then into the vena cava, right atrium, right ventricle, and finally the pulmonary artery. Various hemodynamic parameters (such as pulmonary arterial pressure, cardiac output, and many others) can be directly measured or indirectly calculated. Answer A:A Swan-Ganz catheter directly measures pressure in the pulmonary circulation. Therefore, it does not measure systolic blood pressure. Rather, blood pressure is measured peripherally or with an arterial line that is inserted into an artery. Answer C:A Swan-Ganz catheter does not measure systolic blood pressure. The mean arterial pressure is best measured by an arterial line. Alternatively, a less invasive method would be calculation from the systolic and diastolic blood pressures. Answer D:A Swan-Ganz catheter cannot provide arterial blood gas measurements because it is situated in the right side of the heart, where blood is venous. Arterial blood gas measurements are obtained from blood draws with needle sticks in arteries. Alternatively, arterial blood gas can be obtained from an arterial line. Bottom Line:A Swan-Ganz catheter is a form of invasive monitoring. It is not uncommon in critical care settings, where its use may be indicated for close hemodynamic monitoring of the heart. The presence of this type of catheter is not an absolute contraindication to mobility. However, due to the delicate positioning and significant risks of dislodgement, caution should be exercised when mobilizing patients with this kind of line.

A physical therapist is designing a study to determine whether individuals participating in inpatient rehabilitation therapy prefer music playing or no background noise. The therapist asks every patient who is in the gym on Monday afternoon. WHICH type of sampling methodology is this therapist using? A. Systematic B. Convenience C. Cluster D. Random

Correct answer: B A convenience sample is used when individuals are included in the study because they are the first people available to the researcher. Similar to using a cohort of students for an educational study, using all the patients in the gym at one facility at one time would be considered a convenience sample. Answer A:Systematic sampling uses a sample chosen based on a numerical system. For example, if the therapist chose to ask every fourth person who came into the gym that afternoon, that would be systematic sampling. Answer C:Cluster sampling uses a group of participants, often regional but in this case people working in a certain area within the gym, or more broadly all the patients in the west wing of the facility. Answer D:Random sampling is genuinely random, and everyone has an equal chance of participating. In this case, the therapist would have had to place the names of everyone in the facility participating with therapy into a drawing. Everyone would, therefore, have an equal chance of being included, regardless of whether their therapy was on Monday or in the morning compared to when the sample was chosen. Bottom Line:Different types of sampling are described in research studies, and each has advantages and disadvantages. The greatest rigor in any investigation is pure random sampling, but this is often very challenging to achieve. Convenience sampling is commonly used as groups of subjects are readily available in specific locations. Understanding the type of sampling used can help determine how appropriate the results are for a particular patient population

A physical therapist is treating a patient with a C2 ASIA A spinal cord injury who is in an inpatient rehabilitation facility and is using mechanical ventilation with 40% supplemental oxygen via the tracheal collar. The patient is positioned supine in bed, and they have their compression stockings and abdominal binder on. Blood pressure is 90/60, heart rate is 90, and the pulse oxygen is 95%. While completing a dependent roll with the assistance of a certified nursing assistant to place a sling for an overhead lift transfer, the high pressure alarm on the ventilator goes off, and the patient's pulse oxygen drops to 89%. What is the BEST response that the physical therapist should make? A. Check to see if the ventilator tubing has been disconnected B. Initiate suctioning of the patient C. Increase the rate of supplemental oxygen D. Manually ventilate with a resuscitation bag

Correct answer: B A high pressure alarm is used to indicate that the pressure of the ventilator circuit has increased. Typically, it is caused by secretion buildup, which can be managed by suctioning the patient. Answer A:A low pressure alarm is used to indicate that the pressure of the ventilator circuit has decreased. Typically, it is caused by the ventilator being disconnected due to the patient being moved during functional activities. If the ventilator tube is disconnected, the physical therapist will simply reconnect it to the patient. Answer C:Increasing the supplemental oxygen for the patient would not be effective since the high pressure alarm indicates that the ventilator circuit is unable to get air into the lungs due to a blockage. Answer D:The decision to manually ventilate the patient with a resuscitation bag would occur after the patient was suctioned and sufficient oxygen was still not present . At that time, manual ventilation should be administered to maintain adequate oxygen levels while help was being called to determine the cause of the high pressure alarm. Bottom Line:A high pressure alarm is used to indicate that the pressure of the ventilator circuit has increased. Typically, it is caused by secretion buildup, which can be managed by suctioning the patient.

A patient has a limitation in cervical sidebending to the left. In assessment of joint mobility in the cervical spine to see if there is a deficit at C5/C6, which of the following is MOST likely causing the limitation and should be assessed by the physical therapist? A. Decreased superior glide of C5 on C6 at left facet B. Decreased superior glide of C5 on C6 at right facet C. Decreased inferior glide of C6 on C5 at left facet D. Decreased inferior glide of C5 on C6 at right facet

Correct answer: B A limitation in sidebending that has a mechanical origin is most likely caused by a facet that is not opening on the contralateral side. In order for sidebending to occur to the left, the facet on the right needs to open by having C5 glide superiorly on C6 to open that facet. Answer A:The right side needs to open for a left side bend. Answer C:As the head sidebends, when looking at C5/C6, C5 does glide inferiorly on C6 on the left. This answer says C6 is gliding inferiorly on C5, which is incorrect. In addition, a locked facet that does not open on the contralateral side is most likely the reason for the limitation. Answer D:This would occur during a right sidebend, not left. Bottom Line:A limitation in sidebending that has a mechanical origin is most likely caused by a facet that is not opening on the contralateral side. The facet on the contralateral side needs to open by having the more superior vertebrae glide superiorly on the more inferior vertebrae.

A patient has a limitation in posterior glide of her left humeral head. Which of the following activities of daily living (ADL) would be the MOST difficult to perform with the left upper extremity? a) reaching back to grab the seat belt when preparing to drive her car b) fastening her bra in the back c) reaching forward to grab a doorknob d) brushing the back of her hair

Correct answer: B A limitation in the posterior glide of the humeral head would affect shoulder flexion and shoulder internal rotation. In this question, the motion of fastening a bra in the back would require the full range of glenohumeral internal rotation and therefore would be the most difficult to perform if posterior glide of the humerus were limited. Answer A:Reaching back with the left arm to grab the seatbelt from the driver's seat requires shoulder external rotation, and that motion is not limited by a limitation in posterior glide. With external rotation, the humeral head glides in the anterior direction. Answer C:Reaching forward does involve shoulder flexion, and that motion will be limited if there is reduced posterior glide of the humeral head. However, since the target is the doorknob, the range of shoulder flexion required is used only for the early part of the range, so that motion would not be as difficult to perform. Answer D:Brushing the back of the hair requires a good deal of external rotation and about 90 degrees of abduction, which would not be limited with a limitation in posterior glide of the humeral head. Bottom Line:The humeral head glides posteriorly as a component of shoulder internal rotation and shoulder flexion, such as when fastening a bra in the back.

A patient has deficits in knee flexion due to capsular adhesions. What mobilization is the MOST specific technique to improve the patient's range of motion? a) anterior tibial glide b) posterior tibial glide c) medial tibial glide d) lateral tibial glide

Correct answer: B A limited posterior tibial glide is associated with deficits in knee flexion. Grade III/IV joint mobilizations are indicated to improve the patient's capsular mobility and range of motion. Answer A:An anterior tibial glide would improve knee extension range of motion. Answer C:While a medial tibial glide may improve knee flexion and extension, it is not as specific as the posterior tibial glide. Also, the intercondylar eminence may limit this glide's ability to improve capsular mobility. Answer D:While a lateral tibial glide may improve knee flexion and extension, it is not as specific as the posterior tibial glide. Also, the intercondylar eminence may limit this glide's ability to improve capsular mobility. Bottom Line:A thorough understanding of arthrokinematics is necessary for physical therapists to ensure that joint mobilization techniques, such as posterior tibial glide, are properly applied.

A patient fell off a ladder and landed on a bent knee. X-rays were negative for fracture, but the patient reports significant pain along the medial joint line with weight-bearing, clicking in the knee, and an inability to fully straighten the knee. Based on the mechanism of injury and the patient's symptoms, which of the following structures is MOST likely injured? a) anterior cruciate ligament b) medial meniscus c) medial collateral ligament d) posterior cruciate ligament

Correct answer: B A meniscal tear may occur with a weight-bearing force to the knee and some sort of rotation. That can also be the mechanism for other knee injuries, such as an ACL tear, so it is important to do a full examination and combine it with the patient's symptoms. Knee clicking, pain with weight-bearing, and pain along the joint line all point to a meniscal tear. Range of motion at either end range of flexion or extension is also possible because a portion of the tear can move somewhere it should not be and alter the smooth mechanics of the knee.The mechanism of injury added compression to a weight-bearing knee. Combining that with the symptoms of pain along the medial joint line, clicking, and an inability to achieve full extension leads to suspecting a tear of the medial meniscus. There were no symptoms mentioned regarding instability, which would be more common with a ligamentous injury such as ACL, PCL, or MCL. Answer A:If the ACL was torn, the patient would report symptoms of knee instability. In addition, knee clicking is not indicative of an ACL injury. Answer C:The MCL is injured usually by a valgus type of force. In this case, the patient fell onto a flexed knee with no valgus force described. The symptom of medial joint line pain could indicate MCL; however, the pain would usually be along the MCL and not just at the joint line. In addition, knee clicking is not a symptom associated with an MCL tear. It is more likely a meniscal tear. Answer D:The posterior cruciate ligament is typically injured with a hyperextension injury or with a direct posterior blow to the tibia at 90 degrees of knee flexion. Although it may have been injured with a fall onto a flexed knee, the question asked what is suspected based on the symptoms provided, and a torn PCL would not give medial joint line pain or clicking, and would create some instability. Bottom Line:A meniscal tear may occur with a weight-bearing force to the knee and some sort of rotation. Knee clicking, pain with weight-bearing, and pain along the joint line all point to a meniscal tear. Range of motion at either end range of flexion or extension is also possible because a portion of the tear can move somewhere it should not be and alter the smooth mechanics of the knee.

A physical therapist works at a wellness program for people with types 1 and 2 diabetes. Which of the following statements MOST accurately describes the proper course of action if a patient in the wellness program arrives with a serum glucose reading of 60 mg/dL? a) patients with neither type 1 nor type 2 diabetes should be given carbohydrates prior to exercise b) patients with type 1 or type 2 diabetes should be given carbohydrates prior to exercise c) patients with type 1 diabetes should be given carbohydrates, but patients with type 2 diabetes should not be given carbohydrates prior to exercise d) patients with type 2 diabetes should be given carbohydrates, but patients with type 1 diabetes should not be given carbohydrates prior to exercise

Correct answer: B A normal fasting serum glucose level is 70 mg/dL to 100 mg/dL. A patient whose glucose level is 60 mg/dL should ingest some form of carbohydrate so that the blood sugar is above 100 mg/dL prior to exercise. There is no difference in recommendations for blood glucose levels prior to exercise for patients with type 1 and type 2 diabetes. Answer A:Because exercise would further reduce a patient's glucose levels, instructing patients with either type 1 or type 2 diabetes to start their exercise program without ingesting additional carbohydrates would not be an appropriate response to the situation. Answer C:Because exercise would further reduce a patient's glucose levels, instructing patients with type 2 diabetes to start their exercise program without ingesting additional carbohydrates would not be an appropriate response to the situation. Answer D:Because exercise would further reduce a patient's glucose levels, instructing patients with type 1 diabetes to start their exercise program without ingesting additional carbohydrates would not be an appropriate response to the situation.

A physical therapist is treating a patient with a history of Brown-Sequard syndrome due to a gunshot wound at C7 that affected the right side of his spinal cord. Which of the following functions would be spared below the level of injury? a) light touch of the right leg b) pain of the right leg c) proprioception of the right leg d) temperature of the right leg

Correct answer: B A patient with Brown-Sequard syndrome typically presents with damage to one-half of the spinal cord due to penetrating trauma such as a gunshot wound or stabbing. The patient will present clinically with ipsilateral loss of proprioception, vibration, and light touch and weakness. The patient also will have contralateral loss of pain and temperature.The pattern of deficits in Brown-Sequard syndrome corresponds to the anatomy of the spinal cord. The corticospinal tract, which contains motor commands from the cortex, and the dorsal column-medial lemniscus tract, which contains ascending information regarding proprioception, vibrations, and light, cross at the brainstem. The spinothalamic tract, which contains information regarding pain and temperature, crosses immediately at the level where the sensory nerves enter the spinal cord.Therefore, a patient with Brown-Sequard syndrome at C7 from an injury that affected the right side would have intact pain sensation of his right leg because this information would be on the left side of his spinal cord. Answer A:Information regarding light touch of the right leg goes up the right side of the spinal cord and would be affected. Answer C:Information regarding proprioception of the right leg goes up the right side of the spinal cord and would be affected. Answer D:Information regarding temperature of the left leg goes up the right side of the spinal cord and would be affected.

A physical therapist is completing an initial examination for a patient with a traumatic brain injury with damage to his bilateral frontal lobes. What is the MOST appropriate strategy for the physical therapist to modify their initial examination based on the location of injury? a) approach the patient from the right side of space b) decrease the environment stimuli present c) provide written information to compensate for the receptive aphasia d) require simple yes/no answers to questions

Correct answer: B A patient with damage to the frontal lobes would have poor short term memory, decreased executive functions, and decreased attention, which would make providing feedback and following commands difficult during an initial examination. By decreasing the amount of environmental stimuli present, the physical therapist would be more likely to get an accurate assessment of the patient's abilities. Answer A:The strategy of approaching patients from the right side of space is used with patients with spatial neglect to compensate for their right-sided bias. Since patients with bilateral frontal lobe damage are unlikely to have neglect, this strategy is not appropriate in this scenario. Answer C:The strategy of providing only written information is used for patients with receptive aphasia. Since patients with bilateral frontal lobe damage are unlikely to have receptive aphasia and may have concurrent visual deficits, this strategy is not appropriate in this scenario. Answer D:The strategy of requiring only yes/no responses to questions may be useful in this scenario since patients with bilateral frontal lobe damage often have difficulty with verbal expression. However, this strategy would be more appropriate for a patient with a true expressive aphasia, which is more likely to be caused by damage to the left frontal cortex instead of bilateral frontal lobe damage. The better approach would be to simplify questions and instructions, not necessarily to require shorter answers from the patient. Bottom Line:Patients with damage to the frontal lobes often have difficulty with attention, which affects their ability to follow instructions and listen to feedback. By decreasing the amount of environmental stimuli present, the physical therapist would be more likely to get an accurate assessment of the patient's abilities.

A patient has a medical diagnosis of patellofemoral pain syndrome. Upon examination, the therapist notes a positive Ober's test and abnormal patellar tracking. The therapist wants to apply patellar tape to elongate the tight structures and optimize the position and tracking of the patella. Which of the following patella taping techniques is MOST appropriate? a) inferior glide b) medial glide c) lateral glide d) superior glide

Correct answer: B A positive Ober test indicates tightness of the iliotibial band (ITB), which is on the lateral side of the leg. The ITB also has slips that can connect to the patella and add some lateral tension. The poor patellar tracking is most likely in the lateral direction as well. A medial patellar glide counteracts the pull of the more lateral structures and also elongates the lateral structures. Answer A:This is incorrect as this direction of pull contributes to knee flexion and also does not address the lateral pull or lateral tightness. Answer C:This is incorrect as this is contributing to the problem and adding more lateral pull on the patella. Answer D:This is incorrect as this would help to improve knee extension and not counteract the pull of the ITB. Bottom Line:If the ITB is short and tight, it may contribute to a lateral glide or pull of the patella. To improve this, the patella needs to be stretched/glided medially,which can be done in a variety of ways. This example illustrates the use of tape as the best way to emphasize a medial patellar glide.

The following passive range of motion (PROM) measurements were recorded on the right lower extremity of a cross-country runner. Hip flexion, measured in supine with knee flexed; ( 5 - 120 degrees.) Hip extension measured in prone with knee extended; 5 degrees from neutral. All other hip motions were within functional limits (WFL). Knee flexion/extension measured in prone; (0-130 degrees). When performing the Thomas test on this patient, what should the therapist expect to see? A. The thigh will be off the table, and the knee will be flexed to 90 degrees. When the therapist extends the knee, the thigh will lower down to the table. B. The thigh will be off the table, and the knee will be flexed to 90 degrees. When the therapist extends the knee, the thigh will remain where it is. C. The thigh will be off the table, and the knee will be flexed to 130 degrees. When the therapist extends the knee, the thigh will lower down to the table. D. The thigh will be off the table, and the knee will be flexed to 130 degrees. When the therapist extends the knee, the thigh will remain where it is.

Correct answer: B A thigh that is resting above the level of the table during the Thomas test indicates tightness of the iliopsoas, the rectus femoris, or both muscles. If the tightness is due to the rectus femoris, then extending the knee will place that two-joint muscle on slack, and the thigh will lower back down to the table. In this scenario, the patient's rectus femoris length is WFL, based on the fact that the patient had full knee PROM in prone. Therefore, the iliopsoas is the short/ tight muscle and the thigh will remain off the table. Extending the knee will have no effect on the level of the thigh. In addition, the hip PROM indicates a 5-degree flexion contracture. Answer A:A thigh that is resting above the level of the table during the Thomas test indicates tightness of the iliopsoas and/or rectus femoris. If the tightness is in the rectus femoris, then extending the knee would place that two-joint muscle on slack, allowing the thigh to lower back down to the table. In this scenario, the patient's rectus femoris length is WFL, based on the fact that the patient had full knee PROM in prone. Answer C:The patient has a five-degree flexion contracture and tightness of the iliopsoas but not the rectus femoris. In the Thomas test position, the tibia would hang down due to gravity, and the knee would be in approximately 90 degrees of flexion. It would not rest at the end range position of 130 degrees. In addition, the rectus femoris length is WFL, based on the fact that the patient had full knee PROM in prone, so the thigh would stay where it is and be unaffected by putting the rectus femoris on slack. Answer D:The rectus femoris length is WFL, based on the fact that the knee PROM in prone was full, so the thigh would stay where it is and be unaffected by putting the rectus femoris on slack. However, in the Thomas test position, the tibia would hang down due to gravity, and the knee would be in approximately 90 degrees of flexion. It would not rest at the end range position of 130 degrees. Bottom Line:In a lower extremity with normal muscle length, the thigh should rest on the table and the knee should hang down at 90 degrees over the edge of the table during the Thomas test. If there is tightness of any hip flexor, the thigh will remain elevated off the table. To rule out tightness of the rectus femoris, the knee is extended, putting that muscle on slack. If the leg then drops down, the rectus femoris is the source of the restriction. If the leg remains in the air, the iliopsoas is the most likely source.

A patient who is 1 month status post left transtibial amputation fell and fractured the right femur. The patient is now non-weight bearing (NWB) on the right lower extremity and has not yet been fitted for a prosthesis on the left lower extremity. The physician in the acute care setting would like the patient to sit up in the chair to increase out-of-bed tolerance. The patient requires maximum assist to sit at the edge of bed. Static sitting balance is poor. What is the MOST appropriate way to transfer the patient to the chair? a) dependent lift by two therapists b) hoyer lift c) slide board transfer d) transfer to chair is unsafe and contraindicated for now

Correct answer: B A variety of factors in this scenario indicate that a Hoyer lift is the safest and most effective way to transfer this patient from bed to chair. The patient requires maximum assistance to sit edge of bed, meaning he is unable to assist with any part of the activity. NWB status on bilateral lower extremities (LEs) indicates the patient is unable to stand or pivot in any way. Poor trunk control rules out the use of a slide board. Answer A:A dependent lift by two therapists is not the safest way to transfer as it puts the patient at risk for falls and puts the therapists at risk for injury. Answer C:Due to lack of trunk control by the patient, slide board is not appropriate at this time but could be a future goal. Answer D:Nothing in the scenario indicates a contraindication to the patient sitting up in a chair.

Which of the following BEST describes the physical presentation of a patient found to be in respiratory acidosis? a) confusion and hyperventilation b) altered mental status, hypoventilation, and lethargy c) dizziness and hyperventilation d) Kussmaul breathing

Correct answer: B Acidosis and alkalosis are conditions that are attributed to acid-base imbalances in the body. Acidosis refers to an excess of acid, whereas alkalosis refers to an excess of base. Either can be caused by respiratory or metabolic mechanisms. In the case of respiratory acidosis, the acid-base imbalance is caused by some abnormality in the airway and ventilation that leads to an increased concentration of carbon dioxide (CO2). The underlying cause is typically hypoventilation due to a depressed respiratory drive or lung disease. Hypoventilation is a primary cause of respiratory acidosis. With hypoventilation, decreased clearance of carbon dioxide (CO2) leads to an excess concentration of hydrogen ions in the blood and in turn, lowers the pH. As a result of acidosis, it is common to see lethargy and confusion due to the effects on the central nervous system. Answer A:Although confusion can be present in someone presenting with respiratory acidosis,hyperventilationis not an expected presentation. Rather,hypoventilationwas likely the cause of the respiratory acidosis. Therefore confusion combined with hyperventilation are not expected in respiratory acidosis. Answer C:Dizziness and hyperventilation together are more consistent with arespiratory alkalosis. Increased clearance of carbon dioxide (CO2) leads to a decreased concentration of hydrogen ions in the blood, raising the pH and causing alkalosis. Answer D:Kussmaul breathing specifically refers to deep and rapid respirations that are used as a compensation formetabolic acidosis. In this case, the acid-base imbalance comes from a metabolic cause and the respiratory system is being used to help correct it. It is not the same as respiratory acidosis,where the cause is hypoventilation. Bottom Line:Hypoventilation is a primary cause of respiratory acidosis. With hypoventilation, decreased clearance of carbon dioxide (CO2) leads to an excess concentration of hydrogen ions in the blood and in turn, lowers the pH. As a result of acidosis, it is common to see lethargy and confusion due to the effects on the central nervous system. TrueLearn Insight :An understanding of the effects of hyperventilation versus hypoventilation can help clinicians distinguish between disorders of acid-base imbalances.

A patient fell on his right shoulder when skiing and went to the local PT clinic 2 days later for an evaluation. The patient reported no pain at rest but pain with certain motions. The physical therapist had the patient raise the arm actively in abduction. The patient demonstrated full active range of motion (AROM), with pain anteriorly and superiorly from 85-105 degrees of abduction. With this limited information, what should the PT suspect as the MOST likely explanation for the pain? A. Acromioclavicular joint sprain B. Acute subacromial impingement C. Rotator cuff rupture D. Labral tear

Correct answer: B Acute subacromial impingement is the most likely explanation since there was a traumatic mechanism of injury that can compress the subacromial space. The full active range of motion with a positive painful arc sign where the subacromial space is most susceptible to compression pain is the MOST likely explanation at this point in the examination for a patient with this acute injury. Answer A:The mechanism of injury of falling onto the shoulder could cause a sprain of the acromioclavicular (AC) joint. However, with an AC sprain, the pain would occur closer to the end range of the motion and not at midrange. Answer C:The mechanism of injury of falling onto the shoulder could cause a rupture of the rotator cuff. However, if ruptured, the patient would be unable to abduct the arm full range. Answer D:The mechanism of injury of falling onto the shoulder could tear the labrum. However, a labral tear does not demonstrate a painful arc. The patient would most likelyreport symptoms such as clicking and catching deep within the shoulder and complain of some instability. Bottom Line:The mechanism of injury of falling onto the shoulder can cause a number of different injuries including subacromial impingement, rotator cuff tear, acromioclavicular joint sprain, and labral tear. With subacromial impingement, the patient will demonstrate a painful arc. With AC joint sprain, the pain would occur at the end of the range of motion.

A 62-year-old female underwent quadruple coronary artery bypass graft (CABG) surgery via a median sternotomy. Two days after surgery, physical therapy was consulted to help optimize her mobility. Which of the following describes the MOST appropriate modifications that the physical therapist should make for this patient during the evaluation? a) avoid all shoulder range of motion; maintain slight trunk flexion during all upright activities b) avoid MMT of the upper extremities; no pushing with the upper extremities during sit to stand c) avoid twisting and rotating trunk; log roll only d) evaluation on the first day should be at bed level

Correct answer: B After a median sternotomy, sternal precautions have traditionally been put in place with the belief that avoiding specific arm movements, including using the upper extremities in sit to stand, may help decrease the chances of sternal wound complications. Specific precautions and restrictions vary greatly among hospitals and surgeons, but in general, upper extremity movements are limited at the shoulder. Typically, forceful shoulder motions are avoided as well as range-of-motion testing, which would mean to avoid manual muscle testing of the upper extremities. Answer A:Unless the patient is a high-risk individual or already has signs of sternal wound dehiscence, there is no indication for imposing strict sternal precautions for a patient after a median sternotomy. Therefore, the precautions described here appear to be too strict for a straightforward post-operative case. Answer C:There is no restriction in trunk and spine movements after a median sternotomy. In fact, with sternal precautions after a sternotomy, many patients prefer to log roll and use trunk movements for mobility as compensations for not using the upper extremities. Answer D:Once they are hemodynamically stable, patients after a CABG are encouraged to participate in early mobility as tolerated to avoid the negative sequelae of prolonged bedrest. Therefore, this answer is incorrect since there is no reason to limit activity to bed level. Bottom Line:There is no strong consensus on what defines sternal precautions. Different hospitals and surgeons have varying preferences. In general,forceful upper extremity movements are to be avoidedas well as movement in extreme range of motion. Physical therapists should be aware of these precautions and adapt mobility training so that patients can still achieve optimal functioning given these limitations. TrueLearn Insight :Sternal precautions vary by region, facility, and surgeon preference. There is limited evidence on the effectiveness of sternal precautions in preventing wound complications. Conversely, emerging evidence has supported the benefits of liberalizing sternal precautions. However, this continues to be an area requiring further investigation.

According to the Americans with Disability Act, WHICH of the following questions could be asked of an individual during a job interview? A. How did you sustain your injury? B. Are you able to stand for a prolonged period? C. Are you currently receiving medical or psychiatric care? D. Have you ever had a drug addiction?

Correct answer: B An applicant for a job can be asked questions regarding essential job functions. If the job requires prolonged standing, it is an appropriate question for the applicant to be asked. Answer A:Even if the applicant has an obvious injury, questions about any visible characteristic cannot be asked during any job interview. Answer C:Questions regarding current medical and psychiatric history are not permitted during any job interview. Answer D:Questions regarding prior addiction to drugs are not permitted during any job interview. Bottom Line:Awareness of the regulations within the Americans with Disability Act is critical during a job interview. The interviewer is allowed to ask about essential job functions only, such as the ability to stand for a long period and cannot ask about any obvious physical issues, prior health or psychiatric care, or prior drug addiction. The interviewer can ask about functions performed in a previous job.

An electromyographic (EMG) study is MOST appropriate for which of the following conditions? a) achilles tendinopathy b) muscular dystrophy c) rotator cuff tear d) spinal fusion

Correct answer: B An electromyographic (EMG) study can be used to measure motor nerve function as well as the function and integrity of skeletal muscle fibers. In the case of muscular dystrophy, which is a disease characterized by muscle degeneration, abnormalities coming from muscle fiber damage would be apparent and can contribute to a definitive diagnosis. Answer A:Achilles tendinopathy is an injury to the achilles tendon and is characterized by heel pain and tenderness. Although it can sometimes involve muscle tissue, the primary problem is in the tendon or musculotendinous junction. Changes in the integrity of the muscle fiber or motor units are not likely. Therefore an electromyographic (EMG) study would not provide useful information. Answer C:A rotator cuff tear is an injury in the tendon of one of the four rotator cuff muscles (supraspinatus, infraspinatus, teres minor, and subscapularis). Like all tendon injuries, the primary problem is in the tendon or musculotendinous junction. Neuromuscular changes should not be present in this injury. Therefore an electromyographic (EMG) study would not provide useful information. Answer D:Spinal fusion is an orthopedic surgical technique utilized to stabilize an unstable part of the spine. Therefore an electromyographic (EMG) study would not provide useful information. Bottom Line:An electromyographic (EMG) study can be used to measure motor nerve function as well as the function and integrity of skeletal muscle fibers. It can provide diagnostic information related to neuropathies and myopathies, including muscular dystrophy.

WHY should a patient experiencing an episode of exercise-induced asthma use an inhaler? a) to help get more air into the lungs b) to reduce bronchospasm c) to reduce bronchiectasis d) to provide continuous positive airway pressure

Correct answer: B An inhaler is a quick method of getting medication to the lungs to reduce bronchospasm and encourage bronchodilation, which in turn reduces airway resistance. Answer A:The main issue with asthma is the inability to get airout of the lungs due to bronchospasm. The lungs fill with air and the patient feels they cannot breathe. The inhaler helps get air out. Answer C: Bronchiectasis is a chronic condition in which the walls of the bronchi are thickened. It cannot be reduced with an inhaler. Answer D:Continuous positive airway pressure is provided by a machine for patients with sleep apnea. It provides a stream of pressurized air to prevent sleep apnea by increasing pressure to the throat. Bottom Line:An inhaler contains albuterol or other bronchodilator medications to reduce the airway restrictions that result from bronchospasm, which helps the patient expel more air from the lungs.

A physical therapist is participating in the first care conference of a client who recently sustained a cerebrovascular accident (CVA). The client and their family speak adequate but limited English as a second language. Which of the following educational interventions is the MOST appropriate at this time? a) explain everything to the family giving them as much information as possible b) offer various forms of educational material including books, videos, and brochures c) keep the conference discussion closed and discourage questions to avoid misunderstanding d) using outcome measures, provide the family with specific prognosis to help future planning

Correct answer: B Any discussion and education for individuals after a CVA should provide as much variety of material as possible. Using books, brochures, videos and involving the family as much as possible will assist in family and patient education. Answer A:Education for patients and family after a CVA should be accurate and factual, but should be limited to the amount of information they can assimilate at any one time. Providing everything in the first conference would be too overwhelming for the client and the family. Answer C:All conferences and education should be open and involve extensive communication. If there is a language or literacy barrier, other methods of explaining and communication should be explored. The family must be involved in the decision-making if the education and care are going to be successful. Answer D:Outcome measures can provide some good guidelines for medical personnel concerning prognosis. However, specific predictions should be avoided due to patient differences and the risk of the family developing incorrect expectations. Bottom Line:Education of families and clients after CVA needs to be open, honest, and carefully presented to ensure there are no misunderstandings or inappropriate expectations. Use of various media for education is recommended, but avoiding specific prognosis and being open in the discussion to include the family and client are critical.

A physical therapist is examining a patient with a history of left middle cerebral artery cerebrovascular accident (CVA). The patient can communicate well verbally, can answer all of the physical therapist's questions, and is A & O x 3. When told to wash her hands, she turns the water on, dries her hands, and then applies soap. Her caregiver reports that she makes the same type of errors when at home when she tries to wash her hands without being asked. What is the name of this clinical condition? a) visual agnosia b) apraxia c) agraphia d) receptive aphasia

Correct answer: B Apraxia is a deficit in motor planning that occurs despite intact sensory and motor pathways and the absence of language impairment. It is apparently caused by a deficit in higher-order planning or conceptualization of the motor task. Specifically, this patient has ideational apraxia since she has difficulty performing tasks upon command or spontaneously, which may be caused by a deficit in linking the idea and execution of a motor activity. Answer A:Visual agnosia is the inability to recognize familiar objects despite having normal vision. Visual agnosia may be caused by damage to the association areas of either cerebral hemisphere. Answer C:Agraphia is the loss of the ability to communicate through writing. Answer D:Receptive aphasia is a deficit in language comprehension that is typically found in patients with damage to the left superior temporal lobe. Since verbal comprehension is intact, this patient does not have receptive aphasia. Bottom Line: Apraxia is a deficit in motor planning that occurs despite intact sensory and motor pathways. Ideational apraxia refers to patients who are unable to perform activities upon command or spontaneously.

A physical therapist is treating a patient with a history of multiple sclerosis. The patient exhibits an ataxic gait pattern, hypotonia, and difficulty with maintaining her balance when changing directions. Which of the following intervention strategies would be the LEAST effective with this patient? a) PNF rhythmic stabilization for the trunk b) tone-inhibiting techniques to the lower extremities c) rhythmic auditory stimulation d) weight cuffs on her ankles

Correct answer: B Ataxia is characterized by incoordination of movement in which errors of force, speed, or trajectory occur. There is limited research on the effectiveness of physical therapy interventions for patients with ataxia. However, damage to the cerebellum also causes hypotonia, which is a decrease in muscle tone, so tone-inhibiting techniques would not be effective or necessary in this patient scenario. Answer A:PNF rhythmic stabilization for the trunk is used to increase proprioceptive input and increase co-activation in order to reduce ataxia in patients with cerebellar injuries. Answer C:Rhythmic auditory stimulation is used to focus attention and cue the patient to control the speed of their movements in order to reduce ataxia in patients with cerebellar injuries. Answer D:The use of weight cuffs on the distal lower and upper extremities is used to slow limb movements in order to reduce ataxia in patients with cerebellar injuries. Bottom Line:In this scenario, tone-inhibiting techniques would be the least effective intervention. Ataxia is characterized by incoordination of movement in which errors of force, speed, or trajectory occur. Currently, there is limited research on the effectiveness of physical therapy interventions for patients with ataxia. Physical therapists need to be able to plan their intervention strategies based on lesion location in order to effectively treat patients with neurological injuries.

A patient presents with a diagnosis of L3-L4 moderate-size disc herniation impinging on the nerve root causing lower extremity radicular symptoms. The physician requests that traction be included in the plan of care. What is the LOWEST percentage of body weight that should be used for the initial traction force? a) 15% b) 25% c) 50% d) 75%

Correct answer: B Authors vary in their recommendations regarding the amount of force to be used for traction. However, most agree that the optimal amount of force depends on the patient's clinical presentation, goals of treatment, and position during treatment. For all applications, force should be kept low during initial treatment to gauge patient response.For disc problems, stretch of soft tissue, and decreasing muscle spasms, traction force of 25% of the patient's body weight is used. For joint distraction, 50% is used. Typically traction force to the lumbar spine should not exceed 50% of the patient's body weight.During the initial traction session, force should be kept low to reduce risk of reactive muscle guarding and spasms, and to determine whether traction is likely to aggravate the patient's symptoms. Answer A:Typically at least 25% of the patient's body weight is required to overcome the friction of the body moving horizontally on the table. (Higher forces are required, for example, when lumbar traction is applied without a split table.) Answer C:A force up to 50% of the patient's body weight is used when the goal of the traction is joint distraction. However, this would not be the initial force. The traction should be started closer to 25% of body weight and gradually increased during subsequent sessions as the patient becomes used to the procedure. Answer D:Traction force of the lumbar spine generally should not exceed 50% of the patient's body weight. Bottom Line:For disc problems, stretch of soft tissue, and decreasing muscle spasms, traction force of 25% of the patient's body weight is used. For joint distraction, 50% is used. Typically traction force to the lumbar spine should not exceed 50% of the patient's body weight.

An individual received a significant perturbation from a posterior direction. Which of the following movement strategies is most likely to occur? a) anterior movement of the hips b) forward stepping c) activation of anterior tibialis d) posterior reaching

Correct answer: B Balance strategies occur in response to a displacement of the center of mass from the base of support. The reactions will vary depending on the size of the perturbation. If small, ankle strategies and then hip strategies will occur; larger perturbations produce stepping and reaching reactions, which are done in the direction of the displacement, and not toward the perturbation.A large perturbation normally triggers a stepping reaction, which would be in a forward direction if the perturbation is from behind. This is the only answer that can be correct, as all the others would be reactions related to a perturbation from the front. Answer A:During a moderate perturbation, hip strategies will occur, but a perturbation from behind will push the trunk forward, and the hips will respond with a posterior movement to compensate and maintain the center of mass over the base of support. Answer C:Ankle strategies are the first to occur with a perturbation, but anterior tibialis will respond first when the individual is swaying backward in response to a perturbation from the front. Answer D:Reaching reactions are common during a large perturbation but are done to extend the base of support and stabilize posture. After a large perturbation from a posterior direction, the reaching reaction would be forward, not posterior.

A community-dwelling older adult reports two falls in the past 2 months. The falls happen at night when the patient walks to the bathroom in the dark. Which balance exercise would be the MOST appropriate to target this patient's impairments? A. Eyes open in a tandem stance position B. Eyes closed in a modified tandem position C. Single limb stance on a foam pad D. Double limb stance on a foam pad

Correct answer: B Balance, or equilibrium, is maintained by sensory input from the vestibular, somatosensory (proprioceptive), and visual systems. If an individual has difficulty balancing with the lights off, or with eyes closed, it is because they rely more on the visual system than on the other two systems. Conversely, a person may have an inner ear disorder and be unable to rely on vestibular input and thus rely more on visual and proprioceptive input. Depending on the patient, different balance activities can be implemented in which one of the sensory systems is blocked to train the others. For example, a foam pad would make it harder for the proprioceptive system to give input because a soft surface does not give the same information as a flat firm floor. Covering the head with a dome or asking the patient to move the head around challenges the vestibular system's input.Eyes closed in a modified tandem position is the only option presented where the patient's eyes are closed. Because the falls happen in the dark when the patient cannot rely on their visual system, they need to be able to rely on input from the vestibular and somatosensory (proprioceptive) systems. Practicing balancing with eyes closed in a modified tandem position requires the patient to cue into the other two systems. Answer A:Eyes open in a tandem stance position would address the somatosensory system and not the visual system of balance. Answer C:Single limb stance on a foam pad would address the somatosensory system and not the visual system of balance. Answer D:Double limb stance on a foam pad would address the somatosensory system and not the visual system of balance. Bottom Line:Balance is maintained by sensory input from the vestibular, somatosensory (proprioceptive), and visual systems. Practicing balancing with eyes closed in a modified tandem position requires the individual who relies on visual systems to cue into the other two systems.

A 50-year-old female with hypertension was referred to the physical therapy clinic for a wellness program. The patient reports that she is taking her beta blocker regularly. To be able to develop an appropriate aerobic exercise program, which of the following is BEST to determine the patient's appropriate response during exercise? a) heart rate b) rate of perceived exertion c) blood pressure d) respiratory rate

Correct answer: B Beta blockers are anti-hypertensive medications responsible for lowering the heart rate. Because of this, using a heart rate to monitor exercise response in a patient who is taking beta blockers will not yield reliable results. Using the rate of perceived exertion is a better method of determining exercise tolerance in this case. Answer A:Beta blockers lower the heart rate and therefore will not produce accurate information on an exercise response in a person taking this medication. Answer C:Beta blockers also lower blood pressure and therefore will not provide accurate information on exercise response in a person taking this medication. Answer D:While monitoring the respiratory rate is important to ensure safe exercise levels, it does not directly measure the patient's exercise response. Bottom Line:Beta blockers are antihypertensive medications responsible for lowering the heart rate. Because of this, using heart rate to monitor exercise response in a patient who is taking beta blockers will not yield reliable results. Using the rate of perceived exertion is a better method of determining exercise tolerance in this case. TrueLearn Insight :It is important for the physical therapist to use tools to objectively measure the patient's response to exercise so that the therapist can design and progress appropriate exercise programs.

During an examination, a patient reports he is taking a certain medication. This alerts the physical therapist that using the patient's heart rate (HR) to monitor exercise is not advisable because the medication lowers HR. WHICH of the following medications is the patient taking? A. Ace inhibitor B. Beta blocker C. Digoxin D. Statin

Correct answer: B Beta blockers block the effects of epinephrine and cause the heart to beat more slowly and with less force, decreasing cardiac output and lowering blood pressure. Thus, HR and BP are not reliable vital signs for monitoring the effect of exercise on a patient taking a beta blocker. Answer A:Angiotensin conversion enzyme (ACE) inhibitors decrease fluid production and prevent angiotensin from constricting blood vessels. Both affect blood pressure, not heart rate. Answer C:Digoxin increases heart contractility and would not lower HR. Answer D:Statins reduce levels of triglycerides and cholesterol. Bottom Line:Beta blockers block the effects of epinephrine and cause the heart to beat more slowly and with less force, decreasing cardiac output and lowering blood pressure.

A patient with a tibial fracture and slight concussion is being treated in the hospital after a motor vehicle accident 24 hours ago. The physical therapist is instructing the patient in gait training with crutches when the patient suddenly has onset of right upper extremity (UE) and lower extremity (LE) weakness and difficulty speaking, and the patient becomes labile. When asked what is wrong, the patient attempts to speak but can only repeat the word "penny." The physical therapist suspects a cerebrovascular accident and calls a code. What lobe of the brain does the physical therapist suspect has been affected? a) parietal b) frontal c) occipital d) temporal

Correct answer: B Broca's area is located in the left frontal lobe and is responsible for expressive language. The patient is having difficulty communicating, also known as expressive aphasia, and has impaired emotions, which also occurs with damage to the frontal lobes. The right-sided weakness also indicates that the stroke is located on the left side. Answer A:The parietal lobe is responsible for movement, orientation, and recognition. Answer C:The occipital lobe is responsible for visual processing, and the patient would have demonstrated visual impairment with damage to the occipital lobe. Answer D:The temporal lobe is responsible for memory and processing of speech. An injury to this lobe would not affect the patient's ability to speak but would affect the patient's ability to understand what is being said.

A 64-year-old man was referred to physical therapy with acute-onset neck pain radiating into the left shoulder for 4 weeks. Pain was the worst when running and lifting objects. The patient's goal was to return to running. Past medical history included hypertension, dislipidemia, hypothyroidism, and recent thyroidectomy. Cervical spine imaging revealed degenerative changes. Physical examination of the cervical spine, shoulder, and upper quarter did not reproduce his symptoms. Having the patient run briefly on the treadmill reproduced his neck and shoulder pain. When he stopped running, his pain ceased. Which of the following is the MOST likely explanation for the patient's pain? a) discogenic disorde b) cardiovascular disease c) osteoarthritis d) tumor

Correct answer: B Cardiovascular disease is the most plausible explanation, as the pain was reproduced with running and went away when the running stopped. The increased cardiovascular demands were the only activity that reproduced the pain. None of the other tests and measures that looked for dysfunction with the musculoskeletal or neurological systems were able to reproduce the pain. The pain radiating to the left shoulder is also typical of cardiovascular symptoms. In addition, the pain was not always present; it occurred only when increasing the demands on the cardiovascular system. Answer A:If the pain was neurological in origin, it would have been picked up during the upper quarter screen and also would have included some other neurological signs, such as numbness and tingling. In addition, the provocation tests like axial compression would increase symptoms. In this case, none of the examination tests reproduced the pain. Answer C:If the pain was due to osteoarthritis, the physical therapist would have been able to reproduce it during the examination with a number of the provocation tests and position changes. Answer D:Pain from a tumor would most likely not increase or decrease with running. Bottom Line:It is important to create a working hypothesis from the subjective history and follow with clinical tests and measures before being biased by imaging results. A patient with pain radiating into the neck and left shoulder that has symptoms reproduced only by increasing the demand on his cardiovascular system, and not during any other clinical tests, should be referred back to his primary care doctor for a referral to a cardiologist.

A physical therapist is treating a patient with central cord syndrome. In which of the following locations would the patient demonstrate the greatest weakness? a) distal lower extremity b) distal upper extremity c) proximal lower extremity d) proximal upper extremity

Correct answer: B Central cord syndrome is an incomplete spinal cord injury commonly caused by cervical hyperextension. It usually occurs in patients who are older with some preexisting arthritic changes and spurs in the cervical area. The hyperextension of the cervical spine causes the most damage to the fibers that are medial in the spinal cord. Due to the anatomical configuration of the corticospinal tract, upper extremity (UE) fibers are more medial and lower extremity (LE) fibers are more lateral, thus the UEs are affected more than the LEs. In addition, the distal UE fibers are more medial than the proximal UE fibers, with the greatest weakness shown in the distal UE fibers. Answer A:The distal lower extremity fibers are found more laterally in the spinal cord and are spared with central cord syndrome. Answer C:The proximal lower extremity fibers are found more laterally in the spinal cord and are spared with central cord syndrome. Answer D:The proximal upper extremity fibers are found more laterally in the spinal cord and are less affected than the distal upper extremity fibers in patients with central cord syndrome. The more proximal upper extremity muscle groups are often initially affected by central cord syndrome but are more likely to recover than the distal upper extremity fibers.

A 56-year-old male reports left shoulder pain for the past month of insidious onset that he says "comes and goes." He describes the pain as intermittent and "sharp and burning" with "pins and needles" localized over the lateral deltoid area. The physical therapist examines shoulder PROM, AROM, and mobility and performs special tests for impingement and instability. The therapist is unable to reproduce the symptoms. Which of the following is the MOST appropriate next course of action? a) discuss the possibility of early signs of myocardial infarction and have him ask his physician about referral to a cardiologist b) examine range of motion and movements of the cervical spine to look for cervical radiculopathy c) refer the patient back to the physician for an electromyography to test for muscle disease d) perform tests for thoracic outlet syndrome

Correct answer: B Cervical radiculopathy results from compression of cervical nerve roots, leading to pain radiating to corresponding areas of the shoulders and arms. In cases where shoulder pain or symptoms cannot be reproduced by tests on peripheral structures, involvement of the cervical spine should be suspected and cleared. Answer A:Although pain in the upper extremity can be a symptom of a myocardial infarction, the overall clinical picture of this patient does not suggest cardiac involvement. If the patient's only symptom is intermittent shoulder pain, then there is no indication that this is cardiac related. Therefore, this answer is incorrect. Answer C:The patient's overall clinical picture does not suggest muscle disease. Signs and symptoms of muscle disease may include noticeable atrophy, abnormal swelling, and muscle fasciculations. His symptoms are more consistent with radiculopathy so tests for the cervical spine should be performed for confirmation. Answer D: Thoracic outlet syndrome could be a possibility (although small) since symptoms may include pain and numbness in the shoulder and arm. However, it is not the best answer because the patient's symptoms warrant examination of the cervical spinefirst.Only after cervical spine involvement is ruled out should other causes for symptoms be considered. Therefore, this answer is incorrect. Bottom Line:Musculoskeletal symptoms can often be attributed to a neurologic cause. Therefore, physical therapists must be able to identify whether a patient's symptoms come from a peripheral cause (e.g., shoulder joint, muscle, peripheral nerve) versus a central cause (e.g.,cervical spine, nerve root, referred pain) in order to provide the most appropriate and effective treatment interventions.

A 25-year-old patient in acute rehabilitation with an ASIA A C8 spinal cord injury has a tilt-in-space wheelchair with a head rest and pressure-relieving cushion. After completing a postural assessment using pressure mapping, the physical therapist determines the patient tended to sit in a posterior pelvic tilt, which put excessive pressure on her sacrum. Which of the following should be the FIRST change that the PT should make? a) change the backrest b) change the cushion c) change the headrest d) change the type of wheelchair

Correct answer: B Changing the wheelchair cushion to a style with a firmer surface is often used in patients who sit in a posterior pelvic tilt to improve their sitting posture. Answer A:Changing the backrest may allow the patient to sit in a more upright position, which could improve the pelvic rotation. However, changing the cushion would be the most direct way to change the position of the pelvis. Answer C:Changing the headrest would not affect the position of the pelvis. Answer D:Changing the wheelchair from a tilt-in-space to another type of wheelchair would not affect the position of the pelvis and would require the patient to be able to successfully perform another type of pressure relief (e.g., lateral, push up), or skin breakdown would become more likely. Bottom Line:A physical therapist must decide upon initial examination based on the level and completeness of injury, patient age, skin status, and comorbid factors which type of wheelchair and seating system (headrest, backrest, and cushion) should be initially provided to a patient with a spinal cord injury to improve their sitting posture. Pressure ulcers are likely to develop if the physical therapist recommends an incorrect type of wheelchair and seating system.

A physical therapist is treating a 9-year-old child with spastic diplegic cerebral palsy who has developed bilateral hip dysplasia. The patient has adductor spasticity and a scissoring gait. When considering surgical interventions to address the hip dysplasia, which of the following procedures is the LEAST effective surgical option? a) adductor tenotomy b) hamstring release c) femoral derotational osteotomy d) obturator neurectomy

Correct answer: B Children with hip dysplasia due to cerebral palsy often have tight hamstring muscles, but hamstring release would not address the hip dysplasia. A hamstring release surgery would improve knee extension passive range of motion and increase the amount of anterior pelvic tilt available.Hip dysplasia is a common orthopedic complication of diplegic cerebral palsy because a combination of spasticity and decreased weight bearing may lead to an underdeveloped hip joint. Surgical procedures to prevent it or reduce it include adductor tenotomy and obturator nerve neurectomy, which also weakens the adductor pull. A surgical derotation of the femoral head is also an option. Answer A:A adductor tenotomy would improve the position of the hip by decreasing hip adduction so that the femoral head would be better positioned in the acetabulum. Answer C:A femoral derotational osteotomy would surgically reposition the femoral head into the acetabulum. Answer D:An obturator neurectomy would improve the position of the hip by decreasing hip adduction spasticity but cutting the nerve to the hip adductors so that the femoral head would be better positioned in the acetabulum. Bottom Line:Hip dysplasia is a common orthopedic complication of diplegic cerebral palsy because a combination of spasticity and decreased weight bearing may lead to an underdeveloped hip joint. Surgical procedures to prevent it or reduce it include adductor tenotomy and obturator nerve neurectomy, which also weakens the adductor pull. A surgical derotation of the femoral head is also an option. TrueLearn Insight :Physical therapists who treat children with cerebral palsy must recognize the surgical interventions that are used to address the orthopedic implications of the disease process. Physical therapists often provide postoperative care to these patients, so PTs must be able to explain to parents and their children the purpose of the procedure and what the rehabilitative process will be like.

A physical therapist is completing an initial examination with a patient with a history of Alzheimer's disease. Which of the following classes of medication would the therapist expect the patient to take in order to delay the progression of symptoms? a) anxiolytics b) cholinesterase inhibitors c) dopamine agonists d) selective serotonin reuptake inhibitors

Correct answer: B Cholinesterase inhibitors are a class of medications that have been approved by the FDA and that allow patients to retain quality of life and retain cognitive function during the early stages of Alzheimer's disease. Answer A: Anxiolytics are a common class of medication used to treat anxiety disorder, and are often used as an adjunctive medication since patients with Alzheimer's disease often develop anxiety. Answer C: Dopamine agonists are commonly used to treat Parkinson's disease and are not used in patients with Alzheimer's disease. These medications could exacerbate any psychotic symptoms associated with the Alzheimer's disease. Answer D: Selective serotonin reuptake inhibitors (SSRIs) are a common class of medication used to treat depression. SSRIs are often used as an adjunctive medication for patients Alzheimer's disease who commonly develop depression.

A 59-year-old patient is admitted to the hospital with possible cardiovascular accident (CVA). Brain MRI is pending. The therapist evaluates the patient's mobility and wants to include the test for clonus in the examination. Which of the following is the best way to test for clonus? A. Noxious stimulus "pinprick" to sole of foot B. Quickly dorsiflex the foot and maintain dorsiflexion C. Stroke the lateral ankle up the lateral/dorsal aspect of the foot D. Stroke the sole of foot along lateral aspect and around to the hallux

Correct answer: B Clonus is characterized by cyclical, spasmodic alternation of muscular contraction and relaxation in response to sustained stretch of the spastic muscle. It is examined using a quick stretch stimulus that is then maintained. Clonus is common in the ankle plantar flexors but may also occur in other areas of the body, such as jaw or wrist. Answer A:This is a way to test for flexor withdraw reflex. Flexor withdraw reflex is a primitive reflex present during infancy but integrated by the CNS at an early age. Once integrated, these reflexes are generally not seen in adults. Reappearance of these reflexes in adults following a brain injury are always indicative of neurological involvement. Answer C:This test is called Chaddock's reflex. It produces dorsiflexion of the big toe and is considered a confirmatory response. Answer D:Stroking the lateral aspect of the sole of the foot is the proper procedure when looking for Babinski sign. A normal response to this stimulus consists of flexion of the big toe, sometimes flexion of the other toes, or no response at all. An abnormal response (Babinski sign) consists of dorsiflexion of the big toe and fanning of the four other toes. It is indicative of a corticospinal lesion. Bottom Line:Clonus is characterized by cyclical, spasmodic alternation of muscular contraction and relaxation in response to sustained stretch of the spastic muscle. It is examined using a quick stretch stimulus that is then maintained. Clonus is common in the ankle plantar flexors but may also occur in other areas of the body, such as jaw or wrist.

A physical therapist is completing an initial examination of a patient who had a wrist fracture and is now out of the cast. The patient demonstrates symptoms of severe pain when touched lightly, vascular changes, and limited passive range of motion throughout the affected upper extremity. What is the MOST likely cause of these symptoms? a) heterotopic ossification b) complex regional pain syndrome c) myositis ossificans d) Raynaud's phenomenon

Correct answer: B Complex regional pain syndrome(CRPS) type 1, previously called reflex sympathetic dystrophy syndrome, occurs commonly in patients with a history of stroke. CRPS type 2 occurs after a distinct nerve injury or a mild or severe trauma, such as wrist fracture and casting in this case. The cause of complex regional pain syndrome is unknown, but changes in the autonomic nervous system result in pain, vascular changes, and limited passive range of motion. The nerves controlling blood flow and temperature are damaged in patients with CRPS, so they may display changes in skin temperature, skin color, edema, excessive sweating, and/or hair loss in the affected limb. Answer A: Heterotopic ossification refers to the formation of bone within soft tissues outside normal bone and joint structures. It is not likely to be the cause of the symptoms in the scenario since hypersensitivity to touch and vascular changes are not associated with heterotopic ossification. Answer C: Myositis ossificans refers to the formation of bone within soft tissues such as large muscle, usually due to trauma. It is not likely to be the cause of the symptoms in the scenario since hypersensitivity to touch and vascular changes are not associated with myositis ossificans. Answer D: Raynaud's phenomenon is the result of vasospasms in the arteries of the hand, occurs in response to cold or stress, and is not likely to be the cause of the symptoms in the scenario.

A 65-year-old woman with a history of diabetes comes to the physical therapist with complaints of difficulty with balance, generalized weakness, and horizontal diplopia. When looking at the patient in neutral gaze, the therapist notes the patient's head is turned slightly to the left. To determine cranial nerve VI palsy, the therapist will use the cranial nerve assessment technique that involves: A. assessing upward and downward gaze. B. assessing lateral gaze. C. inspecting eyelid for ptosis. D. assessing visual acuity.

Correct answer: B Cranial nerve VI is responsible for lateral movement of the eyeballs. Assessment of direction of gaze(ability to move eyes out laterally when intact) will assess cranial nerve VI (lateral rectus of the eye). Eyes will maintain a position medially when impaired. The patient might adopt a face turned toward the side of the affected eye with the aim of controlling diplopia and maintaining binocular vision. Answer A:Cranial nerve IV is responsible for upward and downward movement of the eye. Answer C:Cranial nerve III is responsible for eyelid elevation. Answer D:Cranial nerve II is responsible for visual acuity. Bottom Line:Direction of lateral gaze will determine the function of cranial nerve VI. Patients often complain of horizontal double vision with cranial nerve VI palsy.

Which of the following signs and symptoms should alert a physical therapist that the patient may have a diagnosis of delirium? a) slow chronic onset of mental changes b) hypoactive or hyperactive state c) slow decline in attention d) significant changes in all vital signs

Correct answer: B Delirium is significant in that the patient may exhibit both hypoactive and hyperactive states. Delirium can manifest in three ways: a hypoactive state, a hyperactive state, or a mixed state where the patient fluctuates between the two. The onset of delirium is sudden and does not directly or permanently affect short-term or long-term memory. Answer A: All changes in delirium are acute and sudden; slow onset of mental changes would be more specific to dementia or mild cognitive impairment. Answer C: Changes in delirium are sudden; slow decline in attention would be more specific to dementia or mild cognitive impairment. Answer D: There is no change or minimal change in vital signs when a patient is suffering from delirium, which is one of the reasons it is so difficult to diagnose. All signs and symptoms are behavioral and can be hypoactive or hyperactive. Bottom Line: Delirium is a sudden acute-onset condition that is characterized by significant behavior changes and hypoactive or hyperactive state or fluctuation between the two. There is no significant change in vital signs, which makes it much harder to diagnose. Working closely with the caregivers to determine if the behavior is suddenly different is the primary means of diagnosis.

An individual presents to the clinic with a report of pain in the low back region. The physical therapist is examining the sacroiliac (SI) joint to determine if it is the source of the pain. Which of the following tests would stress the SI joint in an effort to reproduce the pain? a) march test b) distraction/gapping test c) supine to long sit test d) standing forward flexion test

Correct answer: B Distraction/Gapping is a pain provocation test that adds evidence, positive or negative, to the hypothesis of an SI joint sprain or dysfunction. This test stresses the anterior sacroiliac ligaments. Answers A & C & D:This would assess for a movement dysfunction of the SI joint but is not attempting to reproduce the pain. Bottom Line:Pain provocation tests are best used as a cluster, but even in isolation should be used to determine if the source of pathology is stemming from the SI joint compared to other tests of the lumbar region or movement tests, as pain provocation tests look for local inflammation within the joint. The Distraction/Gapping testis a pain provocation test that adds evidence, positive or negative, to the hypothesis of an SI joint sprain or dysfunction. This test stresses the anterior sacroiliac ligaments.

During a chart review, a physical therapist notes that the patient is taking diuretics. Which of the following assumptions is MOST accurate and should be considered before working with this patient? a) side effects include orthostatic hypertension and electrolyte imbalance b) side effects include orthostatic hypotension and dehydration c) the diuretic may have been prescribed to help address low BP d) the diuretic may have been prescribed to help address low levels of potassium

Correct answer: B Diuretics prevent the kidneys from reabsorbing sodium and water from the urine. Thus the amount of water and sodium that exits the body is increased, and blood pressure is reduced. Orthostatic hypotension is a side effect, and the therapist should be on the lookout for patient reports of lightheadedness or dizziness. In addition, the patient's blood pressure should be monitored. Dehydration from excessive loss of water is also a possible side effect. Answer A:Diuretics cause more water to be released in the urine, and blood pressure would fall, not rise. The risk of electrolyte imbalance exists because of the risk of excessive loss of sodium or potassium. Answer C:A diuretic helps address high blood pressure by removing more water through the urine. Answer D:A side effect of diuretics may be an excessive loss of potassium. Options are to use a potassium-sparing diuretic, or to prescribe a potassium supplement when the patient is taking diuretics. In any event, a diuretic would not be prescribed solely to correct low levels of potassium.

A physical therapist is examining a patient with a history of multiple sclerosis in outpatient rehabilitation. MRI shows multiple lesions in her cerebellum bilaterally. The patient is unable to brush her teeth or tap her feet due to difficulty with reversing movements of her extremities. What is the name of this clinical condition? a) apraxia b) dysdiadochokinesia c) dysmetria d) intention tremor

Correct answer: B Dysdiadochokinesia is the inability to perform rapid alternating movements. This disorder often causes difficulty with activities of daily living and with walking. Dysdiadochokinesia is typically found in patients with damage to the cerebellum. Answer A:Apraxia is the inability to perform motor activities upon command while the sensory and motor functions are intact. Apraxia is more commonly found in patients with damage to the left cerebral cortex rather than the right cerebral cortex. Answer C:Dysmetria is characterized by incoordination of movement in which overshooting or undershooting occurs while reaching. Dysmetria is typically found in patients with damage to the cerebellum. Answer D:An intention tremor occurs during voluntary movement while the tremor gets worse as it approaches its target. An intention tremor is typically found in patients with damage to the cerebellum.

A physical therapist is working with a patient who experienced a cerebrovascular accident 1 week ago. The patient is ambulating with a quad cane but is struggling with changing direction safely. The therapist decides to add more extrinsic feedback for the patient. Which of the following cues would provide this patient with appropriate extrinsic feedback? a) having the patient turn their head from right to left while walking b) placing items on the floor for the patient to walk around c) adding ankle weights to the patient when walking d) instructing the patient to practice with eyes closed

Correct answer: B Extrinsic cues for motor learning include cues that are not normally received during the movement. In this case, a visual cue on the floor will add an extrinsic cue for the patient when working on direction changes. Proprioceptive, vestibular, and natural visual cues are all intrinsic cues. Answer A:Turning the head from one side to the other will trigger the vestibular system which is an intrinsic cue and will not assist the patient in direction changes. Answer C:Ankle weights will increase intrinsic, not extrinsic, cues by adding proprioceptive input for the patient. Cutaneous input, which is extrinsic, will be added but will not assist in direction changes. Answer D:Visual input as provided by the sensory system in the form of eyes open or closed is a form of intrinsic cue. Bottom Line:Extrinsic cues for motor learning include cues that are not normally received during the movement. Proprioceptive, vestibular, and natural visual cues are all intrinsic cues.

A physical therapist decides to use functional electrical stimulation (FES) on a patient. Which of the following is an APPROPRIATE USE of FES? A. Pain control for a patient status post total knee arthoplasty B. Substitute for an ankle foot orthosis (AFO) to improve ambulation in a patient post stroke C. Muscle strengthening to strengthen the wrist muscles post casting after a distal radius fracture D. Biofeedback to improve activation of the abdominals

Correct answer: B Functional electrical stimulation (FES) is the term used to denote the use of electrical stimulation to enhance or improve functional movements. A typical application of FES is to activate the dorsiflexors in the swing phase of gait to improve walking in patients with foot drag following a stroke. Answer A:Electrical stimulation for pain control is not an example of functional electrical stimulation. Answer C:Neuromuscular electrical nerve stimulation is used to stimulate specific muscles to assist in strengthening. Answer D:Biofeedback does not use electrical stimulation.

When working with a patient over a period of 6 visits, one clinic insists that the same therapist always conduct the goniometer measurements of the affected joint. By doing this, which of the following concepts is being considered? A. Inter rater reliability B. Intra rater reliability C. Inter rater validity D. Intra rater validity

Correct answer: B Goniometry has been shown to have excellent intra rater reliability, that is, when the same therapist repeats the measurements on the same patient over time. Validity is not being considered in this question as validity considers the instrumentation being used rather than the therapist. Answer A:Inter rater reliability is the idea that two different therapists can measure the same joint and get similar results. However, studies have shown that inter rater reliability with goniometry, while good, is not as good as intra rater reliability, which is the same therapist measuring the same patient. Answer C:The validity of goniometry is the consideration of whether the instrument is measuring what it is intended to measure. This is not being considered in this question, which is instead asking whether repeated measures are accurate. Answer D:Validity is not under consideration in this question. The question is looking at reliability, whether measurements can be reliable between therapists over time. Bottom Line:Inter and intra rater reliability issues with respect to goniometry are always being considered. While both have been shown to be good, intra rater reliability is greater than inter rater. Therefore having the same therapist always measure their patient over time will be more reliable than having different therapists perform the measurements.

A 40-year-old overweight male patient presents to the clinic with pain in his right MTP joint. The patient reports that he started himself on a weight loss regimen and recently began running two miles a day and eating a high-protein/no-carbohydrate diet. Prior to this, the patient was very sedentary. Upon examination, muscle strength is within normal limits in both lower extremities. Passive range of motion (PROM) is within normal limits except for bilateral ankle dorsiflexion (0-3) degrees. Right MTP joint has full PROM but is very painful, red, and swollen. Slight hallux valgus was noted bilaterally. The patient reports that the pain came on suddenly at night. What is the most likely cause of this patient's pain? a) hallux valgus/bunion b) gout c) metatarsal fracture d) sesamoiditis

Correct answer: B Gout is a type of arthritis caused by a uric acid buildup in the joints. Commonly affected joints include the feet, ankles, wrist and elbows, and is especially common in the first MTP joint. The body produces uric acid when breaking down purines. Obesity and eating a diet high in purines like red meats are risk factors for this disease. The presentation is a hot, swollen joint that usually occurs suddenly at night after a meal of fatty foods. Answer A:The patient has slight hallux valgus bilaterally, but only one joint is painful. In addition, there was no range of motion limitation in the joint. Answer C:A metatarsal fracture could be possible after this obese patient began a running regimen and pounding on his feet. However, the pain was not localized to the metatarsal, but more at the joint line. In addition, the redness, heat, and swelling are more indicative of an arthritic type of disorder. Lastly, fracture pain would not appear suddenly in the middle of the night, especially if the patient was pain-free just prior to that. Answer D:The sesamoid bones are located under the first MTP and if irritated or fractured would cause pain to that joint. However, the pain would be on the plantar surface only and would be painful upon bearing weight, not beginning suddenly in the middle of the night. Bottom Line:Obesity, a diet high in meat, along with sudden onset of symptoms at night and a hot, swollen first MTP joint, should lead the therapist to suspect gout.

A 55-year-old female with diabetes mellitus (DM) is being treated by a physical therapist 2 times a week for the past 4 months for adhesive capsulitis. Treatment consists of manual therapy, including shoulder joint mobilization. Her shoulder range of motion is slowly returning, and the patient reports 7/10 pain with treatment, and 2/10 pain at all other times. Based on the information provided, which grade(s) of joint mobilization is/are the MOST appropriate to use with this patient? a) Grades 1-2 b) Grades 3-4 c) Grade 5 d) Joint mobilizations are not indicated in this patient due to the past medical history of DM

Correct answer: B Grades 3 and 4 are the most appropriate because they will help to stretch the joint capsule and adhesions. The pain felt during the mobilizations is due to the forces attempting to stretch the capsule and is normal. The fact that the pain subsides and is usually at a very low level at all other times indicates that the pain during treatment is temporary and appropriate. In addition, there are gains being made in ROM, validating the treatment choice. Answer A:Grades 1 and 2 are meant for patients in severe pain to help with some pain reduction and encourage some joint mobility to prevent stiffening. These grades do not increase ROM as effectively. This patient has 2/10 pain at most times, and grades 1 and 2 would not be the most effective at restoring her ROM. Grades 1 and 2 may be appropriate for a patient at the very beginning phases of adhesive capsulitis, but this patient is already 4 months into her adhesive capsulitis. Answer C:Grade 5 is a thrust manipulation at end range and is contraindicated as this can tear a fibrotic capsule. This is performed sometimes under anesthesia but is not appropriate in this case. Answer D: Idiopathic adhesive capsulitis occurs more frequently in females over age 50 and in patients with DM. The patient described here is a typical patient. Joint mobilizations for this diagnosis are indicated and very appropriate. Bottom Line: Adhesive capsulitis is treated with shoulder joint mobilization to restore the capsular mobility and the shoulder range of motion. Grades 1 and 2 are small oscillations at the beginning and beginning to mid-range and are good for early phases, or for any patient in severe pain. Grades 3 and 4 are more effective at increasing range of motion. Grade 3 is large-range oscillation to the restriction of the range, and grade 4 is small oscillations at the point of restriction. Grade 5 is more of a thrust manipulation and not appropriate because it may tear the fibrous adhesions.

A clinician decides to use a moist hot pack on a patient's frozen shoulder before beginning manual therapy. WHICH of the following physiological responses is seen as a result of the superficial heat? a) heat induced vasoconstriction b) increased pain threshold c) temporary increase in muscle strength for approximately 1/2 hour after application d) reduction in soft tissue extensibility

Correct answer: B Heat increases the activity of the cutaneous thermoreceptors, which can have an immediate inhibitory gating effect on transmission of the sensation of pain at the spinal cord level. Answer A:Heat causes vasodilation and thus an increase in the rate of blood flow. It also causes, to a lesser degree, vasodilation in areas distant from the site of application. This distant vasodilative effect may be used to increase cutaneous blood flow to an area where it is difficult or unsafe to apply a heating agent directly. Answer C:Muscle strength and endurance have been found to decrease during the initial 30 minutes after application of deep or superficial heating. It is thought that this is the result of changes in the firing rate of type II muscle spindle efferent, gamma efferent, and type Ib fibers from the Golgi tendon organs caused by heating of the motor nerves. Therefore, it is recommended that muscle strength and endurance always be measured before applying a heating modality. Answer D:When soft tissue is heated before stretching, it maintains a greater increase in length after the stretching force is applied, less force is required to achieve the increase in length, and the risk of tissue tearing is reduced. Bottom Line:The use of heating modalities in rehabilitation is primarily to control pain, increase soft-tissue extensibility, increase circulation, and accelerate healing.

To examine whether playing overhead sports increases the risk of rotator cuff impingement, a physical therapist performs a chart review of previously collected data, including patient histories. She utilizes her clinic's database to identify the patients who were treated for rotator cuff impingement and an age-matched group of patients who came to the clinic for reasons other than shoulder dysfunction. Which of the following research designs is the physical therapist using? a) correlational study b) case-control study c) longitudinal study d) randomized controlled trial

Correct answer: B In a case-control study, groups are selected based on whether they have the condition being studied. If they have the condition, they are the "case," and if they do not have the condition, they are the "control." The two groups are then compared to determine why the condition occurs more frequently in the "cases" than in the "controls." The investigators will look at factors retrospectively to see if frequency of exposure to certain risk factors may have increased the risk of getting the condition. Answer A:Correlational study examines the relationship between variables that seem to interact with each other, so that when you can see one changing, you get a sense of how the other will change. It does not imply causation, it only describes the nature of the relationship. Answer C:In a longitudinal study, subjects are followed over time, and repeated measures are taken at specific intervals. Answer D:A randomized controlled trial has at least two groups. One group gets an intervention, and one acts as the control. Subjects are assigned randomly to the groups. Bottom Line:In a case-control study, groups are selected based on whether they have the condition being studied. If they have the condition, they are the "case," and if they do not have the condition, they are the "control." The two groups are then compared to determine why the condition occurs more frequently in the "cases" than in the "controls." The investigators will look at factors retrospectively to see if frequency of exposure to certain risk factors may have increased the risk of getting the condition.

A patient status post mastectomy with lymph node removal complains of heaviness, tightness, swelling, and sensory changes in the rightupper extremity. During palpation, the therapist notes that the edema is soft and that mild pressure causes pain. When the limb is elevated, the edemadissipates. The therapist is correct in documenting that the patient is in WHICH of the following stages of lymphedema? a) Stage 0 b) Stage 1 c) Stage 2 d) Stage 3

Correct answer: B In lymph edema stage 1, the edema is present and may be variable (e.g., decrease with rest or sleep and increase with activity). The edema is very soft and easily pits with little to no fibrosis. Elevation of the limb results in dissipation of the edema. Patient may report heaviness, tightness, sensory changes, or pain in the limb. Answer A:Lymphedema stage 0 is also known as the subclinical stage. Lymph transport is impaired; however, lymphatic load has not yet exceeded the transport capacity. Swelling is not evident, and the patient may report heaviness, tightness, or sensory changes in the affected limbs. Answer C:In lymphedema stage 2, edema is present, but the edema no longer dissipates with elevation of the limb. Palpation is positive for fibrosis and results in indentation of the skin. Fibrosis ranges from soft and pitting (early stage II) to hard and non-pitting (late stage II). Skin mobility may be decreased, and texture may be brawny. Answer D:In lymphedema stage 3, fibrotic changes of the skin and fat deposits in the subcutaneous tissues continue to increase in severity. When palpating the skin, the limb is hard with little mobility and increased skin folds. Pitting is absent or minimally present with significant pressure and, if present, dissipates rapidly. Hair follicles may be coarse or absent. Other symptoms include loss of joint spaces, papillomas, skin hyperpigmentation, hyperkeratosis, cysts, and fistulas. Bottom Line:In lymphedema stage 1, the edema is present and may be variable (e.g., decrease with rest or sleep and increase with activity). The edema is very soft and easily pits with little to no fibrosis. Elevation of the limb results in dissipation of the edema. The patient may report heaviness, tightness, sensory changes, or pain in the limb. TrueLearn Insight :In stage 1 (spontaneously reversible lymphedema), it is possible for the swelling that typically presents at the end of the day to recede overnight. Furthermore, elevation of the limb may assist in further reduction of the swelling. The involved extremity is soft, and pitting (impressions from pushing on the skin) is easily induced. With proper management in stage 1, it is possible for the patient to expect complete reduction of the involved limb (when compared to the normal limb).

A physical therapist is working with a patient who recently experienced a traumatic brain injury. The therapist has determined that the patient is currently in the cognitive stage of motor learning. Which of the following training strategies would be MOST suitable for this patient? a) variable feedback, open environment, random practice b) blocked practice, low environmental stimuli, closed skills c) occasional feedback, varied environments, massed practice d) blocked practice, open skills, occasional feedback

Correct answer: B Individuals in the cognitive stage of learning require a very structured environment; they require feedback after every trial to improve learning at this early stage and benefit from very low external environmental stimuli. These clients are easily distracted and should start working on closed skills before slowly progressing to open skills. Answer A:These types of intervention are most appropriate for a patient in the associative stage of learning, as they require higher levels of ability from the patient and greater ability to focus on the task. Answer C:This type of intervention is most suited to a client who is in the autonomous stage of learning, is working on refining movement, and is preparing to transition to the home environment. Answer D:Blocked practice is appropriate for the cognitive stage, but in the cognitive stage, skills should primarily be closed with gradual progression to open skills. Feedback at the cognitive stage should be given after every trial, progressing to variable feedback. Bottom Line:The cognitive stage of motor learning is the earliest stage and requires the most structure with feedback after every trial, focus on closed skills, minimal distraction from the environment, and organized practice. The associative stage, and then the autonomous stage, are more advanced stages of motor learning. The therapist should start introducing more environmental stimuli and more variety into the practice patterns, and feedback should decrease.

A physical therapist is assessing an 8-year-old patient status postsurgical repair of a medial epicondyle fracture secondary to a skateboard accident. The following findings are present: swelling and pain in the left arm and forearm, motor deficits including inability to adduct and flex the wrist, inability to flex digits 4 and 5, and inability to abduct and adduct digits and extend interphalangeal joints of digits 4 and 5. Sensory impairment included diminished sensation in skin on the medial aspect of palm and medial 1½ fingers. The sensory and motor deficits described are most consistent with which fracture? A. Fracture of the medial epicondyle of the humerus with radial neuropathy at the axilla B. Fracture of the medial epicondyle of the humerus with ulnar neuropathy C. Fracture of the medial epicondyle of the humerus with axillary neuropathy D. Fracture of the medial epicondyle of the humerus with radial neuropathy

Correct answer: B Injury of the ulnar nerve affects sensation on the medial aspect of the palm and medial 1 1/2 fingers, as well as causing motor deficits of wrist adduction (flexor carpi ulnaris), wrist flexion (ulnar part of flexor digitorum profundus), flexion of digits 4 and 5 (ulnar part of flexor digitorum profundus), abduction and adduction of digits (interossei and adductor pollicis), and extension of IP joints of digits 4 and 5 (interossei and medial two lumbricals). Answer A:Injury to the radial nerve in the axilla can result from compression against the humeral shaft. Sensory deficit locations would include radial side of dorsum of the hand, especially between metacarpals 1 and 2. Motor deficit would include elbow extension, wrist extension, extension of metacarpophalangeal joints, extensionof interphalangeal joints, elbow flexion (brachioradialis), and supination. Answer C:A patient with axillary neuropathy would present with deltoid weakness and sensory loss in the shoulder region. Answer D:The radial neuropathy was diagnosed by weakness in wrist extension ("wrist drop") and sensory deficit from the distribution of the superficial radial nerve. Elbow extension was normal because the motor innervation to the triceps muscle branched from the radial nerve proximal to the fracture. Bottom Line:Injury of the ulnar nerve affects sensation on the medial aspect of the palm and medial 1 1/2 fingers, as well as causing motor deficits of wrist adduction, wrist flexion, flexion of digits 4 and 5, abduction and adduction of digits, and extension of IP joints of digits 4 and 5.

The physical therapist is completing a physical examination of a patient with a suspected visceral disorder. The therapist screens the abdomen for symmetry,alignment, skin color, and scars. WHICH COMPONENT of the physical exam of the abdomen is being performed? a) auscultation b) inspection c) percussion d) palpation

Correct answer: B Inspection is the process of visually looking at the body for symmetry, alignment, skin color and scars present. Answer A:Auscultation is performed to listen for bowel sounds, which are normally made by the movement of food, liquid, and air through the intestines. Answer C:Percussion uses sound to determine size, shape, and density of tissue. Answer D:Palpation in the abdominal region is usually deep palpation (firm pressure) and is used to assess for tenderness and the presence of visceral organs.

A physical therapist is treating a patient with low back pain. An electrical modality in which two currents of medium frequencies are crossed and the resultant current is used to modulate pain is WHICH of the following? A. High volt pulsed current B. Interferential current C. Neuromuscular electrical nerve stimulation D. Iontophoresis

Correct answer: B Interferential current is formed when two medium-frequency currents are crossed, and the resulting current from the interference of the two currents can be used to manage acute or chronic pain. Answer A:High volt pulsed current is a twin-peaked direct current that can be used for wound care and pain control. Answer C:Neuromuscular electrical nerve stimulation is a biphasic current with a duty cycle and is primarily used to assist in strengthening muscles. Answer D:Iontophoresis is a continuous direct current which is used to drive ionized medication for local application. Bottom Line:Interferential current is formed when two medium-frequency currents are crossed, and the resulting current from the interference of the two currents can be used to manage acute or chronic pain. TrueLearn Insight :Knowledge of various properties of electrical currents allows the clinician to select the best modality to use depending on the goals of treatment.

Which of the following lists correctly includes only examples of intrinsic feedback for promoting motor learning in a client? A. Visual, auditory, tactile, and cutaneous B. Proprioceptive, visual, vestibular, and cutaneous C. Proprioceptive, vestibular, tactile, and auditory. D. Visual, cutaneous, tactile, and vestibular.

Correct answer: B Intrinsic feedback is sensory cuing that occurs as a result of the movement of the client and includes proprioception, visual, vestibular, and cutaneous, all listed in the correct answer. Answer A:Auditory and tactile are examples of extrinsic cues, so this answer is not correct as it contains two options that are not specific to intrinsic cues even though visual and cutaneous are intrinsic. Answer C:Proprioceptive and vestibular are intrinsic cues, but tactile and auditory are extrinsic, so this is not correct. Answer D:Tactile in this list is an extrinsic cue, although the others are intrinsic, so this list is not correct.

A physical therapist evaluates a client who suffered a complete spinal cord injury. The client can demonstrate full motion with moderate resistance in elbow flexion/extension, forearm pronation/supination, wrist flexion/extension, finger flexion/extension, and thumb flexion/extension/abduction. The highest level of manual wheelchair activities the client will be able to perform is to: A. propel independently on all indoor and outdoor surfaces. B. propel independently on all indoor surfaces and level outdoor terrain. C. propel independently on all indoor surfaces and some to total assist on outdoor surfaces. D. propel independently to some assist indoors on non-carpet, level surface; some to total outdoor assist.

Correct answer: B It is anticipated that a client with a complete C7-C8 injury will be able to independently propel on all indoor surfaces and level outdoor terrain; some assist will be needed with uneven terrain based on the Consortium for Spinal Cord Medicine clinical practice guidelines. To prescribe a wheelchair appropriately, the physical therapist must access the individual's ability to propel a manual wheelchair and safely navigate a power wheel chair. Functionally relevant muscles innervated for C7-C8 include: latissimus dorsi; sternal pectoralis; triceps; pronator quadratus; extensor carpi ulnaris; flexor carpi radialis; flexor digitorum profundus and superficialis; extensor communis; pronator/flexor/extensor/abductor pollicis; and lumbricals (partially innervated). Movement possible for C7-C8 includes: elbow extension; ulnar/wrist extension; wrist flexion; finger flexions and extensions; and thumb flexion/extension/abduction. Answer A:It is anticipated that a client with a complete T1-T9 injury will be able to propel independently on all indoor and outdoor surfaces in a manual wheelchair. Answer C:It is anticipated that a client with a complete C6 injury will be able to propel independently on all indoor surfaces and some to total assist outdoors in a manual wheelchair. Answer D:It is anticipated that a client with a complete C5 injury will be able to propel independently to some assist indoors on non-carpet, level surface; some to total outdoor assist in a manual wheelchair.

A physical therapist is examining an 8-year-old boy in an outpatient sports physical therapy clinic who presented via direct access with an insidious onset of right hip/groin pain that began 1 week ago and progressively worsened. The patient has a noticeable limp but does not remember falling or hurting the leg. Passive range of motion (PROM) reveals a limitation in abduction and internal rotation on the right. Which of the following should the physical therapist do NEXT? A. Assess the joint play of the hip to determine if the capsule is restricted B. Refer the patient to an orthopedist C. Begin treatment to reduce pain with use of appropriate modalities D. Refer the patient to a therapist who specializes in pediatrics

Correct answer: B Legg-Calves-Perthes disease affects young children due to a lack of blood supply to the head of the femur causing avascular necrosis. Symptoms often are vague and can refer to the hip, thigh, and knee. The individual is often undersized, and has atrophy, weakness, and a noticeable limp. Boys are five times more likely to be affected. Immediate orthopedist follow-up is recommended to conserve as much of the dying bone as possible.The patient is describing symptoms of Legg-Calves-Perthes disease, where blood flow to the femoral head is disrupted, which can lead to necrosis. It is common in boys ages 4-8. The patient is not a candidate for physical therapy at this time and should see an orthopedist. Answer A:There is a lack of blood supply to the femoral head, and the bone may be necrotic. Moving the femur around can cause damage. Answer C:The patient is not a candidate for physical therapy at this time and should be referred to an orthopedist. Answer D:The patient is not a candidate for physical therapy at this time, even if the therapist is a certified specialist in pediatric PT. Bottom Line:Legg-Calves-Perthes disease disrupts blood flow to the femoral head which can lead to necrosis. Physical therapy is not appropriate and the patient should see an orthopedist.

A patient presents to the clinic through direct access with reports of severe low back pain. Observation reveals forward-head, flat-back posture with no muscle atrophy. Sensation is diminished along the lateral aspect of the lower leg and foot. Ankle reflex is diminished, and Babinski reflex is absent. What should the physical therapist do NEXT? a) have the patient go to the emergency room b) continue the examination and evaluate the myotomes c) refer the patient to a physician d) apply a modality that will provide some pain relief and postpone the rest of the examination for next session

Correct answer: B Lumbar radiculopathy presents with specific nerve root impairments, such as fatiguing weakness seen when testing myotomes, distal numbness within aspecific dermatome, negative red flags, and possibly low-level local pain. At times a patient's pain level does not correspond directly to the diagnosis and can include other biopsychosocial factors. Pain is not the only factor in determining if a physical therapist should refer a patient out or send a patient to the emergency department. In this case, it is appropriate to continue the examination and evaluate the myotomes. Answer A:An emergency room visit is not appropriate for someone with lumbar radiculopathy. Answer C:There are no red or yellow flags present. If multiple levels of nerve were affected, then referral to a physician would be the correct answer. Answer D:It is important to get a full picture of the findings with a thorough examination. It is inappropriate to begin treatment before completing the exam. Bottom Line:Lumbar radiculopathy presents with specific nerve root impairments, such as fatiguing weakness seen when testing myotomes, distal numbness within aspecific dermatome, negative red flags, and possibly low-level local pain.

A physical therapist is treating a patient who has a T12 ASIA C spinal cord injury in an acute rehabilitation facility. The physical therapist wants to perform locomotor training on a treadmill with this patient. The gait training technique will modify the amount of assistance provided in order to facilitate the patient's ability to perform aspects of gait as independently as possible. Which of the following options for gait training WOULD BE RECOMMENDED in this scenario? a) orthotic-assisted stepping with a harness b) manual-assisted stepping with a harness c) robot-assisted stepping d) stimulation-assisted stepping with a harness

Correct answer: B Manual-assisted stepping is the most common form of assistance provided during locomotor training on a treadmill for patients with spinal cord injury. One advantage of locomotor training using manual assistance is that the physical therapist may modify the amount of assistance provided on a step-by-step basis within the gait cycle in order to encourage the patient's ability to complete gait as independently as possible. Answer A:The use of orthotics while performing locomotor training on a treadmill has not been reported in the literature and may discourage appropriate sensory feedback during gait, which is hypothesized to be an important factor associated with the successful application of locomotor training. Answer C:Robot-assisted stepping uses technology to assist the a patient in performing a consistent gait pattern during training, which was not the priority of the physical therapist in this scenario. Answer D:Stimulation-assisted stepping uses electrical input to activate spinal reflexes that elicit a withdrawal reflex used for stepping. However, the use of withdrawal reflexes for stepping does not allow variability in the amount of assistance provided, which was the priority of the physical therapist in this scenario. Bottom Line:Physical therapists who work with patients with spinal cord injury need to be able to recommend specific technologies used to complete locomotor training in patients with spinal cord injury while knowing the advantages and disadvantages of each tool. Manual-assisted stepping is the most common form of assistance provided during locomotor training on a treadmill for patients with spinal cord injury. One advantage of locomotor training using manual assistance is that the physical therapists may modify the amount of assistance provided on a step by step basis within the gait cycle in order to encourage the patient's ability to complete gait as independently as possible.

A physical therapist in the hospital is performing a chart review of a 38-year-old female with a history of sudden-onset headache, photophobia, and neck stiffness. Magnetic resonance imaging was normal. A lumbar puncture found an elevated white blood cell count, elevated proteins, and increased cerebral spinal fluid pressure. What is the most likely diagnosis in this scenario? A. Guillain-Barré syndrome B. Meningitis C. Multiple sclerosis D. Hydrocephalous

Correct answer: B Meningitis is an infection that causes inflammation of the meningeal layer of the central nervous system. Meningitis may be caused by bacterial, viral, or fungal infection. The symptoms of meningitis include headache, photophobia (sensitivity to light), and neck stiffness. A lumbar puncture for a patient with meningitis will indicate an elevated white blood cell count, elevated proteins, and increased cerebral spinal fluid pressure. Answer A:Guillain-Barré syndrome is an autoimmune disease that causes demyelination of the peripheral nervous system. A lumbar puncture for a patient with Guillain-Barré syndrome will indicate increased protein levels but no increase in white blood cell count or cerebral spinal fluid pressure, so Guillain-Barré syndrome is an unlikely diagnosis in this scenario. Answer C:Multiple sclerosis is an autoimmune disease caused by demyelination of the central nervous system that typically affects females intheir 30s or 40s. Multiple sclerosis is often diagnosed via evidence of lesions to the central nervous system in two or more areas via magnetic resonance imaging. The finding of normal magnetic resonance imaging makes multiple sclerosis an unlikely diagnosis in this scenario. Answer D:Hydrocephalous is a condition caused by an increase in cerebral spinal fluid in the brain. Symptoms of hydrocephalous include headache, decreased balance, double vision, and decreased cognitive status. Hydrocephalous will cause an enlargement in the ventricles of the brain due to the increased intracranial pressure. Hydrocephalous may be caused by scarring associated with meningitis, and a lumbar puncture would indicate an elevated cerebrospinal pressure. However, a finding of normal magnetic resonance imaging along with elevated proteins and white blood cell counts would make hydrocephalous an unlikely diagnosis in this scenario. Bottom Line:Meningitis is an infection that causes inflammation of the meningeal layer of the central nervous system. The symptoms of meningitis include headache, photophobia, and neck stiffness. Physical therapists must be able to identify different diagnoses associated with neurological dysfunction and adapt their strategies for the treatment and examination according to the diagnosis.

Which of the following procedures describes the BEST method for evaluating a patient's middle and lower chest wall movements? a) both hands of the examiner firmly placed on the patient's shoulders b) both hands of the examiner placed on the lower ribs of the patient while the patient breathes quietly c) both hands of the examiner placed on the clavicles, one on each side while the patient breathes quietly d) one hand of the examiner on the patient's abdomen, and the other on the upper chest wall over the sternum while the patient breathes quietly

Correct answer: B Middle and lower chest wall movements are most evident in the lower ribs anteriorly, posteriorly, and laterally. Placement of hands anteriorly on the lower ribs assesses for middle chest wall movement, while placement posteriorly assesses for lower chest wall movement. Bilateral hand placement on the lower ribs allows the examiner to evaluate for the amount of movement, timing, and symmetry. Answer A:In cases of respiratory distress and with labored breathing, the patient may have excessive chest wall movements extending to shoulder movements. Nonetheless, placing hands on the shoulders is not the best way to evaluate breathing. Answer C:The position described is the proper method for evaluating movements of the upper chest wall. It can be used as part of a comprehensive evaluation. However, the examiner would not be able to discern movements in the middle and lower chest wall in this manner. Therefore, this answer is incorrect. Answer D:The procedure described is a good way of evaluating for a patient's breathing pattern in different areas of the chest wall. It allows the examiner to palpate differences in upper chest movements versus diaphragmatic movement. However, it does not directly assess middle and lower chest wall movements. Therefore, it is not the best answer and is thus incorrect.

A patient has painless weakness of the levator scapula and rhomboid muscles during manual muscle testing. Based on this information, which of the following nerves is MOST likely affected? A. Suprascapular B. Dorsal scapular C. Long thoracic D. Axillary

Correct answer: B Motor nerve injuries will not present with the characteristic sensory changes that are common with nerves that have cutaneous innervation. The clinical examination will reveal painless weakness upon muscle testing, and the pattern of weakness will be specific to the nerve's motor innervation, such as the dorsal scapular nerve.The patient described in the scenario is experiencing painless weakness, which is consistent with a nerve injury. The dorsal scapular nerve has contributionsfrom the C4 and C5 nerve roots, and it supplies the motor innervation for the levator scapula and rhomboid muscles. Since this nerve has no cutaneous sensory branch, the patient will not experience paresthesias such as pins, needles, or numbness. Answer A:The suprascapular nerve innervates the supraspinatus and infraspinatus muscles and provides sensory to the glenohumeral and acromioclavicular joints.An injury to this nerve would present with weakness in these muscles and pain along the sensory branch. Since this nerve has no cutaneous sensory branch, the patient will not experience paresthesias such as pins, needles, or numbness. Answer C:The long thoracic nerve innervates the serratus anterior muscle, and an injury to this muscle would present with weakness to this this muscle and scapular winging. Answer D:The axillary nerve innervates the deltoid and teres minor muscles and provides sensory to the glenohumeral joint and cutaneous innervation to the upper arm along the regimental badge area. An injury to this nerve would present with weakness to these muscles and sensory impairments such as pain, paresthesias,and decreased sensation along the cutaneous innervation. Bottom Line:Motor nerve injuries will not present with the characteristic sensory changes that are common with nerves that have cutaneous innervation. The clinical examination will reveal painless weakness upon muscle testing, and the pattern of weakness will be specific to the nerve's motor innervation.

A physical therapist is assessing running in a patient who complains of pain in the left knee. On examination, the therapist finds left hip abductor weakness and left "in toeing" during the running pattern. Which of the following would be the cause of these findings? a) adaptive shortening of the hip external rotators b) adaptive shortening of the ITB c) femoral retroversion d) weakness of the hip internal rotators

Correct answer: B Muscle weakness of the hip abductors and adaptive shortening of the iliotibial band leads to abnormal internal rotation and a "toe in" running pattern. Answer A:Adaptive shortening of the hip external rotators leads to abnormal external rotation and a "toe out" running pattern. Answer C:Femoral retroversion leads to abnormal external rotation and a "toe out" running pattern. Answer D:Weakness of the hip internal rotators leads to abnormal external rotation and a "toe out" running pattern.

A physical therapist is treating a patient in the ICU. Pre-treatment vitals signs were HR 90 BPM, O2 saturation 95%, respiratory rate 15 breaths per minute, and BP 140/90. The patient transferred to the bedside commode for the first time using a stand pivot transfer, mod assist of two. While sitting on the bedside commode, vital signs are now: HR 120 BPM, O2 saturation 90%, respiratory rate 20 breaths per minute, and BP 180/110. Which vital sign change indicates an abnormal response to exercise/mobilization? a) heart rate b) blood pressure c) respiratory rate d) oxygen saturation

Correct answer: B Normal blood pressure ranges are 85-140 mm Hg for systolic blood pressure and 40-90 mm Hg for diastolic blood pressure. Abnormal responses to exercise include systolic blood pressure increasing more than 20-30 mm Hg from resting level or decreasing more than 10 mm Hg below resting level. The patient in the scenario had a 40 mm Hg increase in systolic blood pressure, showing an abnormal response. Answer A:Normal range for resting heart rate is 50-100 BPM. An abnormal response would be heart rate increase of more than 20-30 bpm above the resting heart rate. Answer C:Normal respiratory rate is 12-20 breaths per minute. An abnormal response to exercise would be a patient becoming short of breath or an increase in the respiratory rate that the patient does not tolerate. Answer D:Oxygen saturation generally should be maintained >90%, or the physician should determine acceptable minimum level. Bottom Line:It is important for a physical therapist to monitor a patient's vital signs during mobilization in the acute care setting. Using parameters such as heart rate, respiratory rate, blood pressure, and oxygen saturation, you can objectively monitor how the patient is tolerating each activity, which can help guide the treatment session. Abnormal responses to exercise include systolic blood pressure increasing more than 20-30 mm Hg from resting level or decreasing more than 10 mm Hg below resting level.

A 68-year-old patient is admitted to the hospital with pneumonia and a long history of COPD. He is currently on 2 liters O2 via nasal cannula, which he normally does not use at home. His oxygen saturation is 98%. The physician asks the therapist to ambulate the patient on room air to see if he qualifies for home oxygen. During ambulation on room air, the patient's saturation drops to 91%. What is the BEST course of action the physical therapist should take? A. Notify physician that patient will require home oxygen B. Continue ambulating the patient and monitor saturation throughout treatment C. Return patient to his room immediately and apply 2 liters O2 D. Find a chair to allow the patient to sit and rest until saturation returns to upper 90s

Correct answer: B Normal range for oxygen saturation is 98% to 100%. Normal response to activity is for the O2 saturation to remain in the normal range. However, patients with chronic pulmonary dysfunction or congestive heart failure often desaturate with activity. Caution should be taken with patients who desaturate below 90% with activity. Exercise should not be continued if oxygen saturation drops to 88% or below.In this scenario, the patient's oxygen saturation is still at an acceptable level, so no immediate action is required and ambulation can continue with monitoring. Answer A:Typically O2saturation must drop to 88% on room air to qualify for home oxygen. Answer C:No immediate action is warranted as O2saturation level is still acceptable. When the normal treatment has concluded, the therapist should then return the O2to the patient. The therapist typically documents how long it takes the oxygen saturation to recover to pretreatment level after activity. Answer D:Finding a place for the patient to rest immediately is not indicated. The patient's saturation is still at an acceptable level to continue ambulating. Bottom Line:Normal response to activity is for the O2saturation to remain in the normal range. However, patients with chronic pulmonary dysfunction or congestive heart failure often desaturate with activity. Caution should be taken with patients who desaturate below 90% with activity. Exercise should not be continued if oxygen saturation drops to 88% or below.

A physical therapist is conducting an initial examination with a patient with a T12 complete spinal cord injury that occurred 1 week ago due to a fall, which resulted in no additional injuries. Which of the following muscle groups should be the focus of therapeutic exercise to facilitate independent lateral transfers by the time of discharge from the acute rehabilitation facility in approximately 4 weeks? a) biceps b) triceps c) pectoralis major d) serratus anterior

Correct answer: B Of the options above, the triceps are the key muscle group to facilitate independence in lateral transfers in patients with complete thoracic spinal cord injuries since they lift the buttocks off the sitting surface along with the latissimus during the transfer. Answer A:While the biceps are used in many functional activities, it is not the key muscle used when completing lateral transfers. Answer C:While the pectoralis major is used to stabilize the shoulder girdle, it is not the key muscle used when completing lateral transfers. Answer D:While the serratus anterior is used to stabilize the shoulder girdle, it is not the key muscle used when completing lateral transfers. Bottom Line:The triceps and latissimus facilitate independence in lateral transfers in patients with complete thoracic spinal cord injuries since they lift the buttocks off the sitting surface during the transfer. The physical therapist must base the initial examination on the level and completeness of injury, patient age, goals, and comorbid factors. These will determine the muscle group that should be the focus of therapeutic interventions when treating a patient with a spinal cord injury.

A physical therapist is treating a patient 3 weeks after right ACL reconstruction and partial medial menisectomy. PROM is 0-130. The patient also has had some pain and swelling. Which of the following interventions is MOST appropriate for quadriceps strengthening? A. Open chain quad sets at 20 degrees of knee flexion B. Open chain quad sets at 60 degrees of knee flexion C. Bilateral squats to 90 degrees only D. Electric stimulation to the quadriceps to create a contraction 10 sec on 10 sec rest x 15 minutes

Correct answer: B Open chain quad sets at 60 degrees of knee flexion is the correct answer as this would help to strengthen the quadriceps in a range that is safe for the ACL graft. Answer A:This would help strengthen the quadriceps but is an unsafe range for the ACL graft as it applies an anterior shear to the tibia. Answer C:Squats strengthen the quadriceps, but due to the concomitant menisectomy, the range should be limited to about 60 degrees at this time 3 weeks post op. Answer D:The patient can already contract the quadriceps and can better strengthen by exercise than by electric stimulation alone. Electric stimulation during therapeutic exercise is also effective, but just passively applying electric stimulation with no active strengthening is not as effective. Bottom Line:ACL protocols differ on type of graft as well as concomitant injuries and repairs. A patient with an ACL repair can safely contract the quadriceps in the 40-90 degree range very early on. A patient status post menisectomy typically should avoid flexion greater than 60 degrees early on.

A 14-year-old boy presents at outpatient physical therapy with pain over the right tibial tubercle. He reports playing a lot of basketball and that the pain is usually worse right after the game. The patient has a history of right chronic shoulder dislocations and a mid shaft fracture of the right tibia at age 5. What should the physical therapist suspect? a) growth plate complication from previous fracture b) Osgood-Schlatter disease c) systemic issue affecting joints on the right side d) stress fracture from repetitive loading

Correct answer: B Osgood-Schlatter disease is apophysitis of the tibial tubercle, where the ligamentum patella inserts onto the tibial tuberosity. The condition occurs during a growth spurt and with activities that demand a lot of quadriceps activity, such as running and jumping. The patient's age of 14, location of pain at the tibial tubercle, and description of activities that provoke the pain all point to Osgood-Schlatter disease. Answer A:The fracture was 9 years ago and is fully healed. If it were not healed, the patient would be unable to bear weight. In addition, the fracture was midshaft, and that is not near a growth plate location. Answer C:There is no systemic disease that would affect only the right side. In addition, shoulder dislocations may indicate some sort of laxity in the patient's connective tissue; however, it is unrelated to the apophysitis at the tibial tubercle also known as Osgood-Schlatter disease. Answer D:Repetitive loading can cause a stress fracture. In this case, however, the patient's history, location of pain, and age all point to Osgood-Schlatter disease. Bottom Line:Osgood-Schlatter diseasehas a very typical presentation. The ligamentum patella attaches there and can pull with excessive quad activity, especially during a growth spurt when the patient is still skeletally immature. The result is apophysitis at the tibial tubercle. Extreme cases can result in avulsion fractures.

The physical therapist is completing a physical examination in a patient with a suspected visceral disorder. The therapist screens the abdomen for tendernessand the presence of visceral organs. WHICH COMPONENT of the physical exam of the abdomen is being completed? A. Percussion B. Palpation C. Auscultation D. Inspection

Correct answer: B Palpation in the abdominal region is usually deep palpation (firm pressure) and is used to assess for tenderness and the presence of visceral organs. Answer A:Percussion is used to determine size, shape, and density of tissue using sound. Answer C:Auscultation is performed to listen for bowel sounds, which are normally made by the movement of food, liquid, and air through the intestine. Answer D:Inspection is the process of visually looking at the body for symmetry, alignment, skin color, and scars.

The physical therapist is completing a physical examination in a patient with a suspected visceral disorder. The therapist uses sound to screen the abdomen to determine size, shape, and density of tissue. Which component of the physical exam of the abdomen is being completed? a) auscultation b) percussion c) inspection d) palpation

Correct answer: B Percussion is used to determine size, shape, and density of tissue using sound. Answer A:Auscultation is performed to listen for bowel sounds, which are normally made by the movement of food, liquid, and air through the intestines. Answer C:Inspection is the process of visually looking at the body for symmetry, alignment, skin color, and scars. Answer D:Palpation in the abdominal region is usually deep palpation (firm pressure) and is used to assess for tenderness and the presence of visceral organs. Bottom Line:Percussion is the use of sound to determine size, shape, and density of tissue. TrueLearn Insight :Percussion is the use of sound to determine size, shape, and density of tissue. Indirect percussion is performed using the examiner's fingers to tap directly on the surface of the body. Indirect percussion requires the therapist to position the middle finger of the non-dominant hand against the patient's skin and strike above and below the IP joint of the of the third digit with the dominant hand.

A patient fell while hiking in a rural area and sustained a posterior dislocation of the glenohumeral joint, which was reduced at a local hospital. After resting for a few days at home, she is coming to see the physical therapist because she noticed she is getting progressively weaker. Manual muscle test (MMT) results include the following: 5/5 elbow and wrist flexion, 4/5 shoulder external rotation, 3/5 shoulder flexion, abduction, and extension; and 1/5 elbow, wrist, and finger extension. Based on the MMT results, which of the following structures does the physical therapist suspect has MOST LIKELY been injured? a) axillary nerve b) posterior cord c) radial nerve d) supraspinous nerve

Correct answer: B Posterior cord is correct as the patient is demonstrating weakness in the muscles innervated by both the axillary and radial nerves. Therefore, the level of injury has to be superior to where they begin, at the posterior cord. The axillary nerve supplies teres minor and deltoid, and in this patient weakness in flexion and abduction from the deltoid and some ER weakness from teres minor are seen. (Other, unaffected muscles contribute to those motions, and that may be why it is not a 1/5 like the forearm muscles). The radial nerve innervates the triceps and the extensor muscles in the posterior compartment of the forearm, and those are also very weak. Answer A:This is incorrect as it would not explain the extensor weakness. Answer C:This answer is incorrect as it would explain only the extensor paralysis. Answer D:This answer is incorrect as it would explain only the abduction weakness. Bottom Line:The posterior cord supplies the radial and axillary nerves. The axillary nerve supplies teres minor and deltoid, which will affect flexion, abduction, and external rotation. The radial nerve affects the extensors of the elbow, wrist, and fingers. If these two nerves are simultaneously affected, the site of the injury is most likely more proximal at the posterior cord.

A physical therapist is completing an initial examination of a patient who was hospitalized due to an acute exacerbation of rheumatoid arthritis and intolerable pain. Past medical history includes depression, type 2 diabetes, and hypertension. While in acute care, the patient's blood sugar has been extremely high, and the patient was started on insulin as a result. Which of the following medications that the patient is currently prescribed is the MOST likely cause of her unstable blood sugar levels? A. Glucophage B. Prednisone C. Prozac D. Tramadol

Correct answer: B Prednisone is a medication used to decrease inflammation in patients with acute rheumatoid arthritis. One common adverse effect of prednisone is elevated blood sugar.Physical therapists must be familiar with the medications that are commonly used to treat specific diseases so that they can recognize the primary and adverse effects of these medications in their patients. Answer A:Glucophage is a medication used to control blood sugar in patients with type 2 diabetes, so it would not be the cause of elevated blood sugar levels. Answer C:Prozac is a medication used to treat depression; elevated blood sugar is not one of its common side effects. Answer D:Tramadol is an opiate medication used to treat pain, and elevated blood sugar is not one of its common side effects.

A physical therapist uses transcutaneous electrical nerve stimulation (TENS) to assist in controlling pain in a patient rehabilitating after total knee arthroplasty. Which of the following PARAMETERS is appropriate for the goal of controlling pain? a) duty cycle of 1:5 b) pulse frequency of 10 pulses per second c) stimulation intensity to maximum motor d) pulse duration of 500 microseconds

Correct answer: B Pulse frequencies of 1-10 pulses per second applied with an intensity to cause mild motor stimulation can be used for endorphin-level pain control using TENS. Answer A:Duty cycle is not needed for electrical stimulation for pain control. Answer C:Stimulation intensity to maximal motor in not indicated for pain control. Answer D:Pulse duration of 100-300 microseconds can be used for electrical stimulation for pain control.

Which of the following intervention strategies would be MOST helpful for a patient with emphysema with airway collapse? A. Teach coughing techniques B. Teach pursed-lip breathing C. Perform postural drainage D. Administer chest percussion

Correct answer: B Pursed-lip breathing keeps weakened bronchiole airways open by creating positive pressure during expiration. Answer A:Teaching proper coughing technique is an important intervention for an acutely ill patient with excessive bronchial secretions. Emphysema patients often do not have signficant secretions and have a minimal or absent cough. Answer C:Postural drainage is the assumption of one or more body positions that allow gravity to assist with draining secretions from each of the patient's lung segments. This is typically not used with emphysema patients as secretions are not as common with emphysema as they are with chronic bronchitis. Answer D:Chest percussion is also aimed at loosening retained secretions, which would not be helpful with this particular patient. Bottom Line:Emphysema is a condition of the lung characterized by destruction of the alveolar walls and enlargement of the air spaces distal to the terminal bronchioles. Clinical manifestations include use of accessory muscle breathing, minimal or absent cough, leaning forward to breathe, and dyspnea on exertion. Pursed-lip breathing is used to decrease a patient's symptoms of dyspnea. Pursed-lip breathing keeps weakened bronchiole airways open by creating positive pressure during expiration.

A rehabilitation department has decided they want to perform some research into patient experience. The department has agreed to utilize a qualitative approach to their research. Which of the following methods is most appropriate for this type of research? A. Closed-ended questions B. Open-ended questions C. Performance data D. Statistical analysis

Correct answer: B Qualitative research is work that is performed to explore or understand the meaning of individuals and their experiences. To achieve this understanding, open-ended questions are often used in surveys or interviews that allow the individuals to describe their experiences. Answer A:Closed-ended questions, such as those that ask for a yes/no answer or a rating of 1 -5, are used in quantitative research as they provide data appropriate for statistical analysis. They are not used in qualitative work. Answer C:Performance data produces data appropriate for statistical analysis and is not used in qualitative work. Answer D:Statistical analysis does not occur in qualitative work, which explores themes or trends from individual interviews or open-ended surveys. Statistical analysis is part of quantitative work. Bottom Line:Qualitative research explores people's experiences and uses open-ended questions and observations, from which themes emerge and are interpreted. There is no numerical data or statistical analysis involved in qualitative research.

A patient's pulmonary function test (PFT) shows FEV 1 = 52% pre albuterol, and 54% post albuterol administration, and the flow-volume loop shows residual volume = 168% and total lung capacity = 117%. Which of the following is the MOST appropriate interpretation of the physical therapist? a) this patient's findings are within normal limits b) this patient has findings consistent with chronic obstructive pulmonary disease c) this patient has a strong asthmatic component to their pulmonary function d) this patient has findings consistent with chronic restrictive pulmonary disease

Correct answer: B Residual volume measures the amount of air that remains in the lungs after a maximum expiration. With chronic obstructive pulmonary disease (COPD), air gets trapped in the lungs, and so residual volume increases. A residual volume that hovers near 100% is normal. This patient's residual volume is 168%. In addition, 75% of the forced vital capacity (FVC), or amount of air expelled in a forceful expiration, should be expelled within 1 second. So a normal FEV1 is about 75%. Here it is less than that, indicating airway obstruction. Answer A:Normal residual volume is close to 100%. Residual volume above 100% may indicate obstruction, with air trapped in the lungs. In addition, the FEV1 percentage, which is the amount of air that can be expelled in 1 second expressed as a percentage of the FVC or total amount of air a person can expel, it is only about 50%, indicating airway obstruction. Answer C:The patient's FEV1, which is the amount they can expel in 1 second, did not change much from the pre albuterol to the post albuterol percentage. If there were an asthmatic component, the FEV1 would have increased a great deal after albuterol, a bronchodilator, was administered. Answer D:In a patient with a restrictive disease, the total lung capacity and the residual volume would be decreased and below normal because of the patient's difficulty inhaling. In this case, the residual volume is much higher than 100% and air is trapped inside, and the total lung capacity is also over 100%.

If a baby continues to demonstrate the asymmetric tonic neck reflex (ATNR) after 6 months of age, which of the following activities would be MOST affected? a) prone lying b) rolling c) suckling d) cervical rotation

Correct answer: B Rolling involves rotating the neck. If the ATNR is retained, then as the child turns the head, the upper extremity (UE) on the side of the head rotation will move into extension, and the other arm will flex. These UE positions make it difficult to roll. Answer A:Even with an ATNR reflex, a baby can maintain the prone position. Answer C:Sucking is not affected by the ATNR reflex. Answer D:Cervical rotation is the motion that causes the extension of one arm and flexion of the other arm in the ATNR reflex. However, the ability to turn the head as well as the cervical range of motion is the same whether the child still retains the ATNR or not. Bottom Line:The asymmetric tonic neck reflex (ATNR) occurs with rotation of the head and causes extension of the upper extremity on the face side and flexion of the upperextremity on the contralateral side. This reflex is usually integrated by 6 months of age, and if retained can make some ADLs, such as rolling or feeding oneself, difficult.

A 32-year-old patient reports insidious shortness of breath, fatigue, and generalized joint pain over the last several months. Chest x-ray shows bilateral hilar lymphadenopathy and pulmonary infiltrates. The patient also notes a new onset of a reddish-purple rash around his ankles. What might the practitioner suspect as the cause? a) bronchiolitis obliterans b) sarcoidosis c) scleroderma d) tuberculosis

Correct answer: B Sarcoidosis is an idiopathic granulomatous inflammatory disorder that affects many organ systems, including lungs, heart, skin, and eyes. The pulmonary system is most often involved (90%). Diagnosis is a matter of exclusion; however, the disease presents with three distinctive features: alveolitis, formation of well-defined round granulomas on the skin, and pulmonary fibrosis. Disease progression is variable between patients. Answer A:Bronchiolitis obliterans is a fibrotic lung disease that affects smaller airways. It is characterized by necrosis of the respiratory epithelium in the affected bronchioles. Chest x-ray typically shows pulmonary edema and bilateral patchy alveolar infiltrates. Answer C:Scleroderma is an autoimmune progressive fibrosing disorder that affects the body by hardening connective tissue. Degenerative changes occur in the skin, small blood vessels, esophagus, intestinal tract, lung, heart, and kidneys. It is four times more common in females and affects ages 35-50 most often. The tissues of involved organs become hard and fibrous, causing them to function less efficiently. Within the lung, scleroderma appears as progressive diffuse interstitial fibrosis. Symptoms include dyspnea on exertion, a nonproductive cough, and clubbing of the digits. Answer D:Tuberculosis is an infectious disease characterized by chronic cough with blood containing sputum, fever, night sweats, and weight loss.

A 6-year-old child with hemiplegic cerebral palsy presents with a plantarflexion contracture of 10 degrees due to spasticity. The contracture causes the child to often trip while walking. The child has no other muscle groups that exhibit spasticity that impairs function. The parent has been reading on the internet about treatment options and asks for the physical therapist's input. Which of the following interventions is the MOST appropriate next step? a) electrical stimulation to the plantarflexors b) serial casting of the plantarflexors c) surgical lengthening of the plantarflexors d) oral baclofen

Correct answer: B Serial casting of the plantarflexors would be indicated in this case to improve passive range of motion of the ankle. The contracture is limited to 10 degrees, so that a limited number of cast changes would be necessary to improve the passive range of motion. Serial casting with or without Botox is often used to decrease contractures of the plantarflexors in children with hemiplegic cerebral palsy. Answer A:The effectiveness of electrical stimulation to improve spasticity is limited. In addition, electrical stimulation may not be tolerated in this patient due to their young age. Answer C:Surgical lengthening of the plantarflexors is unlikely to be considered until less invasive methods of contracture management are attempted since the contracture is only 10 degrees. Answer D:Oral baclofen is unlikely to be recommended in this scenario since the spasticity is limited to a single muscle group. In addition, oral baclofen often causes sedation, which would interfere with the child's participation in school and play activities. Bottom Line:Serial casting with or without botox is often used to decrease contractures of the plantarflexors in children with hemiplegic cerebral palsy. Physical therapists who treat children with cerebral palsy whose primary impairment is spasticity must be able to recognize and explain to parents the common physical therapy, surgical, and pharmacological interventions used for its treatment.

A physical therapist is conducting an initial evaluation of a patient who is 1 day post cerebrovascular accident. The therapist asks the patient to close their eyes and using their affected hand, pick out and identify an object from a box containing several small commonly used items, including a paperclip, pencil, coin, and small comb. What sensory function is the therapist assessing? A. Pressure perception B. Stereognosis perception C. Kinesthesia awareness D. Tactile localization

Correct answer: B Stereognosis is the ability to recognize a familiar object through touch alone. In the test described, the therapist requires the patient to find and identify a commonly used object such as a paperclip without visual input. This test would be the most common method used to test stereognosis perception. Answer A:Pressure perception is the ability of the patient to identify pressure applied directly to their skin, with sufficient depth for the therapist to be testing the deep receptors. Answer C:Kinesthesia awareness is the ability to detect movement and would be tested by the therapist moving the patient's joints through small movements, which the patient has to identify. Answer D:Tactile localization is the ability of the patient to identify the location on their body they are being touched; it is not their ability to identify an item through touch. Bottom Line:Different sensory functions must all be tested during a neurological evaluation. Stereognosis is the ability to identify a common item through touch alone without visual input. In the question, asking a patient to pick an item out of a box and identify it would be testing stereognosis.

A physical therapist is completing an initial examination of a patient with an acute C5 American Spinal Injury Association (ASIA) C spinal cord injury. The patient has deep anal pressure. To be classified as an ASIA C spinal cord injury, which of the following functions must be present? A. voluntary muscle contraction of the wrist extensors B. voluntary muscle contraction of the finger adductors C. light touch sensation in the T1 dermatome D. bladder control

Correct answer: B The American Spinal Injury Association (ASIA) Impairment Scale (AIS) is used to classify the completeness of injury in individuals with spinal cord injury. To be classified as having an AIS C injury, a person would have motor function preserved at the most caudal sacral segments for voluntary anal contraction (VAC). A patient would also be classified as having an ASIA C injury if the patient met the criteria for sensory incomplete status (sensory function preserved at the most caudal sacral segments [S4-S5] by light touch, pin prick, or deep anal pressure), and had sparing of motor function more than 3 levels below the ipsilateral motor level on either side of the body.Since the finger adductors are the key muscle group for T1 and are more than 3 levels below the neurological level of injury, voluntary muscle contraction of the finger adductors would allow this patient to be classified with an ASIA C spinal cord injury. Answer A:Since the wrist extensors are the key muscle group for C6 and are within 3 motor levels from the neurological level, voluntary muscle contraction of the wrist extensors is not necessary to classify a patient with an ASIA C spinal cord injury. Answer C:Although light touch sensation in the T1 dermatome might be present in patients with an ASIA C spinal cord injury, it is not a required function for its classification. Answer D:Although bladder control might be present in patients with an ASIA C spinal cord injury, it is not a required function for its classification. Bottom Line:To be classified as having an American Spinal Injury Association (ASIA) Impairment Scale (AIS) injury, a person would have motor function preserved at the most caudal sacral segments for voluntary anal contraction (VAC). A patient would also be classified as having an ASIA C injury if the patient met the criteria for sensory incomplete status (sensory function preserved at the most caudal sacral segments [S4-S5] by light touch, pin prick, or deep anal pressure), and had sparing of motor function more than three levels below the ipsilateral motor level on either side of the body. TrueLearn Insight :The determination of an accurate American Spinal Injury Association (ASIA) Impairment Scale (AIS) classification is critical to predict the likelihood of neurological recovery for a person with a spinal cord injury. Physical therapists must be able to use the AIS classification as they plan their examination and intervention strategy for their patients with spinal cord injury. In addition, physical therapists must be able to explain an AIS classification to patients and their families so that they better understand the patient's prognosis for recovery.

An 85-year-old patient has just moved into a skilled nursing facility (SNF) due to deconditioning. Which of the following outcome measures is MOST effective for the physical therapist to use to establish a baseline of general functional ability and level of independence with activities of daily living. a) Berg Balance Scale b) Barthel Index c) Fugl-Meyer Assessment d) Timed Up and Go (TUG)

Correct answer: B The Barthel Index is a general measure of functional ability and activities of daily living (ADLs) that measures the degree of assistance required on 10 items, including feeding, ambulation, stair climbing, and toileting. The Barthel Index was developed in 1965 and was one of the first measures of functional mobility and ADLs. Answer A:The Berg Balance Scale is an assessment of balance that is used in many patient populations. Since this test does not include measurement of ADL performance and is specifically designed to measure balance only, it would not be the best choice in this scenario. Answer C:The Fugl-Meyer Assessment measures motor function, sensation, and balance, and was specifically designed for patients who have had a stroke. Answer D:The TUG is an assessment of balance and functional mobility and is frequently used to screen elderly patients for their fall risk. Since this test does not include measurement of ADL performance, it would not be the best choice in this scenario.

Which of the following types of exudate from the Bates Jensen Wound Assessment Tool is the unhealthiest type of exudate listed? a) bloody b) purulent c) serous d) serosanguineous

Correct answer: B The Bates Jensen Wound Assessment Tool (BWAT) is used to grade a wound initially and then at regular intervals to track the progress and healing. The location and shape are documented, and then 13 other items are rated 1 to 5, with 1 being the healthiest and 5 having the most unhealthy attributes. In terms of exudate, purulent is a 4/5, and foul purulent is a 5/5, which can be thick, opaque, tan/yellow, and with foul odor or without odor. Answer A:Bloody is thin, red, and a 1/5. Answer C:Serous is thin, clear, watery, and a 3/5. Answer D: Serosanguineous is thin, pink, watery, and a 2/5. Bottom Line:The Bates Jensen Wound Assessment Tool (BWAT) is used to grade a wound initially and then at regular intervals to track the progress and healing. The location and shape are documented, and 13 other items are rated 1 to 5, with 1 being the healthiest and 5 having the most unhealthy attributes. Then the total is plotted on a continuum and tracked. Some examples of categories are the type of exudate, the amount of exudate, the type of necrotic tissue, the amount of necrotic tissue, the depth, and the granulation tissue. In the scenario described in this question, purulent is the unhealthiest type of exudate.

A physical therapist at a home care visit is examining a 75-year-old patient status post right cerebrovascular accident 6 months ago. The patient scores a 40/56 on the Berg Balance Scale. The patient ambulates with a front-wheeled walker independently for short distances inside the house and uses a wheelchair when outside the home. Which of the following short-term goals would be the MOST appropriate for this patient to try to achieve in the next 2 weeks? a) patient will ambulate independently with a quad cane in the left hand for short distances in the home to improve mobility and allow access to all rooms in the house b) patient will ambulate with a rolling walker for short distances outside with close contact guard to encourage safe ambulation in the community c) patient will ambulate independently inside the home for short distances with use of a rolling walker to prevent fall risk d) patient will ambulate with a quad cane in the left hand with minimal assist for short distances outside to encourage safe ambulation in the community

Correct answer: B The Berg balance scale uses predetermined tasks to objectively determine a patient's balance. Interpreting the results of the Berg Balance Scale will indicate the proper progression of gait and balance training and alert the physical therapist to fall risk. A score of 40/56 indicates a medium fall risk, and a score below 40/56 would also indicate a significant increase in the risk of multiple falls. Research linking assistive devices to Berg scores indicates that the cutoff score for use of a front-wheeled walker is 43/56.Based on all of this information, when setting the short term goals for this home care patient, it would not be wise to prioritize progressing the patient to a device with less support. A patient who is independent inside the home with a rolling walker but uses only a wheelchair outside, would benefit from physical therapy to progress the patient to being able to ambulate outside with a rolling walker to encourage safe ambulation in the community. Answer A:A score of 40/56 indicates a medium fall risk, and a score below 40/56 would also indicate a significant increase in the risk of multiple falls. This patient is right on that border. Research linking assistive devices to Berg scores indicates that the cutoff score for use of a front-wheeled walker is 43/56. Based on all of this information, it would not be wise to prioritize progressing the patient to a device with less support, like a quad cane, in the next 2 weeks, as the risk of falls is great. In addition, expecting the patient to be independent with the quad cane in 2 weeks is unrealistic. Answer C:The patient is already able to ambulate independently for short distances using a rolling walker. The short-term goal to be achieved within the next 2 weeks should indicate that the patient is progressing and must not aim to achieve a status that has already been achieved. Physical therapy care must be effective. Answer D:Progressing to a quad cane, which provides less stability than a walker, is not advisable for this patient, especially outside, because the patient is not yet able to ambulate with a rolling walker outside. A score of 40/56 indicates a medium fall risk.

A physical therapist is reading a stress test report. A 55-year-old male patient achieved stage III of the Bruce protocol. ECG showed one box of S-T depression at peak exercise, occasional isolated premature ventricular contractions (PVCs,) and a max HR of 140. Which of the following is the MOST accurate interpretation of the test results? a) the patient became tachycardic during the test b) the patient was on the treadmill for 9 minutes c) the patient achieved a MET level of three d) the patient requires emergent catheterization

Correct answer: B The Bruce protocol stages are 3 minutes long, and the patient at stage III would have been on the treadmill for 9 minutes. Answer A:Tachycardia is a heart rate (HR) over 100 at rest. The patient's workload increased gradually to stage III of the Bruce protocol, which is a treadmill speed of 3.4 MPH of and an incline of 14%. An HR of 140 is appropriate for the demand placed on the patient during exercise. Answer C:A metabolic equivalent of task (MET) level of 3 is a lower MET level, such as with normal walking speed at a zero grade. It is a goal for patients in stage I of cardiac rehabilitation. This patient made it to stage three, which is 3.4 MPH and 14% incline, for a MET level of about 8-10. Answer D:The ECG showed ST depression and only for one box, so there may be a small amount of reversible ischemia. If there was an ST elevation, then the patient would need to go to the catheterization laboratory immediately. Bottom Line:The Bruce protocol is one of the most widely used methods for assessing cardiovascular health and diagnosing ischemic heart disease. It is a standardized treadmill test with stages where the speed and the incline are increased every 3 minutes. The test is stopped if the patient requests it or if there are abnormal ECG or BP responses. Patients commonly start exercising on a treadmill set at 1.7 miles per hour and a 10% grade (stage I) and could increase to a maximum speed of 6.0 miles per hour and a 22% grade. (stage VII). In this scenario, the patient was on the treadmill for 9 minutes.

A patient is being seen at an outpatient clinic 8 weeks postoperatively for a flexor digitorum superficialis (FDS) reconstruction of the third digit. The physical therapist has been cleared to isolate the FDS for strengthening. Where is the BEST position for the physical therapist to stabilize in digit 3, while keeping digits 2, 4, and 5 in full extension, in order to isolate the flexor digitorum superficialis? A. Stabilize middle phalanx laterally to allow for distal interphalangeal (DIP) flexion B. Stabilize proximal phalanx laterally to allow for proximal interphalangeal (PIP) flexion C. Stabilize proximal phalanx laterally to allow for distal interphalangeal (DIP) flexion D. Stabilize middle phalanx laterally to allow for proximal interphalangeal (PIP) flexion

Correct answer: B The FDS tendon inserts on the base of the middle phalanx and when it shortens it flexes the PIP. Flexor digitorum profundus (FDP) could technically assist as a secondary flexor because it also crosses the PIP joint. However, because the ulnar 3 digits of the FDP share a common muscle belly, if the other fingers are restrained in extension, FDP is not permitted to shorten and so, only the FDS can flex the PIP. Answer A:Stabilize middle phalanx laterally to allow for distal interphalangeal (DIP) flexion is the position to isolate the flexor digitorum profundus. FDS attaches at the middle phalanx and cannot flex the DIP joint. Answer C:FDS attaches at the middle phalanx and cannot flex the DIP joint. In addition, if flexing the DIP were the desired action, the middle phalanx should be stabilized. Answer D:Stabilizing the middle phalanx would block the PIP from being able to move into flexion. Bottom Line:The FDS crosses the palmar side of the wrist, MCP, and PIP and attaches to the middle phalanx. It can therefore flex the wrist and MCP as a secondary flexor and the PIP as a primary flexor. It cannot move the distal phalanx (that is the job of the flexor digitorum profundus).

A physical therapist is examining a patient with a history of incomplete spinal injury. The patient complains of stiffness in their bilateral plantarflexors and hip adductors that limits their ability to walk. Which of the following assessment tools would be MOST appropriate in this scenario? a) Hoffman reflex b) Modified Ashworth Scale c) Pendulum test d) Penn spasm frequency scale

Correct answer: B The Modified Ashworth Scale (MAS) would be an appropriate assessment tool in this scenario since it is a biomechanical assessment of muscle tone and stretch reflexes. The MAS is an ordinal scale in which the evaluator assigns grades based on the amount of resistance and where the resistance occurs in the range of motion. The MAS can be used in any muscle group, which is an advantage of the test in this scenario. Answer A:The Hoffman reflex is an electrophysiological test of the monosynaptic stretch reflex, which would not be appropriate in this scenario because it is used to assess spasticity of the hip adductors. In addition, the technical skills and equipment required for Hoffman reflex testing limits its clinical applicability. Answer C:The pendulum test is a biomechanical measure of spasticity, which would not be appropriate in this scenario because it can be used only to measure spasticity of the knee extensors. Answer D:The Penn spasm frequency scale, which scores spasticity based on the frequency of muscle spasms per hour, would not be appropriate in this scenario because the patient does not report muscle spasms.

A physical therapist is planning a research study investigating the effect of plantar flexor strength training on balance, gait speed and muscle tone in adults with a history of chronic stroke. Muscle tone will be assessed using the Modified Ashworth scale, which grades muscle tone from a 0, defined as no increase in muscle tone, to a 4, defined as rigid. The Modified Ashworth scale is an example of what type of data? a) interval b) ordinal c) nominal d) ratio

Correct answer: B The Modified Ashworth scale is a measure that ranks muscle tone on an ordinal scale of 0 to 4 in which higher numbers indicate greater muscle tone. Though one can know that each successive level is greater than another, the distance between levels is not defined. The order of the values is the important thing. Answer A:The Modified Ashworth scale is not an interval level measure since the numbers do not indicate equal distances between the units of measurement. Answer C:The Modified Ashworth scale is ordinal, not nominal data. Nominal data is more like a label and has no numerical significance, such as being male or female. Answer D:The Modified Ashworth scale is not a ratio level measure since the numbers do not indicate equal distances between the units of measurement and there is no true zero. An example of ratio data is weight. Bottom Line: Ordinal data measures non-numeric items such as level of satisfaction, degree of importance, or amount of muscle tone. One can see that there is more of the item being measured as the numbers go up, but there is no clear definition of the distance between each level. Only certain statistical tests are appropriate to use with ordinal data.

A 58-year-old male presents to the outpatient clinic with low back/hip pain x 4 weeks. He reports gradual worsening of symptoms with radiation into his right lower extremity. He states that symptoms are worse with sitting for prolonged periods, which makes his hour commute to work very difficult. Any bending or lifting also exacerbates the symptoms. Pain wakes him up at night several times. Which of the following functional outcome measures would be MOST appropriate to use with this patient? A. Hendler 10-Minute Screening Test for Chronic Back Pain Patients B. Oswestry Disability Index C. SF-36 Health Survey D. Lower Extremity Functional Scale

Correct answer: B The Oswestry Disability Index is the most specific and best choice for this particular patient. The Oswestry Disability Index includes rating activities such as lifting, sitting, sleeping, and traveling, which are all mentioned by this particular patient in his history. Answer A:The Hendler test is used for low back-pain patients and helps differentiate organic from functional low back pain. This test is used for patients with chronic back pain. The patient in this scenario has had pain only x 4 weeks; therefore would not be considered chronic. Answer C:The SF-36 is a more generalized health questionnaire that can be used with a variety of diagnoses. Although it provides good information about the patient's perceived physical status, it is not the most specific to this patient's impairments. Answer D:The Lower Extremity Functional Scale is used primarily for hip, knee, and ankle pathologies with questions geared toward ADLs that would be impaired with a lower extremity injury. Bottom Line:Injury to the lumbar spine can greatly affect the patient's ability to function. Numerical scoring tables may be used to determine the degree of pain caused by the lumbar spine pathology. Care must be taken when selecting the proper functional outcome measure. The Oswestry Disability Index is most commonly used and is a good functional scale because it deals with the ADLs and therefore is based on the patient's response and concerns affecting daily life.

As part of the evaluation of a new patient with risk factors for peripheral arterial disease (PAD), the physical therapist would like to assess the patient's ankle-brachial index (ABI). Which of the following BEST describes the proper procedure for performing this test? A. Patient is positioned in supine. With a Doppler, blood pressure is taken at either arm and then in both ankles. The ABI for each leg is calculated by dividing the systolic blood pressure obtained from either arm by the systolic blood pressure obtained from that ankle. B. Patient is positioned in supine. With a Doppler, blood pressure is taken at both arms and then in both ankles. The ABI for each leg is calculated by dividing the systolic blood pressure obtained from that ankle by the highest systolic blood pressure obtained from either arm. C. Patient is positioned in upright sitting with legs dangling. With a Doppler, blood pressure is taken at both arms and then in both ankles. The ABI for each leg is calculated by dividing the the highest systolic blood pressure obtained from either arm by the systolic blood pressure obtained from that ankle. D. Patient is positioned in upright sitting with legs dangling. With a Doppler, blood pressure is taken at both arms and then in both ankles. The ABI for each leg is calculated by dividing the systolic pressure obtained from that ankle by the highest systolic blood pressure obtained from either arm.

Correct answer: B The ankle-brachial index (ABI) is a simple and non-invasive screening tool for peripheral arterial disease (PAD). The patient should be allowed to rest in a supine position for about 10 minutes to allow blood pressure to stabilize. All blood pressure measurements should be done in a supine position. The ABI is calculated by dividing the systolic blood pressure obtained from the ankle by the higher brachial systolic blood pressure obtained from either arm.This answer correctly describes the proper procedure for measuring the ABI. The supine position is used so that the extremities remain at the level of the heart. False readings would result during testing if any extremity were positioned too high or too low.

An individual had a hemorrhagic stroke and presents with minimal left-sided weakness of the extremities with the lower extremity more involved than the upper extremity, gait apraxia, and behavioral changes. The physical therapist should suspect a lesion primarily in which of the following locations? A. Left anterior cerebral artery B. Right anterior cerebral artery C. Left middle cerebral artery D. Right middle cerebral artery

Correct answer: B The anterior cerebral artery (ACA)supplies oxygenated blood to the frontal lobes and superior medial parietal lobes. The most common results of a stroke in this area include: Contralateral motor and sensory loss with lower extremity more affected than upper extremity Gait apraxia Personality and behavior changes (frontal lobe involvement) Answer A:The anterior cerebral artery (ACA) is correct; however, a stroke in the left ACA would cause weakness contralaterally in the right extremities, not the left extremities.The most common presentation after a stroke in the ACA includes: Contralateral motor and sensory loss with lower extremity more affected than upper extremity Gait apraxia Personality and behavior changes (frontal lobe involvement) Answer C:Whether the stroke is in the anterior cerebral artery (ACA) or the middle cerebral artery (MCA), the motor and sensory loss would be seen on the contralateral extremities. In this case, the weakness is on the left side, so the lesion would be in an artery on the right side, not the left. In addition, the presentation here reflects a stroke in the ACA. When the MCA is involved, the upper extremity is often more involved than the lower extremity. Personality changes are possible, but more likely with ACA than MCA. Lastly, gait sensory ataxia is more common with MCA, but gait apraxia is more common with ACA involvement. Answer D:The injury would indeed be to an artery on the right side; however, the presentation here reflects a stroke in the anterior cerebral artery (ACA), not the middle cerebral artery (MCA). Motor and sensory loss would be seen on the contralateral extremities with a stroke in both the ACA and MCA. However, with MCA stroke, the upper extremity is often more involved than the lower extremity, which is the opposite of the patient presentation here. Personality changes are possible with MCA stroke, but more likely with ACA than MCA. Lastly, gait sensory ataxia is more common with MCA, but gait apraxia is more common with ACA involvement. Bottom Line:When reviewing the effects of a hemorrhagic stroke, it is important to know:1. The area of the brain supplied by each artery2. The function of each of these areas of the brain3. Whether the presentation would be seen ipsilaterally or contralaterally TrueLearn Insight :Individuals post stroke can present in many differing ways, even with the same lesion. However, it is important to know typical presentations.

A physical therapist is evaluating a patient with shoulder pain and stiffness. When measuring shoulder passive range of motion (PROM), the therapist notes that there is a capsular pattern of restriction. Which of the following descriptions below represents the capsular pattern of the glenohumeral joint? A. Flexion most restricted, abduction next most restricted, then internal rotation next most restricted B. External rotation most restricted, abduction next most restricted, then internal rotation next most restricted C. Abduction most restricted, internal rotation next most restricted, then external rotation next most restricted D. Flexion most restricted, external rotation next most restricted, then internal rotation next most restricted

Correct answer: B The capsular pattern of restriction for the shoulder is external rotation most restricted, abduction next most restricted, then internal rotation next most restricted. Answer A:Flexion > abduction > internal rotation is the capsular pattern of the hip. Answer C:External rotation is limited more than abduction and internal rotation in the capsular pattern of the shoulder. Answer D:Flexion is not one of the motions described in the capsular pattern of the shoulder. Bottom Line:Capsular pattern is the characteristic pattern of limitation of the joint. The capsular pattern of restriction for the shoulder is external rotation most restricted, abduction next most restricted, then internal rotation next most restricted. Capsular patterns are based primarily on empirical findings versus research but can be a useful piece of the diagnostic puzzle.

A patient with a recent fibular (common peroneal) nerve injury is MOST likely to present with a gait deviation that involves: A. decreased push-off on affected side in stance, circumduction in swing. B. a foot slap on affected side in stance, steppage gait in swing. C. a hip drop during swing phase on affected side, leaning backward at heelstrike. D. leaning forward at heelstrike on affected side, steppage gait in swing.

Correct answer: B The common peroneal (fibular) nerve innervates the muscles of the anterior and lateral compartments of the lower leg, which include the dorsiflexors of the foot. Injury to this nerve typically results in significant gait deviations. During stance,foot slap can occur due to the dorsiflexors' decreased ability to provide eccentric lowering of the foot. During swing, foot drop also can occur due to the inability to lift the foot up against gravity. As a compensation, a steppage gait where the hip and knee flex more than normal can be seen in an attempt to clear the foot and avoid dragging during swing. Answer A:Lack of push-off is typically seen in individuals with impaired motor control of the plantarflexors during stance, not the dorsiflexors. Circumduction is a deviation that occurs when the limb is too long and does not have the usual deviation to clear the foot from the floor when it is unable to dorsiflex (steppage gait is more common), although it is a feasible strategy. Answer C:Hip drop is typically associated with weakness in the hip abductors. It is not associated with a peroneal nerve injury. Leaning back at heelstrike is a compensation when the hip extensors are weak on the stance side. Answer D:Leaning forward at heelstrike is a compensation for weak quadriceps to prevent knee buckling. Steppage gait is a compensation for weak dorsiflexors where the hip and knee flex more to clear the foot that is hanging down.

A patient complains of difficulty in walking 6 weeks after undergoing right total hip arthroplasty. Upon gait analysis, you notice that the patient's left hip drops during weight bearing on the right leg. The patient also notes that her hip muscles are easily fatigued upon prolonged walking. Based on this information, what would be the BEST intervention for the patient? a) wall squats with exercise ball at the back b) side lying abduction with ankle weights c) pelvic bridging with ball squeeze d) quadruped leg lifts with 90 degree knee flexion

Correct answer: B The correct answer is sidelying abduction with ankle weights. The symptom described in this scenario is the Trendelenburg sign, where the patient experiences a hip drop on the contralateral side when weight bearing on the affected leg. The Trendelenburg sign is caused by the weakness of the hip abductors (gluteus medius), which are often affected following total hip arthroplasty surgery. To strengthen the hip abductors, the best exercise in this scenario is progressive strengthening exercise of hip abductors (i.e., sidelying hip abduction with ankle weights). Answer A:Wall squats with exercise ball at the back targets the quadriceps muscles. Answer C:Pelvic bridging with ball squeeze targets hip extensors with hip adductors. Answer D: Quadruped leg lifts with 90-degree knee flexion targets hip extensors.

An administrative assistant complains of a tingling sensation on his right posterior arm down to his hands after prolonged desk work. Upon assessment, the patient has poor posture as characterized by increased thoracic curve and a forward head. When the patient was asked to abduct his right shoulder and rotate his head to the left, symptoms worsened and a decreased radial pulse was noted. What is the BEST intervention for this scenario? a) stretch pectoralis minor, strengthen upper trapezius b) stretch pectoralis minor, strengthen rhomboids c) stretch pectoralis major, strengthen upper trapezius d) stretch pectoralis major, strengthen levator scapulae

Correct answer: B The correct answer is: stretch pectoralis minor, strengthen rhomboids. The decreased pulse and tingling sensation down the arms are symptoms of thoracic outlet syndrome (TOS). One cause of TOS is tightness of the pectoralis minor muscle. In this scenario, the posture of the patient is an upper cross syndrome, specifically, round back with forward head posture. Upper cross syndrome can be addressed by stretching the upper trapezius, levator scapulae, and pectoralis muscles while strengthening the neck flexors and rhomboids. Identification of muscular imbalances that may cause thoracic outlet syndrome will result in the development of appropriate treatment strategies to resolve the condition. Answer A: Stretching pectoralis minor may contribute to resolution of the TOS; however, the upper trapezius should be stretched as well. Answer C: While the upper trapezius should be stretched, stretching the pectoralis major does not necessarily resolve the TOS. Answer D: While the levator scapulae should be stretched, stretching the pectoralis major does not necessarily resolve the TOS.

A 20-year-old runner came to the physical therapy clinic complaining of pain on the right hip and thigh. He reports that the symptoms started after an intensive work-out a week ago. He states that sidelying on the right reproduces the pain. On palpation, the physical therapist noted tenderness of the area of the greater trochanter. Based on this information, which structure must be assessed NEXT? A. Rectus femoris B. Iliotibial band C. Biceps femoris D. Sartorius

Correct answer: B The correct answer is:iliotibial band. With inflammation of the trochanteric bursa, pain is experienced over the lateral hip and possibly down the lateral thigh to the knee when the iliotibial band rubs over the trochanter. Answer A:This muscle does not attach to or pass through the greater trochanter. Answer C:This muscle is located in the posterior thigh and do not attach to or pass through the greater trochanter. Answer D:This muscle runs in an inferomedial direction obliquely across the upper and anterior part of the thigh. It does not attach to or pass through the greater trochanter. Bottom Line:With inflammation of the trochanteric bursa, pain is experienced over the lateral hip and possibly down the lateral thigh to the knee when the iliotibial band rubs over the trochanter.

The physical therapist is assessing a full-term infant for primitive reflex activity. The therapist places the infant in supine and firmly strokes the sole of the foot. The therapist notes flexion in the stimulated leg and extension of the opposite leg with adduction. The reflex demonstrated is: A. Babinski reflex B. Crossed extension reflex C. Moro reflex D. Galant reflex

Correct answer: B The crossed extension reflex appears at 28 weeks' gestation and is integrated at 1-2 months of age. The stimulus to elicit the response is a noxious stimulus to the foot. The response consists of flexion in the stimulated leg and then extension of the opposite leg with adduction. Answer A:The Babinski reflex appears at birth and is integrated at approximately 6-9 months of age but can last as long as 24 months of age. The stimulus to elicit the response is firmly stroking the sole of the foot from heel toward the toes. The response consists of the big toe extending and the toes fanning out. Answer C:The Moro reflex appears at 28 weeks' gestation and is integrated at 3-5 months of age. The stimulus to elicit the response is dropping the head backward. The response consists of abduction and extension of arms, and splaying of fingers, and may be followed by arm flexion and adduction. Answer D:The Galant reflex appears at 28 weeks' gestation and is integrated at approximately 3 months of age. The stimulus to elicit the response in prone is stroking of paravertebral skin. The response consists of lateral curvature of the trunk on the stimulated side. Bottom Line:The crossed extension reflex appears at 28 weeks' gestation and is integrated at 1-2 months of age. The stimulus to elicit the response is a noxious stimulus to the foot. The response consists of flexion in the stimulated leg and then extension of the opposite leg with adduction.

A physical therapist is performing the following special test on a patient. The patient is sitting. The therapist puts one hand on the posterior portion of the patient's right scapula and the other hand on the patient's right elbow. The therapist then passively moves the patient's right arm through the range of shoulder flexion while keeping the arm in internal rotation and looks to see if the patient displays a facial grimace during this motion. Which of the following subjective complaints from the patient would be the MOST likely to prompt the therapist to perform this test? a) reports of shoulder pain that is present at all times with rest and with motion b) reports of shoulder pain when lifting the arm overhead c) a feeling of shoulder instability popping in and out with certain motions d) difficulty elevating the arm all the way due to weaknes

Correct answer: B The description presented is of the Neer test. This special test is performed to check for subacromial impingement. Symptoms of subacromial impingement include shoulder pain when lifting the arm overhead. Answer A:The text describes the Neer test for shoulder impingement. With subacromial impingement, there is usually pain in the midrange of shoulder motion when the subacromial space is smaller and structures get impinged, which causes the pain. Pain at rest is less likely until the late stages, when other complications such as partial tearing may develop. If the patient reports shoulder pain at all times, the therapist should consider other diagnoses like bursitis. Answer C:The text describes the Neer test for subacromial impingement. It does not test for shoulder instability. Answer D:The text describes the Neer test for shoulder impingement. A patient who reports weakness in the shoulder may have a torn rotator cuff, which can be tested with other special tests and not the Neer test. Bottom Line:During the Neer test, the physical therapist passively flexes the patient's arm while maintaining it in internal rotation and looking for the presence of shoulder impingement. Classic shoulder impingement signs are a painful arc; pain in the shoulder with overhead activities, especially while the shoulder is in internal rotation; and tenderness when palpating the greater tuberosity.

Which of the following conditions is due to a hormone deficiency? a) Acromegaly b) Addison disease c) Cushing syndrome d) Graves disease

Correct answer: B The following diseases are all endocrine disorders: Addison disease is a disorder of the adrenal gland resulting in insufficient cortisol release. Acromegaly is a condition in adults of hypersecretion of human growth hormone from the anterior pituitary gland. Cushing syndrome is an excess of cortisol in the body and is due to the pituitary over-secreting adrenocorticotropic hormone (ACTH). Graves disease is a form of hyperthyroidism with an increase in T4 production. Addison disease is a disorder of the adrenal gland resulting in insufficient cortisol release. There is resultant weakness, hypotension, weight loss, nausea, and vomiting. With chronic cortisol insufficiency, more melanocyte stimulating hormone is released as well, resulting in a tanned pigmentation. Answer A: Acromegaly is a condition in adults in which hypersecretion of human growth hormone from the anterior pituitary gland results in increased bone thickness and hypertrophy of soft tissues, primarily in the facial features, hands, and feet. Answer C: Cushing syndrome is an excess of cortisol in the body and is due to the pituitary over-secreting adrenocorticotropic hormone (ACTH) either due to a tumor, or due to the hypothalamus dysfunction causing the pituitary to over-secrete. The ACTH stimulates the adrenal glands to release cortisol. Answer D: Graves disease is a form of hyperthyroidism with an increase in T4 production. Symptoms can include tremors, tachycardia, heat intolerance, and occasionallyexophthalmos (protruding eyes).

A 71-year-old male was admitted to the hospital with a diagnosis of congestive heart failure exacerbation. He was referred to physical therapy during his hospitalization to improve his activity tolerance. During auscultation of heart sounds, the physical therapist hears an S1 heart sound followed by an S2 heart sound. What does each sound represent? a) S1 represents closing of the pulmonic and aortic valves; S2 represents closing of the tricuspid and mitral valves b) S1 represents closing of the tricuspid and mitral valves; S2 represents closing of the pulmonic and aortic valves c) S1 represents closing of the tricuspid and pulmonic valves; S2 represents closing of the mitral and aortic valves d) S1 represents opening of hear valves; S2 represents closing of heart valves

Correct answer: B The heart is divided into four chambers (two atria and two ventricles), each with a valve to ensure blood travels in a single direction and to prevent regurgitation. Blood is pumped from the heart to the rest of the body through these four chambers in a sequence of events collectively called the cardiac cycle, which consists of two phases, systole (when the ventricles are pumping blood out) and diastole (when the ventricles are filling with blood from the atria). Systole ends when the semilunar valves (pulmonic and aortic valves) close to prevent blood from backing up into the ventricles after it has been ejected, creating the S2 sound. Diastole ends when the atrioventricular valves (tricuspid and mitral valves) close to prevent blood from backing up into the atria, creating the S1 sound. The S1 and S2 heart sounds represent the synchronized closing of heart valves during the cardiac cycle. The two sounds create the characteristic "lub-dub" during auscultation of a normal heart. S1 is heard first, at the end of diastole, when blood has filled the ventricles and the atrioventricular valves (tricuspid and mitral) close to prevent regurgitation back into the atria. The S2 sound is then heard shortly after, at the end of systole, when blood has left the right and left ventricles and enters the pulmonary and systemic circulation, respectively. The sound being heard indicates closing of the semilunar valves (pulmonic and aortic) to prevent regurgitation back into the ventricles. Answer A:This answer is incorrect because it describes the closing of the wrong pair of valves when each heart sound is heard. The semilunar valves, the pulmonic and aortic valves, do close together, as do the atrioventricular valves, the tricuspid and mitral valves. However, S1 represents closing of the atrioventricular valves, and S2 represents closing of the semilunar valves, not the reverse. Answer C:This answer is incorrect because it does not describe what actually happens during the cardiac cycle. S1 represents closing of the atrioventricular valves (tricuspid and mitral), and S2 represents closing of the semilunar valves (pulmonic and aortic). Each pair of valves contains a valve from the right side of the heart with the other from the left side of the heart. It is easy to confuse the function of the valves with the correct side on which they are located. In the case of this answer, the tricuspid and pulmonic valves are located on the right side of the heart, but they do not close at the same time, meaning S1 cannot be produced from these two valves. Similarly, the mitral and aortic valves are located on the left side of the heart but they also do not close at the same time either, meaning S2 cannot be produced from these two valves. Answer D:This answer is incorrect because it does not describe what heart sounds actually represent. The opening of heart valves does not create a sound unless an abnormality exists. Both S1 and S2 heart sounds indicate the normal closing of heart valves. Bottom Line:The S1 and S2 heart sounds represent the synchronized closing of heart valves during the cardiac cycle. S1 is heard first at the end of diastole, then the S2 sound is then heard shortly after, at the end of systole.

Following an athlete's baseball season, the individual presents to physical therapy with a report of stiffness when trying to fully extend the elbow. Examination of range of motion (ROM) reveals a limitation both actively and passively with elbow extension. Which joint mobility assessment should the physical therapist perform at the humeroradial joint to determine if hypomobility is limiting the ROM? A. Anterior glide at the radial head B. Posterior glide at the radial head C. Medial glide at the radial head D. Lateral glide at the radial head

Correct answer: B The individual has difficulty extending their elbow. The appropriate glide for elbow extension is to glide the concave radial head posteriorly against the convex capitulum of the humerus. Answer A:Anterior glide of the radius on the humerus would improve elbow flexion. Answer C:A medial glide of the radial head would be used to improve stability if there was splaying at the radioulnar joint. Answer D:A lateral glide of the radial head is not an accessory motion that contributes to humeroradial extension (or flexion). Bottom Line:When the therapist assesses the radiohumeral joint at the elbow, the concave radial head needs to glide posteriorly, in the same direction as the roll, along the convex capitulum of the humerus, to improve elbow extension. An anterior radial glide would be a component motion of elbow flexion.

A physical therapist positions a 25-year-old patient in supine with the upper extremity in 90 degrees of external rotation and 90 degrees of abduction. The therapist then applies a force to the posterior humeral head in an anterior direction. The patient reacts with muscle guarding and comments that the force applied makes him feel uncomfortable. He also reports some pain. Which of the following structures is MOST likely to be stressed during this examination procedure? a) acromioclavicular ligament b) glenohumeral ligament c) posterior capsule d) supraspinatus tendon

Correct answer: B The inferior glenohumeral ligament complex (a component of the anterior capsule) stabilizes against anteroinferior shoulder dislocation, especially when the shoulder is positioned in abduction and external rotation. The apprehension test involves an anteriorly directed force applied to the posterior humeral head while the upper extremity (UE) is positioned in 90 degrees of external rotation and 90 degrees of abduction. In a patient with shoulder instability, the force creates a feeling of apprehension because the humerus may sublux or dislocate in an anterior inferior direction. The apprehension test stresses the anterior structures of the shoulder, including the anterior capsule, glenohumeral ligaments, and labrum. While the test does not accurately differentiate the structure involved, the structures presented for the other options in this question do not get stressed with an anteriorly directed force on the humerus. Answer A:The acromioclavicular ligament connects the acromion and clavicle. It is not one of the structures assessed with the apprehension test and does not stabilize the humerus in the glenoid. Answer C:The force applied during the apprehension test is in a posterior-to-anterior direction, stressing the anterior portion of the capsule, not the posterior portion. Answer D:The supraspinatus tendon attaches on the superior portion of the humeral head on the greater tubercle. It is part of the rotator cuff musculature and does give some dynamic stability to the glenohumeral joint. However, in the question above, an anteriorly directed force on the humeral head while the UE is in 90 degrees of abduction and external rotation would MOST likely stress the anterior capsule and glenohumeral ligament. Bottom Line:For all special tests, one must identify which test is being performed based on the position of the extremity and the direction of the force applied to the joint. The reader can then apply knowledge about the purpose of the test to the question being asked. In this case, the questions asks what structure is stressed with this test. Once the reader recognizes that the test being performed is the apprehension test, and knowing that the purpose of the test is to assess anterior shoulder stability, the reader can arrive at the correct answer of the glenohumeral ligament.

A physical therapist is completing an initial examination with a patient who has a history of an ASIA A cervical spinal cord injury. What is the LOWEST level of injury that would allow the use of a tenodesis grasp for functional activities? a) C5 b) C6 c) C7 d) C8

Correct answer: B The key muscle for assessing function at the C6 level is the wrist extensors. Patients with an ASIA A C6 spinal cord injury will use a tenodesis grasp by extending their wrist in order to achieve passive finger flexion, which allows them to hold objects during functional activities. Answer A:A patient with a C5 ASIA A spinal cord injury would not have active wrist extension so that a tenodesis grasp would not be available. Answer C:For a patient with a C7 ASIA A spinal cord injury, the wrist extensors can be used with tenodesis to allow for limited grasp function, but this skill is also possible for patients with a C6 injury. Answer D:A patient with a C8 ASIA A spinal cord injury would have available finger flexor activity so that a tenodesis grasp would not be necessary.

A physical therapist is completing an initial examination with a patient who has a history of an ASIA A cervical spinal cord injury. The physical therapist asks the patient to extend their wrist against gravity and then against resistance. Which myotome is the physical therapist assessing? a) C5 b) C6 c) C7 d) C8

Correct answer: B The key muscles for assessing function at the C6 level are the wrist extensors. For a patient with a C6 ASIA A spinal cord injury, the wrist extensors can be used with tenodesis to allow for limited grasp function. Answer A:The key muscles for assessing function at the C5 level are the elbow flexors. For a patient with a C5 ASIA A spinal cord injury, the elbow flexors can be used to assist with activities of daily living and bed mobility. Answer C:The key muscles for assessing function at the C7 level are the elbow extensors. For a patient with a C7 ASIA A spinal cord injury, the elbow extensors can be used to assist with lateral transfers and push-up weight shifts for pressure management. Answer D:The key muscles for assessing function at the C8 level are the finger flexors. For a patient with a C8 ASIA A spinal cord injury, the finger flexors can be used for grasp to assist with activities of daily living, transfers, bed mobility, and weight shifts. Bottom Line:The key muscles for assessing function at the C5 level are the elbow flexors. The key muscles for assessing function at the C6 level are the wrist extensors. The key muscles for assessing function at the C7 level are the elbow extensors. The key muscles for assessing function at the C8 level are the finger flexors.

A physical therapist is treating a patient with shoulder pain during upper extremity (UE) elevation. Mobility testing was normal. Upon examination of active shoulder elevation, the patient's humeral head was observed to glide superiorly after approximately 40 degrees of elevation. Weakness in which of the following muscles would MOST likely explain this observation? a) deltoid b) infraspinatus, teres minor c) supraspinatus d) teres major

Correct answer: B The line of pull for the infraspinatous and teres minor allow them to depress the humeral head. When elevating the arm, the deltoid's line of pull would cause the humeral head to translate in a superior direction unless opposed by the inferior line of pull of the infraspinatous and teres minor. These two forces allow the arm to be raised without the humeral head impinging on the subacromial arch. If there is an imbalance in strength, and the rotator cuff muscles are weak, the deltoid may overpower the humeral head and cause it to glide superiorly when it is contracting. Answer A:The deltoid is most likely the reason for the superior glide and abnormal mechanics during arm elevation in this patient. Answer C:Although the supraspinatus is an important rotator cuff muscle, its main role is in the initiation of abduction. It also does not have as good of a line of pull to move the humeral head in an inferior direction as the infraspinatous and teres minor do, since they originate lower on the scapula than the supraspinatous. The problem also occurred later in the range, so weakness of the supraspinatous is not the best choice. Answer D:Teres major extends, adducts, and internally rotates the glenohumeral joint. It has no role in humeral head depression. If this muscle was weak, it would not result in excessive superior glide of the humeral head during arm elevation. Bottom Line:One of the roles of the rotator cuff muscles (primarily the infraspinatus and teres minor) is to depress and rotate the humeral head to clear the subacromial space during arm elevation. When they are weak or there is an imbalance in the force couple with the deltoid during arm elevation, shoulder mechanics may be altered, and the humeral head will glide superiorly.

A physical therapist is performing a manual muscle test on a patient with longstanding carpal tunnel syndrome. Besides flexor pollicus longus and flexor pollicus brevis, which of the following muscles will also demonstrate weakness? a) abductor pollicus brevis and abductor pollicus longus b) lateral lumbricals and abductor pollicus brevis c) lateral lumbricals and adductor pollicus brevis d) medial lumbricals and abductor pollicus longus

Correct answer: B The median nerve is compressed in the carpal tunnel with carpal tunnel syndrome. Patients often present with wasting of the thenar emminence and weakness in all muscles innervated by the median nerve. Abductor pollicus brevis and the lateral lumbricals (those closest to the thumb, which is the lateral part of the hand when in anatomical position) are innervated by the median nerve and will become weak when nerve function is affected. Answer A:Abductor pollicus brevis is an intrinsic hand muscle and is innervated by the median nerve. However, abductor pollicus longus is an extrinsic muscle originating outside of the hand and is innervated by the posterior interosseus nerve, which is a deep branch of the radial nerve. Answer C:The lateral lumbricals (those closest to the thumb, which is the lateral part of the hand when in anatomical position) are innervated by the median nerve. Adductor pollicus brevis is an intrinsic hand muscle but is not part of the thenar eminence. It is innervated by the ulnar nerve. Answer D:The medial lumbricals are located toward the ulnar side of the hand (which is the medial side in anatomic position) and are innervated by the ulnar nerve. Abductor pollicus longus is an extrinsic muscle originating outside of the hand and is innervated by the posterior interosseus nerve, which is a deep branch of the radial nerve. Bottom Line:The median nerve innervates flexor pollicus brevis and longus, opponens pollicus, abductor pollicus, and the lateral lumbricals in the hand and can be weakened when the median nerve is compressed in the carpal tunnel.

A physical therapist is treating a patient who had a left middle cerebral artery stroke. The patient can produce many words with grammatically correct sentences but has difficulty communicating with the therapist because their speech output is disorganized. They also have difficulty comprehending others' speech. The physical therapist should employ all of the following strategies during treatment EXCEPT which of the following? a) therapist should slow down their rate of speech b) therapist should ask open-ended questions c) therapist should increase the use of non-verbal cues d) therapist should use pictures for the patient to point to

Correct answer: B The patient described in this scenario is experiencing symptoms of aphasia, which are consistent with the diagnosis of a left middle cerebral artery stroke. The left hemisphere of the brain is responsible for most of language function in the majority of people. The inability to understand verbal commands along with disorganized speech is consistent with a lesion to Wernicke's area, which causes receptive aphasia. Receptive aphasia is also known as Wernicke's aphasia. The use of open-ended questions increases the complexity of communication and is unlikely to be effective in communicating with patients with expressive aphasia. Answer A: Slowing down the rate of speech allows patients with expressive aphasia more time to process language and may lead to an improvement in communication with the therapist. Answer C:The use of non-verbal cues allows patients with expressive aphasia the ability to use intact non-verbal communication skills and may lead to an improvement in communication with the therapist. Answer D:The use of pictures allows patients with expressive aphasia the ability to use intact non-verbal communication skills and may lead to an improvement in communication with the therapist. Bottom Line: Receptive aphasia causes deficits in language comprehension and language expression, which make communication difficult during physical therapy. Asking open-ended questions is unlikely to be effective. Communication strategies include slowing down speech, and using pictures and non verbal commands where possible.

A patient fell while ice skating and had mild shoulder pain. The radiograph was negative. Physical exam by the physician showed moderate instability, and the patient was referred to physical therapy 6 weeks ago. The patient's health insurance provider permits a maximum of 30 physical therapy visits per year. The patient has been attending PT twice a week for 6 weeks with no change in the shoulder pain or improved strength of the shoulder musculature. The BEST course of action is to: A. continue physical therapy until at least 8 weeks to allow ample time for improvement as the patient is making progress. B. contact the patient's physician to discuss the patient's progress. C. reduce visits to once a week with a comprehensive home exercise program to extend the physical therapy benefits. D. perform a re-evaluation and design a new plan of care to speed the recovery.

Correct answer: B The patient has had no change in her pain for 6 weeks. At this point, there should have been a change if the shoulder was healing and if the physical therapy was effective. Therefore, there may be something else going on, especially with the moderate instability. The patient might benefit from an MRI to see if there is damage to some of the soft tissue structures that was not obvious on the radiograph. The therapist should contact the patient's physician. Answers A & C & D:There has been no change in the patient's pain after 6 weeks of conservative treatment, so the therapist should consult with the patient's physician to determine next steps. Bottom Line:If a patient is not achieving the short-term or long-term goals, and the patient was referred by a physician, it is wise to discuss the patient's progress with the referring physician to determine the best course of action. It is not advisable to continue the treatment program if it is ineffective.

A 65-year-old patient presents with a 3-year history of Parkinson's disease. The patient is now in the moderate stages of the disease and has had an increasing number of falls while using his cane in the last few months. Which of the following interventions is the MOST appropriate intervention at this stage of the disease? a) prepare the caregiver for the prevention of contractures and pressure wounds b) begin gait training with use of rolling walker c) consider transitioning the patient to a wheelchair d) begin gait training with use of a light-weight standard walker

Correct answer: B The patient in the scenario is currently using a cane. The next step would be progression to a walker since the patient is suffering from an increased number of falls. A rolling walker is optimal because initiating motion with Parkinson's disease is hard, so the wheels on the bottom are less likely to cause freezing of gait and have less impact on slowing gait speed. Answer A:Prevention of contractures, pressure wounds, pneumonia, and other impairments that are a result of immobility is usually performed when the patient is in the late stages of Parkinson's disease. Patients with an advanced disease process are typically wheelchair or bed bound most of the day and need assistance with all ADLs. Answer C:Transitioning to a wheelchair is usually reserved for the late stages of Parkinson's disease. At this point, the patient has a great deal of difficulty walking, but if the therapist can provide safety and stability with the rolling walker instead of taking away the patient's ability to actively ambulate, the rolling walker is in the patient's best interests. In the late stages, a wheelchair would be appropriate. Answer D:Because the cane no longer provides enough stability, a walker is a better option now. However, because it is hard to initiate motion with Parkinson's disease, a rolling walker is a better option than a standard walker because it has less of an impact on gait freezing and does not slow gait as much as a standard walker. Bottom Line:A comprehensive evaluation is required with a Parkinson's patient to determine the level of impairment and degree of function. During the early and middle stages of the disease, rehabilitation is focused on preventive and restorative strategies with some compensatory strategies as needed. In the late stages of the disease, rehabilitation shifts to more preventive and compensatory options that focus on safety and maximizing remaining mobility.

A physical therapist is treating a patient with type 1 diabetes who is in cardiac rehabilitation in an outpatient setting. Prior to the session, the patient's breath smells fruity, she reports nausea, and he has a serum glucose level of 450 mg/dL. The MOST appropriate course of action is to: a) ask the patient to inject her emergency glucagon b) call 911 for emergency medical services c) instruct the patient to begin her exercise program d) wait 5 minutes and then have the patient recheck her glucose levels

Correct answer: B The patient in this scenario is showing symptoms of ketoacidosis, which is a life-threatening medical emergency that occurs when a high level of ketones is produced since fat is used for energy because of a lack of carbohydrates. The most appropriate response in this scenario is to call foremergency medical servicesso that blood glucose levels can be managed with IV insulin. Answer A:Glucagon is an emergency medication used to treat low blood sugar, so this would be an inappropriate response in this scenario. Answer C:Because exercise would further reduce the patient's glucose levels, instructing the patient to start her exercise program would not be an appropriate response to the situation. Answer D:Because a blood glucose level of 450 mg/dL and symptoms of ketoacidosis represent a life-threatening medical emergency, waiting and rechecking glucose levels would not be an appropriate response to the situation. Bottom Line: Ketoacidosis is a life-threatening medical emergency that occurs when a high level of ketones is produced since fat is used for energy because of a lack of carbohydrates. Physical therapists must be able to recognize ketoacidosis and be able to monitor blood glucose levels in their patients with diabetes to ensure that the therapist safely provides interventions.

A physical therapist is observing the gait of a patient with right hip osteoarthritis. The patient reports stiffness and minimal pain. After midstance, when the right leg is supposed to move into the push-off phase of gait, the limb stays directly under the body. Some increased lordosis is also noted. Based on the missing range of motion at the right hip demonstrated during this gait pattern, which of the following mobilizations would be MOST appropriate to increase that range? A. Grade 2 anterior glide B. Grade 4 anterior glide C. Grade 2 posterior glide D. Grade 4 posterior glide

Correct answer: B The patient is demonstrating a lack of hip extension. An anterior mobilization of the convex femoral head is appropriate direction for the arthrokinematics of the hip and a grade of 4 the most appropriate grade from the options presented to improve ROM. Answer A:The patient is demonstrating a lack of hip extension, and an anterior mobilization of the convex femoral head is an appropriate direction for the arthrokinematics of the hip. However, a grade of 2 is more suited toward pain and not as effective in increasing range of motion. Answer C:A posterior glide would best address a limitation in hip flexion, not extension. In addition, a grade of 2 is not the most effective for restoring ROM. Answer D:A posterior glide would best address a limitation in hip flexion, not extension. Bottom Line:During normal gait, maximum hip extension ROM is achieved just before heel off/terminal stance. If there is a lack of extension, the patient may skip that phase or may increase extension at the lumbar spine to compensate. The joint mobilization to increase hip extension would be an anterior glide of the proximal femur.

A physical therapist is treating a patient with a C5 ASIA A spinal cord injury who is in an inpatient rehabilitation facility. The patient is positioned in bed with the head of the bed elevated to 30 degrees, and they have their compression stockings and abdominal binder on. The patient has an indwelling catheter due to a lack of bladder control. The physical therapist is about to complete an overhead lift transfer to a wheelchair, but the patient reports that their head "feels funny." The physical therapist checks the patient's blood pressure, which is 200/100. Which of the following actions should the physical therapist NOT immediately perform? a) check the indwelling catheter to determine whether it is draining b) lay the patient flat in bed c) remove the abdominal binder d) remove the compression stockings

Correct answer: B The patient is experiencing an episode of autonomic dysreflexia, which is found in patients with a spinal cord injury that occurs at T6 and above. A painful stimulus below the level of injury causes a dangerous rise in blood pressure. When a person is experiencing autonomic dysreflexia, the physical therapist needs to decrease the patient's blood pressure while assessing for the cause of the dysreflexia. Laying the patient flat in bed would increase the patient's blood pressure and would be dangerous. If the patient were experiencing hypotension, lowering the head of the bed would be indicated. Answer A:A non-draining catheter is a common cause of autonomic dysreflexia. Nursing should be notified if the catheter is not draining because intermittent catheterization may be required. Answer C: Removing the abdominal binder would help lower blood pressure and should be done immediately when someone is experiencing autonomic dysreflexia. Answer D: Removing the compression stockings would help lower blood pressure and should be done immediately when someone is experiencing autonomic dysreflexia. Bottom Line: Autonomic dysreflexia is a common medical emergency for patients with spinal cord injury. Physical therapists must be able to identify its symptoms and initiate its management when it occurs. Laying the patient flat in bed would increase the patient's blood pressure and would be dangerous. Patient and family education regarding the symptoms and treatment of autonomic dysreflexia is a critical area to address during rehabilitation of patients with spinal cord injury.

An elderly obese patient with diabetes and high blood pressure is attending outpatient physical therapy for treatment of knee pain due to osteoarthritis. While riding the bicycle, the patient becomes diaphoretic, begins acting confused, and reports feeling dizzy. What should the physical therapist do NEXT? ` A. Take the patient's blood pressure B. Have the patient drink a cup of orange juice C. Lie the patient down and elevate the legs D. Call 911

Correct answer: B The patient is showing signs of hypoglycemia, when blood sugar levels are too low. If levels get too low, the brain does not get enough glucose and stops functioning normally. In addition, low blood glucose levels trigger the release of epinephrine which can cause "fight or flight" symptoms like rapid heart rate and sweating. Administering carbohydrates will raise the blood glucose level back up and should be initiated immediately. If done quickly, an emergency can be avoided. Answer A:Confusion is not usually associated with blood pressure issues. The signs and symptoms here indicate hypoglycemia, and it is important to address it as soon as possible. The profuse sweating is also more indicative of hypoglycemia. Answer C:The patient is not demonstrating signs of low blood pressure or orthostatic hypotension from riding the bicycle. Answer D:This is not a medical emergency at this stage. Providing some sort of snack or juice with carbohydrates should take care of the problem and restore the glucose levels. Bottom Line:Hypoglycemia results from insulin levels that are too low and should be addressed immediately by intake of carbohydrates such as juice, a snack. Signs and symptoms include weakness, dizziness, feeling of rapid heart rate, and anxiety and/or confusion.

A patient reports 1 month of intermittent toothache-like pain of insidious onset in the right shoulder and neck. The patient also reports mild numbness down the arm that is present some days and not other days. He states his arm just feels heavy when he's doing yard work. Tests are negative for cervical radiculopathy. Which of the following special tests is the MOST appropriate for the physical therapist to perform? A. Patient's forearm rests on table in supination. The therapist taps on the wrist and looks for signs of tingling and symptom reproduction B. Patient positioned in 90 degrees of bilateral shoulder AB and ER, and 90 degrees of elbow flexion; patient slowly makes a fist and releases for 3 minutes; radial pulse is palpated; therapist looks for symptom reproduction C. Patient standing holding involved arm at 90 degrees of abduction; therapist applies inferiorly directed force to the humerus and looks for symptom reproduction D. Patient positioned in sitting with shoulder in 90 degrees of flexion and elbow in 90 degrees of flexion; therapist internally rotates arm and looks for symptom reproduction

Correct answer: B The patient's symptoms appear to be due to thoracic outlet syndrome (TOS). The borders of the thoracic outlet are the anterior scalene muscle anteriorly, the middle scalene muscle posteriorly, and the first rib inferiorly. TOS is described as compression of the neurovascular structures as they exit through the thoracic outlet. Patients can present with a combination of neurological and vascular signs, or the signs of each individually, such as vague pain or heaviness in the shoulder with certain activities of the arm. For this reason, TOS is usually a diagnosis of exclusion. Many of the tests are vascular in nature, however, thoracic outlet tests must also reproduce the patient's symptoms to be considered positive.The Roos test, in which the patient is positioned in 90 degrees of bilateral shoulder AB and ER, and 90 degrees of elbow flexion, and then slowly makes a fist and releases for 3 minutes,is one of the special tests for TOS. If the patient is unable to maintain the position or suffers ischemic pain, heaviness, profound weakness, or numbness/tingling of the hand during the 3 minutes, the test is considered positive for thoracic outlet syndrome. The radial pulse may also weaken or be obliterated during the maneuver. Answer A:In the Tinel test, the examiner taps over the location of a peripheral nerve suspected of having a compression neuropathy. The test is positive if the patient experiences tingling as the examiner taps. At the wrist, a Tinel test is performed to test for carpal tunnel syndrome or compression of the median nerve. Answer C:This position describes the drop-arm test, which is used to diagnose a suspected rotator cuff tear. Inability to lower the arm slowly where it suddenly drops is positive for rotator cuff pathology. The symptoms described by this patient are not consistent with rotator cuff tear. Answer D:The test described here is the Hawkins-Kennedy test for impingement of the rotator cuff. The patient's symptoms are not consistent with rotator cuff impingement.

A tennis instructor is referred to physical therapy for left shoulder pain. Postural examination is within normal limits except for downward rotation and prominent inferior angle of the left scapula with slight adduction. Which of the following shortened muscles contribute to this patient's posture and should be ASSESSED by the physical therapist? a) pectoralis major, upper trapezius, serratus anterior b) pectoralis major, rhomboids, levator scapulae c) pectoralis major, pectoralis minor, rhomboids d) pectoralis minor, lower trapezius, levator scapulae

Correct answer: B The pectoralis minor originates on ribs 3-5 and inserts on the coracoid process. When shortened, it pulls on the coracoid process, causing the scapula to tip in an anterior direction, causing the inferior angle to move posteriorly and appear as a prominence. The levator scapulae originates from the transverse processes of C1-C4 and inserts on the superior angle of the scapulae. When shortened, it pulls the scapulae into downward rotation and elevation. The rhomboids line of pull also causes downward rotation along with scapular retraction. Answer A:The pectoralis major does not attach to the scapula and cannot pull it into a tipped position or a downwardly rotated position. A shortened pectoralis major would pull the humerus into a forward shoulder posture with some internal rotation of the humerus. A shortened upper trapezius would pull the scapula into an elevated and upwardly rotated position. The serratus anterior would pull the scapula into abduction or protraction. Answer C:The pectoralis minor if shortened would tip the scapula anteriorly, causing the inferior angle to move posteriorly and become more prominent. The rhomboids if shortened would adduct and downwardly rotate the scapula. However, a shortened pectoralis major would not cause these postural deviations. the pectoralis major inserts on the humerus and pulls it forward with internal rotation. Answer D:A shortened pectoralis minor would pull on the coracoid process and cause the scapula to tip anteriorly, making the inferior angle more prominent posteriorly. Levator scapulae if shortened would pull the scapula into slight adduction and downward rotation. However, the lower trapezius would pull the scapula into adduction and upward rotation if shortened. Bottom Line:Shortened or tight musculature can pull a mobile bone into an altered position and affect posture. Knowing the origin, insertion, and line of pull can help the therapist identify muscles that are too short (or too long) when observing postural deviations.

Which of the following describes the MOST appropriate location for auscultation of breath sounds? a) supraclavicular area, near acromion b) between the scapula and the spine on either side, at the level of T7-T9 c) on the sternal body, at the level of the 5th rib d) midway between medial border of the scapula and the axilla, inferior to the spine of the scapula

Correct answer: B The posterior and superior portions of the lower lobe is auscultated between the scapula and the spine on either side, at the level of T7-T9. Auscultation of lung sounds allows the physical therapist to assess the respiratory status of the patient and to screen for cardiopulmonary dysfunction.It is important to make note of landmarks and have a good knowledge of surface anatomy during auscultation, since lung sounds can be heard only over areas where there is lung tissue and where airway conduction occurs. Answer A: The supraclavicular area, near acromion, is not over lung tissue and is not where airway conduction occurs. Depending on the size of the stethoscope compared to the patient, the stethoscope may overlap the area where the apices of the lungs can be heard. A better way to auscultate the apices would be to move the stethoscope more medial and toward midline. Answer C: On the sternal body, at the level of the 5th rib corresponds to an area that is below the level of the mainstem bronchus as well as where the left and right bronchi bifurcate. Hence there is no airway conduction occurring at this location, and it would be inappropriate to auscultate for breath sounds here. Answer D: Midway between the medial border of the scapula and the axilla is close to some lung tissue and conducting airways. However, since it is over a bony surface in addition to layers of scapular muscle, it is not a good area for auscultation of breath sounds. Bottom Line: The posterior and superior portions of the lower lobe is auscultated between the scapula and the spine on either side, at the level of T7-T9. Auscultation of lung sounds allows the physical therapist to assess the respiratory status of the patient and to screen for cardiopulmonary dysfunction.

A physical therapist has been consulted to see a patient at home 3 weeks status post right total hip replacement with a posterior approach. The patient's functional status is as follows: independent in bed mobility; sleeps on his back since the surgery but does not sleep well because he is used to sleeping on his left side; able to dress independently using a reacher to help get his right leg into the pant leg; able to go up and down the three steps that lead to the house using a reciprocal pattern and one railing. Which of the following is the BEST activity modification advice to help this patient? A. Go up the stairs with a step-to pattern for safety B. Try sleeping on left side with pillow between knees C. Start trying to put pant leg on without use of reacher to improve ROM D. Try doing steps without holding the railing to challenge and improve balance

Correct answer: B The primary goals following a joint replacement are to restore function, decrease pain, and gain muscle control to enable the patient to return to previous or improved level of function. The treatment plan is shaped by a variety of factors including perioperative complications, type of prosthesis, surgical technique, and approach. Typical total hip replacement precautions include no flexion past 90°, no adduction or internal rotation past neutral. Due to the patient having difficulty sleeping, it would be acceptable for him to try to sleep on his left side with a pillow between his knees. The pillow would prevent adduction of the right lower extremity, which is a common precaution following posterior total hip replacement. Answer A:There is no need to have the patient revert back to step-to pattern. The fact that he is already going up with step-through pattern is good. Answer C:Following a posterior hip replacement, patients usually have post surgical precautions. Most often the precautions include no hip flexion greater than 90 degrees, adduction, and internal rotation of the operated lower extremity. The patient uses the reacher to put on his pants to avoid bending down, which would break his hip precautions by allowing greater than 90 degrees of hip flexion. The patient should continue this method until released from precautions by surgeon. Answer D:The patient should continue to hold the railing for safety. It is unnecessary to make the activity more difficult for him at home and put him at risk for falling.

A physical therapist is treating a patient with a right transtibial amputation that occurred 3 days ago. WHICH of the following positions should be encouraged to prevent contractures in the right lower extremity that could interfere with future prosthetic training? a) sidelying on the right b) prone c) seated in a wheelchair d) supine

Correct answer: B The prone position encourages both hip extension and knee extension of the residual limb, which would prevent hip and knee flexion contractures, which often interfere with prosthetic fit and training. Answer A:Sidelying is an acceptable position for patients with transtibial amputation if hip flexion and knee flexion are avoided, but it would not be actively encouraged. Answer C:Sitting in a wheelchair may encourage hip and knee flexion contractures, which often interfere with prosthetic fit and training. Extending the knee while seated in a wheelchair by using a limb support would be recommended to prevent knee flexion contractures. Answer D:Supine is an acceptable position for patients with transtibial amputation if hip flexion and knee flexion are avoided by not elevating the head of the bed or using pillows under the knee, but it would not be actively encouraged. Bottom Line:The prone position encourages both hip extension and knee extension of the residual limb, which would prevent hip and knee flexion contractures, which often interfere with prosthetic fit and training. Physical therapists must recommend proper positioning for their patients with amputations in order to prepare their residual limbs for prosthetic use.

A patient with a history of atrial fibrillation (A-fib) and other cardiovascular issues displays a normal sinus rhythm (NSR) on telemetry. The physicaltherapist asks if there have been any changes in his medical care. The patient reports that in the past month he has started taking an angiotensinreceptor blocker, had a right heart catheterization, and has undergone radio frequency ablation. Which of the three interventions mentioned is MOST likely responsible for the change from A-fib to NSR? a) the medication b) the radio frequency ablation c) the right heart catheterization d) all three interventions could have been responsible for the change from A-fib to NSR

Correct answer: B The radio frequency ablation procedure directly addresses A-Fib. The physician locates the area on the heart causing the A-fib and ablates that area. Answer A:Angiotensin receptor blockers address blood pressure issues, not A-fib. An antiarrhythmic would be a more appropriate medication. Angiotensin in the body narrows blood vessels and stimulates sodium transport, increasing water retention by the body. Both result in increased blood pressure. Angiotensin receptor blockers block the receptor sites that produce angiotensin, and so the blood vessels can dilate. Less fluid is retained, and blood pressure is reduced. Answer C:This procedure monitors pressure in the heart and is not a treatment for A-fib. Answer D:Only the radio frequency ablation directly addresses A-Fib. The physician locates the area on the heart causing the A-fib and ablates that area. The medication reduces blood pressure, and the right heart catheterization monitors blood pressure. Bottom Line:Atrial Fibrillation (A-Fib) is an erratic twitching of the atrial muscle caused by ectopic foci in the atria sending erratic signals. It can be controlled by antiarrhythmicmedications. Another option is radio frequency ablation surgery, which destroys the area of heart tissue responsible for the irregular electrical signals.

A physical therapist is assessing a patient with complaints of right flank and low back pain over the last year. The patient has a history of cholecystitis. The therapist decides to perform a physical examination of the abdominal region and begins with Murphy's sign for cholecystitis. Murphy's test is performed in WHICH ABDOMINAL QUADRANT? a) left upper quadrant b) right upper quadrant c) right lower quadrant d) left lower quadrant

Correct answer: B The right upper quadrant contains the gallbladder, liver, pancreas, and right kidney. Murphy's test is performed with the patient in the supine position and the examiner placing his or her hands on the right upper abdominal quadrant at the inferior costal margin. Answer A:The left upper quadrant contains the spleen and stomach. Answer C:The right lower quadrant contains the small intestine, appendix, and large intestine. Answer D:The left lower quadrant contains the small and large intestine.

A 66-year-old male with a history of hypertension and diabetes mellitus (type II) presents for an initial evaluation prior to starting a general health and wellness program. The physical therapist notices signs of impaired circulation in the lower extremities and decides to perform some screening tests for vascular disease. Which of the following describes the proper procedure for performing the RUBOR OF DEPENDENCY TEST? A. Patient lying prone, with knee flexed to 90 degrees B. Patient lying supine, with the leg straight and lifted to 45 degrees of hip flexion for 2 minutes, followed by hanging the leg over the edge of the bed C. Patient sitting at the edge of the bed, hanging leg for 2 minutes D. Patient sitting at the edge of the bed, leg maintained in knee extension for 2 minutes, followed by hanging the leg over the edge of the bed

Correct answer: B The rubor of dependency test, or Buerger's test, assesses for the presence of lower limb ischemia. It is performed with the patient lying supine, with the leg straight and lifted to 45 degrees of hip flexion for 2 minutes in order to take away the assistance of gravity in bringing circulation to the lower limb. Then the leg is placed hanging over the edge of the bed. The test is positive if there is a clear decrease in foot perfusion when the leg is elevated or if there is reactive hyperemia (appearance of excess circulation, i.e. "rubor" or redness) in the dependent position. Answer A:This test position has the potential for decreasing arterial supply to the lower limb and thus show signs of arterial insufficiency. However, it is not the standard position for the rubor of dependency test. Answer C:In this test position, the lower limb is dependent. Gravity would actually assist in providing arterial circulation to the lower limb. While signs of arterial insufficiency may be present in this position, it does not describe the rubor of dependency test. Answer D:This test position has the potential for decreasing arterial supply to the lower limb and thus show signs of arterial insufficiency. However, since the lower extremity is below the level of the heart, gravity would still assist in providing arterial circulation. In addition, when the lower limb is hanging over the edge of the bed, some reactive hyperemia could be present. However, this is not the proper procedure for the rubor of dependency test

An infant is being evaluated by the physical therapist for the presence of primitive reflexes. The therapist notes the presence of the symmetric tonic neck reflex (STNR). The therapist would be correct in performing WHICH of the following movements to elicit the STNR response? a) lateral head turning b) neck flexion and neck extension c) pushing on the foot d) passive flexion of one leg

Correct answer: B The symmetric tonic neck reflex is elicited by neck flexion, which leads to upper extremity (UE) flexion and lower extremity (LE) extension; neck extension leads to UE extension and LE flexion. Answer A:Lateral head turning leads to flexion (less extension) of the extremities on the skull side and extension on the face side, This is the description of theasymmetric tonic neck reflex (ATNR). Answer C:Pushing on the foot elicits a strong extension of the leg. This is the description of the extensor thrust response. Answer D:Passive flexion of one leg is associated with extension of the other leg. This is the description of the crossed extensor reflex. Bottom Line:The symmetric tonic neck reflex is elicited by neck flexion which leads to UE flexion and LE extension; neck extension leads to UE extension and LE flexion.

A physical therapist is reviewing a patient's medical file before the initial examination. The X-ray report indicates a Hill-Sachs lesion on the right side. This finding is associated with which of the following types of trauma? a) acromioclavicular separation b) glenohumeral dislocation c) humeral neck fracture d) avulsion fracture of supraspinatus

Correct answer: B The therapist should suspect that the patient has experienced at least one glenohumeral dislocation. During an acute traumatic dislocation of the glenohumeral joint, the humeral head most commonly dislocates from the glenoid in an anterior and inferior direction. The posterior portion of the humeral head can impact the inferior glenoid rim as it dislocates and create a compression fracture known as a Hill-Sachs lesion. Answer A:The Hill-Sachs lesion is seen with glenohumeral dislocations, not acromioclavicular separations. Answer C:The Hill-Sachs lesion is seen with glenohumeral dislocations and is located on the posterior humeral head. It is not present with a humeral neck fracture. Answer D:The Hill-Sachs lesion is a compression fracture on the posterior humeral head and is not an avulsion type of fracture. It is seen with glenohumeral dislocation.

A patient has reached a point in the rehabilitation of an anterior talofibular ligament sprain where sport-specific training and practice is permitted. Which tape technique would be the MOST appropriate for this patient? A. Open basket-weave B. Closed basket-weave C. Low-Dye D. McConnell

Correct answer: B The use of an eternal support, including tape and bracing, is beneficial for the prevention of first-time and recurrent ankle sprains. These interventions will allow the patient to resume higher-demand activities while decreasing the risk of reinjury.The closed basket-weave tape technique was designed to provide support for the ankle in patients who have sustained a lateral ankle sprain injury. Answer A:The open basket-weave tape technique can be used during the acute phase of injury to control swelling. It will not be beneficial for a patientreturning to competition. Answer C:The low-Dye tape technique is used for patients with plantar fasciitis. Answer D:McConnell is a type of tape rather than a specific technique. Bottom Line:The use of an eternal support, including tape and bracing such as the closed basket-weave, is beneficial for the prevention of first-time and recurrent ankle sprains. These interventions will allow the patient to resume higher-demand activities while decreasing the risk of reinjury.

Therapeutic ultrasound is indicted for which of these patients? A. Patient with lung cancer, with pain in the hip from arthritis B. Patient status post clavicle fracture 2 weeks ago C. Pregnant patient with low back pain D. Patient status post total knee replacement 2 weeks ago with knee pain

Correct answer: B The use of ultrasound over fractures is a PRECAUTION versus a contraindication. Low-dose ultrasound has been show to accelerate fracture healing. The application of high-intensity ultrasound over a fracture generally causes pain. Therefore, only low-dose ultrasound should be applied over the area of a fracture. Answer A:Ultrasound is contraindicated in patients with tumors due to the possibility of increasing the rate of tumor growth or metastasis. Answer C:Maternal hyperthermia has been associated with fetal abnormalities. It is recommended that therapeutic ultrasound not be applied at any level in areas where it may reach a developing fetus, including the abdomen, low back, and pelvis. Answer D:Joint cement and plastic materials used for fixation or as components of prosthetic joints are rapidly heated by ultrasound. Therefore, it is generally recommended that ultrasound not be applied over a cemented prosthesis or in areas where plastic components are used. Bottom Line:Contraindications for therapeutic ultrasound include malignant tumors, pregnancy, and joint replacement. Low-dose ultrasound has been shown to speed the rate of healing of fresh fractures in some studies.

Researchers studied limb coordination following Botox injection in patients with post-stroke spasticity. They performed a randomized controlled trial with an experimental group and a control group. Which of the following is the dependent variable? a) botox injection b) limb coordination c) spasticity d) patients post-stroke

Correct answer: B The value of the dependent variable will depend on how the independent variable is manipulated. Thus, if one is looking to see the effect of treatment A on outcome B, outcome B is the dependent variable. The outcome will depend on the effect independent variable A has on it. For example, a dependent variable can be test scores, and the independent variable can be the type of music playing during the test. Limb coordination is the dependent variable. The mean for whichever method is used to measure limb coordination (e.g., a coordination test score or measurement of gait parameters) might "depend" on whether the patient received the Botox or not. Answer A:Botox injections are the independent variable. Answer C:All patients in this study have spasticity. It is not a variable. Answer D:All subjects in this study are post stroke. The sample population of patients is not a variable.

Which of the following descriptions is correct for locating the brachial pulse? a) elbow flexed laterally in the antecubital fossa b) elbow flexed medially in the antecubital fossa c) elbow extended medially in the antecubital fossa d) elbow extended laterally in the antecubital fossa

Correct answer: B There are two options for locating the brachial pulse, the distal medial aspect of the humerus and medially in the antecubital fossa. The elbow should be flexed and supported to ensure biceps are not contracted. Answer A:The brachial pulse is located medially in the antecubital fossa and not laterally. Answer C:The brachial pulse is located medially in the antecubital fossa, but the elbow must be flexed and supported to avoid contraction in the biceps, which would limit the ability to locate the pulse. Answer D:The brachial pulse is located medially in the antecubital fossa, and the elbow should be flexed and supported to avoid contraction in the biceps. Bottom Line:Locating the brachial pulse can be done by palpating on the distal medial aspect of the humerus or locating it medially in the antecubital fossa. The elbow should be flexed and supported to avoid the biceps contracting, which would limit access to the pulse.

A physical therapist at a hospital is asked to give an inservice about positioning to prevent deformities to a group of medical residents. Which of the following is the BEST course of action? a) before beginning the inservice, ask the participants a few questions and a request a "show of hands" to determine their knowledge on the subject b) have the residents complete a pre-inservice survey at least 1 week before the lecture to assess the residents' knowledge c) design the lecture at an introductory level to make sure the content is appropriate for everyone to follow d) design the lecture at a more advanced level since this group has already had coursework in the subject

Correct answer: B To plan an optimal educational session, the therapist must know the level of the audience and some things the audience hopes to get out of the session by performing a needs assessment in advance. Doing this at least a week before the lecture allows time to adapt the presentation accordingly instead of doing it on the spot. Answer A:Asking at the start of the session is better than not asking at all but does not really provide the time to adapt the presentation that would have been gained by surveying the attendees a week in advance. Answers C & D:Planning a lecture that is very simplified or very advanced without knowing the level of the learner is inappropriate. Bottom Line:To produce an optimal educational session, the therapist must know the level of the audience and some things the audience hopes to learn at the session by performing a needs assessment in advance. Doing this at least a week in advance gives the therapist time to adapt the presentation much better than adapting it on the spot.

A patient who has been referred to physical therapy due to recent neck pain also presents with weakness and numbness in the right hand and fingers. The physical therapist notes numbness specifically on the dorsal surface of the index and middle fingers as well as weakness in the wrist flexors and triceps. Strength in the elbow flexors, wrist extensors, and finger flexors is normal. Which of the following conditions is MOST likely to be present in this patient? a) compression of the C6 nerve root b) compression of the C7 nerve root c) compression of the radial nerve d) compression of the musculocutaneous nerve

Correct answer: B Weakness or numbness that corresponds to involvement of a certain dermatome can be similar to symptoms of a peripheral nerve injury. In this patient scenario, symptoms are consistent with involvement of the C7 dermatome (dorsal surface of index and middle fingers) as well as the C7 myotome (triceps extension and wrist flexion). In addition, radial nerve involvement is ruled out by the presence of intact wrist extensors. Therefore, the most likely condition present is a C7 injury. Answer A:This answer is incorrect because C6 is unlikely to be involved. The C6 myotome (wrist extension) is intact. In addition, the pattern of numbness in this patient scenario is more consistent with involvement of the C7 dermatome. Answer C:The radial nerve is unlikely to be involved due to several reasons. Presence of weakness in the triceps but not the wrist extensors (both innervated by the radial nerve) suggests that the radial nerve is intact distally, which means it should be intact proximally as well. Therefore, the source of weakness in the triceps is most likely a structure other than the radial nerve. Answer D:The biceps are innervated by the musculocutaneous nerve, and their strength is normal, so this answer is incorrect. Bottom Line:Weakness or numbness that corresponds to involvement of a certain dermatome can be similar to symptoms of a peripheral nerve injury. For instance, muscles innervated by the radial nerve often overlap those that are innervated by the C7 nerve root.

Which of the following fitness ball exercises best strengthens the abdominals? A. Seated on therapy ball, roll body forward and shift weight onto the "sit bones" (ischial tuberosities) B. Lie prone on ball and walk hands out as ball moves toward the feet until it is near the tibias, then walk back in, keeping spine in neutral C. Start seated on ball, then lie back and walk feet out until ball is under thoracic spine, keeping spine in neutral, then walk back in until seated again D. Lie prone over ball with ball under stomach and alternate lifting right arm with left leg, then left arm and right leg while keeping spine in neutral

Correct answer: B When the patient is in a prone position where the hands are out front and the ball is providing support only under the tibias, the force of gravity is in the direction to cause the spine to sag into a lordotic position. To maintain the spine in neutral, the abdominal muscles must contract. Answer A:When seated on the therapy ball, rolling forward onto the ischial tuberosities increases the lumbar lordosis and creates an anterior pelvic tilt, which does not involve abdominal muscles. A posterior tilt would be caused by contracting the abdominals and rolling backward on the ball to move the weight off the ischial tuberosities and more onto the sacrum. Answer C:If a patient is supine, the therapy ball is under the thoracic spine, and the feet are on the floor, the force of gravity would try to flex the hips. The resistance needed to maintain the neutral spine position would involve contraction of the hip extensors. Answer D:Prone alternating arm and leg lifts over a ball does not strengthen the abdominals. Gravity is applying a force in the direction of trunk flexion. In this exercise, the arms and legs are opposing that force, and this exercise strengthens the trunk extensors. Bottom Line:Abdominal muscle strengthening will occur with exercises that require the trunk to flex against gravity, such as a sit up, but it can also occur by putting the body in a position where the force of gravity is in the direction to extend the trunk or cause a lordosis, and holding the trunk in neutral while resisting that force, such as in a plank. Another example is having a therapy ball is under the legs while the arms are on the floor supporting the rest of the body off of the floor.

A 5-year-old boy with spastic diplegia has been attending physical therapy for the past 4 weeks. A long-term goal is for the patient to have independent sitting balance on a chair or bench. Currently he can sit independently in a chair with a chair back and no armrests, but on an unsupported surface, he requires close contact guard. The BEST exercise for today's session is to have him sit: A. on the floor with legs crossed and try and to throw a foam basketball into a nearby low hoop. B. on a bench and give "high 5's" to the therapist while the therapist moves the hand/target in various directions to challenge his balance. C. on a bench while the therapist applies gentle perturbations for 30 seconds, followed by 1 minute of rest, for 10 repetitions. D. on a therapy ball while the therapist applies gentle perturbations for 30 seconds, followed by 1 minute of rest, for 10 repetitions.

Correct answer: B When treating a pediatric patient, the therapist should use play activities that focus on treatment goals. Throwing a basketball or reaching to give a "high 5" can be fun activities, whereas holding a position for 30 seconds and then sitting still for 1 full minute, and repeating that activity 10 times, would be less easily tolerated by a 5-year-old child. Answer A:The goal is to achieve independent sitting balance on a chair or bench, and so this is less task-specific and not the best choice. In addition, the spasticity in the lower extremities might make it difficult to arrange them in a cross-legged position on the floor, although all patients differ in their level of spasticity. Answer C:This activity will help improve sitting balance, but it might be difficult to get a 5-year-old patient to comply because, as described here, this activity is long and monotonous. Answer D:This activity will help improve sitting balance but might be too advanced because he cannot sit independently on a firm surface yet. In addition, it could be difficult to get a 5-year-old to comply because, as described here, this activity is long and monotonous. Bottom Line:When treating a pediatric patient, the therapist should use play activities that focus on treatment goals and that are fun, such as reaching to give a "high 5."

Postural drainage would be LEAST effective for which of the following patients? a) A patient with bacterial pneumonia b) A patient with a pleural effusion c) A patient with cystic fibrosis d) A patient with bronchiectasis

Correct answer: B With a pleural effusion, fluid is outside of the lungs, not in the bronchial tree, and so postural drainage positions would not alter the location of the fluid. Answer A:A patient with a bacterial pneumonia would benefit from postural drainage because they have secretions. Answer C:A patient with cystic fibrosis would benefit from postural drainage because they have secretions. Answer D:A patient with bronchiectasis would benefit from postural drainage because they have secretions. Bottom Line:Postural drainage is an intervention for drainage of secretions from one or more lung segments to the central airways using the effect of gravity. It is indicated when there is difficulty with secretion clearance, high sputum production, or the presence of certain diseases like cystic fibrosis and bronchiectasis. For fluid outside of the bronchial tree, such as pleural effusion, where the fluid is between the lungs and the pleural cavity, postural drainage is not effective.

A physical therapist is working with a patient who has had a stroke. The patient reports difficulty engaging the brakes on the wheelchair because of motor control and strength impairments on the more involved upper extremity. Which of the following is the wheelchair option that would maximize the patient's ability to engage the brakes? a) push-to-lock brake b) brake lever extension c) pull-to-lock brake d) scissor brake

Correct answer: B Abrake lever extensionwill increase the patient's leverage and thus make it easier for them to engage the brakes. Answer A:A push-to-lock brake does not make it easier or harder to engage the brake compared to other brake types. Answer C:A pull-to-lock brake does not make it easier or harder to engage the brake compared to other brake types. Answer D:A scissor brake positions the brakes under the seat, which may make propulsion easier since the brakes are not in the way of the wheels. However, it may be harder for a patient to reach a scissor brake, so they may be more difficult to engage compared to other brake types.

If the treatment approach for a non-infected wound is to liquefy the necrotic tissue in the wound, which of the following is the MOST appropriate type of debridement? a) maggots b) autolytic c) wet-to-dry dressings d) soft abrasion

Correct answer: B Autolytic debridement uses moisture-retentive dressings to seal the wound closed and allow the body's own endogenous enzymes to break down and liquefy the necrotic tissue. It takes longer than other methods but is less invasive and requires less skill. Dressings include hydrogels, hydrocolloids, transparent films, and semipermeable foams. Answer A:Maggots ingest necrotic tissue and spare viable tissue. Answer C:Wet-to-dry dressings are applied to a wound. When the necrotic tissue adheres to the most gauze and the gauze is dry, the gauze is pulled away. This method is not recommended for wounds that have greater than 50% granulation tissue as the gauze removes the healthy granulation tissue as well. Answer D:Soft abrasion is a type of mechanical debridement that uses a piece of gauze or swab to remove non-adherent, moist necrotic tissue.

A physical therapist is completing an initial examination with a patient who has a history of stroke. The patient's right eyelid is droopy and unable to open fully. The patient also is unable to move their eye medially during an oculomotor screen. Which cranial nerve was affected to cause these symptoms? a) I b) III c) IV d) VI

Correct answer: B Cranial nerve III is the oculomotor nerve, which controls the extraocular eye movements of adduction, elevation, and depression, as well as elevation of the eyelid. Answer A:Cranial nerve I is the olfactory nerve, which allows for the sense of smell. Answer C:Cranial nerve IV is the trochlear nerve, which controls the extraocular eye movement of depression of the adducted eye. Answer D:Cranial nerve VI is the abducens nerve, which controls the extraocular eye movement of abduction. Bottom Line:Cranial nerve III is the oculomotor nerve, which controls the extraocular eye movements of adduction, elevation, and depression as well as elevation of the eyelid. Physical therapists must be able to identify impairments and functional limitations associated with injuries to the cranial nerves in order to adjust their examination strategy and treatment plans accordingly.

A 54-year-old patient presents with onset of acute low back pain (LBP) rated as 8/10 after lifting a heavy box at home. She denies radicular symptoms and has significant muscle spasms throughout the lumbar paraspinals. Of note, she was diagnosed with uterine cancer 3 months ago and is actively undergoing radiation to shrink the tumor. What modality is MOST appropriate to incorporate into the plan of care? a) electrical stimulation to the affected area b) cold pack to the affected area c) ultrasound to the affected area d) moist hot pack to the affected area

Correct answer: B Cryotherapy is most appropriate for this patient. A cold pack can help to control acute inflammation, decrease pain, and decrease muscle spasms. The malignancy would not be affected by the use of ice on the low back.History of malignancy would exclude any heating modality, ultrasound, and use of electrical stimulation from the plan of care in most cases. Answer A:The use of electrical currents is a precaution for a patient with a malignant tumor. Although no research has explored the effect of electrical stimulation on tumors, because electrical current can enhance tissue growth, it should be avoided. Electrical stimulation should not be applied to any area of the body of a patient with a known malignancy due to risk of metastasis. Answer C:It is recommended that ultrasound not be applied to malignant tumors. A physical therapist should consult the referring physician before applying ultrasound to a patient with a history of malignancy within the last 5 years. Answer D:Heat is contraindicated with this patient. Thermotherapy may increase the growth rate or rate of metastasis of malignant tissue by increasing circulation to the area or by increasing the metabolic rate.

An individual is seen in an outpatient clinic following a distal femur fracture that was placed in a long leg cast for 8 weeks and is fully healed. There is now a 10 degree flexion contracture of the knee. Which of the following manual techniques would be MOST effective to treat this patient's dysfunction? a) anterior glide of the proximal tibia with internal tibial rotation b) anterior glide of the proximal tibia with external tibial rotation c) posterior glide of the proximal tibia with internal tibial rotation d) posterior glide of the proximal tibia with external tibial rotation

Correct answer: B In order to extend the knee, the concaveproximal tibia glides anteriorly and also externally rotatesin open chain in the final degrees of knee extension. This option addresses both components of the motion and is the most effective technique to restore full knee extension after development of a knee flexion contracture. Answer A:Anterior glide of the proximal tibia will increase knee extension, but the tibia needs to externally rotate in the final degrees of knee extension, not internally rotate. Answer C:A posterior glide of the proximal tibia with internal tibial rotation will increase knee flexion, not extension. Answer D:A posterior glide of the proximal tibia will increase flexion, so this would not be the most effective technique even though the tibia does externally rotate in open chain as a component of knee extension.

A therapist reads in a chart that the patient has a burn over the entire surface of the right upper extremity. Which of the following is the correct PERCENTAGE of body surface area burned, according to the rule of nines? a) 4.5% b) 9% c) 18% d) not enough information to calculate

Correct answer: B One upper extremity including the dorsal and ventral surfaces totals9%. Answer A:The anterior or posterior surface only of the upper extremity (UE) is 4.5% Answer C:Both upper extremities together would total 18%. Answer D:There is sufficient information to utilize the rule of nines to estimate total body surface area burned. Bottom Line:The rule of nines estimates the total body surface area burned and thus assists in calculating appropriate fluid resuscitation measures. The entire head is estimated as 9% (4.5% for anterior and posterior). The entire trunk is estimated at 36% (18% for anterior trunk and 18% for posterior). The groin is estimated at 1%. The upper extremities total 18% and thus 9% for each upper extremity. Each upper extremity can be further divided into anterior (4.5%) and posterior (4.5%). The lower extremities are estimated at 36%, 18% for each lower extremity (9% anterior and 9% posterior)

A physical therapist is examining a patient with a complete spinal cord injury due to a fracture at C5. The ASIA exam reported a motor level of C6, a sensory level of C7, and a zone of partial preservation of C8. What is the patient's NEUROLOGICAL LEVEL? a) C5 b) C6 c) C7 d) C8

Correct answer: B The neurological level of injury for a complete spinal cord injury is the most cephalad of the motor or sensory levels. In this case, the motor level is C6 and the sensory level is C7, so the neurological level is C6. Answer A:The location of mechanical injury often does not represent where the spinal cord injury occurs in many patients. Answer C:The sensory level is the most caudal intact dermatome for both sides of the spinal cord with respect to light touch and pin prick. Answer D:The zone of partial preservation in the most caudal level with any sensory or motor function for a patient with a complete spinal cord injury.

A 72-year-old male with congestive heart failure (CHF) has been cleared for exercise by his cardiologist. The physical therapist performs a 6-minute walk test (6MWT) at the initial visit to assess the patient's aerobic capacity. The patient achieved a distance of 88 m (288 feet). Which of the following is the MOST accurate interpretation of the results? a) the patient achieved a distance that is much higher than the normative value for his age group which is predictive of a positive outcome b) the patient achieved a distance that is lower than the normative value for his diagnosis which is predictive of poor outcomes c) the patient achieved a normal distance for his diagnosis which is predictive of a positive outcome d) normative values have not been established for his age group

Correct answer: B The normative value for the 6MWT varies depending on age and diagnosis. However, a distance of 88 m is considered to be quite low. For a 72-year-old male with heart failure, the average distance should be around 300 m. Of note, one study (Tabata et al 2014) found that 6MWT distance is an independent predictor of readmission to the hospital for heart failure. In that study, a cut-off distance of less than 390 m was found to be predictive of readmission. Answer A:This answer is incorrect because 88 m is well below the normative value of the 6MWT for this patient's age group. Answer C:This answer is incorrect because 88 m is well below the normative value of the 6MWT for this patient's age group. It is not predictive of good outcomes. In fact, for individuals with a diagnosis of heart failure, a 6MWT distance of less than 390 m has been shown to be predictive of readmission to the hospital. Answer D:This answer is incorrect because normative values for the 6MWT have been well established for different age groups. In one study (Steffan et al 2002), the average distance achieved for males 70-79 years was found to be 527 m. Bottom Line:The 6MWT is a very useful tool for the assessment of aerobic capacity for individuals with various diagnoses. It also has specific clinical utility for cardiac patients. Although it is not always necessary to know the exact normative values or cut-off values for this test, it is helpful to be aware of a typical range for different age groups. Regardless, it is important to understand that a distance of only 88 m is very low and is abnormal in most cases.

A 70-year-old woman was referred to physical therapy with a 6-month history of progressive gait difficulties, left-leg numbness, and urinary problems. Previously the patient was able to walk 2 to 3 miles a day until she noticed gait difficulties and bilateral leg stiffness. She complained that she was not able to fully control her legs. The therapist would like to test the lower extremities for vibration sense. What pathway carries this information? a) lateral corticospinal b) posterior column medial lemniscal c) anterolateral d) spinothalamic

Correct answer: B The posterior column-medial lemniscal pathway conveys proprioception, vibration sense, and fine, discriminative touch. This would be the appropriate pathway to test vibration. Answer A:The lateral corticospinal pathway controls movement of the extremities and not vibration sense. Answers C & D:The anterolateral pathway includes the spinothalamic tract, which conveys pain, temperature, and crude touch. This would not test for vibration sense.

A physical therapist is reviewing a patient's medical file before the initial examination. The x-ray report indicates a Hill-Sachs lesion at the right shoulder. This finding is associated with which of the following types of trauma? a) acromioclavicular joint separation b) avulsion of the supraspinatous c) glenohumeral dislocation d) humeral neck fracture

Correct answer: C A Hill-Sachs lesion is a compression fracture of the posterior surface of the humeral head seen after an anterior/inferior glenohumeral joint dislocation where the posterior humeral head impacts the inferior glenoid rim. Answer A:A Hill-Sachs lesion occurs with a glenohumeral joint dislocation, not an acromioclavicular (AC) separation. Answer B:A Hill-Sachs lesion occurs after a glenohumeral joint dislocation. Answer D:A Hill Sachs lesion occurs after a glenohumeral joint dislocation and is a compression fracture of the posterior humeral head, not the humeral neck.

A physical therapist is planning a research study investigating the relationship between plantar flexor strength and gait speed. Gait speed will be measured with motion analysis equipment, and strength will be measured with a hand-held dynamometer. What type of statistical test would be used to determine if there was a linear association/relationship between gait speed and plantar flexor strength? a) chi square b) independent t test c) Pearson r d) repeated measures analysis of variance

Correct answer: C A Pearson r correlation is used to determine the relationship between two variables that are interval or ratio level data. Since both gait speed and plantar flexor strength are ratio level data and the research question was attempting to determine the relationship between these variables, a Pearson r would be an appropriate statistic to utilize. Answer A:A Chi square is used to analyze nominal level data to determine if the proportion of categories of data differ from what you would expect to find by chance. Since both gait speed and plantar flexor strength are ratio level data and the research question was attempting to determine the relationship between these variables, a Chi square would be an incorrect statistic for this analysis. Answer B:An independent t test is used to compare the means of interval or ratio data from two independent groups of individuals. Since both gait speed and plantar flexor strength are ratio level data and the research question was attempting to determine the relationship between these variables, an independent t test would be an incorrect statistic for this analysis. Answer D:A repeated measures analysis of variance is used to compare three or more measures of interval or ratio level data collected from the same researchsubject over time. Since both gait speed and plantar flexor strength are ratio level data and the research question was attempting to determine the relationship between these variables, a repeated measures analysis of variance would be an incorrect statistic for this analysis. Bottom Line:A Pearson r correlation is used to determine the relationship between two variables that are interval or ratio level data.

A chest radiograph (x-ray) would be MOST appropriate for assessing which of the following conditions? a) aortic stenosis b) coronary artery disease c) pneumothorax d) pulmonary embolism

Correct answer: C A chest radiograph, or x-ray, uses radiation to produce a two-dimensional image of a patient's anatomical structures. Air appears black on an x-ray due to its lack of density while bone appears white since it is very dense. Therefore, with a full inspiration, normal lung tissue should appear to be black where the lungs should sit anatomically. In the case of a pneumothorax, where air leaks out of the lungs and into the pleural cavity, and subsequently collapses the lung, reduced lung volumes on the affected side should be apparent on a chest x-ray compared to the normal side. Answer A:Aortic stenosis (AS) is the narrowing of the aortic valve, which is where blood exits from the left ventricle to the aorta. AS is best diagnosed by heart catheterization or echocardiography. Although with AS changes in the heart and the chest may be present in a chest radiograph, it does not confirm its diagnosis. Therefore, this answer is incorrect. Answer B:Coronary artery disease (CAD) is the narrowing of the coronary arteries that supply the heart with oxygenated blood. It cannot be seen in a chest radiograph because the affected tissues are too small and cannot be differentiated in a single two-dimensional view. CAD is typically evaluated by a heart catheterization, where a long thin tube is inserted via an artery and eventually into the vessels of the heart. Newer technologies such as cardiac magnetic resonance imaging (MRI) and cardiac computed tomography (CT) also can be used to evaluate for CAD. Answer D:A pulmonary embolism (PE) is a blood clot in the lung. Although some abnormal signs can sometimes be appreciated on a chest radiograph, it is not diagnostic of a PE. In many cases, a work-up of a PE may be ordered as a result of abnormal findings on a chest radiograph. However, a chest radiograph alone does not confirm a diagnosis. Therefore this answer is not the most appropriate choice. Bottom Line:Chest radiographs, or x-rays, can provide plenty of useful information for the physical therapist. However, it is important to be aware of the limitations of x-rays as a diagnostic tool. The main purpose of a chest x-ray is to screen for anatomical abnormalities and pathological processes in the chest and thorax. A pneumothorax, where air leaks out of the lungs and into the pleural cavity, and subsequently collapses the lung, shows reduced lung volumes on the affected side compared to the normal side on x-ray.

Which of the following recommendations regarding proper positioning is MOST appropriate to tell a patient with a transfemoral amputation who is being discharged home? A. Whenever you lie supine, place a small pillow under most distal portion of limb to elevate slightly B. Try to continuously increase amount of time sitting in a chair C. Try to lie prone for a few minutes throughout the day D. When in the wheelchair, elevate the leg rest to provide support for the thigh

Correct answer: C A common complication after transfemoral amputation is a hip flexion contracture. To avoid getting a contracture the patient needs to avoid positions where the hip is flexed for long periods of time, and try to stretch the muscles out by lying prone. Answers A & B:This position encourages hip flexion and will feed into the contracture. Answer D:The patient has a transfemoral amputation, and the thigh will be supported by the seat of the chair. An elevated leg rest will not necessarily make much of an improvement in limb support. Bottom Line:After transfemoral amputation, it is important to avoid positions of prolonged hip flexion and also hip abduction.

A physical therapist is completing a chart review for a patient who was in a motor vehicle accident. The chart reported that the patient had a traumatic brain injury when their skull hit the windshield, causing a contusion to the frontal lobe. The neurosurgeon described the mechanism of injury as a coup-contrecoup injury. What area of the brain would you also expect to be damaged with this coup-contrecoup scenario? a) basal ganglia b) midbrain c) occipital lobe d) temporal lobe

Correct answer: C A coup-contrecoup injury occurs when the brain is damaged in a high-speed accident. The initial injury (coup to the frontal lobe) causes the brain to rapidly go in the opposite direction and hit the skull (contrecoup), which causes additional injury to the occipital lobe. Answer A:The basal ganglia may be affected in a high-velocity impact due to diffuse axonal injury but would not be the site of the contrecoup injury. Answer B:The midbrain may be affected in a high-velocity impact due to diffuse axonal injury but would not be the site of the contrecoup injury. Answer D:The temporal lobe would not be the site of the contrecoup injury as it is located more laterally than posteriorly. Bottom Line:Understanding the mechanism of traumatic brain injury is important for physical therapists to be able to predict impairments and plan their inital examinations with this patient population. In a coup-contrecoup injury, the initial injury (coup to the frontal lobe) causes the brain to rapidly go in the opposite direction and hit the skull (contrecoup), which causes additional injury to the occipital lobe.

The physical therapist examines a patient following a stroke for the presence of an upper motor neuron lesion. The therapist documents the grade of the patellar deep tendon reflex as 3+. The response characteristic of a 3+ deep tendon reflex grade is: A. strong muscle contraction with sustained clonus. B. slight muscle contraction with slight joint movement. C. clearly visible, brisk muscle contraction with moderate joint movement. D. strong muscle contraction with 1 to 3 beats of clonus.

Correct answer: C A deep tendon response (DTR) with a grade of 3+ would be consistent with hyperreflexia. In hyperreflexia, the therapist should see a clearly visible, brisk muscle contraction with moderate joint movement. Answer A:Strong muscle contraction with sustained clonus is consistent in an abnormal response and would be graded a 5+. Answer B:Slight muscle contraction with slight joint movement is consistent in a normal response and would be graded a 2+. Answer D:Strong muscle contraction with 1 to 3 beats of clonus is consistent with an abnormal response and would be graded a 4+. Bottom Line:A deep tendon reflex with a grade of 3+ would be consistent with hyperreflexia. In hyperreflexia, the therapist should see a clearly visible, brisk muscle contraction with moderate joint movement.

Researchers at a medical center hypothesized that stress would be correlated with oxygen uptake and power measured during cycle ergometry. Identify the type of hypothesis described. A. Statistical hypothesis B. Directional hypothesis C. Nondirectional hypothesis D. Null hypothesis

Correct answer: C A nondirectional hypothesis states that there will be a difference between the variables but does not specify what that difference will be. Answer A:This is the term to describe the two types of hypotheses, null and alternative. Answer B:A directional hypothesis specifies the expected direction of the difference between the means of the variables being studied. For example, A will be significantly greater than B. Answer D:The null hypothesis predicts that there will be no treatment effect (i.e.; that there will be no significant difference in the means of the variables being studied). Bottom Line:A nondirectional hypothesis states that there will be a difference between the variables but does not specify what that difference will be. A directional hypothesis specifies the expected direction of the difference between the means of the variables being studied. For example, A will be significantly greater than B.

A physical therapist is treating a man with type 1 diabetes who is in cardiac rehabilitation. Prior to the session, the patient's serum glucose reading is 55 mg/dL. The MOST appropriate course of action is to: A. ask the patient when he last took insulin. B. instruct the patient to begin his exercise program. C. give the patient 4 ounces of juice. D. wait 5 minutes and then have the patient recheck his glucose levels.

Correct answer: C A normal fasting serum glucose level is 70 mg/dL to 100 mg/dL. A patient whose blood glucose level is 55 mg/dL should ingest some form of carbohydrate, such as juice, before exercise so that the serum glucose is above 100 mg/dL. Answer A:Although asking the patient when he last took insulin would provide information about the likely direction of his serum glucose levels, a serum glucose level of 55 mg/dL requires immediate additional carbohydrates to bring the glucose level to higher than 100 mg/dL prior to exercise. Answer B:Because exercise would make the patient's glucose levels decrease further, instructing the patient to start his exercise program would not be an appropriate response to the situation. Answer D:Because a blood glucose level of 55 mg/dL requires additional immediate carbohydrates to get the glucose levels above 100 mg/dL prior to exercise, waiting and rechecking glucose levels would not be an appropriate response to the situation.

A physical therapist is performing the initial evaluation on a patient in their home. The medical notes indicate that the patient was diagnosed as having a deep vein thrombosis 2 days before their recent discharge from the hospital. The patient is alert and oriented, they are wearing compression stockings, and they are eager to commence therapy. Which of the following actions is MOST appropriate for the physical therapist to perform? a) complete the history but do not do any physical testing or mobility testing at this time; wait another 24 hours b) do not initiate the evaluation; call the physician to clear activities c) mobilize the patient after confirming they are on anticoagulation therapy d) recommend that the patient should be on full bed rest for another 5 days

Correct answer: C A patient who has been diagnosed with a deep vein thrombosis (DVT) should immediately receive anticoagulation such as heparin, then continue with long-term oral anticoagulants such as warfarin. Bed rest is recommended only for the first 24 hours, after which, provided the anticoagulation therapy has started, and ideally with the support of compression stockings, the patient should be mobilized. Answer A:After the diagnosis of a DVT, anticoagulant therapy should commence, but only 24 hours of bed rest is recommended before mobilization. This patient is now at least 3 days post initial diagnosis and, provided anticoagulation therapy has been started, it is safe to mobilize the patient. Answer B:Unless the patient is showing active signs of a new DVT, such as swollen calf, pain, and redness, the situation does not require a call to the physician or 911. Provided the patient is on anticoagulant therapy, they can initiate mobilization within 24 hours of initial diagnosis. Answer D:The recommended rest for a DVT is 24 hours, during which time anticoagulation therapy is commenced. Provided the therapy has been started, the patient should then be mobilized. Seven days of bed rest is not appropriate for this patient. Bottom Line:Deep vein thrombosis (DVT) is a potentially life-threatening situation. However, once it has been diagnosed, anticoagulation therapy should commence immediately, and then the patient should be mobilized after 24 hours. After that, provided the therapist confirms that the patient is on anticoagulant therapy as prescribed, it is safe to get the patient up and moving.

A physical therapist is practicing sitting balance activities at the edge of a mat with a patient with a C6 American Spinal Injury Association A spinal cord injury. When the patient is positioned with bilateral upper extremity support on the mat, which of the following positions would interfere with sitting balance? a) knees above the hips b) shoulders anterior to the hips c) trunk extended posterior to the hips d) full contact of the thigh against the mat

Correct answer: C A patient with a C6 American Spinal Injury Association (ASIA) A spinal cord injury has no trunk stability or active trunk musculature. So, a position of slight trunk extension would place the patient's center of mass outside of the base of support and would lead to a loss of balance. The position of true balance often is perceived by the patient as more posterior than its actual position, but the trunk must remain slightly anterior over the base of support. Practice and feedback structured by the physical therapist are required to teach the patient that the trunk should be more flexed to allow for improved balance. Answer A:For a patient with a C6 ASIA A spinal cord injury, the knees should be positioned slightly above the hips to maximize the patient's base of support. Answer B:For a patient with a C6 ASIA A spinal cord injury, the shoulders should be positioned slightly anterior to the hips so that the center of mass would be within the base of support. In addition, the patient would be able to use intraabdominal pressure and tension in the posterior longitudinal ligament to provide additional support. Answer D:For a patient with a C6 ASIA A spinal cord injury, the posterior thigh should have full contact against the mat to maximize the patient's base of support. Bottom Line:For a patient with a C6 American Spinal Injury Association (ASIA) A spinal cord injury, there is no trunk stability or active trunk musculature, so trunk extension is not indicated, and the patient must learn to lean forward slightly and keep the center of mass above the base of support. Leaning forward slightly also allows use of intraabdominal pressure and tension in the posterior longitudinal ligament to provide additional support. TrueLearn Insight :Physical therapists must be able to teach patients with tetraplegia due to a complete spinal cord injury how to position themselves with maximum stability while in a seated position so that they can be as independent as possible with their functional activities.

A 91-year-old patient is admitted to the hospital following a fall resulting in multiple rib fractures. He also suffered a pneumothorax and now has a chest tube. A physical therapy consult is requested. What should the physical therapist do FIRST? A. Hold the evaluation until the chest tube is removed B. Evaluate the patient, but keep treatment limited to bed mobility only C. Evaluate the patient and progress as tolerated per the patient's symptoms D. Evaluate the patient and begin resisted strengthening exercises

Correct answer: C A patient with a chest tube can be progressed as tolerated based on their symptoms, vitals, and comorbidities just as any other patient would. The only special precaution taken is to ensure that the drainage collection system does not tip and to keep it in a gravity-dependent position. Answer A:The presence of a chest tube does not preclude physical therapy. The distal end of the drainage tube may or may not be attached to a sealed drainage system, which may or may not be attached to suction. Due to the nature of the drainage collection system, clinicians are cautioned against tipping the collection system so as not to disrupt the drainage level reading. Answer B:There is no precaution with a chest tube that prevents progressing past only bed mobility. Answer D:Based on the age of the patient, presence of rib fractures, and a chest tube, the best course of action would not be to start with resistance exercises. The patient should be progressed as tolerated with general mobility, including bed mobility, transfers, and gait. There is no precaution with a chest tube preventing resistive exercises, but these would be best utilized further down the line in treatment. Bottom Line:There are many thoracic and cardiovascular surgical procedures that involve placing one or more chest tubes into the pleural space or mediastinum to evacuate air or fluid. Surgery, acute trauma, a disease, or pathological progression can all necessitate a chest tube, especially when it has triggered excess fluid, blood, or air to accumulate and leak into the chest cavity. The presence of chest tubes does not preclude the patient participation in physical therapy. In fact, coughing and deep breathing exercises can help with lung expansion and drainage. The therapist must remember to keep the collection system from tipping and also to keep it in a gravity-dependent position.

A physical therapist is completing an initial examination with a 21-year-old patient who had an ASIA A T6 spinal cord injury 3 weeks ago. She has no precautions on her activity and an unremarkable past medical history. Which of the following is the MOST appropriate pressure-relief technique for this patient to practice? a) anterior b) lateral c) push up d) tilt

Correct answer: C A patient with an ASIA A T6 spinal cord injury would have 5/5 motor strength in the triceps so that they would be able to weight shift by pushing up from the wheelchair and locking their arms into extension to completely clear their ischial tuberosities for approximately 30 seconds. A push-up relief requires the least amount of time to complete and requires no additional equipment. Therefore, a push up weight shift is an appropriate discharge goal for pressure relief for a young patient without comorbidities. Pressure ulcers are likely to develop if an incorrect method of pressure relief is recommended by the physical therapist. Answer A:An anterior weight shift is completed by leaning forward onto a supporting surface to take pressure off the ischial tuberosities for a period of approximately 2 minutes. Due to the time it takes to complete the weight shift, an anterior weight shift would not be an appropriate discharge goal for a young patient with intact triceps and no comorbidities. Answer B:A lateral weight shift is completed by leaning to each side one at a time to take pressure off the ischial tuberosities for a period of approximately 2 minutes. Due to the time it takes to complete the weight shift, a lateral weight shift would not be an appropriate discharge goal for a young patient with intact triceps and no comorbidities. Answer D:A tilt weight shift is performed by using a specialized wheelchair to remove pressure from the ischial tuberosities by tilting the seat and back of the wheelchair to a reclined position for a period of approximately 5 minutes. Due to the need for a specialized wheelchair and the time it takes to complete the weight shift, a tilt weight shift would not be an appropriate discharge goal for a young patient with intact triceps and no comorbidities. Bottom Line: The push-up method is the method of choice for an SCI patient with intact triceps innervation. Pressure ulcers are likely to develop if an incorrect method of pressure relief is recommended by the physical therapist

A 21-year-old patient is being seen in an outpatient physical therapy clinic for pain in the mid-back area. All red flags have been cleared. The patient has thoracic hypomobility and a gross decrease in trunk strength. Observation of forward flexion revealed a rib hump on the right side. Which of the following is the MOST appropriate intervention for this patient? A. Perform prone thoracic unilateral posterior-to-anterior mobilizations on the left transverse processes B. Perform prone thoracic central posterior-to-anterior mobilizations on the spinous processes C. Instruct the patient in a quadruped thoracic rotation exercise reaching under and across the body with the right arm D. Instruct the patient in a quadruped thoracic rotation exercise reaching under and across the body with the left arm

Correct answer: C A rib hump on the right means that the thoracic vertebrae in that region are rotated to the right. As the vertebrae rotate right, because the ribs are attached, the left side of the ribs would protrude more anteriorly, and the right side of the ribs would protrude more posteriorly, creating a rib hump seen when the patient flexes forward. Scoliosis interventions should focus on counter balancing the curve, whether with a manual technique or therapeutic exercise. Therefore, an exercise that encourages left thoracic rotation, by reaching across with the right arm, is most appropriate. Answer A:A posterior-to-anterior mobilization force on the left transverse process would cause right rotation, and so it is not appropriate. Answer B:A central posterior-to-anterior mobilization over the spinous process would improve thoracic extension, which is not a part of the scoliotic pattern and therefore would not be the most appropriate intervention at this time. Answer D:A rib hump on the right revealed right thoracic rotation; therefore, a right thoracic rotation exercise is not appropriate. Bottom Line:A rib hump on the right means that the thoracic vertebrae in that region are rotated to the right. As the vertebrae rotate right, because the ribs are attached, the left side of the ribs would protrude more anteriorly, and the right side of the ribs would protrude more posteriorly.

A physical therapist is trying to determine the best intervention for a patient. The therapist has located four different research articles regarding the intervention they are considering. When the therapist is reading the results section, which of the following statistical statements would give the therapist the greatest confidence that the intervention should be considered? A. p = .3 B. effect size = .13 C. Significance level of .01 D. Alpha level of 1.0

Correct answer: C A significance level of .01 demonstrates the most substantial level of significance and demonstrates that the occurrence of the results was not due to chance. Most statistical analyses use a significance level of .05 or .01 with the latter being the most stringent statistical cutoff. Answer A:A report of p = .3 is the method of reporting the significance or alpha levels, and .3 would not be considered a good result. This would indicate that there is a higher probability of the results being due to chance rather than being significant. A p level of .01 or .05 is considered appropriate. Answer B:The effect size is the statistical indicator of the impact the intervention had in comparison to the control group. An effect size of .13 is considered small and is therefore not as strong an indicator that the intervention is appropriate, even though it may have been statistically significant. A large effect size, statistically significant, would be the stronger support for the intervention. Answer D:Alpha levels are another way of saying "significance level." The two are the same, but researchers choose one term or the other. In both cases, .05 or .01 is normal, with .01 being the most significant. An alpha level of 1.0 is very high and indicates a strong probability that the result was by chance. Bottom Line:Understanding what the researcher is reporting is an important step to correctly interpreting the research. In this question, the concepts of statistical significance were presented in several ways (alpha, p values, significance levels), all of which are showing the same thing, the best of which is .01. Effect size demonstrates the impact of the experiment or intervention, and .13 is small.

A physical therapist is treating a patient with back pain and notices the patient has a limp. The therapist suspects that the leg lengths may be unequal. The physical therapist wants to assess the leg lengths for any anatomical discrepancies. Which of the following is the MOST accurate method to determine if there is a true leg-length discrepancy? A. Perform a supine to long sit test and monitor how the malleoli line up B. Tape measure from umbilicus to lateral malleolus C. Tape measure from ASIS to medial malleolus D. Tape measure from umbilicus to medial malleolus

Correct answer: C ASIS to medial malleolus will give the measurement of the leg length without the influence of a change in pelvic position. It does not determine whether the discrepancy is in the femur or tibia, but it will identify a total difference. Answer A:The supine to long sit test would identify a functional leg-length discrepancy due to a posteriorly rotated innominate. Answers B & D:This technique looks for an apparent or functional leg-length discrepancy.

A 50-year-old male has been referred by his primary care physician to physical therapy for management of chronic knee pain from osteoarthritis. A review of the patient's medical history reveals hypertension, hyperlipidemia, diabetes mellitus (type II), and current tobacco use. Prior to arriving to the clinic today, the patient fell onto his knee in the shower this morning. He ambulates with an antalgic gait and rates his pain as a 9/10. Upon physical examination, the therapist notes diffuse swelling of the knee and lower limb. The calf and foot are cold with absent dorsalis pedis and posterior tibial pulses. Which of the following should be the NEXT course of action?

Correct answer: C Acute compartment syndrome occurs when there is an excessive buildup of pressure within a compartment of a limb or an extremity, typically caused by an acute injury such as fracture or loss of blood supply. The anterior portion of the lower limb does not have room to accommodate a lot of swelling as the fascia is quite inflexible. Signs and symptoms include the 6 P's: pain, parasthesia, paralysis, poikilothermia, pallor, and pulselessness. Findings of a cold limb with absent pulses are very concerning, especially in the setting of a recent injury. Therefore, the current history combined with the patient's physical presentation indicate a good possibility for acute compartment syndrome, which requires prompt medical attention as soon as possible. If left untreated, acute compartment syndrome can cause permanent muscle or nerve damage. Answer A:This answer is incorrect because it delays the most appropriate medical treatment. The most immediate concerns in this patient's case are ischemic pain and pulselessness in the setting of an acute injury, and they must be addressed immediately. While the patient may have peripheral vascular disease based on his risk factors, the current symptoms nonetheless should necessitate referral to the emergency department. Answer B:In this patient's case, having a cold limb with absent pulses after an acute injury, excessive swelling and 9/10 pain should be considered an emergency. Like the other incorrect answers, this answer is incorrect because it delays the MOST appropriate medical treatment. Answer D:Like the other incorrect answers, this answer is incorrect because it delays the MOST appropriate medical treatment. A cold limb with absent pulses and extreme ischemic pain needs to be treated in the emergency department, most likely with a fascial release. Bottom Line:Acute compartment syndrome occurs rarely. However, when it occurs, it is a time-sensitive emergency, so it must be ruled out if found in the right setting and physical presentation. Patients present with the 6 P's: pain, parasthesia, paralysis, poikilothermia, pallor, and pulselessness. Findings of a cold limb with absent pulses are very concerning, especially in the setting of a recent injury. Understanding of differential diagnosis can help physical therapists in screening for medical emergencies in order to best protect patient safety and prevent harm.

A physical therapist is assessing running in a patient who complains of pain on the inside of the right knee. On examination, the therapist finds right "out toeing"during the running pattern. Which of the following would be the cause of these findings? a) adaptive shortening of the hip internal rotators b) adaptive shortening of the ITB c) adaptive shortening of the hip external rotators d) adaptive shortening of quadratus lumborum

Correct answer: C Adaptive shortening of the hip external rotators leads to abnormal external rotation and a "toe out" running pattern. Answer A: Adaptive shortening of the hip internal rotators leads to abnormal internal rotation and a "toe in" running pattern. Answer B: Adaptive shortening of the iliotibial band leads to abnormal internal rotation and a "toe in" running pattern. Answer D: Adaptive shortening of the quadratus lumborum leads to pelvic drop during stance.

A 90-year-old patient is working with physical therapy following a recent fall with a traumatic hip fracture. The patient underwent open reduction and internal fixation (ORIF) surgery and is currently weight-bearing as tolerated and working on functional transfers. To date, the patient has always transferred to the stronger side. Which of the following statements would be correct if the patient requested to start transferring into their weaker or affected side? A. It would not prove beneficial, and all transfers should be done to the strong side B. There is a greater risk of damage to the surgical site if transfers are done toward the affected side C. It will increase proprioception and kinesthesia and would benefit the weaker side D. It will make the stronger side work harder and is therefore a good strengthening activity

Correct answer: C Although transfers are initially practiced into the strong side for safety, working toward the affected side will increase proprioception and kinesthesia in the weaker side. Transferring both ways also prepares the patient for more functional transfers when they might have to move in either direction. Answer A:Transfers are often done to the strong side initially for safety but should always be practiced in both directions for improved proprioception and kinesthesia and also to ensure functional levels when transfers may be made in either direction. Answer B:There is no known additional risk to the surgical site particularly with open reduction and internal fixation (ORIF) where the only contraindication is usually a weight-bearing restriction. Answer D:Strengthening is more likely to occur through the increased use of the weaker side during a transfer into the weak side, so this is not a strengthening activity for the already stronger side. Bottom Line:Transfers must be practiced in both directions to ensure that the patient can be functionally successful in their home, where they may need to transfer in either direction. Initially, transfers are usually practiced toward the stronger side for safety and success, but transferring into the weaker side increases proprioception and kinesthesia and increases the function of the weaker side.

A physical therapist is teaching a patient with a history of a T6 spinal cord injury several methods for assisted cough. Which of the following describes the BEST method for this patient to cough? a) anterior to posterior compression of the upper chest wall at the level of the sternum as the patient initiates cough b) forceful pressure at the upper back in a forward and downward direction as the patient initiates cough c) forceful pressure at the abdomen in a posterior and upward direction as the patient initiates cough d) huffing

Correct answer: C An individual with a T6 spinal cord injury is likely to lack innervation to the abdominal muscles. This answer is correct because it describes a method of assisting the patient in building enough abdominal pressure to generate increased expiratory flow for an effective cough. Answer A:An individual with a T6 spinal cord injury is likely to have innervation to muscles of the upper trunk. Therefore, support in the form of compression at this level is likely not necessary. Answer B:The method of assistance described may provide increased forward lean, leading to compression of the abdomen for the patient and in turn assist in generating increased intraabdominal pressure. However, it is not the best answer because a better method of assistance would be direct support for the abdominal muscles. Answer D:Huffing is a method of bringing up mucus from the lower airways of the lungs. It requires repeated forced exhalation from a combination of increased abdominal pressure and sometimes upper trunk compression with upper extremity adduction. This answer is incorrect because an individual with a spinal cord injury is unlikely to be able to generate enough abdominal pressure to perform huffing. Bottom Line:Although a spinal cord injury is primarily a neuromuscular diagnosis, cardiopulmonary compromise is not uncommon. Depending on the level of injury, these patients likely will lack innervation to muscles that support the trunk and/or abdomen and will be unable to generate enough pressure for an effective cough. Therefore, it is important for them to have strategies for assisted coughing or other methods of airway clearance. In this case,forceful pressure at the abdomen in a posterior and upward direction as the patient initiates couch assists in airway clearance.

A physical therapist is treating a patient who has left hemiplegia that occurred 1 year ago. During gait, the left knee tends to buckle occasionally during stance. After approximately 75 feet, the patient also has difficulty clearing the foot during swing due to an inability to recruit the ankle dorsiflexors. Which of the following orthoses will IMPROVE the patient's pattern of gait and optimize function? A. KAFO (knee ankle foot orthosis) with the ankle set in 5 degrees of plantarflexion B. MAFO (molded ankle foot orthosis) with the ankle set in 5 degrees of dorsiflexion C. MAFO (molded ankle foot orthosis) with the ankle set in neutral dorsiflexion D. MAFO (molded ankle foot orthosis) with the ankle set in 5 degrees of plantarflexion

Correct answer: C An orthosis would be indicated in this scenario since the patient is 1 year post injury, so that further recovery is less likely than if the injury was more recent and because the patient exhibits functional deficits in ambulation. A MAFO (molded ankle foot orthosis) would be preferred to a KAFO (knee ankle foot orthosis) since the left knee is largely stable during stance so that a larger, harder to don/doff KAFO would not be recommended. A setting of the ankle at neutral dorsiflexion would assist with foot clearance during swing while preventing excessive dorsiflexion during stance, which would help prevent knee buckling. Answer A:A KAFO would not be recommended in this scenario since it would be larger and harder to don/doff than a MAFO and thus less likely to be used. An ankle setting of 5 degrees of plantarflexion would make foot clearance during swing difficult. Answer B:An ankle setting of 5 degrees of dorsiflexion would make the knee more likely to buckle during stance due to knee instability than a neutral ankle setting. Answer D:An ankle setting of 5 degrees of plantarflexion would make foot clearance during swing difficult. Bottom Line:Difficulty with isolated ankle dorsiflexion is a common issue in the hemiplegic population. An orthotic that allows the foot to safely clear the floor during gait will always position the ankle at neutral or some degree of dorsiflexion, but never plantarflexion, because there is a possibility of tripping. A molded AFO provides support to an unstable ankle in the frontal and sagittal planes.

During a functional assessment, a patient performs a squat but is unable to go down very far. The physical therapist applies a posteriorly directed force to the talus and then asks the patient to squat again. The patient is now able to squat further with increased range. Based on this response, which of the options is the BEST explanation for the initial restriction?

Correct answer: C Ankle dorsiflexion requires a posterior glide of the talus and/or an anterior glide of the distal tibia and fibula. If the bone is out of position, normal biomechanics will be disrupted. Capsular restriction also can limit the range of motion. To determine the true cause, a full examination including flexibility testing and joint mobility testing are needed. Additional information can be gained from the patient's response to a specific mobilization.A squat requires ankle dorsiflexion. In this scenario, the mobilization of the talus in a posterior direction led to immediate improvement in the amount of dorsiflexion. if the talus is out of position and sitting slightly anterior, once it is put back in place there should be an improvement noted.The tibia and fibula move anteriorly over the talus in a squat, so if the talus is out of position slightly, the normal biomechanics are thrown off. In addition, if the source of the problem was a restrictive posterior capsule that did not allow the talus to move posteriorly, then one mobilization in the posterior direction should not have such a sudden improvement because the restriction would most likely still be there. Answer A:Though excessive posterior capsule tightness limiting posterior glide of the talus does limit ankle dorsiflexion, it would not immediately improve with one posterior force to the talus. A more plausible explanation would be a talus that was shifted slightly out of position and was put back in place with the posteriorly directed force. Answer B:A shortened or inflexible Achilles tendon can limit a squat's range of motion but would not be improved with a mobilization to the ankle. Answer D:The talus glides posteriorly with ankle dorsiflexion, not anteriorly. A reduced anterior glide would limit ankle plantarflexion. Bottom Line:Ankle dorsiflexion requires a posterior glide of the talus and/or an anterior glide of the distal tibia and fibula. If the bone is out of position, normal biomechanics will be disrupted. To determine the cause, a full examination including flexibility testing and joint mobility testing are needed.

A 79-year-old patient is 1 day post-right cerebrovascular accident in the acute care setting. The therapist enters for an examination and must first assess the patient's mental status. She is drowsy initially but is able to answer some questions. She falls back asleep between questions but arouses to light touch. She is able to state her name and thinks she is at the nursing home where she resides. She is unsure of the date. How would the therapist document this patient's mental status in the chart? a) alert and oriented x1 b) alert and oriented x3 c) lethargic and oriented x1 d) lethargic and oriented x2

Correct answer: C Arousal is the physiological readiness of the human system for activity. It is described by using traditionally accepted terms to identify the patient's level of consciousness. Alert describes a patient who is awake and attentive to normal levels of stimulation. Lethargic describes a patient who is drowsy and may fall back asleep if not stimulated in some way. Orientation refers to the patient's awareness of person, place, and time. The patient is only oriented to self (oriented x 1) and is lethargic, requiring stimulation to stay on task. Answers A & B: The patient in this scenario is not alert, as she falls back asleep between questions and requires stimulation to stay on task. She is oriented x 1. Answer D: Orientation typically refers to the patient's awareness of person, place, and time. This patient is oriented only to self.

A patient with a traumatic brain injury needs a wheelchair for mobility. As part of prescribing a wheelchair, the therapist records the following measurements during the examination: pelvic width = 18 inches, lower leg length = 18 inches, upper leg length = 18 inches. Which of the following is the appropriate wheelchair seat dimension (width x length) that should be ordered for the patient? A. 18 inches wide by 16 inches long B. 18 inches wide by 18 inches long C. 20 inches wide by 16 inches long D. 20 inches wide by 18 inches long

Correct answer: C Assistive devices such as wheelchairs must be accurately prescribed to allow for maximal patient comfort and mobility while preventing skin breakdown. Physical therapists are critical members of the team of professionals who prescribe wheelchairs for patients with functional mobility deficits.In order to allow adequate seat width, 2 inches is typically added to the measurement of pelvic width. In order to allow for appropriate seat depth, 2 inches is typically subtracted from the measurement of upper leg length. A patient with pelvic width of 18 inches and upper leg length of 18 inches would require a wheelchair 20 inches wide by 16 inches long. Answer A:A wheelchair that is 18 inches wide when the pelvic width is also 18 inches would be too narrow for the patient to comfortably sit in and would place the patient at risk for skin breakdown. Answer B:A wheelchair that is 18 inches wide when the pelvic width is also 18 inches would be too narrow for the patient to comfortably sit in and would place the patient at risk for skin breakdown. A wheelchair that is 18 inches deep when the patient's upper leg length is also 18 inches would be too long for the patient to comfortably sit in and would place the patient at risk for skin breakdown. Answer D:A wheelchair that is 18 inches deep when the patient's upper leg length is also 18 inches would be too long for the patient to comfortably sit in and would place the patient at risk for skin breakdown.

An 11-year-old boy is being examined in physical therapy following an injury to his right elbow during a wrestling match. The boy fell on an outstretched hand with the elbow straight and a valgus stress applied by the other wrestler. The patient has tenderness to palpation and swelling on the ulnar side of the elbow. Passive range of motion (PROM) was within normal limits (WNL), with pain at end range elbow extension and wrist extension. Resisted motions were WNL except for pain with wrist flexion. Which of the following is the most likely diagnosis? a) medial epicondylitis b) grade 3 ulnar collateral ligament sprain c) avulsion fracture of the medial epicondyle d) elbow dislocation

Correct answer: C Avulsion fractures of the medial epicondyle are common in skeletally immature children. With a valgus stress to the elbow in a child who is skeletally immature, the growth plate at the medial epicondyle would get pulled away before having a tear of the ulnar collateral ligament. Repetitive wrist flexor actions, a fall on an extended elbow with a valgus stress, or excessive throwing/pitching at a young age can cause an avulsion at the medial epicondyle. Using the muscles that attach to the medial epicondyle, such as with wrist flexion, or stretching the muscle, such as with passive wrist extension, would pull on the fracture site and be painful. Answer A:The patient sustained a fall and a valgus force to the elbow. Medial epicondylitis is more of an overuse injury with gradual onset. Answer B:Ulnar collateral ligament sprains are not common in this age range, as the epicondyle would pull apart more easily than the ligament would pull away. In addition, the pain with resisted wrist flexion is more of a sign of the avulsion fracture as the common wrist flexor attachment is on the medial epicondyle. Answer D:The mechanism of injury could have caused an elbow dislocation; however, the patient has full range of motion, which would be unlikely if the elbow were dislocated. Bottom Line: An avulsion fracture is common in adolescent children because the growth plates can separate easily with a tensile force. A medial epicondylar fracture can occur with overuse, such as excessive throwing or pitching, or from a traumatic injury with some valgus stress. The common signs and symptoms are local tenderness to palpation, pain with resisted wrist flexion, and passive wrist extension

A patient is 4 months status post anterior cruciate ligament (ACL) repair via patellar tendon graft and is currently in the return-to-play phase of rehabilitation. While performing squats, the patient complains of anterior knee pain halfway through set two out of three sets. Which of the following options is the BEST course of action for the physical therapist? a) stop the exercise and apply ice to prevent inflammation and swelling b) alter the exercise, have the patient perform resisted seated knee extension instead c) modify the exercise, have the patient perform a more shallow squat d) alter the exercise, have the patient perform standing terminal knee extension in close chain with a resistance band instead

Correct answer: C Because the pain is felt in the anterior knee, it is suggestive of patellofemoral pain. A squat can be an excellent functional strengthening exercise when performed properly status post ACL repair, when in the return-to-sport phase. It is a functional activity that strengthen the quadriceps and hip extensors and also maintains closed chain loading through the knee joint. Thus instead of dropping the exercise altogether at this complaint of pain, a modification may solve the problem. By having the patient perform a more shallow squat, there is less knee flexion ROM, limiting excessive compression forces on the anterior knee joint. Answer A:In the return-to-play phase, the patient's goal is to get stronger. It important to first attempt to modify the exercise before having them stop altogether. In addition, the patient has already performed half of the repetitions prescribed without pain. It is unlikely that after two sets the patient is already getting inflammation. Answer B: Strengthening in the return-to-play phase should focus on function. Open chain knee extension not only compresses the patellofemoral joint but also causes anterior shear of the tibia on the femur. Activities performed in weight-bearing positions, and avoidance of excess strain on the anterior knee joint, are most beneficial in this stage of recovery. Answer D:Closed chain terminal knee extension is an excellent exercise to improve loading through the knee joint, while controlling anterior shear and patellofemoral compression. However, at this stage of recovery, terminal knee extension with a therapy band is too easy and not as functional for return-to-play. Bottom Line:Weight-bearing, closed chain exercises are important to improve strength of the core, hip, and knee when in the return-to-play phase of an ACL repair. In that phase, modifying the exercise to decrease pain is important, but it is equally important to continue to progress and challenge the patient.

A patient presents with diagnosis of right-sided Bell's Palsy. Which of the following would the therapist MOST likely see during the examination? A. Inability to open the right eye lid B. Loss of taste on the posterior two-thirds of the tongue C. Inability to close the right eye lid D. Inability to chew on the right side

Correct answer: C Bell's Palsy is the paralysis of the facial nerve (CN VII). Cranial nerve VII controls the muscles of facial expression, including the orbicularis oculi muscle, which allows control of closing the eye. Patients with Bell's palsy lose this ability, or it is at least weakened during the episode. They often have to tape the eye shut at night to keep it closed. Most will recover fully in a matter of weeks or months. Answer A:Cranial nerve III is responsible for opening of the eye, not cranial nerve VII. Answer B:The facial nerve functions in the conveyance of taste from the anterior two-thirds of the tongue (not posterior). Answer D:The muscles that control chewing (muscles of mastication) are controlled by the trigeminal nerve (cranial nerve V) and therefore would not be affected by Bell's palsy. Bottom Line:Bell's palsy is a lower motor neuron lesion involving paralysis of the facial nerve (CN VII). It is usually the result of an acute inflammatory process that causes compression on the nerve where it exits the stylomastoid foramen. The patient will usually present with facial droop,inability to close eye lid, weakness or paralysis of muscles of facial expression, and loss of taste on anterior two-thirds of tongue. 90% of patients recover fully in 2-8 weeks.

An individual presents to physical therapy with a report of pain when holding his head up to look at the computer screen at work. The physical therapist suspects hypomobility in the cervical spine. Which of the following joint mobility tests would best identify his functional deficit if limited? a) up glide b) side glide c) central posterior to anterior glide d) unilateral posterior to anterior glide

Correct answer: C Central posterior to anterior glide is the correct answer, as the patient needs symmetrical extension at both facets in order to hold the head up. Answer A:This answer is incorrect as it would address the ability to flex the spine. Answer B:This answer is incorrect as it would address quadrant/combined motions such as sidebending, rotation, or unilateral flexion/extension.Answer D:This answer is incorrect as it would address quadrant/ combined motions such as sidebending, rotation or unilateral flexion/ extension. Bottom Line:A central posterior to anterior glide is used to assess extension at the spine and can be performed with pressure through the spinous process or via bilateral transverse processes at the same time.

A physical therapist is examining a patient diagnosed with an incomplete thoracic spinal cord injury at T10. Which of these is the BEST anatomical location for measurement of middle chest wall excursion? a) sternal angle b) midpoint between the xiphoid process and umbilicus c) xiphoid process d) midpoint between the sternal angle and the xiphoid process

Correct answer: C Chest wall excursion measurements made at the xiphoid process site represent middle chest wall motion. Answer A:Chest wall excursion measurements made at the sternal angle may best represent upper chest wall motion. Answer B:Chest wall excursion measurements made at the midpoint between the xiphoid process site and the umbilicus represent lower chest wall motion. Answer D:Chest wall excursion measurement made at midpoint between the sternal angle and the xiphoid process is not a standard measurement of chest wall excursion.

A physical therapist is treating a patient who had a left middle cerebral artery stroke. When working on increasing right ankle dorsiflexion range of motion, the therapist quickly moves the patient's right ankle into dorsiflexion while the knee is extended and finds that the ankle bounces back and forth in a phasic pattern. What is the name of this clinical condition? A. associated reaction B. Babinski sign C. clonus D. rigidity

Correct answer: C Clonus is a cyclical alternation of muscular contraction and relaxation that occurs in response to sustained sudden stretch of a spastic muscle. Clonus typically occurs in the plantarflexors but also may be found in other areas of the body, such as the wrist or patella.Physical therapists who treat patients with stroke must be able to recognize the common impairments associated with central nervous system injury. Answer A:An associated reaction is an involuntary movement resulting from activity occurring in other parts of the body, such as a sneeze, yawn, or movement of another extremity. Answer B:The Babinksi sign is dorsiflexion of the great toe along with fanning of the other toes when the lateral sole of the foot is stimulated. Answer D:Rigidity is a nonvelocity-dependent increase in muscle tone. Bottom Line:Clonus is a cyclical alternation of muscular contraction and relaxation that occurs in response to sustained sudden stretch of a spastic muscle. Clonus typically occurs in the plantarflexors but also may be found in other areas of the body, such as the wrist or patella.

Which of the following environmental modifications is most appropriate for challenging advanced adaptive balance control? A. Quiet hallway B. Busy therapy gym C. Community-based program D. Simulated community setting

Correct answer: C Community-based programs, such as tai chi, produce the most advanced challenges to adaptive balance control. Adaptive balance is the ability to modify or change the balance response based on the situation by increasing either the task or the environment. Community programs provide the greatest challenges for patients. Answer A: Environmental challenges can be progressively adapted to challenge adaptive balance control. A quiet hallway would be one of the first locations due to the lack of excessive stimuli that this environment would provide. Answer B:A busy therapy gym contains multiple stimuli, but a therapy clinic is still a controlled environment compared to public places and real community settings. In general, the therapy gym would be one of the earlier locations that adaptive balance control would be practiced. Answer D:A simulated community setting allows a lot of stimuli for the client but remains controllable and therefore is still not the most advanced setting for practicing adaptive postural control. Bottom Line:Adaptive postural control is the ability of the patient to modify their balance response, either to the task, which becomes increasingly difficult, or to the environment, which becomes increasingly unpredictable. Environmental modifications progress from closed to open and would start in a quiet hallway or private room, advance to a therapy gym, and then simulate community settings. The most advanced modification would be real community settings, which provide the lowest levels of predictability and therefore the highest levels of adjustment by the patient.

A physical therapy clinic changed its hours 1 year ago. The clinic conducted a retrospective study to determine the effect of staying open an extra hour each evening on patient compliance with attending physical therapy sessions. The method they used: A. asked current patients whether keeping the clinic open later was beneficial and then calculated the percentage. B. compared patient outcomes before and after instituting the schedule change to see if there was significant improvement. C. compared the number of cancellations before and after the schedule change. D. compared compliance with home exercise programs before and after the schedule change now that there was more time to review them in the evenings.

Correct answer: C Comparing cancellations before and after the schedule change is the correct answer and describes 1 method of looking at data retrospectively to see whether the change in schedule had a significant effect on the attendance at the clinic.In any research study, it is important to align the research question with the proper independent and dependent variables that can help answer that question. If a study is looking to see if X affects Y, then measuring Z, even if it provides interesting information, will not answer the research question. In this case, the only dependent variables are whether patients cancelled or did not cancel after the schedule change. Answer A:The study was examining attendance, only whether patients did or did not attend, and not how the patients felt about it. In addition, this incorrect option did not include looking at the attendance data pre- and post-schedule change; it just mentioned asking current patients. Answer B:The dependent variable here was the attendance, not patient outcomes. Answer D:This study was examining attendance as the dependent variable to see whether a change in hours led to a change in attendance. The study did not mention examining compliance with home exercise programs. Bottom Line:In any research study, it is important to align the research question with the proper independent and dependent variables that can help answer that question. If a study is looking to see if X affects Y, then measuring Z, even if it provides interesting information, will not answer the research question. TrueLearn Insight :Do not be tempted by the word compliance in the answer just because that word was also in the question. In this case, the uses of the word compliance was unrelated. Read the question and try to answer it in your mind before looking at the tempting distractors.

An 8-year-old boy was playing dodge ball and was hit by the rubber ball in the face. On examination he is unable to close his eyes tightly or raise and lower his eyebrows. Based on the findings, the physical therapist suspects injury to which nerve? A. cranial nerve V B. cranial nerve VI C. cranial nerve VII D. cranial nerve IX

Correct answer: C Cranial nerve VII(facial) is assessed by looking for facial expression symmetry (frown, smile, raise and lower eyebrows, puff out cheeks, close eyes tightly). The patient examination findings are consistent with loss of cranial nerve VII function. In this case, it is most likely that the inflammation due to the ball hitting the face caused cranial nerve VII (facial) impairment. Answer A:Cranial nerve V (trigeminal) provides the motor nerve for the muscles of the jaw (mastication). Testing this nerve is achieved by asking the client to clench teeth together as the therapist palpates the muscles over the temples (temporal muscle) and jaw (masseter) on each side. Answer B:Cranial nerve VI (abducens) is responsible for lateral movement of the eyeballs. The therapist assesses direction of gaze (ability to move eyes out laterally when intact). Eyes will maintain a position medially when impaired. Answer D:Cranial nerve IX glossopharyngeal provides taste, gag, and the ability to swallow. Bottom Line:Cranial nerve VII (facial) is assessed by looking for facial expression symmetry (frown, smile, raise and lower eyebrows, puff out cheeks, close eyes tightly).

A physical therapist is performing an initial evaluation on an 81-year-old African-American female with a history of hypertension, congestive heart failure (CHF), chronic obstructive pulmonary disease (COPD), anemia of chronic disease, chronic kidney disease, and diabetes mellitus (type II) in the patient's home. The physical therapist performs a thorough examination of all extremities as well as the skin integrity and skin color. Which of the following BEST describes the clinical rationale for examining skin color? A. There is no indication for examining skin color in this patient B. The physical therapist is concerned about jaundice due to the patient's multiple co-morbidities C. The physical therapist is looking for signs of cyanosis due to cardiovascular and pulmonary risk factors D. The physical therapist is looking for signs of impaired wound healing due to the patient's history of diabetes

Correct answer: C Cyanosis is the bluish discoloration of the skin resulting from poor circulation. It can also result from inadequate oxygenation of the blood. It is most easily seen where the skin is thin, such as the lips, mouth, earlobes, and fingernails, and can be noticed in the hands and feet turning a bluish color. Due to the significant cardiovascular and pulmonary risk factors of this patient, it would be reasonable to suspect compromise in the circulatory system. Therefore, it would be indicated to look for signs of cyanosis in order to be able to identify these changes before, during, and after physical therapy interventions. Answer A:Due to the significant cardiovascular and pulmonary risk factors of this patient, it would be reasonable to suspect compromise in the circulatory system. Therefore, it would be indicated to look for signs of cyanosis in order to be able to identify these changes before, during, and after physical therapy interventions. If the patient consistently presents with cyanosis, it may be a sign that she is not receiving adequate oxygenation. Answer B:When suspecting jaundice, examination of skin color would be important (looking for a yellow discoloration of the skin). However, based on the presented history of this patient, there is no indication of liver involvement. Therefore, this is not the best answer and is incorrect. Answer D:In all patients with diabetes, it is important to perform skin inspections, especially on weight-bearing surfaces. However, this is not the clinical rationale for looking at skin color. When examining the skin for patients with diabetes, the examiner might look for skin integrity and protective sensation, but not necessarily skin color. Therefore, this answer is incorrect because it is not the best answer. Bottom Line:In patients with cardiovascular and pulmonary disease, cyanosis is a possible finding and is indicative of impaired circulation and oxygenation. There can be a number of reasons for a physical therapist to inspect the skin. However, when examining skin color in a cardiac or pulmonary patient, the clinical rationale is to look for and look for signs of cyanosis.

A physical therapist evaluates a client with traumatic brain injury (TBI) in an acute care hospital. The client presents with hyperactive extensor reflexes in both the upper and the lower extremities in response to a noxious stimulus. The therapist is correct in recognizing that this positioning represents: a) decorticate posturing b) opisthostonos posturing c) decerebrate posturing d) decerebellate posturing

Correct answer: C Decerebrate posturing is a stereotyped response to noxious stimuli. It consists of extension, adduction, and hyperpronation in the upper extremity and extension with plantar flexion in the lower extremity. Comatose patients with lesions below the red nucleus but above the vestibular nucleus might have decerebrate posturing.Abnormalities of movement depend on the area of the brain injured. This posturing reflects injury at the superior border of the pons, resulting in the loss of inhibitory control of the cortex and basal ganglia. The progression of posturing from decorticate to decerebrate is seen as a negative sign, as lower levels of the brain are affected. Answer A:Decorticate posturing is a stereotyped response to noxious stimuli. In the upper extremity, it consists of flexion of the arm, wrist, and fingers; in the lower extremity, it consists of extension, internal rotation, and plantar flexion. This response is common initially in traumatic brain injury (TBI) and reflects the lost cortical control with motor patterns similar to those seen in cortical stroke. The client has an extensor position of the upper and lower extremities. Answer B:Opisthotonos posturing consists of spasm of the muscles causing backward arching of the head, neck, and spine, as in severe tetanus, some kinds of meningitis, and strychnine poisoning. The client has an extensor position of the upper and lower extremities. Answer D:Decerebellate posturing is caused by a lesion in the cerebellum. The function of the cerebellum is to coordinate muscular activity. Decerebellate posturing includes opisthotonos. The client has an extensor position of the upper and lower extremities. Bottom Line:Decerebrate posturing is a stereotyped response to noxious stimuli. It consists of extension, adduction, and hyperpronation in the upper extremity and extension with plantar flexion in the lower extremity.

A child arrives for physical therapy due to a burn from a hot liquid. The physical therapist is assessing for depth of penetration to classify the burn using capillary refill. The therapist presses down on the skin for 4 seconds and then watches how long it takes for the burn to regain color. The therapist notes that the burn has a mottled white appearance and does not blanch with pressure. How does the therapist classify this burn? a) superficial burn b) superficial partial thickness burn c) deep partial thickness burn d) full thickness burn

Correct answer: C Deep partial-thickness injuries extend into the reticular dermis and are insensate, have a mottled white appearance, and do not blanch with pressure as the result of impaired vascularity and capillary refill. Answer A:Superficial injuries are burns limited to the epidermis without disruption of epithelial integrity. These injuries are characterized by erythema of the skin secondary to vasodilation of local capillaries and have brisk capillary refill. Sunburns are examples of superficial burns. Answer B:Superficial partial thickness burns are dermal injuries that extend into the papillary dermis. These wounds are sensate, blanch with pressure, and typically result in blistering because of the local inflammatory process. Answer D:Full thickness burns extend through the entire dermis and into the subcutaneous tissue. The appearance is leathery brown or black eschar with no capillary refill. The description in this question does not describe the wound as leathery or with eschar.

Which of the following information from the patient interview is MOST impactful on cardiovascular risk? A. Family history B. Height and weight C. History and management of type II diabetes D. History of low back pain

Correct answer: C Diabetes is a significant risk factor for cardiovascular disease. Adults with diabetes have a 2-to-4-fold higher risk for cardiovascular morbidity and mortality than adults without diabetes. If a patient has a history of diabetes, it is essential that this controllable cardiovascular risk factor be addressed. Answer A:A family history of cardiovascular disease is definitely a risk factor but is an uncontrollable risk factor. Nonetheless, it is important information, but not as important as the management of diabetes. Therefore, it is not the correct answer for this question. Answer B:Obesity is a cardiovascular risk factor. Taking note of height and weight enables the physical therapist to determine the body mass index (BMI), which is a measure of obesity. However, it is not always a reliable indicator of cardiovascular risk, since there are definite variations in body types and compositions that can skew the BMI. Nonetheless, it is important information, but not as important as the management of diabetes. Therefore it is not the correct answer for this question. Answer D:Although low back pain may limit an individual's activity levels in everyday life and probably limit physical exercise, it is not a direct risk factor for cardiovascular disease. Nonetheless, physical therapists often treat low back pain in order to allow an individual to participate in activities that lower their risk for cardiovascular disease. However, it is not the correct answer for this question. Bottom Line:Being aware of cardiovascular risk factors allows the physical therapist to make decisions on how to best manage a patient's overall well-being. In the case of diabetes, which is a significant cardiovascular risk factor, it is imperative for this condition to be properly managed. If left untreated, this condition puts patients at increased risk for a host of other complications and negative outcomes. Adults with type 2 diabetes have a 2-to-4-fold higher risk for cardiovascular morbidity and mortality than adults without diabetes.

While the therapist is taking a history during the examination of a 31-year-old male patient with chronic low back pain, the patient mentions that he has been regularly taking oxycodone for pain relief, with increasing doses recently. He would like to try physical therapy to reduce his pain. The patient reports no history of smoking and an alcohol use of one to two beers per weekend. He also reports mild abdominal pain. Upon physical examination and palpation, there is minor, but noticeable abdominal distention. The patient is MOST likely to also have which of the following conditions? a) ascites from cirrhosis b) peptic ulcer disease c) opioid-induced constipation d) early signs of prostate cancer

Correct answer: C During physical examination, it is important to have a well-rounded understanding of all body systems that could impact a patient's physical functioning and overall health status. Problems in the gastrointestinal system can severely reduce a patient's physical function as well as their pain experience. Almost half of all patients on long-term opioid therapy will experience some form of opioid-induced constipation. Given the patient's report of chronic pain and opioid use as well as noticeable abdominal distention, it is reasonable to suspect the presence of opioid-induced constipation. Answer A:Although during physical examination abdominal distention is present, there is no indication from the given medical or social history that would point to cirrhosis. The patient's alcohol use is not at a level that would cause cirrhosis, especially at age 31. In addition, opioid use is not directly related to cirrhosis. Answer B:The use of opioids such as oxycodone is not associated with peptic ulcer disease as in the case of nonsteroidal antiinflammatory drugs (NSAIDs) such as aspirin, ibuprofen, and naproxen. Peptic ulcer disease would be a reasonable suspicion if the patient mentions the use of NSAIDs. Answer D:The patient's gender is a risk factor for prostate cancer; however, his young age makes it unlikely. Based on the information provided in this scenario, there is no other indication for the patient to have prostate cancer, such as difficulty with urination. Constipation is a much more likely reason for the abdominal discomfort, given the patient's longstanding opiod use and abdominal distention.

What is the correct sequence of sensations in response to cryotherapy to explain to a patient getting an ice massage with an ice cup? A. Intense cold, aching, burning, numbness, analgesia B. Intense cold, burning, aching, numbness, analgesia C. Intense cold, burning, aching, analgesia, numbness D. Intense cold, numbness, burning, aching, analgesia

Correct answer: C During the application of cryotherapy by any means, the patient will usually experience the following sequence of sensations: intense cold burning aching analgesia numbness These sensations are thought to correspond to the increased stimulation of thermal receptors and pain receptors followed by blocking of sensory nerve conduction as tissue temperature decreases.Cryotherapy may be applied through a variety of materials, including cold or ice packs, ice cups, controlled cold compression units, vapocoolant sprays, frozen towels, ice water, and contrast baths. Different materials cool the patient at different rates and to different degrees and depths. Answer A: Burning typically occurs before aching. Answer B: Analgesia occurs before numbness. Answer D: Numbness is typically the last sensation felt. Bottom Line: During the application of cryotherapy by any means, the patient will usually experience the following sequence of sensations: intense cold, burning, aching, analgesia, and numbness. Cryotherapy may be applied through a variety of materials, including cold or ice packs, ice cups, controlled cold compression units, vapocoolant sprays, frozen towels, ice water, and contrast baths.

A physical therapist is treating a patient with a history of multiple sclerosis. The patient has good verbal expression and understands the therapist's commands. However, the patient is difficult to understand because their voice is hoarse, with a variable pitch. What is the name for this clinical condition? a) aphasia b) apraxia c) dysarthria d) dysphagia

Correct answer: C Dysarthria is a speech disorder that affects the motor aspect of speech and causes impairments in articulation, resonance, and phonation. Language comprehension and expression are not affected by dysarthria, but patients are often difficult to understand. Dysarthria is found in patients with multiple sclerosis due to damage to the cranial nerve nuclei, corticobulbar tract, or cerebellum. Answer A:Aphasia is a deficit in language comprehension (Wernicke's/receptive aphasia) or generation (Broca's/expressive aphasia) that is usually caused by damage to the left cerebral cortex. Answer B:Apraxia is a deficit in motor planning that occurs despite intact sensory and motor pathways. The lesion location for apraxia is variable, but the lesion often occurs with damage to the left cerebral cortex. Answer D:Dysphagia is a deficit in swallowing function due to weakness or decreased coordination of the oropharyngeal musculature or due to cognitive deficits. Dysphagia may lead to choking or aspiration pneumonia. Dysphagia is found in patients with multiple sclerosis due to damage to the cerebral cortex, cranial nerve nuclei, corticobulbar tract, or cerebellum. Bottom Line: Dysarthria is a common consequence of CNS disorders such as multiple sclerosis. Physical therapists must recognize that patients with dysarthria have a deficit in the motor aspects of language alone while langauge comprehension and expression are not affected.

A 14-year-old female gymnast presents with low back pain. On postural examination, a noticeable increase in lumbar lordosis and anterior pelvic tilt are seen. Which of the following tests BEST assesses the length of a muscle that could be contributing to her anterior pelvic tilt? a) 90-90 hamstring test b) PROM measurement of hip flexion c) Ely's test d) Ober's test

Correct answer: C Ely's test is the correct answer because it specifically tests for the length of the rectus femoris, which is a muscle that crosses both the hip and the knee anteriorly. Tightness in the rectus femoris commonly contributes to anterior pelvic tilt. During the test, the patient is positioned prone with the pelvis neutral. The examiner then passively flexes the test knee into end range. The test is positive if, during passive knee flexion, the pelvis tilts anteriorly. This happens in a tight or shortened rectus femoris because as the knee flexes from movement of its distal attachment (patellar tendon), the muscle will pull on the proximal attachment (anterior inferior iliac spine), causing anterior tilt of the pelvis. Answer A:The 90-90 hamstring test is a test of the length of the hamstrings (biceps femoris, semitendinosus, and semimembranosus). This test places the hamstring muscles, which cross both the hip and the knee posteriorly, in a lengthened or stretched out position. Because of their proximal attachment to the ischial tuberosity, if they were shortened, they would pull the pelvis into a posterior tilt. The 90-90 test does not directly provide information about the iliopsoas or rectus femoris. Answer B:Moving the hip in a direction of hip flexion will place the muscles in the anterior portion of the hip on slack. When testing for the length of a muscle or muscle group, passive range of motion is performed in the direction opposite to that muscle's action. The muscles that would be shortened in a lordotic posture with anterior tilt posture are the iliopsoas and rectus femoris. To assess the length of these hip flexors, the hip needs to be moved in the direction of extension. In the case of the rectus femoris, a two-joint muscle, the knee would also need to be flexed to elongate the muscle at the knee. The Thomas test is one such test that assesses hip flexor length but was not an option in this question. Answer D:The Ober's test is a test for tightness of the iliotibial band. A positive test may indicate tightness of the tensor fasciae latae, which is a hip flexor just like the rectus femoris. However, this test does not directly affect muscles that may contribute to anterior pelvic tilt.

A patient has a great deal of pain when rotating their head to the left. The physical therapist determines that the facet on the right side between C5-C6 is not opening. Which intervention is MOST appropriate for this patient? A. Grade 1 posterior to anterior glide on the transverse process of C6 B. Grade 4 posterior to anterior glide on the transverse process of C6 C. Grade 2 posterior to anterior glide on the transverse process of C5 D. Grade 4 posterior to anterior glide on the transverse process of C5

Correct answer: C Grade 2 posterior to anterior glide on the transverse process of C5 is an appropriate grade to help reduce pain, and because a higher grade may not be tolerated. The direction of the glide is correct, and the application on the C5 level transverse process is also appropriate because C5 is being flexed off of C6 to open the facet joint line between C5 and C6. During cervical rotation, the contralateral facet needs to open (flexion, upglide) while the ipsilateral facet needs to close (extension, downglide). To flex a segment in the spine, the superior segment must be mobilized posteriorly to anteriorly in the direction of the facet (45 degrees) which is toward theeyeballs. Answer A:Grade 1 posterior to anterior glide on the transverse process of C6 is appropriate for pain, but the mobilization would cause extension at C5-C6, which is the opposite of what is required for contralateral rotation. Answer B:Grade 4 posterior to anterior glide on the transverse process of C6 is incorrect because the grade may not be tolerated due to the patient's current pain level, and C5 is the level that needs to glide forward on C6 for the rotation. The mobilization would cause extension at the C5-C6 level, which is the opposite of what is required for contralateral rotation. Answer D:Grade 4 posterior to anterior glide on the transverse process of C5 is incorrect. Although the direction of mobilization is appropriate, the grade is better suited for ROM/stiffness, but not with this patient's pain level. Bottom Line:During cervical rotation the contralateral facet needs to open (flexion, upglide) while the ipsilateral facet needs to close (extension, downglide). To flex asegment in the spine the superior segment must be mobilized posteriorly to anteriorly in the direction of the facet (45 degrees) which is toward theeyeballs. Grade 1-2 mobilizations are used for pain, while grades 3-4 are used for range of motion/ stiffness.

A 30-year-old female is being treated in physical therapy for back pain and stiffness of a mechanical origin. The patient finds it difficult to straighten up symmetrically after sitting at her desk all day. Observation reveals a restriction in the left extension quadrant. The patient has minimal pain and mostly stiffness. What is the BEST intervention for the physical therapist to perform? A. Grade 2 UPA (unilateral posterior to anterior) to the left facet B. Grade 3 UPA (unilateral posterior to anterior) to the right facet C. Grade 3 UPA (unilateral posterior to anterior) to the left facet D. Grade 4 CPA (central posterior to anterior) to the spinous process

Correct answer: C Grade 3 UPA is the correct answer as this will improve extension, specifically to the left facet, which will improve the left extension quadrant. The grade is appropriate for stiffness with very little pain.The vertebral facet joints have a right and a left. When mobilizing centrally, it will affect both sides. To be more specific, if the motion deficit is unilateral, the therapist must emphasize either the right or left facet with the direction of force. For pain a grade of 1-2 should be used, and for ROM a grade of 3-4 should be used. Answer A:Grade 2 UPA (unilateral posterior to anterior) to the left facet is incorrect as the location is correct but the grade of 2 is more for pain relief and not for improving joint mobility. Answer B:Grade 3 UPA (unilateral posterior to anterior) to the right facet is incorrect as this will improve the right quadrant and not the left. Answer D:Grade 4 CPA (central posterior to anterior) to the spinous process is incorrect as the grade is appropriate for stiffness but will focus on bilateral extension and will not specifically address the left extension quadrant.

A patient status post left total hip replacement complains of difficulty moving, swelling, and sensory changes in the left lower extremity. During palpation, the therapist notes skin fibrosis and skin pitting in various areas of the extremity. Skin mobility is decreased, and skin texture is brawny. The legs are of unequal size, with the left being significantly larger. When the limb is elevated, the edema does not dissipate. The dorsum of the second toe elicits a positive Stemmer sign. The therapist is correct in documenting that the patient is in which of the following stages of lymphedema? a) stage 0 b) stage 1 c) stage 2 d) stage 3

Correct answer: C In lymphedema stage 2, edema is present, but the edema no longer dissipates with elevation of the limb. Palpation is positive for fibrosis and results in indentation of the skin. Fibrosis ranges from soft and pitting (early stage II) to hard and non-pitting (late stage II). Both are commonly present in various locations along the involved limb. Skin mobility may be decreased, and texture may be brawny. A positive Stemmer sign is present. There is an increased risk of skin infection.Stemmer's sign is a thickened fold of skin at the base of the second toe or second finger that can be gently pinched and lifted. The presence of this sign is most often an early diagnostic indication of primary lymphedema; however, it can also develop later in secondary lymphedema.In lymphedema stage 2, edema is present but the edema no longer dissipates with elevation of the limb. Palpation is positive for fibrosis and results in indentation of the skin. Fibrosis ranges from soft and pitting (early stage II) to hard and non-pitting (late stage II). Skin mobility may be decreased, and texture may be brawny. A positive Stemmer sign is present. Answer A:Lymphedema stage 0 is also known as the subclinical stage. Lymph transport is impaired; however, lymphatic load has not yet exceeded the transport capacity.Swelling is not evident, and the patient may report heaviness, tightness, or sensory changes in the affected limbs. Answer B:In lymphedema stage 1, the edema is present and may be variable (e.g., decrease with rest or sleep and increase with activity). The edema is very soft and easily pits with little to no fibrosis. Elevation of the limb results in dissipation of the edema. The patient may report heaviness, tightness, sensory changes, or pain in the limb. Answer D:In lymphedema stage 3, fibrotic changes of the skin and fat deposits in the subcutaneous tissues continue to increase in severity. When the PT palpates the skin, the limb is hard with little mobility and increased skin folds. Pitting is absent or minimally present with significant pressure and, if present, dissipates rapidly. Hair follicles may be coarse or absent. Other symptoms can include loss of joint spaces, papillomas, skin hyperpigmentation, hyperkeratosis, cysts, and fistulas.

A physical therapist is working on gait training with a patient 4 weeks post cerebrovascular accident. The patient is ambulating with a quad cane and is performing the Timed Up and Go (TUG) test. At the end of the test, which of the following remarks from the therapist is give the patient knowledge of results? A. "You need to place the cane further away from your side" B. "You need to increase stride length on the left" C. "You took 22 seconds to complete the test" D. "You need to decrease your turning radius"

Correct answer: C Knowledge of results provides information about the outcome of the movement, in this case,the amount of time the patient took to complete the TUG test. Knowledge of performance provides cues about the way the task was performed, such as correct use of cane or gait pattern corrections. Answer A:Correct placement of the cane would be a performance issue, not a result. In this case, the outcome of the TUG test is knowledge of result, not how well the patient used the cane. Answer B:Performance issues include gait pattern deficiencies such as stride length and are not results. The knowledge of result would be the time taken for the TUG test. Answer D:Decreasing turning radius is a performance issue, not a result, and so would not be considered knowledge of result feedback. Bottom Line:Knowledge of results is the sharing with the patient the overall outcome of the movement performed, whereas knowledge of performance is how well they did the actual movement, such as their use of the cane, stride length, and turning radius.The time taken to complete the TUG test is the only knowledge of result feedback given in these answers.

A patient has been working with their physical therapist on ankle strategies and balance training. Which of the following interventions would demonstrate the HIGHEST level of challenge for this patient? a) single-leg stance on a solid floor b) transition from heel standing to toe standing c) marching on a foam pad d) standing on foam with eyes open

Correct answer: C Marching on a foam pad is one of the highest level exercises for improvement of ankle strategies due to the unpredictable surface and the time in single-leg stance due to the marching. Answer A:Single-leg stance on a solid floor is considered an intermediate level challenge to the ankle strategies due to the predictability of the surface. Single-leg stance is challenging for the ankle strategies but would be greater on a compliant surface. Answer B:Transitioning from heel to toe standing is an intermediate challenge due to the bilateral nature of standing. Even without upper extremity (UE) support, this activity is considered at an intermediate level. Answer D:Standing on foam on both feet with eyes open is an entry-level activity for ankle strategies as it provides a challenge due to the compliant nature of the foam but is bilateral so limits the challenge. Bottom Line:Ankle strategies are critical for successful balance and can be used progressively from bilateral stance, including on a compliant surface, to unilateral stance, which is at its greatest on a compliant surface. Adding movement, such as marching, that include single-leg stance on a compliant surface is one of the greatest challenges that can be given for the ankle strategies.

A physical therapist is treating a new patient with a history of multiple sclerosis. During the treatment, the patient begins crying for a few minutes for no apparent reason and then starts laughing for another few minutes. After the episode, the patient returns to normal, but she was embarrassed and told the PT it happens frequently. The PT plans to contact the physician. What should be DOCUMENTED in the patient's chart? a) will contact physician as patient may have bipolar disorder b) will contact physician as patient may have conversion disorder c) will contact physician as patient may have pseudobulbar affect d) will contact physician as patient may have schizophrenia

Correct answer: C Multiple sclerosis is an autoimmune disease caused by demyelination of the central nervous system. It typically affects females in their 30s or 40s. Patients with multiple sclerosis may exhibit pseudobulbar affect (PBA), which is characterized by sudden displays of crying, laughing, or other emotional outbursts without an apparent cause. PBA may be caused by damage to the corticobulbar tract and is estimated to affect up to 10% of patients with multiple sclerosis. Answer A:Bipolar disorder is characterized by alternating cycles of manic episodes and depressive episodes that last for days to weeks. Based on the rapid changes of mood in the described scenario, bipolar disorder is an unlikely diagnosis. Answer B: Conversion disorder is characterized by the presence of symptoms affecting motor or sensory function that suggests a neurological injury that cannot be explained by a neurological diagnosis. Patients with conversion disorder are not intentionally feigning their symptoms, and this disorder is usually triggered by an extremely stressful psychological event. Based on the defined neurological injury in the described scenario, conversion disorder is an unlikely diagnosis. Answer D: Schizophrenia is a severe mental illness that is characterized by delusions, hallucinations, emotional disturbance, impaired memory, and decreased executive function and is often diagnosed in young adulthood. Based on the symptoms in the described scenario, schizophrenia is an unlikely diagnosis.

A 75-year-old patient reports chronic right knee stiffness over the last several years. Upon examination, right knee flexion passive range of motion (PROM) is (0-95) and active range of motion (AROM) is (0-95) with patient reporting 2/10 knee pain. Which of the following statements MOST accurately describes the interpretation of the findings of the ROM testing? a) AROM and PROM are limited and there is an empty end feel b) AROM is WNL, PROM is limited, and there is a firm end feel c) AROM and PROM are limited, and there is a hard end feel d) AROM and PROM are limited, and there is a soft end feel

Correct answer: C Normal knee flexion PROM is (0-135). At the end of the motion the normal end feel is soft due to approximation of the posterior thigh and posterior calf. In addition, PROM is normally a few more degrees than AROM due to the external force applied by the examiner. In this scenario, the AROM and PROM were exactly the same and occurred at only 95 degrees of flexion. Therefore, there is no opportunity for soft tissue approximation of the thigh and calf.The barrier to further motion of each joint is called the end feel. Soft, firm, and hard end feels can be normal or abnormal, depending on the joint. An empty end feel is always abnormal and occurs when a patient stops the therapist from completing the ROM due to pain, although there is no true restriction. The end feel is most likely hard, which would explain the finding of AROM and PROM being identical in this patient. A hard end feel can be normal, like elbow extension, or it can be abnormal, as in this scenario. Based on the patient's age and complaints of chronic pain, the hard end feel could be related to arthritis. Answer A:An empty end feel means the therapist did not get to the point in the ROM where the motion came to an end, because pain prevents reaching the end of ROM. No resistance is felt except for the patient guarding from further motion. The patient in this scenario has only 2/10 pain, therefore does not have an empty end feel. In addition, it would be unlikely to find an identical AROM and PROM for a person guarding the motion due to pain, but without a true restriction. Answer B:The normal PROM is (0-135), so AROM is not within normal limits. In addition, a firm end feel can result from a muscular, capsular, or ligamentous stretch, such as the end feel in extending the MCP joint to its end range. If the end feel was firm, there would be some give, and most likely the PROM measurement would be greater than the AROM because the examiner could move the joint further than the patient actively could on their own. The patient in this scenario had PROM and AROM ranges that were identical, most likely due to some sort of hard body in the joint or arthritic changes. Answer D:A soft end feel can be normal, such as soft tissue approximation during knee flexion of a normal knee, or it can be abnormal, such as a knee that is very swollen and has a boggy feeling when the patient tries to extend it. In this instance, the PROM and AROM both stop at 95 degrees, so the joint does not get to the point in the flexion ROM where soft-tissue approximation would stop the motion.

An individual presents to the clinic with a report of dull, achy pain along the low back and tingling into the right foot. The patient feels most comfortable in sitting position and has difficulty standing for long periods of time or lying in prone. Dermatomal testing revealed diminished sensation along the big toe on the right. The physical therapist noted fatiguing weakness along the extensor hallicus longus on the right. Repeated motion testing revealed improvement in all symptoms with repeated flexion, and an increase in low back pain with repeated extension. Which of the following medical diagnoses should the physical therapist most likely suspect? a) central stenosis b) herniated nucleus pulposus c) foraminal (lateral) stenosis d) retrolisthesis

Correct answer: C Of the four choices, foraminal stenosis explains all the findings. The stenosis creates a narrowing of the intervertebral foramen where the nerve root exits and may cause nerve root impingement on that side. The level in which it impinges will affect the associated nerve root and therefore that level's dermatome and myotome. In addition, if the issue was a herniated disc impinging on the nerve root, that condition usually responds positively to extension, but in this case extension worsened the pain. Answer A:Central stenosis is a narrowing of the spinal canal and would cause bilateral symptoms as the spinal cord is the structure being impinged. Answer B:A herniated disk would typically respond to extension and not flexion. Answer D:This is a posterior translation of the vertebra and would not typically cause impingement of the nerve root. Bottom Line:Foraminal (lateral) stenosispresents with unilateral tingling/numbness along the distal dermatome of the affected nerve root. Myotome testing would reveal fatiguing weakness along a specific myotome and key muscle(s) group, and in this case the extensor hallicus longus is the key muscle for the L5 myotome. Those with foraminal (lateral) stenosis relieve pressure on the nerve root by flexing, which helps symptoms. Pain worsens with extension.

A physical therapist is conducting an initial examination for a male patient with a T4 complete spinal cord injury that occurred 1 week ago from a fall that resulted in no additional injuries. Discharge from the acute rehabilitation facility is expected in approximately 4 weeks. What is the MOST likely method the patient will use to perform pressure relief in the wheelchair after discharge? A. anterior weight shift B. lateral weight shift C. seated push up D. tilt via tilt-in-space wheelchair

Correct answer: C Patients achieve a push-up weight shift by fully extending their arms to clear their buttocks from the sitting surface for approximately 30 to 90 seconds. Since this patient has a T4-level injury, he would be able to achieve this transfer since his upper extremities are fully innervated. Many patients prefer a push-up weight shift because it requires the least amount of time to perform compared to the other methods of pressure relief; it eliminates the need to reposition their wheelchair; and it does not require a tilt-in-space wheelchair to complete.A physical therapist must choose the type of pressure relief to teach the patient with a spinal cord injury at initial examination based on the level and completeness of injury, patient age, and comorbid factors. Answer A:The patient performs an anterior weight shift by leaning forward to clear the buttocks from the sitting surface for approximately 30-90 seconds. Depending on body size, the buttocks might not be fully un-weighted using this transfer, and some patients may need a surface in front of them to aid in regaining an upright position. As a result, an anterior weight shift is not the preferred method of weight shift for patients with paraplegia and is unlikely to be the method of choice at discharge. Answer B:The patient performs a lateral weight shift by leaning laterally onto another surface to clear one of their buttocks from the sitting surface for approximately 30-90 seconds. Depending on body size, the buttocks might not be fully un-weighted using this transfer, and the patient might require extra time to complete due to the need to weight shift each side separately. Further, the patient might need a surface to the side on which to complete this weight shift. As a result, a lateral weight shift is not the preferred method of weight shift for patients with paraplegia and is unlikely to be the method of choice at discharge. Answer D:A tilt weight shift is achieved by the patient's wheelchair tilting posteriorly between 25 and 65 degrees using either a power or manual tilting mechanism for approximately 2-5 minutes. A patient with a T4 injury is unlikely to require a tilt weight shift or power wheelchair at discharge since the upper extremities are fully innervated.

A physical therapist is treating a patient with a C8 spinal cord ASIA D injury. Which of the following methods of gait training on a treadmill will provide a consistent/uniform amount of assistance to the lower extremities in order to recreate as normal a gait pattern as possible? a) orthotic-assisted stepping b) manual-assisted stepping c) robot-assisted stepping d) stimulation-assisted stepping

Correct answer: C Physical therapists who work with patients with spinal cord injury must be able to recommend technologies for locomotor training, knowing the advantages and disadvantages of each tool. Robot-assisted stepping uses technology to assist a patient in performing a consistent gait pattern during training since the machine is providing the assistance, and it remains uniform throughout the training session. A disadvantage of robot-assisted stepping is the cost and training associated with the technology. Advantages include the need for fewer therapists during gait training and a potentially lower risk of musculoskeletal injury for the therapists providing the locomotor training. Answer A:The use of orthotics while performing locomotor training on a treadmill has not been reported in the literature and may discourage appropriate sensory feedbackduring gait, which is hypothesized to be an important factor associated with the successful application of locomotor training. It would also not affect the amount of assistance provided. Answer B:Manual-assisted stepping is the most common form of assistance provided during locomotor training on a treadmill for patients with spinal cord injury. It can require two to three therapists. One advantage of locomotor training using manual assistance is that the physical therapists may modify the amount of assistance provided on a step-by-step basis within the gait cycle in order to encourage the patient's ability to complete gait as independently as possible. However, it will be difficult for the physical therapists to facilitate a consistent gait pattern due to variability in patient performance and therapist assistance, so for this question, because the amount of assistance is not uniform, this answer is incorrect. Answer D:Stimulation-assisted stepping uses electrical input to activate spinal reflexes that elicit a withdrawal reflex used for stepping. However, the use of withdrawal reflexes for stepping does not facilitate a normal gait pattern, which was the priority of the physical therapist in this scenario. Bottom Line:Robot-assisted stepping uses technology to assist a patient in performing a consistent gait pattern during training since the machine is providing the assistance.

A physical therapist is treating a patient with an incomplete spinal cord injury with severe spasticity of all four extremities. Which of the following positioning aids is NOT appropriate for use with this patient? a) hip adductor wedge between the legs b) multipodus boots on the feet c) pillows under knees d) wedge for positioning in sidelying

Correct answer: C Pillows under the patient's knees would encourage the development of knee flexion contractures, so they should not be used in this scenario. Answer A:A hip adductor wedge would position the patient in hip abduction and would be recommended since increased hip adductor tone/spasms are often found in patients with incomplete spinal cord injuries and could lead to skin breakdown or contractures. Answer B:Multipodus boots would position the patient's ankles in neutral dorsiflexion and would be recommended since increased plantarflexor tone/spasms are often found in patients with incomplete spinal cord injuries and could lead to skin breakdown or contractures. Answer D:A wedge for positioning in sidelying would be recommended for turning the patient every 2 hours while they are in bed to prevent skin breakdown. Bottom Line:Positioning aids can serve many purposes, such as preventing contractures and reducing spasticity. They can also assist position changes to prevent skin breakdown. For a patient with spasticity, it is contraindicated to place pillows under the knees, which will feed into a flexion contracture position. Once a contracture develops, even with reduction of spasticity, the contracture may persist and need to be surgically released.

An individual presents to physical therapy with a report of shoulder stiffness. A quick functional screen reveals the patient can raise the arms fully and can place both arms behind the head, but when asked to place both arms behind the back, the patient could not get the left arm any higher than the back of the buttock. The patient mentioned noticing that motion was stiff as well, making them unable to place their cell phone in the back pocket of their pants. Which joint mobility assessment should the physical therapist perform at the glenohumeral joint? a) anterior b) inferior c) posterior d) superior

Correct answer: C Posterior assessment, as in reaching behind the back, requires internal rotation (IR) of the humerus in the glenoid. During IR, the humerus glides posteriorly. Answer A:Anterior glide is a component motion of shoulder extension. Answer B:An inferior glide occurs as a component motion of shoulder elevation. Answer D:A superior glide occurs with shoulder adduction. Bottom Line:Reaching behind the back, as with fastening a bra, putting a wallet in the back pocket of pants, or zipping a skirt in the back, requires glenohumeral internal rotation. If the ROM restriction wis limited actively and passively in the same direction of IR, the appropriate directional glide to assess mobility at the glenohumeral joint is posterior.

A patient in a skilled nursing facility with a medical history of high blood pressure and high cholesterol presents 7 days status post right hip ORIF with abdominal pain. Medications include Lotensin, Percocet, and Lipitor. He reports significant abdominal pain and cramping as well as nausea and lack of appetite. The patient has decreased but present bowel sounds and can pass gas but has not had a bowel movement since surgery. What is the MOST likely cause of the symptoms? a) cholecystitis b) postoperative ileus c) opioid constipation d) small bowel obstruction

Correct answer: C Postsurgical patients are typically on a narcotic pain medication to manage the acute pain of surgery. Opioid drugs like Percocet are known to inhibit gastric emptying and peristalsis in the GI tract. The fact that the patient does have some bowel sounds and is able to pass gas would lead the clinician to believe it is more of a constipation issue versus ileus or bowel obstruction. Answer A:Cholecystitis is inflammation of the gallbladder, which typically causes right upper quadrant pain. It can also cause nausea, vomiting, or fever. Answer B:An ileus is the inability of the intestine to contract. Symptoms include nausea/vomiting, abdominal cramping, constipation, difficulty passing gas, and difficulty tolerating a normal diet. Typically with an ileus, no bowel sounds are heard. Answer D:A small bowel obstruction is partial or complete blockage of the small intestine. With a small bowel obstruction, patients typically cannot pass gas. Bottom Line: Opioids are known to cause constipation.Constipation results from decreased intestinal peristalsis, which is probably mediated by effects on opioid receptors in the enteric nervous system. Stomach motility may decrease, but tone may increase. In the large intestine, peristalsis is diminished, and tone is increased. This delays the passage of fecal mass and allows for increased absorption of water, which leads to constipation.

Which of the following manual muscle testing positions BEST isolates activation of the gluteus maximus? A. Patient stands at the end of the treatment table and then leans forward to allow trunk to rest on table with feet on the floor; patient then extends one lower extremity backward and upward, maintaining the knee in extension; examiner places a hand on the patient's distal posterior thigh; the patient is then asked to provide resistance upward into the examiner's hand while the examiner presses downward toward the floor in the direction of hip flexion. B. Patient is placed in a supine position with both lower extremities extended; examiner then passive flexes the test hip and knee to approximately 30 degrees of flexion at both joints; the patient is then asked to provide resistance through the heel, pushing out into the examiner's hand C. Patient is positioned in prone with examiner maintaining a neutral pelvis by stabilizing at the iliac crest of the test side; the lower extremity being tested is then placed in end range knee flexion and the patient is asked to extend the hip; resistance is given in a downward direction from the posterior surface of the distal thigh D. Patient is positioned in prone with examiner maintaining a neutral pelvis by stabilizing at the iliac crest of the test side; the patient is then asked to raise the lower extremity being tested off the table while maintaining the knee in full extension; resistance is given in a downward position from the posterior surface of the distal tibia.

Correct answer: C Prone hip extension with the knee positioned in flexion is the correct answer. In a prone position, hip extension would require the gluteus maximus to contract against gravity. However, the hip extension would be accomplished by contributions from the gluteus maximus and the hamstrings. Both muscles extend the hip. By flexing the knee to end range while asking the patient to extend the hip, the hamstring muscles become actively insufficient. The muscle is shortened over both the hip and the knee, and the overlapping of actin and myosin reduces the contributions from the hamstrings for hip extension. Answer A:This test position places the patient in some hip flexion and describes a modified method of testing the overall strength of the hip extensors for a patient with a hip flexion contracture who cannot be tested from the range of 0 degrees into hip extension, or for a patient who cannot tolerate the prone position. However, it elicits contraction from essentially all hip extensor muscles. With the knee maintained in extension, the hamstrings are lengthened enough to effectively contribute to movement of the hip into extension. In addition, in this modified position, the hip is in some flexion at the start of the test, which places the hamstrings in a very good length/tension relationship and increases their effectiveness at generating tension. The gluteus maximus will not be isolated in this test as described. Answer B:This test position describes a modified method of testing the overall strength of the hip extensors as well as knee extensors in a supine position. This is an incorrect answer because the resisted motion in this position does not isolate the gluteus maximus. A significant contribution to the overall resistance would likely also come from the quadriceps. Answer D:This test position describes a method of testing the overall strength of the hip extensors. However, it is not selective for which hip extensors due to the lengthened position of the hamstrings. In this position, both the gluteus maximus and the hamstrings contribute to hip extension. Bottom Line:All of the answer choices describe test positions that would elicit activation from the gluteus maximus, making them all valid ways of testing the action of hip extensors. However, multiple muscles typically act on a single joint in its degrees of freedom. In order to target a specific muscle, other muscles that also act on the joint would have to be placed in positions that make action or contraction less effective. One of those methods is placing muscles in active insufficiency, which makes muscles too short to allow cross-bridges to form between actin and myosin. Understanding this important anatomical and biomechanical concept is essential in effective musculoskeletal examination.

A physical therapist is reading a research article reporting on 2 groups of participants who were not randomly assigned. One group received a novel intervention that was being tested, and both groups took a pre-test and post-test. Which of the following research designs was used in this study? a) pre-experimental b) exploratory sequential c) quasi-experimental d) convergent parallel

Correct answer: C Quantitative research is experimental work that tests a theory through the examination of the relationship between variables. One design that is used in quantitative work is the quasi-experimental design, which involves groups that take a pre-test and a post-test but without random assignment. Only the experimental group receives the treatment.Different types of experimental design are used in quantitative work. A person reading the research should understand the different kinds of experiments that can be used to better understand the rigor of the research being performed. True experimental research is stronger than quasi-experimental because it includes randomization of subjects, pre-tests and post-tests, and the intervention is provided to 1 group. When there is no randomization of the subjects, it is considered a quasi-experimental design. Answer A:A pre-experimental study design involves only 1 group that is exposed to the treatment and then measured. There is no comparison to a second group as indicated in the question. Answer B:An exploratory sequential design is used in mixed-method research and is used to develop better measurement instruments. The design initially involves qualitative work to determine what is needed, followed by quantitative research on a new measure. Answer D:A convergent parallel is a design used in mixed-method work that compares data from both quantitative and qualitative work at the same time. Two different databases are used and compared to see how the different data compare or diverge.

During a chart review at a hospital-based outpatient clinic, a physical therapist learns that a new patient coming in for an initial examination has rheumatoid arthritis (RA). The chart lists multiple medications, some current, some used in the past, including celecoxib, ibuprofen, methotrexate, and prednisone. Which one of those medications is the MOST effective medication to treat RA? a) Celecoxib b) Ibuprofen c) Methotrexate d) Prednisone

Correct answer: C Rheumatoid arthritis is a systemic autoimmune disease affecting the synovial joints of the body that can also cause inflammation and injury in other organs. Common symptoms include joint pain, most commonly symmetrically and most commonly in the fingers and toes, as well as swelling and inflammation. The treatment of choice is to slow the progression of the disease and its destruction. Disease-modifying antirheumatic drugs (DMARD) are the most effective treatment for rheumatoid arthritis (RA). Methotrexate is part of the DMARD family and is a commonly prescribed medication for RA, an autoimmune disease. NSAIDs and short-term steroid use also help reduce inflammation but are not directed at slowing the disease itself. Answer A: Celecoxib is an over-the-counter nonsteroidal antiinflammatory (NSAID). Like ibuprofen, it is used to decrease pain and inflammation. It can be used to treat RA, but it is not as effective as methotrexate in addressing the root cause of the disease. Answer B: Ibuprofen is an NSAID, used to treat pain and decrease inflammation. While effective for treating symptoms of RA, methotrexate is the most effective at slowing the progression of the disease itself. Answer D:As a steroid, prednisone is used for symptomatic management to decrease inflammation. Given the chronic nature of the disease, prednisone is not the most effective way to treat RA, as it is not often prescribed for long-term use. In addition. it does not slow the progression of RA like methotrexate. Bottom Line: Disease-modifying antirheumatic drugs (DMARD) are the most effective treatment for rheumatoid arthritis (RA). Methotrexate is a commonly prescribed DMARD. TrueLearn Insight :For questions involving pharmacology, it is helpful to recognize the classification of a medication. In that manner, it may be possible to rule out some choices if more than one have similar properties, such as in this case, where two of the choices were NSAIDs and, therefore, could both be ruled out.

Based on knowledge of risk factors, which of the following patients has the HIGHEST RISK of developing type II diabetes? a) African American teacher, BMI of 20, sibling with type I DM b) Chinese high school student with BMI of 25 c) Hispanic accountant with BMI of 30 d) White construction worker with BMI of 30

Correct answer: C Risk factors for type II diabetes include a sedentary lifestyle, obesity, higher ages, and being of certain ethnicities, including African American, Native American, Asian and Hispanic. Of these options, the Hispanic accountant with a BMI of 30 has the highest risk because of the at-risk ethnic group, the sedentary job, and the BMI fo 30, which is in the obese category. Answer A:The ethnicity here is a risk factor, but the BMI is normal, the profession is active, not sedentary, and type I DM does not increase risk of type II DM. Answer B:Asian ethnicity is a risk factor, but the age of a high school student is younger than the other options, and risk increases with age. BMI of 25 is overweight but not obese. Answer D:The BMI here is at the obese level, but the race is less of a risk factor, and the job of construction worker is fairly active.

A patient's radial nerve was completely severed during an open mid-shaft humeral fracture 8 weeks ago. The fracture is now healed. The physical therapist is about to conduct a manual muscle test (MMT). Based on the level of the injury, where would the physical therapist MOST LIKELY expect to find complete motor loss during the MMT? a) shoulder extension, elbow extension, wrist extension, and finger extension b) elbow extension, wrist extension, and finger extension c) wrist extension and finger extension d) finger extension

Correct answer: C Since the shoulder extensors are not innervated by the radial nerve, and the level of injury occurred below the point at which the elbow extensors are innervated, weakness would be seen only in muscles innervated by the radial nerve below that level. Those muscles are the wrist and finger extensors. The patient would demonstrate a wrist drop as well. Answer A:Shoulder extensors would be intact, as shoulder extensors are not innervated by the radial nerve. Elbow extensors would be intact, as the triceps are innervated above the level of the radial groove. Answer B:Elbow extensors are still intact, as they are above the level of injury at the radial groove. Answer D:The level of injury would result in paralysis of both the wrist and finger extensors, not just finger extensors. This is not the best answer. Bottom Line:A radial nerve injury at the radial groove (spiral groove), such as with a mid-shaft humeral fracture, will affect the motor function of the wrist extensors and finger extensors. Other radially innervated muscles such as the triceps are spared because they are innervated above the level of injury.

A physical therapist is examining a patient for a wellness screening. The patient has no significant past medical history. While examining their lab values in the medical records, the therapist notes that the patient's A1C is 8.0%. All other lab values are normal. What does the physical therapist LEARN from finding this A1C value on the lab test? A. The patient is slightly anemicB. The patient is at increased risk for diabetesC. The patient has type II diabetesD. The patient has metabolic syndrome a) the patient is slightly anemic b) the patient is at increased risk for diabetes c) the patient has type II diabetes d) the patient has metabolic syndrome

Correct answer: C The A1C test reflects average blood glucose levels over the past 3 months. A1C values above 6.5% indicate that the patient will be diagnosed with type II diabetes. Answer A:Anemic patients have low levels of hemoglobin. A1C values are unrelated to anemia. Answer B:A patient whose A1C is between 5.7% and 6.5% is considered at risk for diabetes, which means that they are likely to develop the disease without intervention. Answer D:Metabolic syndrome includes multiple risk factors for atherosclerosis, including dyslipidemia, obesity, hypertension, and insulin resistance. The patient would not be diagnosed with metabolic syndrome since other lab values are normal.

A physical therapist is completing an initial examination with a 60-year-old patient with a left CVA in a skilled nursing facility. Upon inspection of the patient's skin, the therapist noted that there was an area on the patient's right ischial tuberosity that had a full-thickness loss of skin that exposed underlying adipose tissue but did not reveal bone or muscle. What stage of pressure ulcer is described in this scenario? a) Stage 1 b) Stage 2 c) Stage 3 d) suspected deep tissue injury

Correct answer: C The National Pressure Ulcer Advisory Panel and the Agency for Healthcare Research and Quality recommend the use of a standardized staging system for the description of pressure ulcers. A stage 3 pressure ulcer is a full-thickness tissue loss that exposes subcutaneous fat. However, bone, tendon, or muscle are not exposed in a stage 3 pressure ulcer. A pressure ulcer is often caused by shear or pressure that occurs on the ischial tuberosities as a result of inadequate weight shifting or positioning. Answer A:A stage 1 pressure ulcer describes an area of intact skin with nonblanchable redness, which is usually found over a bony prominence. Answer B:A stage 2 pressure ulcer is a partial-thickness loss of skin that appears as a shallow open ulcer with a red-pink wound bed but without slough tissue. Answer D:A suspected deep tissue injury (DTI) is a purple or maroon local area of discolored skin that results from damage to underlying soft tissue. The DTI may have a softer feel or a difference in temperature in comparison to the surrounding tissue. Bottom Line:The National Pressure Ulcer Advisory Panel and the Agency for Healthcare Research and Quality recommend the use of a standardized staging system for the description of pressure ulcers. A stage 3 pressure ulcer is a full-thickness tissue loss that exposes subcutaneous fat. However, bone, tendon, or muscle are not exposed in a stage 3 pressure ulcer. Physical therapists must be able to accurately classify pressure ulcers in order to develop effective treatment plans for their management.

A patient who ambulates with bilateral axillary crutches following foot surgery reports numbness and tingling in the right posterior forearm. On examination the physical therapist notes that the patient also has a wrist drop on the right. What structure is the MOST likely cause of the patient's complaints? A. Median nerve B. Axillary nerve C. Radial nerve D. Ulnar nerve

Correct answer: C The correct answer is radial nerve. Complaints of numbness and tingling in the right posterior forearm and weakness of wrist extensors may be due to prolonged compression of the radial nerve when ambulating with ill-fitted crutches. This condition is also called crutch palsy. The radial nerve innervates the wrist extensors and controls sensation in parts of the forearm. Answer A:The median nerve innervates the forearm flexors and pronators, which are not affected in this scenario. Answer B:The axillary nerve innervates the deltoid and teres minor muscles, which are not affected in this scenario. Answer D:The ulnar nerve innervates the fine motor muscles of the hand, mainly flexors, which are not affected in this scenario. Bottom Line:Compression of the radial nerve caused by constant pressure on the nerve when using ill-fitting crutches is the most likely cause. This causes symptoms such as numbness and tingling in the forearm and weakness of the wrist extensors.

A physical therapist performs a movement analysis on a patient with a weak lower trapezius. The patient will MOST likely demonstrate limitation in which of the following movements? a) shoulder extension b) shoulder ER c) shoulder abduction d) shoulder horizontal abduction

Correct answer: C The correct answer is shoulder abduction. In full abduction with scapular retraction, all parts of the trapezius are recruited: retraction of the shoulder girdle by the entire muscle, and upward rotation of the scapula by the upper and lower portions of the muscle. The upper trapezius produces scapular elevation, while the lower trapezius produces scapular depression. Answer A:Upward rotation of the scapula is not needed with shoulder extension.Answer B:Upward rotation of the scapula does not occur with shoulder external rotation. Answer D:Upward rotation of the scapula does not occur with shoulder horizontal adduction. Bottom Line:Shoulder elevation occurs in a precisely coordinated series of synchronous motions termed scapulohumeral rhythm. The lower trapezius depresses the scapula during shoulder abduction.

A patient with a left L5 nerve root compression is having difficulty walking. Upon gait analysis, you notice that the patient exhibits hip drop on the left, and lateral trunk lurch to the right side when weight bearing on the right leg. The patient reports that the symptoms worsen throughout the day upon prolonged walking. Based on this information, which structure is MOST likely affected? a) inferior gluteal nerve b) obturator nerve c) superior gluteal nerve d) sciatic nerve

Correct answer: C The correct answer is superior gluteal nerve. The symptoms described in this scenario are called Trendelenburg gait, where the patient experiences hip drop on the contralateral side to the affected muscle when weight bearing on the affected side. To compensate, the patient lurches the trunk laterally to the same side as the affected muscle. Trendelenburg gait is caused by weakness of the hip abductors (gluteus medius). The hip abductors are innervated by the superior gluteal nerve, which arises from nerve roots L4, L5, and S1. With the compression of the nerve root L5, the nerve supplying the hip abductors produce symptoms such as muscle weakness, causing problems with gait. Answer A:Inferior gluteal nerve is supplied by L5 as well but mainly innervates gluteus maximus, which is the primary hip extensor. Answer B:Obturator nerve is supplied by L2-L4, which is not affected in this scenario. Answer D:Sciatic nerve is supplied by L4-S3 but does not innervate the hip abductors. Bottom Line:An L5 nerve root compression may cause problems with the hip abductor muscles, which are supplied by the superior gluteal nerve. Weakness of the hip abductors could result in a Trendelenburg gait pattern.

A patient reports severe vertigo for the past week that occurs when rolling over in bed to the right side and lasts about 30 seconds. During the examination, the therapist wants to test balance prior to the positional testing for vertigo, which will likely provoke her symptoms. What is the proper SEQUENCING for balance testing for modified Clinical Test for Sensory Integration and Balance (CTSIB)? A. Feet apart eyes open, modified tandem eyes open, uneven surface eyes open, feet apart eyes closed, modified tandem eyes closed, uneven surface eyes closed B. Uneven surface eyes open, uneven surface eyes closed, modified tandem eyes open, modified tandem eyes closed, feet apart eyes open, feet apart eyes closed C. Feet apart eyes open, feet apart eyes closed, modified tandem eyes open, modified tandem eyes closed, uneven surface eyes opened, uneven surface eyes closed D. Feet apart eyes closed, feet apart eyes open, modified tandem eyes closed, modified tandem eyes open, uneven surface eyes closed, uneven surface eyes open

Correct answer: C The correct sequencing for balance testing should progress from easier to more difficult activities. Balance is maintained by sensory input from three systems: vision, somatosensory, and vestibular. Conducting balance testing in the correct order can progressively isolate each system to help the therapist identify the patients area of weakness. For instance, if the patient is okay with feet apart eyes open, then falls with feet apart eyes closed, one can assume this patient is very visually dominant with balance (because nothing with somatosensory or vestibular changed). In another example, if the patient is okay on all tests except for the last condition (uneven surface with eyes closed), the therapist can assume there is a vestibular component to the balance deficit. During that condition, vision and somatosensory are altered or removed and the patient is relying mostly on the vestibular system to maintain upright posture. A fall on the condition is strongly correlated with a vestibular dysfunction.A test such at the CTSIB is a low-tech sensory organization test to determine which system is at fault leading to impaired balance and postural stability. Answer A:Each position should be tested eyes open immediately followed by eyes closed in the same position in order to get a good comparison and systematically eliminate a particular balance system. Answer B:This is an incorrect order of testing as it begins with the more difficult activities and progresses to easier ones. Answer D:This is incorrect due to starting each activity with eyes closed before eyes open. Bottom Line:A test such at the CTSIB is a low-tech sensory organization test to determine which system is at fault leading to impaired balance and postural stability. The correct sequencing for balance testing should progress from easier to more difficult activities.

A physical therapist is instructing a patient in pursed lip breathing. Which of the following are the BEST instructions to give the patient? a) inspiration and expiration should be of the same duration b) expiration should be forceful c) expiration should be twice as long as inspiration d) inspiration should be twice as long as expiration

Correct answer: C The expiration phase should be twice as long as the inspiration phase, preventing premature closing of the airways and allowing more CO2to be expelled. Answer A:Even in normal breathing, inspiration and expiration are not equal since inspiration is active and expiration is passive as the diaphragm floats back down, and takes a little longer. With pursed lip breathing (PLB) to address dyspnea, the expiration should be twice as long. Answer B: Expiration does not need to be done forcefully. It should be done through pursed lips in a slow controlled fashion and last twice as long as inspiration allowing for greater expulsion of CO2. Answer D:The exact opposite is true, expiration should be twice as long as inspiration so the body can expel the CO2. Bottom Line:Pursed lip breathing (PLB) is a strategy for dyspnea. Dyspnea occurs when carbon dioxide levels rise and the brain tells the body to breather faster. With PLB, a patient inhales through the nose, which allows the air to be warmed, humidified, and filtered, and then breathes out through pursed lips which creates a pressure that keeps the airways open longer, allowing for greater expulsion of CO2. The expiration phase should be twice as long as the inspiration phase, preventing premature closing of the airways, which happens with obstructive pulmonary diseases.

Which of the following BEST describes the proper procedure for measuring ankle girth in a patient with pitting edema in the lower extremities? a) press your finger on the edema between the malleoli and remove, then measure the time until the depression disappears b) using a tape measure, obtain girth measurement by wrapping around widest point of the ankle joint c) using a tape measure, obtain girth measurement with a figure-of-eight pattern around the ankle and foot d) use a string to wrap around the widest part of the ankle to obtain the circumference and then measure the length of that string

Correct answer: C The figure-of-eight measurement method is a standardized way of obtaining a measurement of ankle swelling. It uses anatomical landmarks to guide a standard figure-of-eight pattern in the following order beginning midway between the tibialis anterior tendon and the lateral malleolus: tuberosity of the navicular, medial longitudinal arch of the foot, base of the 5th metatarsal, tibialis anterior tendon again, distal medial malleolus, Achilles tendon, distal lateral malleolus, and then back to the starting point. Answer A:This technique identifies pitting edema but does not measure ankle girth. Answer B:This method is one way of obtaining an objective measurement, but it is not the most reliable method. Due to the uneven surfaces of the ankle anatomy and the transient nature of ankle edema, the widest part of the ankle joint is not consistent. A more reliable method would be the figure-of-eight method that utilizes standard anatomical landmarks. Answer D:This method describes an indirect measurement of ankle girth. It is not a reliable method because of the possibility of user error. In addition, the widest part of the ankle joint is not always consistent. Therefore, a more reliable method would be the figure-of-eight method that utilizes standard anatomical landmarks. Bottom Line:The presence of ankle swelling is easily noticeable with visual inspection and with palpation. However, objective measurements can be difficult due to the uneven nature of the ankle anatomy as well as transient changes due to position. The figure-of-eight method has been found to be highly reliable in obtaining measurements of ankle swelling.

A patient with a traumatic brain injury is able to perform the motor skills during a physical therapy session, but is demonstrating deficits in attention. Which of the following areas of the patient's brain were MOST likely affected? a) basal ganglia b) cerebellum c) frontal cortex d) hippocampus

Correct answer: C The frontal cortex is commonly damaged in patients with traumatic brain injury and results in deficits in attention. Sustained and divided attention may both be affected after traumatic brain injury and contribute to deficits in executive function in these patients.Answer A:The basal ganglia is not often affected by traumatic brain injury, while damage may cause deficits in the acquisition of motor skills. Answer B:The cerebellum may be affected by a traumatic brain injury due to both a coup-contrecoup effect and diffuse axonal injury. Damage to the cerebellum after a traumatic brain injury typically causes ataxia and problems with motor learning. Answer D:The hippocampus is largely responsible for the storage of long-term memory. While long-term memory deficits are found in patients with traumatic brain injury, these deficits are usually caused by damage to the frontal lobe

A physical therapist is conducting an initial examination with a patient who has a history of an ASIA Impairment Scale A cervical spinal cord injury. The patient has 5/5 muscle strength in the elbow flexors, elbow extensors, finger flexors, and wrist extensors but 0/5 muscle strength in the finger abductors. What is the patient's motor level? a) C6 b) C7 c) C8 d) T1

Correct answer: C The motor level is defined by the lowest key muscle function that has a grade of at least 3 providing the key muscle functions represented by segments above that level are graded as a 5. Since the key muscle for assessing function at the C8 level is the finger flexors, and the patient has 0/5 strength in the finger abductors, the motor level for the patient in this scenario would be C8. Answer A:The key muscle for assessing function at the C6 level is the wrist extensors. The patient's motor level would not be classified as C6 since the patient has 5/5 strength in lower spinal levels. Answer B:The key muscle for assessing function at the C7 level is the elbow extensors. The patient's motor level would not be classified as C7 since the patient has 5/5 strength in lower spinal levels. Answer D:The key muscle for assessing function at the T1 level is the finger abductors, so the patient's motor level would not be classified as T1 since the patient has 0/5 strength.

A physical therapist asks a postpartum patient with back pain to lie supine and lift one leg to perform the active straight leg test (ASLR). She describes the action as very difficult. The physical therapist provides an anterior pelvic compression force and repeats the test with no change. The physical therapist then provides a posterior pelvic compression force and repeats the test, and the patient reports that raising the leg is now much easier. Which muscle group should the physical therapist strengthen based on this patient's findings? A. Transversus abdominis B. Iliopsoas C. Multifidi D. Rectus abdominis

Correct answer: C The multifidus is a deep spinal stabilizer that should help keep the pelvis stable during a straight leg raise in supine. If weak or dysfunctional, its stabilizing actions can be assisted via a posterior pelvic compression force. If applying that force makes the leg raise easier, an exercise program that emphasizes the timing and strength of the multifidus is recommended. Answer A:The transversus abdominis muscle would be targeted with an anteriorly directed pelvic compression. Answer B:Hip flexor function is not affected by the posterior or anterior compressive force through the pelvis. Answer D:The ASLR test does not describe ways of targeting the rectus abdominis for stability. Bottom Line:The ASLR test is a functional test that is used primarily to diagnose pregnancy-related pelvic pain. The ASLR test grades the leg raise by difficulty and then repeats the test while compression is applied in either a posterior or anterior direction to assist force closure. The multifidus is a common area of dysfunction that can be supported with a posterior pelvic compression force. The transversus abdominis is targeted if anterior compression makes the leg raise less difficult.

A patient has been in the ICU for 5 days with acute respiratory failure and bilateral pneumonia. The patient is on 4 L O2 via nasal cannula with O2 saturation at 98%. During initial examination in the ICU, the physical therapist assists the patient to transfer from supine to sitting at edge of bed. O2 saturation is at 90%. Next, the physical therapist assesses standing, and the oxygen saturation is at 82%. What is the most appropriate action for the physical therapist to do NEXT? a) continue the examination, assess standing tolerance, and monitor that O2 saturation stays above 80% b) seat the patient into the closest chair and see if the saturation comes up c) return the patient to sitting at the edge of the bed and see if the saturation comes up d) call the patient's nurse to the room immediately

Correct answer: C The next appropriate and safe option is to return to patient to sitting at the edge of bed. The physician determines a minimal acceptable saturation, which is usually 90-94%. The saturation rate when the patient was sitting at the edge of the bed was 90% originally, so returning the patient to sitting at the edge of the bed should allow the opportunity for the saturation level to rise. If the patient's saturation does not recover or continues to drop, this patient is already in the bed and can be returned to supine. Answer A:The saturation rate has dropped too low, at 82%, and the main objective is to get it back up. For this patient, standing is too taxing on the cardiopulmonary system. With such a large drop, and a level of 82%, now is not the time to assess standing tolerance. Anything below 90% is a concern. Answer B:Assisting the patient to the chair would allow the patient to rest and hopefully allow the saturation to recover. However, if the saturation continues to drop, having the patient already sitting at the edge of the bed would allow them to return immediately to supine instead of having to come back to the bed Answer D:Calling the patient's nurse is appropriate after getting the patient to a safe position to allow oxygen saturation to recover. The first thing to do is always to ensure the safety of the patient. Informing the nurse of the patient's response to activity should be done after every treatment in the ICU. Bottom Line:In the acute setting, as the patient begins functional progression, it is important to remember that even the smallest amount of movement can cause abnormal exercise response in patients. Patient vitals such as HR, BP, and oxygen saturation give the therapist important information regarding how the patient is tolerating the activity. Pulse oxymetry measures oxyhemaglobin saturation. Normal range for O2 saturation is >95%, and the recommendation is to keep the rate over 90% during exercise. In this scenario,returning the patient to the edge of the bed and seeing if the oxygen saturation comes up is the best next step.

A patient with right knee synovitis is being examined by the physical therapist. The swelling makes it impossible for the patient to obtain full knee extension. Which of the following end feels should the physical therapist expect to feel during passive range of motion for right knee extension? a) firm b) capsular c) soft d) hard

Correct answer: C The normal end feel for knee extension is a hard end feel, but in this case the edema does not allow the knee to fully extend and have that hard end feel. The swelling makes it a soft boggy end feel. A soft end feel may occur normally when a joint has reached its end range and is limited by contact of soft tissue on soft tissue, such as at end range knee flexion or elbow flexion. However, a soft end feel also can be felt when there is swelling in the joint, as in the patient case described here. Answer A:A firm end feel occurs when the structure limiting the range of motion is a capsule, ligament, or tendon. In this patient case, the edema is limiting the motion. Answer B:A capsular end feel is a firm end feel and occurs when the joint capsule limits the motion. Answer D:A hard end feel is classified as a bone-on-bone end feel and is the normal end feel for knee extension. However, in this case the edema limits the full range, and the knee does not fully extend into the screw-home position with a hard end feel. Bottom Line:The end feel of a joint during PROM can provide feedback as to the reasons for restriction of joint range of motion. Edema of a joint will usually result in a soft end feel.

A physical therapist is examining a patient who has difficulty ambulating due to consequences of diabetic neuropathy. Upon examination, the patient's right foot is red, swollen, and warm to the touch. In addition, the patient's arch appears collapsed and the patient reports pain in the foot while sensation is absent. Which of the following interventions would be CONTRAINDICATED in this scenario? A. axillary crutches to allow non-weightbearing activity B. knee scooter to allow non-weightbearing activity C. transcutaneous electrical nerve stimulation D. total contact casting

Correct answer: C The patient described in the scenario has a Charcot arthropathy, which is a complication of diabetic neuropathy. Charcot deformities often cause foot ulcerations that eventually can lead to an amputation for patients with diabetes. The treatment for Charcot arthropathy typically consists of trying to decrease the inflammation of the joint by offloading the joint while managing pain.The use of transcutaneous electrical nerve stimulation(TENS) is contraindicated due to the lack of sensation in this patient. Answer A:The use of axillary crutches to allow non-weightbearing activity on the right lower extremity would be a safe and acceptable treatment option for a patient with a Charcot arthropathy. Answer B:The treatment for Charcot arthropathy typically begins with off-loading the foot so use of a knee scooter would be appropriate. Answer D:The use of total contact casting to reduce pressure on the foot would be a safe and acceptable treatment option for a patient with a Charcot arthropathy. Bottom Line:The treatment for Charcot arthropathy typically consists of trying to decrease the inflammation of the joint by offloading the joint while managing pain. Physical therapists who work with patients with diabetes need to be able to apply safe and effective interventions to manage the common complications associated with the disease.

A 12-year-old boy with a report of bilateral knee pain is being examined in an outpatient physical therapy office. He plays on the town's travel basketball team, and the pain is limiting his ability to play. Physical observation revealed tenderness and mild swelling at the tibial tuberosities. Ligamentous tests and meniscal tests are negative. Which of the following initial interventions would be BEST based on the above information? A. Kinesiotaping to properly align the patella and allow the quadriceps to pull at the correct angle. B. Orthotics to prevent overpronation, which can cause genu valgus C. Patient education to avoid jumping until pain starts to improve, and moderate-intensity quadriceps strengthening D. Neuromuscular re-education for proper jumping/landing form since basketball causes the pain, and ice 2-3 times per day

Correct answer: C The patient has Osgood-Schlatter disease, also known as apophysitis of the tibial tubercle. It occurs in children around the time of the growth spurt if there is an excessive amount of jumping and explosive quadriceps activity, causing the quadriceps tendon to apply a traction force to the tubercle. The intervention should include activity modification to allow the area to heal, with moderate-intensity quadriceps strengthening. Ice and anti-inflammatories are also helpful in reducing the pain. Answer A:The alignment of the patella was not mentioned in the examination findings, and a malaligned patella does not contribute to apophysitis of the tibial tubercle. The mechanism of injury is the force of the quadriceps pulling at their attachment. Answer B:There is not enough information to know if the patient overpronates, and genu valgus does not cause apophysitis of the tibial tubercle. Answer D:This answer is incorrect as the patient is still in the acute stage of healing. This should be performed after successful healing and strengthening has occurred. It is contraindicated to continue with a lot of jumping while there is an inflamed growth plate at the tibial tubercle. Bottom Line:Osgood-Schlatter disease is an apophysitis of the tibial tuberosity due to traction from the quadriceps. This injury occurs in adolescent children who perform a lot of running and jumping activities. Flexibility, ROM, strength, and balance are all important, but initially activity modification must occur to allow the area to heal, followed by moderate-intensity quadriceps strengthening.

A patient has right greater trochanteric pain when running. Leg length measurements in supine position indicate the right leg is longer by 2.5 cm. The therapist asks the patient to stand up for a postural evaluation. In standing, the iliac crest heights, anterior superior iliac spine (ASIS) height, posterior superior iliac spine (PSIS) height, and knee crease heights all are level. Which of the following should the physical therapist do NEXT? a) perform a quadrant/scour test b) measure the leg lengths of tibia and femur seperately c) examine the posture of the foot and ankle d) watch the patient run on the treadmill

Correct answer: C The patient has a longer right leg, therefore, when he stands up, the right landmarks should be higher. If they are not, some sort of compensation is occurring. This calls for examination of the foot and ankle posture. In this case, the iliac crest and knee heights are level, so the compensation must be at the ankle, most likely in the form of excessive pronation on the longer side. In a runner with a longer limb and excessive pronation to compensate, there can be a number of biomechanical issues, such as excessive internal rotation of the limb over the pronating foot, in this case causing pain at the greater trochanter. Answer A:The pain is not in the hip joint but at the greater trochanter. In addition, the leg-length discrepancy that is compensated for in standing may be a contributing factor to poor running biomechanics. The quadrant test, or scour test, identifies issues within the coxofemoral joint itself. When an axial load is applied, it will also be able to identify a labral tear if there is catching and clicking within the joint. This is not the next best action in this patient scenario. Answer B:There is no new knowledge to gain by measuring leg lengths of each bone individually. It is already established that the right side is longer. The fact that the right hip and knee landmarks are not higher than the left in standing should lead the therapist to locate the compensation and look at the foot and ankle next. Answer D:The therapist should absolutely perform a gait evaluation in walking and running, but of the options provided, the NEXT best thing to do would be to continue the postural examination and determine why the right landmarks appear level at the hip and knee even thought the right leg is longer. That will help identify where the body is compensating and the effect on the running biomechanics. Bottom Line:A patient with a longer leg should demonstrate that difference during a postural examination of foot and ankle while standing. In this example, the patient compensated for the longer leg in standing by pronating, making the hip and knee landmarks appear level.

A physical therapist is conducting an initial examination with a patient with an incomplete spinal cord injury due to fall. As a result of the injury, the patient has no sensory and motor function at the 4th lumbar vertebrae and below. What type of orthotics would be MOST LIKELY to be recommended for this patient for ambulation? a) HKAFO b) KAFO c) AFO d) PLS

Correct answer: C The patient in the described scenario would be expected to have no motor function in the dorsiflexors, plantarflexors, and extensors due to the lesion affecting motor function at L4 and below. In addition, the patient would have no sensation starting at the L4 dermatome, so that they would have sensory deficits below the knee as well as the posterior thigh. As a result of these deficits, the patient would require bilateral ankle-foot orthoses (AFO) to compensate for these impairments and allow for ambulation. Answer A:A HKAFO (hip-knee-ankle-foot orthosis) is unlikely to be recommended for this patient since the patient has intact function at L2 and L3, so that compensation for the weakness of the hip flexors and knee extensors with an orthotic would not be necessary. Answer B:A KAFO (knee-ankle-foot orthosis) is unlikely to be recommended for this patient since the patient has intact function at L3, so that compensation for weakness of the knee extensors with an orthotic would not be necessary. Answer D:A posterior leaf spring (PLS) is a thin plastic ankle foot orthosis that is placed into a shoe. During early stance, the PLS will bend slightly and then recoil during late stance to assist with foot clearance and to allow activity of the dorsiflexors during swing. A PLS does not enclose any of the ankle joint so that it does not control ankle inversion and eversion, so that it is not likely to be recommended for a patient with absent ankle sensation and motor function. Bottom Line:An injury at L4 would paralyze all ankle and foot musculature. A solid ankle-foot orthosis (AFO) with full ankle support is required in order to support the foot during gait and allow stability and ground clearance. TrueLearn Insight :The decision to recommend an orthotic device is based on a number of factors, including patient's impairments, functional deficits, and prognosis. Physical therapists must be able to able to base their orthotic prescriptions by balancing the advantages and disadvantages of different types of orthotics as well as the specific effect of the orthotic on a patient's gait based on their knowledge of biomechanics and kinesiology.

A physical therapist is examining a patient who has difficulty walking without losing their balance, especially when changing direction. The patient walks with a wide base of support and exhibits irregular step lengths bilaterally. Which of the following areas of the nervous system is the MOST likely location of injury for this patient? a) basal ganglia b) brainstem c) cerebellum d) frontal lobe

Correct answer: C The patient in the scenario described is demonstrating an ataxic gait pattern which is found in patients with damage to the cerebellum. The etiology of cerebellar injuries include stroke, brain tumor, multiple sclerosis, and other neurological disorders. Answer A:Patients with damage to the basal ganglia show a variety of gait deficits based on the etiology of the injury. For example, the gait pattern typically found in patients with Parkinson's disease often includes a shuffling, festernating series of steps with little trunk rotation and arm swing. Answer B:Patients with damage to the brainstem show an unpredictable pattern of gait deficits based on the etiology of the injury and which aspects of the motor system are affected. Answer D:The motor cortex is located in the rear part of the frontal lobe. Patients with damage to the motor cortex often show a hemiparetic gait pattern in which one side of the body is much more affected than the other. Bottom Line:Physical therapists need to be able to identify lesion location, such as cerebellum, based on observational gait analysis in order to plan their examination and intervention strategies in patients with neurological injuries.

Which of the following scenarios would DESCRIBE a postsurgical female cardiac patient 2 days post op with hemoglobin (Hgb) and hematocrit (Hct) levels of 8.2 g/dL and 25%, respectively. a) sharp decrease in heart rate upon standing; BP normal and stable b) bradycardia at rest; with standing BP drops lower c) sharp increase in heart rate upon standing; able to walk only a short distance (20 feet) prior to requiring test d) walking and reaching a 6-minute walk distance of 950 ft but with sharp increase in heart rate

Correct answer: C The patient in this case is presenting with lab values that are below the normal range for females (Hgb of 12-16 g/100mL and Hct of at least 37%). The lab values indicate postsurgical anemia, which is common among patients after cardiac surgery, such as coronary artery bypass graft (CABG). This type of acute anemia decreases the oxygen-carrying capacity of blood and in turn increases the demand on the heart. Common symptoms include elevated heart rate (especially a sharp increase from rest), activity intolerance, and generalized weakness and fatigue. Patients with acute anemia are likely to tolerate only light activity and may require frequent rest. Answer A:The patient is this case is presenting with postsurgical anemia, which is unlikely to cause a drop in heart rate. Rather, it may actually cause a drop in blood pressure with positional changes (orthostatic hypotension). A decrease in heart rate with activity is likely attributed to an arrhythmia or a heart block. Therefore, it is not likely based on the information in this patient case. Answer B:A patient presenting with postsurgical anemia is not likely to present with a slow heart rate (bradycardia) unless there is additional evidence of an arrhythmia or heart block. Even with a baseline heart rate that is very slow, the heart should at least increase with standing activity. Therefore, the presentation in this answer choice does not match the described scenario. Answer D:A patient presenting with postsurgical anemia is unlikely to be able to tolerate a 6-minute walk test (6MWT). In addition, a 6MWD of 950 feet is approaching the normative value for this patient. Bottom Line:A hallmark sign of postsurgical anemia is activity intolerance. Heart rate rises disproportionately to the activity demand. Although it is possible that patients with acute anemia may be asymptomatic, it is important to be able to recognize the expected physiologic changes and limitations in activity tolerance that many of these patients may experience in order to appropriately and safely progress functional activity.

A 75-year-old patient is admitted to the hospital with acute urinary tract infection causing significant weakness and confusion. The patient lives in an assisted living facility and was independent in ambulation with a rolling walker prior to admission. Upon evaluation, the PT provides about 60% assistance for the patient to perform sit to stand. The patient is then able to ambulate with the walker about 10 feet with the PT keeping one hand on the patient's waist. Which of the following statements BEST describes the level of assistance provided by the PT? A. Minimal assistance for transfer, contact guard for gait B. Minimal assistance for transfer and for gait C. Moderate assistance for transfer, contact guard for gait D. Moderate assistance for transfer, supervision for gait

Correct answer: C The patient requires moderate assistance for the transfer. For minimum assistance, the patient can do most of the activity, and for maximum assistance the patient cannot assist at all with the activity. Here, the patient needs the PT to do at least 60% of the lifting. And so, the transfer is graded as moderate assistance.As for the gait, the therapist is contacting the patient and staying very close for safety, and that is graded as contact guard. Answer A:For minimum assistance, the patient can do most of the activity. During the sit-to-stand transfer, this patient can do only 40% of the effort required, so it is a moderate assist. Contact guard is correct for gait. The clinician has a hand on the patient's waist and remains close to patient to assist immediately if needed. Answer B:Minimal assist means the patient can perform most of the activity without assistance. During the sit-to-stand transfer, this patient can do only 40% of the effort required, so it is a moderate assist. The assistance during gait is graded as contact guard. The clinician has a hand on the patient's waist and remains close to the patient to assist immediately if needed. Answer D:The patient requires moderate assistance for the transfer. For minimum assistance, the patient can do most of the activity, and for maximum assistance the patient cannot assist at all with the activity. Here, the patient can do about 40%, which is less than half of the effort required. And so, the transfer ismoderateassistance.As for the gait, the therapist is contacting the patient and staying very close for safety, and that is graded as Contact Guard. In supervision, someone is nearby, but it is unlikely the patient will need assistance. This patient has weakness and confusion, and needed moderate assist to stand up, so "supervision only" would be unsafe. Bottom Line:When differentiating among the levels of assistance, independent is a term used when the patient can perform the skill consistently with no one present.Supervision requires someone within arm's reach as a precaution, but the patient is unlikely to need assistance. In contact guard assist, someone is positioned close to patient with hands on to be ready to provide assist if needed.Minimal assistance means the patient is able to complete MOST of the activity without assistance.Moderate assistance means the patient is able to perform PART of the activity without assistance.Maximum assistance means the patient is unable to assist with ANY part of the activity.

A 65-year-old male is referred to a physical therapist by his orthopedist for treatment of bilateral adhesive capsulitis. The patient reports that his arms feel very stiff, and he has trouble raising them. The therapist performed a full examination. During the examination, the therapist noted the patient's posture was kyphotic with rounded shoulders. PROM was within functional limits in both shoulders with stiffness and cogwheeling. Bilateral GH joint mobility was WNL. Strength was grossly 4/5 for all musculature. Sensation was intact in bilateral upper extremities. What should the physical therapist do NEXT? A. Apply a thermal modality to bilateral anterior shoulders in preparation for stretching the stiff shoulders B. Perform grade I and II posterior glenohumeral joint mobilizations to reduce the forward shoulder posture C. Contact the patient's orthopedist to discuss the findings of the examination D. Provide the patient with some simple self-stretches he can do daily to improve his kyphotic posture

Correct answer: C The physical therapist should recognize the signs of Parkinson's as a possible reason for the patient's complaints of shoulder stiffness. There was no limitation of motion and no capsular pattern to support a diagnosis of adhesive capsulitis, yet there was cogwheeling upon PROM. In addition, it is rare to have bilateral adhesive capsulitis. It is likely a more systemic issue. The patient also has a kyphotic posture, which may or may not be related to the onset of Parkinson's; however, this patient needs to be examined by a neurologist before the physical therapist begins treatment. The therapist should discuss the findings with the orthopedist and refer the patient back to his doctor for follow-up. Answer A:At this time the main concern is that the patient may have been misdiagnosed and is showing signs of Parkinson's disease. The patient has full range of motion in his shoulders and does not need stretching of the shoulders. Answer B:At this time the main concern is that the patient may have been misdiagnosed and is showing signs of Parkinson's disease. Therefore, addressing the forward shoulder posture is not a priority. Answer D:At this time the main concern is that the patient may have been misdiagnosed and is showing signs of Parkinson's disease. Daily postural stretching is a goodidea for anyone with poor posture; however, this patient needs to be examined by a neurologist before the physical therapist begins treatment. Bottom Line:Cogwheeling during passive range of motion is never a normal finding and should lead a physical therapist to suspect Parkinson's disease.

Which of the following is NOT a physiological effect of hydrotherapy when the body is immersed in water? a) increased total work of breathing b) decreased weight bearing c) decreased urine output d) increased cardiac output

Correct answer: C The physiological effects of water are the result of its physical properties. These properties include a relatively high specific heat and thermal conductivity as well as the ability to provide buoyancy, resistance, and hydrostatic pressure to the body. Immersion of an individual up to the neck in water has been shown to INCREASE urine production and urinary sodium and potassium excretion. It is proposed that these effects are the result of increased renal blood flow and decreased antidiuretic hormone and aldosterone production. Answer A:Immersion of the whole body in water increases the work of breathing because the shift of venous blood from the peripheral to the central circulation increases the circulation in the chest cavity. Hydrostatic pressure on the chest wall increases resistance to lung expansion. Immersion in water up to the neck has been shown to decrease expiratory reserve volume by about 50% and to decrease vital capacity by 6% to 12%. These effects, when combined, increase the total work of breathing by about 60%. Answer B:The buoyancy of water unloads weight-bearing joints and thus allows patients with load-sensitive joints to perform exercises with less trauma and pain. For example, at 75% immersion, weight bearing on the lower extremities is reduced by 75%. Answer D:The cardiovascular benefits of hydrotherapy are primarily due to hydrostatic pressure. The hydrostatic pressure exerted on distal extremities with upright immersion in water displaces venous blood proximally from the extremities and thus enhances venous return by shifting blood from the periphery to the trunk. With immersion to the neck, central blood volume increases by about 60%, and cardiac volume increases by about 30%. The heart, in turn, responds with an increased force of cardiac contraction and increased stroke volume. This results in approximately 30% increased cardiac output.

A patient reports medial ankle pain localized around the medial malleolus that worsens during ambulation and prolonged standing and can be felt deep along the posterior leg. Observation reveals bilateral pes planus. In single-limb stance, the individual's frontal plane stability is poor, and the therapist notices increased sway. Gait evaluation reveals excessive pronation. Which of the following options is the MOST likely explanation for the pain? a) deltoid sprain b) gastrocnemius sprain c) posterior tibialis dysfunction d) tarsal tunnel syndrome

Correct answer: C The posterior tibialis tendon runs posteriorly behind the medial malleolus and is a source of medial ankle pain. It begins proximally on the posterior tibia and is a deep muscle that can provide posterior leg pain. It attaches to the navicular and other medial bones of the foot and is a support structure for the medial longitudinal arch. Because the tendon is a static and dynamic arch stabilizer, it is frequently irritated when standing for prolonged periods, especially in individuals with pes planus. Those with poor arch dynamic stability also have difficulty controlling inversion/eversion (frontal plane) balance in single-limb stance. Answer A:Deltoid strain is a source of medial ankle pain but typically presents with a trauma and will not cause any posterior leg pain. Answer B: Gastrocnemius strain would be more localized to pain posteriorly and with push-off only. Answer D: Tarsal tunnel syndrome may present as medial ankle pain, but there would also be neurological symptoms like numbness and tingling in the foot, due to compression of the posterior tibial nerve in the tunnel. Bottom Line:The posterior tibialis is a dynamic supporter of the medial longitudinal arch and can concentrically raise the arch and eccentrically lower the arch. In patient's with pes planus, the posterior tibialis can be overworked and develop tendinitis, causing pain in the medial foot and ankle as well as the deep posterior calf.

Which of the following lists correctly contains three of the core values from the Physical Therapist Code of Ethics? a) excellence, education, and integrity b) education, compassion, and consultation c) accountability, excellence, and professional duty d) professional duty, consultation, and altruism

Correct answer: C The seven core values that are part of the PT code of ethics are accountability, altruism, compassion/caring, excellence, integrity, professional duty, and social responsibility. There are five roles of the PT within the code of ethics that are separate from the core values; these are the management of patient/client, consultation, education, research, and administration. This list of accountability, excellence, and professional duty is the only one that includes core values solely. Answer A:Education is considered one of the roles of a PT and is not a core value. Excellence and integrity are two of the core values. Answer B:Education and consultation are two of the roles of a PT and not core values. Compassion/caring is considered a core value. Answer D:Consultation is one of the roles of a PT, not a core value. Professional duty and altruism are core values.

A patient presents with complaints of low back pain with radicular symptoms into the right leg at times. During the examination, the patient is asked to sit with poor, rounded posture with shoulders hunched forward. The therapist then flexes the patient's neck down while extending one knee and passively dorsiflexing the foot. The patient has a reproduction of painful symptoms. WHAT TEST did the therapist perform? A. Straight leg raise/Lasegue's test B. Spurling's test C. Slump test D. Long sitting test

Correct answer: C The therapist performed the slump test,which is a common neurological test for the lower limb. It is considered a sitting dural stretch test as it places increased tension on the neuromeningeal tract. The test is performed on each leg to look for asymmetry. The test is considered positive if the patient complains of sciatic pain or there is reproduction of painful symptoms implicating restriction or impingement of the dura, spinal cord, or nerve roots. Answer A:The straight leg raise test, or Lasegue's test, also assesses lower extremity (LE) neurological symptoms; however, it is performed with the patient in supine. Answer B:Spurling's test (foraminal compression test) is a cervical spine test used to detect nerve compression or radiculitis into the upper extremity. Answer D:The long sitting test, or "supine to sit" test, is used to identify dysfunction of the sacroiliac joint that may be the cause of functional leg-length discrepancy. Bottom Line:The slump test is a common neurological test for the lower limb. It is considered a sitting dural stretch test. The patient is asked to sit in a "slumped position." The next steps are as follows: flexing the patient's neck, extending the patient's knee, and passively dorsiflexing the patient's foot. Symptoms are assessed after each step. The process is repeated on both legs. Symptoms of sciatic pain or reproduction of the patient's symptoms indicates a positive test for dura, spinal cord, or nerve root restriction or impingement.

A 53-year-old chef complains that recently he has been dropping items and having weakness of the hand and forearm after working for an hour or so in the kitchen. He reports being right hand dominant and performing tasks of lifting heavy pots and pans, chopping, and stirring during preparations. He also reports numbness and tingling in the right arm at times. During the examination, the therapist would like to screen cervical level 5-6 (C 5-6) deep tendon reflex (DTR) and key muscle strength at the same level. Screening should include which of the following combinations? a) biceps DTR with wrist flexion strength b) triceps DTR with elbow flexion strength c) brachioradialis DTR with wrist extension strength d) biceps DTR with elbow extension strength

Correct answer: C The therapist would like to screen cervical level 5-6 (C 5-6) deep tendon reflex (DTR) and key muscle strength at the same level.Screening should includebrachioradialis DTRtesting at the level of C 5-6. Wrist extension strength testing is also at the level of C 6. This is the combination of testing that meets the criteria of testing a DTR at C 5-6 and a key muscle at C 5-6. Answer A:Biceps DTR testing is at the level of C 5-6. Wrist flexion strength testing is at the level of C 7. This combination of testing does not meet the criteria of testing a DTR at C 5-6 and a key muscle at C 5-6. Answer B:Triceps DTR testing is at the level of C 7-8. Elbow flexion strength testing is at the level of C 5. This combination of testing does not meet the criteria of testing a DTR at C 5-6 and key muscle strength at C 5-6. Answer D:Biceps DTR testing is at the level of C 5-6. Elbow extension strength testing is at the level of C 7. This combination of testing does not meet the criteria of testing a DTR at C 5-6 and a key muscle at C 5-6.

A patient with right shoulder pain demonstrates an abnormal scapular position. The right scapula is resting in downward rotation. The therapist believes muscle strength imbalances are contributing to this issue. Which of the following exercises would be MOST appropriate for this patient? a) seated row with therapy band b) standing face the corner of a wall, arms out in 90 degrees of abduction and external rotation, 90 degrees of elbow flexion, and lean in until a stretch is felt c) prone lying with arms overhead, raise arm up off the plinth d) sidelying external rotation with weight in hand

Correct answer: C The upper and lower trapezius and parts of the serratus are the muscles that upwardly rotate the scapula. If a scapula rests in downward rotation, there is an increased risk of instability as the glenoid is facing downward. In addition there is greater risk of impingement. Prone lying with arms overhead and raise arms up off the plinth strengthens the lower trapezius, which is the only exercise of the four options presented that strengthens an upward rotator of the scapula. Answer A: Seated rows strengthen the rhomboids (which are downward rotators) and middle trapezius (which adducts the scapula). It also strengthens the posterior deltoid, which has no line of pull on the scapula. Answer B: Stretching the pectoralis major does not affect the rotation of the scapula. Answer D:The external rotators act on the humerus and externally rotate and depress the humeral head while also stabilizing the glenohumeral joint. They do not upwardly rotate the scapula. Bottom Line:The upper and lower trapezius and parts of the serratus are the muscles that upwardly rotate the scapula. Prone lying with arms overhead and raise arms up off the plinth strengthens the lower trapezius, which strengthens an upward rotator of the scapula.

A patient presents to the clinic with a report of cervical pain and dizziness. The physical therapist is concerned that the patient could have altered blood supply contributing to the dizziness. WHAT TECHNIQUE should the physical therapist perform to determine if this patient's symptoms are related to altered blood supply? A. Passive extension followed by lateral flexion B. Passive flexion followed by rotation C. Passive extension followed by rotation D. Passive flexion followed by lateral flexion

Correct answer: C The vertebral artery test is designed to narrow the lumen of the vertebral artery by changing the position of the neck and head and, if positive, reproduce symptoms. Though its value in the literature for detecting insufficiency is questionable and there are many variations, all involve extension and rotation. None involve flexion. Answer A:The most common positions involve passive extension and rotation. Answers B & D:Flexion would not compress the vertebro-basilar artery. Bottom Line:One possible cause of dizziness is vertebro-basilar insufficiency. This occurs when there is occlusion of the artery by altered neck movements. This can be tested by utilizing the vertebral artery test, which places the head passively into extension, followed by rotation, and then in some descriptions but not all, lateral flexion. This position is held for 10-30 seconds as the physical therapist monitors the patient for changes or symptoms. The diagnostic value is questionable, and insufficiency should not be ruled out even if the test is negative.

A physical therapist observes a patient walking with a steppage gait, where right hip and knee flexion are greater than normal during swing phase. The patient's medical diagnosis is a herniated nucleus pulposus (HNP). In order to plan the dermatome and myotome testing, at which LEVEL does the physical therapist suspect the disk herniation to be? a) L2-L3 b) L3-L4 c) L4-L5 d) L5-S1

Correct answer: C This is the correct answer as a steppage gait is a compensation for the inability to dorsiflex (to prevent dragging the foot and tripping during swing phase), and L4/L5 is the level that innervates the anterior tibialis. Answer A:This is incorrect as this would affect the hip flexors/quadriceps, which would not cause a steppage gait. Answer B:This is incorrect as it would affect the quadriceps, which would not cause a steppage gait. Answer D:This is incorrect as damage to these nerves would affect the ankle musculature (evertors), hip abductors/extensors, and plantarflexors, but not the dorsiflexors.

During a gait assessment of a patient with degenerative disc disease, the patient presents with right foot drop and a forward-leaning trunk at initial contact, hyperextension of the knee in midstance, and increased hip and knee flexion in swing. During myotome testing, which of the following myotomes should the therapist suspect will be MOST affected? a) L1/2, L2/3 b) L2/3, L3/4 c) L3/4, L4/5 d) L4/5, S1/2

Correct answer: C This question requires you to first identify the muscles involved in this gait deviation. Once you know the muscles, you must identify the correct myotome that innervates each of those muscles. When evaluating gait, the therapist must understand how weakness of each lower extremity (LE) muscle group affects gait. The gait deviations described involve impaired motor function of the tibialis anterior as well as quadriceps.The quadriceps are innervated by L3/4, and the tibialis anterior is innervated by L4/5.When the tibialis anterior is weak, a patient will exhibit foot drop during initial contact and will compensate for the foot hanging down in plantarflexion during swing by flexing the hip and knee more than usual to help clear the ground. Weak quadriceps cause a patient to lean forward at initial contact to prevent knee buckling by keeping the ground reaction force in front of the knee. During midstance with prolonged quadriceps weakness, the knee rests in a position of hyperextension to again keep the ground reaction force anterior to the knee and prevent knee buckling. Answer A:The L2/3 mytome tests for hip flexion strength and is not related to the deviations seen in this patient. The L3/4 myotome will test the quadriceps and show weakness in this patient. The L4/5 level is assessed by testing the strength of the tibialis anterior, which is also weak in this patient. Answer B:The L2/3 mytome tests for hip flexion strength and is not related to the deviations seen in this patient. The L2/3 and the L3/4 myotome will test the quadriceps and show weakness in this patient. The L4/5 level is not listed here, and that is assessed by testing the strength of the tibialis anterior, which is also weak in this patient. Answer D:The L4/5 level tests the strength of the tibialis anterior, which is weak in this patient, and great toe extension. However, the S1/2 levels test ankle plantarflexion and knee flexion and are not related to this patient's deviations.

A ten-year-old boy fell and hurt his ankle after landing on someone else's foot when coming down from a jump in basketball during summer camp. The camp physical therapist examined the patient and found moderate swelling of the lateral ankle with tenderness to palpation superior to the lateral malleolus. The patient has pain with ankle inversion both passively and actively and pain with weight-bearing. Which of the following structures should the physical therapist suspect is injured? A. Anterior talofibular ligament B. Anterior tibiofibular ligament C. Fibula D. Posterior talofibular ligament SHOW EXPLANATION

Correct answer: C This scenario of landing on an uneven surface (like someone else's foot after coming down from a jump) is a typical mechanism of injury for an ankle sprain. However, when given a patient's age, consider if it relates to the case. In this situation, because the child is still growing, the growth plate may fail sooner than a ligament would tear during the motion of excessive ankle inversion. The location of the tenderness is also very important to consider.The location of the tenderness, superior to the lateral malleolus, should lead the physical therapist to suspect a fibular fracture at the growth plate. Answer A:The anterior talofibular ligament is usually the first to sprain during an ankle sprain in a position of ankle plantarflexion and inversion; however, the patient would feel pain at the area of insertion on the anterior portion of the lateral malleolus, and this patient was tender superior to that location. Answer B:The anterior tibiofibular ligament connects the distal tibiofibular joint and if sprained is referred to as a "high ankle sprain." However, in this case the age of the patient and location of the tenderness suggest a fracture of the fibula at the growth plate. Answer D:The posterior talofibular ligament can tear in a more severe ankle sprain in a position of ankle plantarflexion and inversion; however, the patient would feel pain at the area of insertion on the posterior border of the lateral malleolus, and this patient was tender superior to that location. Bottom Line:In a child who is still growing, the growth plate may fail sooner than a ligament would tear during the motion of excessive ankle inversion. The location of the tenderness is also very important to consider when determining the location of the injury.

A 64-year-old patient sustained a left femur fracture and left wrist fracture falling in the shower. The patient underwent ORIF to repair the fractured hip and a closed reduction of the wrist fracture. Per MD orders, patient is partial weight-bearing (WB) through left lower extremity (50%) and non-weight-bearing through left wrist (but cleared to WB through elbow). Which of the following assistive devices is MOST appropriate for this patient to use in the home? A. Rolling walker B. Hemi walker C. Platform walker D. Standard wheelchair

Correct answer: C Three major categories of ambulatory assistive devices are canes, crutches, and walkers. A primary function of assistive devices is to eliminate weight-bearing fully or partially from a lower extremity. The unloading occurs by transmission of force from the upper extremity to the floor by downward pressure on the device. Prescribing the appropriate device is individual to each patient and requires knowledge of the patient's WB status.A platform walker is most appropriate for this patient for short distances. Platform walkers function to transfer body weight through the forearm to the assistive device. They are used when WB is contraindicated through the wrist or hand. Answer A:A rolling walker is appropriate to use for a patient that is NWB or partial WB through a lower extremity. However, due to the WB restriction on the wrist, the patient would be unable to use a rolling walker without modification. Answer B:Hemi walkers, like canes, have evidence to support their effectiveness to improve balance and stability. However, they are not designed for use with NWB or partial WB restrictions. Answer D:A wheelchair would be indicated for this patient to use when going longer distances, such as in the community. Going any distance with 2 limbs being partial WB or NWB would be difficult, and the energy expenditure would be too high to be effective. Bottom Line:Three major categories of ambulatory assistive devices are canes, crutches, and walkers. A primary function of assistive devices is to eliminate weight-bearing fully or partially from a lower extremity. The unloading occurs by transmission of force from the upper extremity to the floor by downward pressure on the device. Platform walkers function to transfer body weight through the forearm to the assistive device. They are used when weight-bearing is contraindicated through the wrist or hand.

An infant is being evaluated for the presence of primitive reflexes. The therapist determines the asymmetric tonic neck reflex (ATNR) is present. The therapist would be correct in describing the STIMULUS for the ATNR as: A. Flexion of the head in prone B. Extension of the head in supine C. Rotation of the head in supine D. Quickly moving the child from an upright sitting position backward toward the support surface

Correct answer: C To elicit the ATNR, the infant is placed in supine, and the stimulus is head rotation. A positive response would be extension of the face-side extremities and flexion of the skull-side extremities. Answer A:Neck flexion is the stimulus for the symmetrical tonic neck reflex. A positive response leads to upper extremity (UE) flexion and lower extremity (LE) extension. Answer B:Neck extension is the stimulus for the symmetrical tonic neck reflex. A positive response leads to UE extension and LE flexion. Answer D:Quickly moving the child from an upright sitting position backward toward the support surface would elicit the Moro response. Bottom Line:To elicit the ATNR, the infant is placed in supine, and the stimulus is head rotation. A positive response would be extension of the face-side extremities and flexion of the skull-side extremities.

A physical therapist is treating a patient status post total knee arthroplasty. The patient demonstrates weakness of the quadriceps and extensor lag. The physical therapist decides to use neuromuscular electrical nerve stimulation (NMES) to assist in strengthening the knee extensors. Which of the following parameters is APPROPRIATE for the desired goal? a) continuous motor level stimulation for 15 minutes b) 40 seconds on, 10 seconds off for 10 minutes c) 10 seconds on, 50 seconds off for 10 minutes d) 30 seconds on, 30 seconds off for 15 minutes

Correct answer: C Using electrical stimulation for strengthening requires a longer off time so that the muscle will not quickly fatigue, so the NMES is set for 10 seconds on and 50 seconds off for 10 minutes. Answer A:Continuous stimulation is not indicated for strengthening muscles. Answer B:A significantly longer on time will quickly fatigue the muscle. Answer D:An equal on-off time will quickly fatigue the muscle. Bottom Line:The appropriate parameters for using electrical stimulation for strengthening requires a longer off time so that the muscle will not quickly fatigue.

A physical therapist is scheduled to work with a patient with Parkinson disease in the outpatient clinic at a regularly scheduled appointment of 10 a.m. The patient arrives at 11 a.m. having been stuck in traffic. The patient agrees to wait and is finally seen by the therapist at noon. The patient indicates they are not tired, they took their medication before breakfast as usual, and they will get some lunch immediately after the session. Throughout the session, the therapist notices a significant decrease in functional ability compared to the prior sessions. Which of the following conditions is MOST likely causing this decline? A. On-off phenomenon B. Festination C. Wearing-off phenomenon D. Freezing

Correct answer: C Wearing-off phenomenon is the decline of symptoms toward the end of the medication time effectiveness. In this case, the patient is working 2 hours later than usual and is, therefore, closer to the end of the medication effectiveness time compared to their usual treatments. Answer A:On-off phenomenon consists of abrupt random fluctuations in motor performance and response. Performance can fluctuate throughout the medication cycle and is not specific to the end of effectiveness time frame. Answer B:Festination is a classic parkinsonian symptom that is noted as progressive increase in speed with shortened stride length and can occur throughout the medication cycle. Answer D:Freezing is the inability to initiate movement, often as a result of a real or perceived barrier such as a doorway. It is not related to medication cycles. Bottom Line:All the issues listed are classic symptoms for patients with Parkinson disease. The on-off phenomenon is rapid changes in motor ability and can occur throughout the medication cycle. The wearing-off phenomenon occurs when medication becomes less effective toward the end of a medication cycle and is the concept described in the questions.

A patient status post tibial fracture is out of the cast and ready to begin physical therapy. The PROM of the affected extremity is 0-110 degrees. When asked to perform a straight leg raise (SLR), as the patient lifts the leg, the knee immediately flexes about 10 degrees and remains at that angle during the lift. Which of the following is the reason that the knee is flexed? a) weak iliopsoas b) short/tight hamstrings c) weak quadriceps d) knee flexion contracture

Correct answer: C When a patient performs an SLR, the hip flexes and the knee should remain in extension. In order to remain in extension, the quadriceps must contract to prevent gravity from trying to flex the knee. The quadriceps are especially challenged to generate the amount of force at end range to keep the knee in extension because they are in a shortened/bunched-up position. When there is weakness, such as atrophy after being in a cast, the quadriceps may be unable to maintain extension, and a lag is seen whereby the quadriceps cannot attain full extension against the force of gravity and the knee flexes a little at end range. Answer A:This answer is incorrect as the individual was able to flex the hip and perform the SLR. In addition, the iliopsoas does not cross the knee. Answer B:Tight hamstrings do make the knee harder to fully extend but only when they are lengthened. This patient's knee flexed right at the start of the SLR, which is not a position that puts the hamstrings on stretch. In addition, the patient has full extension PROM. Answer D:The patient's PROM was 0-110, indicating no knee flexion contracture. Bottom Line:The quadriceps extend the knee. As the knee approaches full extension, the quadriceps must use even more force as they shorten to create enough tension to get the knee fully straight. During a straight leg raise exercise, gravity is trying to flex the knee. If the quadriceps are weak, the knee may flex the first few degrees until the quadriceps are at a length where they can hold the tibia. The knee flexion as the leg is lifted into an SLR is referred to as a quadriceps lag, or extensor lag.

Which of the following observations on a 12-lead ECG would indicate that ISCHEMIA is present and the patient should be referred out for consultation? a) pathological Q waves b) ST segment elevated, and T wave inverted c) ST segment depressed, and T wave inverted d) a single absent P wave

Correct answer: C When ischemia is present, the ST segment will be depressed, and the T wave may be inverted. Active ischemia should be referred to the physician for further evaluation. Answer A:Pathological Q waves will occur hours to days following an acute myocardial infarction and are not present when ischemia is present. Answer B:ST elevations will occur after a large acute myocardial infarction (MI) with subsequent injury to the myocardial tissue. When ischemia is actively present, the ST segments will be depressed. Answer D:An absent P wave is the sign of a premature ventricular contraction (PVC). A single PVC will not compromise cardiac output, but if they increase with activity, then the activity should be stopped and the patient examined for compromised cardiac output. One absent P wave is not an immediate referral issue. Bottom Line:Knowing the implications of ECG changes enables the therapist to make appropriate decisions. The presence of active ischemia can be the precursor to an MI, and the client should be referred back to their physician. Present ischemia will depress the ST segment one to two small boxes beyond the J point, and the T wave may be inverted. TrueLearn Insight: When ischemia is present, the ST segment will be depressed, and the T wave may be inverted. These changes will return to normal when the ischemia is resolved. Irreversible changes will occur 20 minutes to 2 hours from the onset of myocardial ischemia. Ischemia in a client post-MI indicates the possibility of vulnerable myocardium that could enlarge the MI. Active ischemia should be referred to the physician for further evaluation.

A patient whispers "1, 2, 3" while a physical therapist is auscultating different portions of the lungs. The words sound like a whisper on the patient's entire right side but sound louder and more discernible on the left side in the lower lobe. What is the MOST probable explanation for these findings? a) the patient has a left apical neoplasm b) the patient has a left lower lobe pneumothorax c) the patient has a pneumonia centered in the lingula d) the patient has a pleural rub

Correct answer: C When there is consolidation of lung tissue, such as with a tumor or a build-up of mucus, a whisper can sound louder than normal when auscultating that part of the lung. The term for this is whisper pectoriloquy. Space-occupying lesions reduce the amount of air, and so the sound is louder and more easily discernible than in normal lung tissue.The lingula is in the location where the abnormal sound was heard, and pneumonia consolidates lung tissue, creating the abnormal sound on auscultation. Answer A:A neoplasm would explain a whisper pectoriloquy, but the apical area is at the top of the lung and the abnormal sound was at the lower portion. Answer B:A pneumothorax would decrease the sound, not increase it, because there is more air. Answer D:The abnormal sound described here was louder and clearer than normal speech. A pleural rub is a creaking or grating sound caused by pleurisy heard on each inspiration and exhalation. Bottom Line:When there is consolidation of lung tissue, such as with a tumor or a build-up of mucus, a whisper can sound louder than normal when auscultating that part of the lung. The term for this is whisper pectoriloquy. Space-occupying lesions reduce the amount of air, and so the sound is louder and more easily discernible than in normal lung tissue.

After ambulating a patient with a patella-tendon-bearing (PTB) socket, the physical therapist inspects the limb and notices redness over the medial tibial flare and the patella tendon. Which of the flowing explains these observations? a) socket too large b) socket too small c) weight distributed properly d) patient ambulated for too long

Correct answer: C With a patella-tendon-bearing (PTB) socket, the areas that can tolerate weight-bearing and are designed to support the body weight include the patella tendon and the medial tibial flare, thus it would be normal to see some redness in those areas. If the redness does not diminish within 10 minutes, it may be problematic, and the patient may have been weight-bearing too long, but that is normally not the case. Answers A & B:The socket fits properly if the redness is on the areas that are designed to tolerate weight-bearing. Answer D:The redness is in the appropriate areas and is expected after weight-bearing and ambulating. If the redness persists for many hours, then one can assume the patient ambulated for too long of time, but that was not specified here. Bottom Line:With a patella-tendon-bearing (PTB) socket, the areas that can tolerate weight-bearing and have been designed to support the body weight include the patella tendon and the tibial flares. It is normal to see some redness in those areas.

A patient has a burn inhalation injury affecting the lung parenchyma. Which of the following should the physical therapist expect to see on the pulmonary function tests? a) an increase in FEV1 post albuterol administration b) a diffusing capacity for carbon monoxide that is higher than normal c) no change in the ratio of forced expiratory volume/forced capacity volume d) a forced vital capacity that is larger than normal

Correct answer: C With lung scarring both FEV1and FVC decrease, so the ratio remains the same. Answer A:FEV1is the amount of air one can forcefully expel in 1 second. Albuterol is a bronchodilator, and in an asthmatic patient where the airways narrow, FEV1 would increase post albuterol. This patient has scarring of the lungs, and so albuterol should not have much of an effect on FEV1. Answer B:In a normal lung, carbon monoxide binds strongly and can cross the membrane and remain in the body. A burn inhalation results in lung scarring, and DLCOis decreased because carbon monoxide cannot bind well. Answer D:Scarring is a type of restrictive lung disease. Patients with chronic restrictive lung disease have a decreased FVC as compared to normal. Bottom Line:In a chronic restrictive pulmonary disease, forced vital capacity (FVC) or the amount of air one can expel forcefully, is less than normal. In addition, the amount of air one can forcefully expel in 1 second (FEV1) is less than normal. The ratio of FEV1 to FVC, however, can remain the same since both decrease.

A physical therapist is completing the initial examination for a 65-year-old patient in acute rehabilitation with an ASIA A T6 spinal cord injury due to a fall that occurred 1 week ago. The patient has a history of type 2 diabetes, obesity, and a stage II pressure sore on her sacrum. What type of pressure-relieving cushion is MOST likely to be used initially to prevent pressure ulcers? a) foam b) gel c) air d) thermoplastic

Correct answer: C Air inflation cushions are typically used with patients with spinal cord injury who have skin breakdown or are at high risk for skin breakdown. Air inflation cushions demonstrate improved distribution of pressure compared to other cushion designs. One disadvantage of air inflation cushions is that they require monitoring to maintain proper air pressure in the cushion. Answer A:Foam cushions are made of various types of material that are lightweight and require little maintenance. However, foam cushions do not provide adequate relief of pressure in patients with skin breakdown. Answer B:Gel cushions are made of various types of material that are lightweight and require little maintenance. Gel is often incorporated into foam cushion designs to increase the distribution of pressure. However, gel cushions provide less relief of pressure and may increase skin temperature compared to air cushions, so they are less likely to be used in patients with skin breakdown. Answer D:Thermoplastic cushions are made of various types of material that are lightweight and require little maintenance. However, thermoplastic cushions do not provide adequate relief of pressure in patients with skin breakdown. Bottom Line:Air inflation cushions are typically used with patients with spinal cord injury who have skin breakdown or are at high risk for skin breakdown. A physical therapist must choose the type of cushion upon initial examination of the patient with a spinal cord injury, based on the level and completeness of injury, patient age, skin status, and comorbid factors. Pressure ulcers are likely to develop if an incorrect type of pressure-relieving cushion is recommended by the physical therapist.

A twelve-year-old gymnast reports pain on the posterior surface of the right calcaneous. Pain increases with weight-bearing activities. If she rests for a day the pain goes away but returns as soon as she goes back to gymnastics. Manual muscle test scores and passive range of motion are all within normal limits. Gait exam reveals an antalgic gait with pain at heel strike. Which of the following is the most likely diagnosis? a) plantar fasciitis b) achilles tendinopathy c) calcaneal apophysitis d) heel spur

Correct answer: C Calcaneal apophysitis is the most common cause of heel pain in children. It is an inflammation of the growth plate at the calcaneus in growing children caused by repetitive stress to the heel.Children who are very active, especially with a lot of forceful use of the achilles tendon, or who land from heights (like a gymnast) or any jumping athlete are more susceptible to this condition. This condition is also called Sever's disease, though it is not actually a disease. Answer A:Plantar fasciitis pain is felt on the inferior surface of the calcaneus where the plantar fascia inserts, not on the posterior surface. This patient also has pain at heel strike. With plantar fasciitis the pain would not occur at heel strike. Answer B:The patient's age and activity level and the location of the pain point toward calcaneal apophysitis. Answer D:A heel spur would cause pain on the inferior surface of the calcaneus, not on the posterior portion. In addition, the patient's age and activity level and the location of the pain point toward calcaneal apophysitis.

In order to be classified as an ASIA Impairment Scale B injury, which of the following functions needs to be spared? a) bladder control b) light touch more than three levels below the neurological level of injury c) light touch, pin prick at S4-S5, or deep anal pressure d) movement more than three levels below the neurological level of injury

Correct answer: C The ASIA (American Spinal Injury Association) Impairment Scale (AIS) is used to classify the completeness of injury in individuals with spinal cord injury. To be classified as having an AIS B injury, a person would have sensory function below the neurological level of injury, which would includelight touch, pin prick at S4-S5, or deep anal pressure. In addition, no motor function can be preserved more than three levels below the motor level on either side of the body in order to be classified as an AIS B.The determination of an accurate AIS classification is critical in order to predict the likelihood of neurological recovery for a person with a spinal cord injury. Physical therapists must be able to use the AIS classification to their plan their examination and intervention strategy accordingly for their patients with spinal cord injury. In addition, physical therapists must be able to explain an AIS classification to patients and their families so that they have a better understanding of the patient's prognosis for recovery. Answer A:While a person with an AIS B classification is more likely to have bladder control than a person with an AIS A classification, bladder control is not part of the classification criterion for the AIS. Answer B:While a person with an AIS B classification is more likely to have light touch more than three levels below the neurological level of injury than a person with an AIS A classification, light touch is not part of the classification criterion for the AIS except for the assessment of the S4-S5 level. Answer D:The requirement of movement more than three levels below the neurological level of injury is used to establish the classification of a spinal cord injury as either an AIS C or AIS D.

A patient with localized gangrene on the heel of the foot would be characterized as which stage of the Wagner Classification scale? a) 2/5 b) 3/5 c) 4/5 d) 5/5

Correct answer: C The Wagner classification scale categorizes dysvascular ulcers (usually diabetic ulcers) based on wound depth and presence of infection from 0 to 5.Grade 4 is localized, partial foot gangrene.

A physical therapist is working with a patient who recently experienced a traumatic brain injury. The therapist has been advised that the patient is at a Rancho Los Amigos level 5. Which of the following behaviors is the therapist MOST likely to observe? a) limited and nonpurposeful reactions to stimuli b) showing carryover for self-care tasks, incorrect but appropriate responses c) responding to simple commands, inappropriate verbalization d) bizarre behavior with incoherant or inappropriate verbalization

Correct answer: C The Rancho Los Amigos levels of cognitive functioning scale scores a patient on a scale of 1 to 8, where 1 is no response, and 8 is purposeful and appropriate response. A level 5 patient is considered confused-inappropriate and would respond fairly consistently to simple commands; has gross attention to the environment but is highly distractible; and has verbalization that often is inappropriate and confabulatory. Answer A:Level 2 on the scale is generalized response and is manifested in the patient as reacting inconsistently and nonpurposefully to stimuli. Patient responses are limited and often the same regardless of the stimuli. Answer B:Level 6 on the scale is confused-appropriate, and the patient shows some goal-directed behavior with external input or direction. The responses may be appropriate but incorrect due to memory issues. Answer D:Level 4 on the scale is confused-agitated, and the patient will demonstrate bizarre and nonpurposeful behavior with incoherent or inappropriate verbalization.

A patient is being treated by a physical therapist for neck and arm pain after a car crash. The patient has improvement in cervical range of motion and reduction in cervical pain. However, symptoms of pain in the elbow radiating into the 4th and 5th digits of the left hand persist. The therapist suspects dysfunction of the peripheral nervous tissue. Which of the following neurodynamic test positions examines the peripheral neurologic structures contributing to the patient's symptoms? A. Shoulder girdle depression, shoulder abduction, shoulder external rotation, elbow extension, forearm supination, wrist extension B. Shoulder girdle depression, shoulder abduction, shoulder internal rotation, elbow extension, forearm pronation, wrist flexion C. Shoulder girdle depression, shoulder abduction, shoulder external rotation, elbow extension, forearm supination, wrist flexion D. Shoulder girdle depression, shoulder abduction, shoulder external rotation, elbow flexion, forearm pronation, wrist extension

Correct answer: D

A patient reports pain in the thumb after a fall while skiing 3 weeks ago when her thumb got caught in her ski pole. The patient did not seek medical attention at the time and iced the thumb daily. There is no longer pain at rest, but there is pain when using the thumb, especially when gripping large objects. Which of the following examination procedures should the physical therapist perform to identify the MOST likely source of the pain? a) palpation for tenderness of the scaphoid in the snuff box b) valgus stress of the ulnar collateral ligament of the first CMC joint c) palpation for tenderness of the trapezium d) valgus stress of the ulnar collateral ligament of the first MCP joint

Correct answer: D "Skier's" or "Gamekeepers" thumb is classified by an acute injury to the ulnar collateral ligament (UCL rupture) of the first MCP joint. It occurs most commonly if the thumb is pulled into abduction with a valgus force, causing stress and tear to the UCL of the thumb MCP joint. By applying a valgus stress to the joint, the physical therapist can look for motion greater than 30 degrees into valgus, indicating a ligament tear. If the joint is injured, motions that stress the joint, such as trying to wrap the thumb around a large object when gripping, can cause pain and instability at the joint. Answer A:The scaphoid can be fractured with a fall on an outstretched hand and is on the same side of the hand as the thumb, so for those reasons it may be a consideration. However, in this case, the patient did not fall on the hand, rather the thumb was pulled too far by the ski pole handle. In addition, the patient does not have pain at rest and has had a reduction in pain over time. With a fracture, the pain would be present at rest and would persist even after 3 weeks. Answer B:A valgus stress test of the ulnar collateral ligament (UCL) must be done at the metacarpophalangeal (MCP) joint, not the carpal metacarpal (CMC) joint. The MCP joint is the one most likely to be injured with the mechanism of injury described in this question. A valgus force to the thumb would cause the MCP UCL to fail long before the range of motion required for the CMC joint to overstretch. Answer C:The trapezium is the carpal bone just proximal to the first metacarpal, with those two bones forming the first CMC joint. It can be fractured with a fall on an outstretched hand, and it may be a consideration. However, in this case, the patient did not fall on the hand, rather the thumb was pulled too far by the ski pole handle. In addition, the patient does not have pain at rest and has had a reduction in pain over time. With a fracture, the pain would be present at rest and would persist even after 3 weeks. Bottom Line:Skier's thumb, also known as gamekeeper's thumb, is an injury to the ulnar collateral ligament of the first MCP joint and results in excessive valgus motion of the thumb at the MCP joint. TrueLearn Insight :When trying to decide on what structures are injured, pay close attention to the mechanism of injury to give some sense of what structures were stressed. In addition, listening to the patient's symptoms such as whether there is pain only at rest, pain only with motion, pain all the time etc., can help further rule in or rule out the cause of the pain.

A 42-year-old female with diabetes underwent a transtibial amputation on her right leg. During her stay in the hospital, she was referred for physical therapy. An essential part of client education for amputee patients is proper positioning. What is the BEST position for the patient to maintain to prevent limb contractures? a) semi-fowler position b) side-lying on the right side with right hip and knee extended c) hook lying position d) side-lying on the left side with right hip and knee extended

Correct answer: D A common contracture of transtibial amputation is hip flexion contracture. Recommended positions for transtibial amputees are: (a) supine with no pillow under the involved leg, (b) side-lying on the stronger leg with the involved hip and knee extended, and (c) prone. Prolonged sitting is contraindicated. Answer A:In Semi-Fowler position, the head of the bed is elevated at ~30-45 degrees. This may not be the optimal position as this promotes hip flexion as well. Answer B:Side-lying should be on the left side. Side-lying on the involved side should be avoided. Answer C:Hook lying position is not recommended because it involves flexion of the hips and knees. Bottom Line:A common contracture of transtibial amputation is hip flexion contracture. Recommended positions for transtibial amputees are: (a) supine with no pillow under the involved leg, (b) side-lying on the stronger leg with the involved hip and knee extended, and (c) prone. Prolonged sitting is contraindicated.

A patient has deficits in thumb abduction due to capsular adhesions. Which mobilization is the MOST specific carpometacarpal joint technique to improve the patient's range of motion? a) palmar glide of the first metacarpal on a stable trapezoid b) radial glide of the first metacarpal on a stable trapezium c) ulnar glide of the first metacarpal on a stable trapezoid d) dorsal glide of the first metacarpal on a stable trapezium

Correct answer: D A dorsal glide of the first metacarpal on a stable trapezium is indicated to improve thumb abduction. Answer A:A palmer glide of the first metacarpal links with restricted thumb adduction. Moreover, the stabilization listed is incorrect as the trapezium is the proximal bone. Answer B:While the stabilization description is correct, the direction of the glide is incorrect. A radial glide of the first metacarpal links with restricted thumb extension. Answer C:An ulnar glide of the first metacarpal links with restriction thumb flexion, and the stabilization listed is incorrect as the trapezium is the proximal bone.

A 25-year-old office worker was referred to physical therapy for management of neck pain. On postural assessment, the physical therapist noticed a forward head posture. Based on this finding, the physical therapist may expect greater limitation of motion in WHICH CERVICAL MOVEMENT? a) rotation b) lateral flexion c) forward flexion d) extension

Correct answer: D A forward head involves increased flexion of the lower cervical and the upper thoracic regions, increased extension of the upper cervical vertebra, and extension of the occiput on C1. There also may be temporomandibular joint dysfunction with retrusion and depression of the mandible.This abnormal posture could be due to occupational or functional postures requiring leaning forward or tipping the head backward for extended periods and faulty sitting postures, such as working at an improperly placed computer keyboard or screen. Answer A:Limitations in cervical rotation may not necessarily be due to a forward head posture. Answer B:Limitations in cervical lateral flexion may not necessarily be due to a forward head posture. Answer C:Cervical flexion will likely not be affected, as the forward head posture involves increased flexion of the lower cervical region. Bottom Line:A forward head involves increased flexion of the lower cervical and the upper thoracic regions, increased extension of the upper cervical vertebra, and extension of the occiput on C1.

A physical therapist has been working on gait training with several older adults with visual impairments. The therapist has found that tactile cues generally have produced better results than verbal cues and has decided to conduct some research to explore this idea. WHICH of the following statements would be considered appropriate as their hypothesis? A. There is no significant difference between the effects of tactile or verbal cues when working with older adults with visual impairments B. How do older adults with visual impairments respond to verbal versus tactile cues C. What is the difference between the use of verbal and tactile cues when working with older adults with visual impairments D. Tactile cues will be more effective than verbal cues when working with older adults with visual impairments

Correct answer: D A hypothesis is a particular form of question used in quantitative research. It includes all the variables and the prediction that the researcher believes will be demonstrated, based on their literature review or prior research.The null hypothesis is the approach that there will be no difference in the influence of the variables in the population being studied.Research questions are more general and exploratory and usually start with terms such as what or how. In this case, the therapist develops a directional hypothesis, that tactile cues will prove more effective.The hypothesis for a study states the prediction regarding the anticipated outcomes. In this example, the therapist thinks that tactile cues will prove more effective and therefore introduces the idea into the hypothesis. In this case, this would be considered a directional hypothesis, as the researcher is suggesting a potential outcome. Answer A:This statement, "there is no significant difference between" would be considered the null hypothesis for the study. The null represents the approach that in the population under consideration, there will be no difference between the variables, in this case, tactile and verbal cues. Answers B & C:Statements that begin with what or how are looking at the relationship between the variables and are considered research questions, but not the hypothesis. A hypothesis must include predictions about the expected outcomes of the research. Bottom Line:A hypothesis is a particular form of question used in quantitative research. It includes all the variables and the prediction that the researcher believes will be demonstrated, based on their literature review or prior research. The null hypothesis is the approach that there will be no difference in the influence of the variables in the population being studied. Research questions are more general and exploratory and usually start with terms such as what or how.

A physical therapist is performing an initial examination with a patient who had an ischemic right middle cerebral artery stroke 1 week ago. The patient currently exhibits no active motion or sensation in the left upper and lower extremities and is dependent in all functional activities. When thinking about prognosis and how much recovery this patient might experience, knowing the patient will most likely still have improvement in his motor recovery after today is indicated by the fact that: A. it was an ischemic stroke. B. it was on the right side. C. this patient's stroke is quite severe. D. the stroke was 1 week ago.

Correct answer: D A patient who is in the acute stage of recovery is more likely to show motor recovery than patients who are already in a more chronic stage. Patients do, however, exhibit large amounts of individual variability in amount and rate of recovery. Answer A:The type of stroke (ischemic vs hemorrhagic) is not a predictor of motor recovery from stroke. Answer B:The side of lesion is not a predictor of motor recovery from stroke. Answer C:According to the available literature, increased stroke severity as measured by decreased functional ability, no active motion in the more affected side, and no sensation on the more affected side is a predictor of a poor motor recovery from stroke. Bottom Line:Prediction of motor recovery from stroke depends on a number of factors. A patient who is in the acute stage of recovery is more likely to show motor recovery than patients who are more chronic. In addition, patients with very severe strokes are less likely to have as much recovery as someone with a less severe stroke.

A physical therapist is completing the initial examination for a 25-year-old patient in acute rehabilitation with a traumatic brain injury due to a fall that occurred 1 week ago. The patient is currently dependent in all functional activities and is classified as Rancho Cognitive Levels of Functioning Level III. What type of wheelchair and pressure relief strategy will the physical therapist MOST likely use initially to prevent pressure ulcers? a) power wheelchair with weight shifting to each side once per hour b) standard manual wheelchair with weight shifting to each side once per hour c) reclining wheelchair with the seat back reclined for 5 minutes once per hour d) tilt-in-space wheelchair with the chair tilted back for 5 minutes once per hour

Correct answer: D A tilt-in-space wheelchair is indicated in this scenario since this type of wheelchair can take pressure off the ischial tuberosities by tilting the seat and back of the wheelchair together with the seat angle, putting the pressure onto the lower back temporarily. Although the patient has limited cognitive functioning, the patient can easily attain this position through simple actions such as pressing a button or moving a joystick on the chair. Answer A:A power wheelchair is not indicated in this scenario since the patient has limited cognitive functioning since they are classified as a Rancho Level 3. They also could not perform the leaning weight shifts well. Answer B:A standard manual wheelchair is not indicated in this scenario since the patient has no easy method to perform a weight shift in this wheelchair due to their limited physical and cognitive functioning. Answer C:A reclining wheelchair is not indicated in this scenario since the patient has no easy method to perform a weight shift in this wheelchair due to their limited physical and cognitive functioning. Reclining the wheelchair allows for a partial method of weightshift but also increases shear forces, which should be avoided in patients at risk for skin breakdown. Reclining changes the hip angle but does not fully take pressure off the ischial tuberosities and buttocks. Bottom Line:A tilt-in-space wheelchair is recommended for patients with extremely limited mobility at high risk for skin breakdown. Tilt-in-space maintains the hip angle but shifts the pressure from the buttocks and ischial tuberosities to the low back.

Which of the following tests is the MOST appropriate for diagnosing a pulmonary embolism (PE)? a) chest radiograph b) doppler ultrasound of a lower extremity vein c) pulmonary function tests (PFTs) d) V/Q scan

Correct answer: D A ventilation-perfusion scan, more commonly known as a V/Q scan, compares the amount of ventilation in the lungs to the amount of perfusion in the pulmonary vasculature. If there is normal ventilation in the lungs with abnormal perfusion, then a pulmonary embolism (PE) might be present. Although a V/Q scan is not the gold standard for diagnosing a PE, it is the only answer out of the choices that measures perfusion of the lungs, which is impaired when there is a PE. Therefore, it is the BEST answer choice. Answer A:A chest radiograph, or chest x-ray, cannot provide a diagnosis of a pulmonary embolism (PE). The image obtained from a chest x-ray provides information on the bony anatomy and structure of the chest but does not provide enough details to show impairments in the circulation or vasculature of the lungs. Answer B:A Doppler ultrasound of a lower extremity vein can reveal the presence of a deep vein thrombosis (DVT), which can break off and eventually lead to a pulmonary embolism (PE). However, the presence of a DVT alone does not confirm a PE. Therefore, this answer is incorrect. Answer C:Pulmonary function tests (PFTs) evaluate lung volume and capacity. They provide useful information about an individual's overall pulmonary function. However, it is not useful in diagnosing a pulmonary embolism (PE) because it does not measure the perfusion of the lungs. Therefore, it is not the correct answer. Bottom Line:In the case of a pulmonary embolism (PE), the acute medical issue is an impairment in the circulation and perfusion of the pulmonary vasculature. The type of diagnostic test used must evaluate perfusion, which can be done via a V/Q scan. However, it is important to remember that the gold standard diagnostic test for a PE is computed tomography pulmonary angiography.

An incentive spirometer may be used for all of the following reasons EXCEPT? a) to practice diaphragmatic breathing b) to prevent or reverse atelectasis c) to stimulate a cough d) to clear secretions

Correct answer: D An incentive spirometer is a medical device used exercise the lungs to increase lung capacity. It is an effective way to practice diaphragmatic breathing and to prevent or reverse atelectasis. It is used for a variety of lung conditions, and with post surgical patients who are immobile. It is not used to clear secretions directly but can help to stimulate a cough. Answer A:Patients are instructed to perform deep breathing with the incentive spirometer every hour. It helps to have the patient place a hand on the abdominal area to feel the diaphragm working correctly. Answer B:Atelectasis, which is caused by hypoventilation and collapse of the alveoli in the lungs, can be prevented and reversed with deep breathing. Answer C:The incentive spirometer helps to stimulate a cough through the deep breathing exercises. Bottom Line:An incentive spirometer is a medical device used exercise the lungs to increase lung capacity. It is an effective way to practice diaphragmatic breathing and to prevent or reverse atelectasis. It is used for a variety of lung conditions, and with post surgical patients who are immobile. It is not used to clear secretions directly but can help to stimulate a cough.

A physical therapist is examining a child with ataxia due to a rare genetic disorder. When the child performs a finger-to-nose test, the child's hand and arm shake, and the shaking gets worse the closer her hand gets to the target. What is the name of this clinical condition? a) apraxia b) chorea c) dysdiadochokinesia d) intention tremor

Correct answer: D An intention tremor occurs during voluntary movement and the tremor gets worse as it approaches its target. An intention tremor is typically found in patients with damage to the cerebellum. Answer A:Apraxia is the inability to perform motor activities upon command while the sensory and motor systems are intact. Apraxia is more commonly found in patients with damage to the left cerebral cortex rather than the right cerebral cortex. Answer B:Chorea is characterized by jerky involuntary movements, usually of the face, shoulders, and hips. Answer C:Dysdiadochokinesia is the inability to perform rapid, alternating movements. This disorder often causes difficulty with activities of daily living and with walking. Dysdiadochokinesia is typically found in patients with damage to the cerebellum.

A patient has 2/5 strength in the right hamstrings after a partial substance tear of the semi tendinosis. Which of the following is the MOST appropriate strengthening exercise? a) prone knee flexion b) standing knee flexion c) supine heel slides d) sidelying knee flexion

Correct answer: D At this point the patient is unable to flex the knee against gravity but has the ability to flex the knee in a gravity-eliminated position. A sidelying position will allow the patient to contract the muscle and flex the knee to strengthen it and is the appropriate level of difficulty at this phase of the rehabilitation. Answers A & B:This position requires the ability to flex the knee against gravity and at this time is too difficult for the patient with 2/5 strength. Answer C:Supine heel slides require dragging in the heel and raising the knee onto a position of flexion while fighting the forces of gravity that are trying to keep the knee in an extended position on the plinth. Bottom Line:When muscle strength is below a 3, as a general rule the patient should exercise that limb in a gravity-eliminated position where the gravity vector is not adding resistance to the motion. For motions in the sagittal plane, like hip flexion or elbow flexion, that position is usually sidelying. For motions in the coronal plane like shoulder or hip abduction, that can be done in supine or prone.

During the initial physical therapy evaluation of a patient with a known history of atrial fibrillation (Afib), which of the following would be the MOST appropriate way to assess the pulse rate? a) two fingers lightly over the patient's brachial pulse; count the number of beats in 10 seconds, then multiply by 6 b) two fingers lightly over the patient's radial pulse; count the number of beats in 15 seconds, then multiply by 4 c) thumb lightly over the patient's brachial pulse; count the number of beats in 60 seconds d) two fingers lightly over the patient's radial pulse; count the number of beats in 60 seconds

Correct answer: D Atrial fibrillation is a common arrhythmia that is characterized by fast and irregular heart beats. During palpation of a peripheral pulse, an irregular rhythm may be detected, which can present as very fast beats followed by slow regular beats, or vice versa. Therefore, in atrial fibrillation, it is more appropriate to manually assess the total number of beats during one minute to have a more accurate representation of heart rate. In this case, it would mean placing two fingers lightly over the patient's radial pulse and counting the number of beats in 60 seconds. Although all of the counting intervals described in each answer choice (10 seconds, 15 seconds, or 60 seconds) are acceptable for taking a pulse, only one of them is the most appropriate in this case. In certain situations, such as in tachycardia (heart rate greater than 100 beats/min) or with arrhythmias, such as atrial fibrillation, counting the number of beats in 60 seconds would yield the most accurate heart rate in beats per minute. Answer A: Assessing the brachial pulse for palpation is correct. However, in the setting of Afib, where irregular beats often occur, the counting interval should be a full minute in order to accurately capture heart rate in beats per minute. Counting the number of beats in 10 seconds would not be adequate; it would likely yield a result that is either too high or too low. Answer B:In the setting of Afib, where irregular beats often occur, the counting interval should be a full minute in order to accurately capture heart rate in beats per minute. Counting the number of beats in 15 seconds would not be adequate; it would likely yield a result that is either too high or too low. Answer C:The thumb should not be used to palpate a patient's pulse due to the presence of a strong pulse in most people's thumb (from the princeps pollicis artery). It would be easy to mistake the examiner's pulse for the patient's pulse. Counting the number of beats in 60 seconds is appropriate as it would yield the most accurate heart rate in beats per minute for this patient with an arrhythmia. Bottom Line:In atrial fibrillation, it is appropriate to manually assess the total number of beats during one minute to have a more accurate representation of heart rate.

A 65-year-old female reports having dizziness, headaches, ringing in the ears, nausea, and a feeling of unsteadiness for about ten minutes after getting her hair shampooed at the beauty parlor every Friday. The physical therapist recreates the position and symptoms return. Upon objective assessment, the physical therapist finds a torsional nystagmus, which can be corrected when asking the patient to fixate on an object. What is the MOST likely explanation for the cause of this patient's symptoms?

Correct answer: D Benign paroxysmal positional vertigo (BPPV) can be diagnosed via subjective assessment as well as by the Dix-Hallpike maneuver. BPPV is generally classified by a combined horizontal and torsional nystagmus that is inhibited by fixation of the eyes onto an object. The symptoms usually fade within a few minutes. It is usually brought about by position changes such as looking up or looking down. Patients may have severe vomiting, dizziness, and mild difficulty walking. Answer A:A migraine has many of the same symptoms as BBPV including headache, dizziness, and nausea lasting a few hours. A migraine may also be accompanied by light sensitivity. A patient with a migraine does not display nystagmus or report instability when ambulating. The Dix-Hallpike test would also not provoke the symptoms. Answer B:Meniere's disease involves recurrent episodes of vertigo, fluctuating hearing loss, tinnitus, or aural fullness caused by increased volume of endolymph in the semicircular canals. While the vertigo may last hours, it is not often provoked by any specific movements and cannot be diagnosed on objective assessment. No nystagmus would be observed. Answer C:Acoustic neuroma is an inflammation of the vestibular nerve, usually caused by viral infection. Episodes occur spontaneously and are often accompanied by headache, aural fullness, ear pain, and tinnitus. Hearing loss is often progressive and unilateral as opposed to fluctuating. Bottom Line:Many of the symptoms for BBPV, Meniere's disease, migraines, and acoustic neuromas overlap. However, some are specific to only one of those diagnoses. The onset due to position change, and the presence of nystagmus upon objective testing using the Dix-Hallpike maneuver and correction via gaze stabilization leads to the diagnosis of BBPV.

A physical therapist is working with a patient on stand pivot transfers. The therapist decides to work in a quiet area of the gym and works solely on transfer techniques, with minimal rest periods, and performing the entire transfer. Which type of practice schedule is being utilized in this treatment session? A. Random B. Serial C. Distributed D. Blocked

Correct answer: D Blocked practice is the sequence of practice specifically on one task with minimal interruption and low interference. This description is of someone working on a blocked practice sequence. Answer A:Random practice has no obvious sequence to the practice session and actions are practiced in a random order. Answer B:Serial practice is a predictable repeating method of practicing tasks. For example, three tasks are repeated in sequence throughout the practice session. Answer C:Distributed practice includes equal amounts of rest and practice, so in this case, where there are minimal rest periods, it would not be considered distributed. Bottom Line:Different sequence patterns make up the different types of practice. In this case, blocked practice is where the therapist works on one task with minimal interference and not interrupted by other tasks.

Which of the following static positions would produce the HIGHEST systolic blood pressure reading in a normal, healthy adult. a) semi-reclined b) sitting c) standing d) supine

Correct answer: D Blood pressure for a normal, healthy adult is highest in supine. In this position, there is less resistance to blood flow due to gravity. Answer A:Systolic BP is highest in supine, where there is less resistance to blood flow due to gravity . Answers B & C:Systolic BP is highest in supine where there is less resistance to blood flow due to gravity. Bottom Line:Systolic blood pressure is the pressure exerted against the arteries during the ejection cycle. Diastolic blood pressure is the pressure against the arteries during rest. Several factors can affect blood pressure, including blood volume, cardiac output, blood viscosity, and peripheral resistance. Blood pressure changes with body position may reflect the influences of vasomotor tone, venous return, the hydrostatic effects of gravity, or all three. Systolic blood pressure is usually highest in supine where there is less resistance to blood flow due to gravity. One should always record the patient position when taking blood pressure.

A physical therapist is examining a patient in the cardiac intensive care unit and notices the patient is exhibiting Cheyne-Stokes breathing. How should the therapist document the patient's breathing? a) patient able to breathe best when sitting upright and has great difficulty breathing when supine b) patient exhibits a pattern of slow shallow breaths with poor perfusion c) patient able to breathe best when lying supine, with great difficulty breathing when sitting upright d) patient exhibits a cyclic pattern of fast breathing followed by a period of slow breathing and then a period of apnea/no breathing

Correct answer: D Cheyne-Stokes breathing is a cyclic form of breathing where the patient exhibits an increase in frequency and volume followed by a decrease in frequency and volume, ending with periods of apnea that can last 10-30 seconds between cycles. It can cause abnormalities of blood gases and acid-base balance. It is usually seen during sleep and is common in patients with congestive heart failure. It can also occur with encephalitis, brain tumor, stroke, head trauma, or diabetes. Answer A:Patients who breathe best in an upright position and cannot tolerate lying down have a condition called orthopnea. The intolerance to supine is usually due to pulmonary edema associated with congestive heart failure. Answer B:Slow shallow breaths describe a pattern of hypoventilation where air exchange is not effective. Answer C: Breathing easier in supine than in upright sitting is called platypnea and is not very common. It is due to a cardiovascular defect.

A 37-year-old male had physical therapy for 4 weeks after a healed radius fracture with open reduction and internal fixation (ORIF). He was discharged with a home program. He has returned 1 month later because he says his pain is still at least 9/10. Examination findings are within normal limits (WNL) except for: grip strength 3/5, hyperalgesia to the entire hand, hyperhidrosis, and skin that was warm to palpation with some edema and hair loss on the fingers. What does the physical therapist suspect? a) ORIF hardware possibly compressing median nerve b) fracture not yet healed c) injury to the vasculature at the time of injury not properly addressed d) complex regional pain syndrome

Correct answer: D Chronic regional pain syndrome (CRPS)previously referred to as reflex sympathetic dystrophy (RSD), is a form of chronic pain that usually affects an extremity after injury, surgery, or stroke. It is characterized by the presence of signs indicating obvious autonomic and inflammatory changes in the region of pain. Patients may display severe sensitivity to pain as well as sensitivity to non-painful stimuli, such as touch or cold). There are also changes to skin color, skin temperature, and sweating; edema, altered patterns of hair, skin, or nail growth; and reduced strength. Answer A:The symptoms do not show dermatomal or myotomal patterns of the median nerve distribution, nor would a median nerve injury explain all of the symptoms, including the edema and hair loss. Answer B:The fracture was healed even before the patient began the initial bout of physical therapy. In addition, a fracture would not produce all of the symptoms of this patient. Answer C:If there was involvement of the vasculature, that would not explain all of the symptoms, such as decreased grip strength and hyperalgesia. In addition, the injury was 3 months ago, and if there was an issue with blood supply to the musculature, by now there would have been more severe ischemia. Bottom Line:Complex regional pain syndrome (CRPS), also know as reflex sympathetic dystrophy (RSD), is a form of chronic pain that usually affects an extremity after injury, surgery, or stroke. It is characterized by disproportionate pain relative to the severity of the injury and by autonomic and inflammatory features. Initial symptoms can include pain, swelling, redness, and hypersensitivity. As the disease progresses, symptoms can change, causing the affected limb to be cold and pale. A variety of vasomotor, sudomotor, and trophic changes can often accompany CRPS.

A physical therapist is treating a patient with a history of seizures due to traumatic brain injury that is being managed by medication. Which of the following adverse effects is common with antiseizure medications? a) dry mouth b) hypertension c) immunosuppression d) sedation

Correct answer: D Common classes of antiseizure medications, such as barbiturates, benzodiazepines, and hydantoins, have sedation as one of their most common adverse effects. Answer A:Dry mouth is not a common adverse effect for the most common classes of antiseizure medications. Answer B:Hypertension is not a common adverse effect for the most common classes of antiseizure medications. Answer C:Immunosuppression is not a common adverse effect for the most common classes of antiseizure medications.

During auscultation of the lungs, WHERE should the stethoscope be placed if the physical therapist is trying to evaluate the left lower lobe? a) anterior surface of the left side of the chest lateral to the sternal border at the level of the 2nd or 3rd intercostal space b) anterior surface of the left side of the chest lateral to the xiphoid process c) posterior surface of the left chest wall lateral to the spinous processes at the level of T3 d) posterior surface of the left chest wall midway between the axillary line and the spinous process at the level of T8 or T9

Correct answer: D Due to the orientation of lung tissue, the majority of the left lower lobe of the lung can be auscultated in the posterior chest wall; the left lower lobe and the bases would be located distally and more lateral, typically at the level of T8 or lower, depending on the patient's anatomy. Answer A:This answer is incorrect because it does not describe where the left lower lobe can be heard. Instead, this location corresponds with the anterior segment of the left upper lobe. Answer B:This answer is incorrect because the area described typically does not contain a lung field. Rather, the heart is underneath this area. Therefore, lung sounds cannot be heard here. Answer C:At this level, it is possible to auscultate the upper portions of the lower lobe of the left lung. However, this location is very close to the border with the upper lobe, depending on the patient's anatomy. In order to better target the lower lobe, the therapist should auscultate lower in the posterior chest wall. Therefore, this is not the best answer and is incorrect. Bottom Line:The majority of the left lower lobe of the lung can be auscultated in the posterior chest wall midway between the axillary line and the spinous process at the level of T8 or T9. During auscultation of the lungs, the stethoscope should be placed on areas corresponding to the respective lung fields. Knowledge of surface anatomy and chest wall structures would allow the physical therapist to better recognize appropriate areas for auscultation.

A patient with low back pain is being examined by a physical therapist. The therapist notices that each time the patient extends the right hip during gait the patient's back goes into a lordosis. The therapist suspects tight musculature. Which of the following tests would BEST identify the tight muscle(s)? a) Thomas test and 90/90 test b) FABER test and Thomas test c) SLR and 90/90 test d) Ely's test and Thomas test

Correct answer: D During gait, tight hip flexors can limit hip extension and cause a patient to compensate by extending at the lumbar spine. The Thomas test can identify a tight iliopsoas and/or rectus femoris. The Ely test can identify a tight rectus femoris. Answer A:The Thomas test will identify tightness of the hip flexors, but the 90/90 test assesses hamstring length, which is not the issue in this scenario. Answer B:The FABER test screens for hip dysfunction, not muscle length. The Thomas test will identify tightness of the hip flexors. Answer C:The SLR test and the 90/90 test both assess hamstring length, not hip flexor length. Bottom Line: Ely's test is used to examine the muscle length of the rectus femoris. The patient is placed prone, and the knee is passively flexed on the affected side. If the muscle is tight and the patient does not stabilize their core, the pelvis will tilt anteriorly because the rectus pulls at its attachment on the AIIS. The lumbar spine may follow and increase the lordosis as well.The Thomas test involves holding one knee into the chest to hold the pelvis in neutral and letting the other leg hang over the foot of the plinth. If the leg does not lower to the level of the table, the hip flexors are tight.

A physical therapist is assessing the gait of a 42-year-old male construction worker with an injury to the right femoral nerve 3 months ago with resultant weakness. Which of the following gait deviations is this patient MOST likely to demonstrate? a) compensated Trendelenburg gait leaning to the right side b) compensated Trendelenburg gait leaning to the left side c) leaning trunk backward at initial contact (heel strike) on the right side d) leaning trunk forward at initial contact (heel strike) on the right side

Correct answer: D During initial contact (heel strike), the quadriceps muscles contract eccentrically to help maintain stability at the knee, hips, and trunk, or else the knee would buckle. In the case of a femoral nerve injury, where quadriceps weakness is expected, a common compensation used to prevent collapse at the knee is to lean the trunk forward. Doing so would shift the center of mass forward over the base of support, allowing the ground reaction force to create that extension moment and prevent buckling. Answers A & B:This is incorrect because a compensated Trendelenburg gait results from weakness in the gluteus medius. Injury to the femoral nerve causes quadriceps weakness. In a compensated Trendelenburg gait, the patient leans over the leg with the weak gluteus medius to move their line of gravity closer to that hip joint and reduce the forces trying to drop the opposite side of the pelvis. Answer C:This gait deviation is typically used to compensate for weakness in the hip extensors. During initial contact (heel strike), the hip extensors contract eccentrically to assist in maintaining a stable and upright trunk. Weakness would cause instability at the trunk and pelvis in the sagittal plane with a fall into trunk-forward flexion. As a compensation, leaning backward moves the ground reaction force (GRF) posterior to the hip, adding an extension force to compensate for weak extensors having to fight a flexion force of the GRF.

A patient status post healed left clavicle fracture reports generalized stiffness and some hesitancy to move the arm. Postural examination reveals kyphotic posture with left forward shoulder. When the physical therapist asks the patient to stand more upright, bring the shoulders back, and squeeze the shoulder blades together, the patient reports a feeling of stiffness on the left. Based on joint mechanics at the sternoclavicular (SC) joint, which of the following joint MOBILITY ASSESSMENTS should the physical therapist perform at the SC joint? a) superior b) inferior c) anterior d) posterior

Correct answer: D During protraction and retraction, the concave end of the clavicle moves on the convex sternum. During retraction, which is the motion that occurs with scapular retraction, the head of the clavicle glides in a posterior direction. Answer A:When moving in the frontal plane, the end of the clavicle is convex, and so a superior glide would occur during depression. Answer B:When moving in the frontal plane, the end of the clavicle is convex, and so an inferior glide would occur during elevation. Answer C:Anterior glide occurs during protraction. Bottom Line:During protraction and retraction, the concave end of the clavicle moves on the convex sternum. To examine the joint mobility of retraction, the therapist needs the patient to perform a posterior clavicular glide at the sternoclavicular joint.

A 30-year-old male patient who was involved in a motor vehicular accident was diagnosed with traumatic brain injury. The physical therapist noted that the patient is exhibiting signs of lower extremity dysmetria. As part of the treatment plan, what is the BEST intervention for the patient? a) alternating heel raises and dorsiflexion b) side lying hip abduction c) walking in sync with a metronome d) placing feet on floor markers alternately

Correct answer: D Dysmetria is described as inability to judge distance or range of movement. Having an activity that incorporates ability to judge distance or range, such as placing feet on floor markers, is the most appropriate for the patient. Answers A & B:This activity is appropriate for lower extremity strengthening. Answer C:A metronome is usually used in activities where improving/refining speed or rhythm is the target. In this case, judgment of distance or range is the focus.

A physical therapist is assessing gait in a child with cerebral palsy. On examination, the therapist finds bilateral "in toeing" during the gait pattern. Which of the following would be the CAUSE of these findings? a) femoral retroversion b) weakness of the hip internal rotators c) adaptive shortening of the hip external rotators d) femoral anteversion

Correct answer: D Femoral anteversion produces a "toe in" gait pattern. Answer A:Femoral retroversion produces a "toe out" gait pattern. Answer B:Weakness of the hip internal rotators leads to a "toe out" gait pattern. Answer C:Adaptive shortening of the hip external rotators leads to a "toe out" gait pattern.

During an electromyographic study, fibrillation potentials are seen with the tested muscle at rest. Which of the following diseases is MOST consistent with this finding? a) CVA b) C5 SCI c) fibromyalgia d) muscular dystrophy

Correct answer: D Fibrillation activity at rest in an electromyographic study is consistent with a neuropathy, myopathy, or both. It is caused by a disruption in motor nerve function in skeletal muscles. It is important to keep in mind that this type of abnormality is indicative of nerve and muscle disease and not typically associated with conditions involving the central nervous system.Muscular dystrophy is a disease of the muscle with compromise to the skeletal muscle tissues as well as to motor nerve function. It is common to see fibrillation potentials at rest in an electromyographic study in muscle diseases such as muscular dystrophy due to changes that occur in the muscle fibers as well as changes in the sensitivity to neurotransmitters.Answer A:A cerebral vascular accident, or stroke, is a condition that affects the central nervous system. There is typically no indication for an electromyographic study. If this type of study is done, fibrillation activity at rest would not be present. Answer B:A spinal cord injury is a disruption of the spinal cord resulting in impairment or loss of motor and sensory nerve function. It is an upper motor neuron lesion and involves the central nervous system. Usually there is no indication for an electromyographic study for this condition. If this type of study is done, fibrillation activity at rest would most likely not be present. Answer C:Fibromyalgia is a neurologic problem most commonly associated with chronic pain and musculoskeletal dysfunction. It is considered a rheumatological condition and is classified as a musculoskeletal and connective tissue disease. However, it is not associated with neuropathies or myopathies. Therefore, fibrillation activity in an electromyographic study would most likely not be present.

Which of the following audio or visual cues would be most appropriate to use to improve foot placement during gait for a client with Parkinson's disease (PD)? A. Parallel visual cues B. Brisk marching music C. Transverse visual-spatial cues D. Floor markers

Correct answer: D Floor markers or footprints on the floor have been shown to be particularly useful for training in foot placement in patients with PD. Answer A:Parallel visual cues have not shown to be beneficial for foot placement and are not the best option for gait velocity and stride length when transverse cues are more effective. Answer B:Brisk marching music is a good audio cue for enhancing pace in patients with PD. Answer C:Transverse visual-spatial cues have been shown to be effective for gait velocity, stride length, and percent of leg stance time, but not specifically for foot placement, for which floor markers are the most effective. Bottom Line:Individuals with Parkinson's disease benefit from external cues, particularly for gait and gait pattern. Different cues can be more useful for different goals, and floor markers or footprints are most effective for foot placement. Other cues such as music can assist with pace or cadence, and transverse cues can improve stride length.

Which of the following MOST accurately describes the purpose of the percussion (tap) test for the great saphenous vein? a) test for pitting edema caused by venous insufficiency b) evaluate for arterial causes of resting leg pain c) evaluate for capillary refill in the lower extremities d) evaluate for competence of the valves in the vein

Correct answer: D For all tests involving arterial or venous involvement, it is important to understand the anatomical and physiological basis for the tests. The percussion (or tap) test for the great saphenous vein evaluates for the competence of the valves within the vein. This test is performed with the patient standing. The examiner places one hand over the sapheno-femoral junction and the other hand distally over the great saphenous vein. The examiner then taps the vein with the distal hand. If a palpable impulse is produced proximally, then the test is positive and is suggestive of incompetent valves along the great saphenous vein near the level where tapping or percussion was performed. Answer A:Pitting edema is assessed by placing firm pressure in an area of swelling and noting the presence of indentation that does not reverse and the amount of time required for the indentation to return to normal. Although pitting edema is caused by venous insufficiency, the percussion test is meant for testing the competence of the valves of the great saphenous valves, not the presence of pitting edema. Answer B:The percussion test does not involve the arterial system. The rubor dependency test is an example of a test to evaluate for arterial insufficiency. Answer C: Capillary refill is assessed by placing deep pressure on a nail bed and noting the amount of time required for color to return to the blanched area. The percussion test involves the valves inside the veins that prevent blood from pooling and does not involve the capillaries. Bottom Line:The percussion (or tap) test for the great saphenous vein evaluates for the competence of the valves within the vein. TrueLearn Insight :A good understanding of the vascular system allows for better critical thinking when the physical therapist is presented with disorders of peripheral circulation, especially when examining the extremities.

A physical therapist is performing an initial examination with a patient who had a right middle cerebral artery stroke and who is demonstrating decreased attention to the left side of space. For WHAT clinical condition should the physical therapist screen due to the patient's presentation? a) ataxia b) hemiparesis c) apraxia d) hemianopsia

Correct answer: D Hemianopsia refers to a cut in a visual field that may occur after a stroke. Homonymous hemianopsia refers to the loss of the contralateral visual field, which frequently is caused by a middle cerebral artery stroke. For patients who had a right middle cerebral artery stroke, homonymous hemianopsia may occur simultaneously with spatial neglect since the middle cerebral artery is involved with both deficits. Answer A:Ataxia is characterized by incoordination of movement in which errors of force, speed, or trajectory occur. Ataxia is typically found in patients with damage to the cerebellum. Answer B:Hemiparesis is paralysis of one half of the body. Answer C:Apraxia is the inability to perform motor activities upon command while the sensory and motor systems are intact. Apraxia is more commonly found in patients with damage to the left cerebral cortex rather than the right cerebral cortex.

Which of the following interventions will produce the BEST training effect for a patient's aerobic capacity status post myocardial infarction 2 weeks ago? A. Light bilateral dumbbell curls, weight only at 30%, of 1-repetition maximum, 3 sets of 10 repetitions B. Lower extremity strength training with light resistance bands C. Stationary bicycle for 10 minutes, rate of perceived exertion on modified-Borg scale maintained at 1/10 D. Treadmill walking with a 1% grade at 2 mph for 10 minutes, rate of perceived exertion on modified-Borg scale maintained at 3/10

Correct answer: D In exercise prescription, it is important to keep in mind the specificity of exercise when selecting the mode, frequency, duration, and intensity. The type of exercise that most directly addresses aerobic capacity is one that involves large and functional movements (such as walking) as well as one that has a high enough intensity. Treadmill walking with a 1% grade at 2 mph for 10 minutes, RPE on modified-Borg scale maintained at 3/10 is the correct answer because activity at this level involves large and functional movements, such as walking uphill. In addition, the intensity that is being maintained would be considered moderately difficult, which corresponds to a rate of perceived exertion (RPE) at 3/10. Of all the answer choices, this one would produce the largest training effect in aerobic capacity. Answer A:Resistance exercise training, such as with dumbbell curls, can be part of a comprehensive physical conditioning program for cardiac patients. In fact, strength training has been shown to improve exercise tolerance in these patients. However, performing this exercise as a primary intervention would not lead to the best training effect in aerobic capacity. Answer B:Resistance exercise training should be introduced gradually into a comprehensive physical conditioning program. Using light resistance bands is one effective way of doing that. However, this type of training would likely not produce a large training effect in aerobic capacity. Answer C:Pedaling on a stationary bicycle is a good mode of exercise since it can produce an adequate heart rate response while stressing the lower extremity muscles. However, with the RPE at 1/10, the intensity is likely not enough to create a noticeable aerobic effect. Bottom Line:In exercise prescription, it is important to keep in mind the specificity of exercise when selecting the mode, frequency, duration, and intensity. The type of exercise that most directly addresses aerobic capacity is one that involves large and functional movements (such as walking) as well as one that has a high enough intensity.

A physical therapist is evaluating a patient with left foot pain when walking. The patient has a history of arthritis. During examination of gait, the therapist finds excessive pronation during mid stance through terminal stance. This gait deviation is caused by which of the following? a) pes cavus of the foot b) plantar flexed first ray c) limited calcaneal eversion d) insufficient ankle dorsiflexion (less than 10 degrees)

Correct answer: D Insufficient ankle dorsiflexion(less than 10 degrees) leads to excessive pronation during mid stance through terminal stance of gait. Answer A:Pes cavus of the foot (high arch) leads to excessive supination during initial contact through mid stance. Answer B:A plantar flexed first ray leads to excessive supination during initial contact through mid stance. Answer C:Limited calcaneal eversion leads to excessive supination during initial contact through mid stance.

A patient reports left elbow pain that has gradually increased over the last 4 weeks. The therapist suspects lateral epicondylitis. The patient would be most symptomatic trying to hold which of the following positions one resistance was applied by the therapist? A. Supination of the forearm, ulnar deviation and flexion of the wrist B. Pronation of the forearm, ulnar deviation and extension of the wrist C. Supination the forearm, radial deviation and flexion of the wrist D. Pronation of the forearm, radial deviation and extension of the wrist

Correct answer: D Lateral epicondylitis (or tennis elbow) usually occurs in patients 35 years or older and those with a great deal of wrist flexion and extension in their occupation or activities. It is a chronic overuse injury to the common extensor tendon at the elbow from repeated microtrauma to the tendon leading the inflammation.The common extensor tendon is the origin for the following muscles: extensor carpi radialis brevis (ECRB) extensor carpi ulnaris extensor digitorum extensor digiti minimi Placing the forearm in pronation, radial deviation, and wrist extension will put stress on the extensor carpi radialis brevis (ECRB) which is the most commonly affected muscle in tennis elbow. Sudden and severe pain in the area of the lateral epicondyle with this test is a positive sign for lateral epicondylitis. Answer A: Placing the arm in supination and resisting flexion and ulnar deviation will place stress through the flexor carpi ulnaris, which does not originate at the common extensor tendon. Answer B: Placing the arm in this position will place stress through the extensor carpi ulnaris, which does originate from the common extensor tendon but is less likely than the ECRB to be the source of the inflammation. Answer C: Placing the arm in supination and resisting flexion and radial deviation will place stress through the flexor carpi radialis, which does not originate at the common extensor tendon. Bottom Line: Placing the forearm in pronation, radial deviation, and wrist extension will put stress on the ECRB, which is the most commonly affected muscle in tennis elbow. Sudden and severe pain in the area of the lateral epicondyle with this test is a positive sign for lateral epicondylitis.

A physical therapist is performing a grade three mobilization to the distal femur in the posterior direction. Which of the following motions is the therapist trying to increase? a) hip flexion b) knee flexion c) hip extension d) knee extension

Correct answer: D Mobilization to the distal femur in a posterior direction increases knee extension. The distal femur is convex and glides posteriorly during knee extension. Answer A:To increase hip flexion, the therapist would need to mobilize the proximal end of the femur in a posterior direction. Answer B:To increase knee flexion, the therapist would need to mobilize the distal end of the femur in an anterior direction, or the proximal end of the tibia in a posterior direction. Answer C:To increase hip extension, the therapist would need to mobilize the proximal end of the femur in an anterior direction. Bottom Line:The distal end of the femur is convex and articulates with the proximal tibia for knee flexion and extension motions. Mobilizing the distal femur in a posterior direction increases knee extension, and mobilizing the distal femur in an anterior direction increases knee flexion.

A research study examined the difference between the effectiveness of treatment A (a new, experimental treatment) and treatment B (conventional treatment) in reduction of spasticity. The p value is set at 0.05. One group received treatment A. One group received treatment B. After analyzing the data, the researchers found a difference in mean outcome scores between the two groups with a p value of p = 0.086. Which of the following conclusions can be made? A. Treatment A is a more effective treatment than treatment B B. Treatment B is a more effective treatment than treatment A C. One treatment is more effective, but there is not enough information provided here to know which one D. There is no difference in the effectiveness of the two treatments

Correct answer: D Researchers typically set the alpha level at 5%, (or lower) meaning, there is only a 5% or less probability that the results are due to chance.If the p value of the statistical test results is equal to or less than 0.05, the null hypothesis can be rejected and the findings are most likely due to whatever treatment was used. The p value for this study's results was greater than 0.05, meaning the difference in the two treatments is not statistically significant. Answers A & B:The p value is greater than 0.05, meaning there is no significant difference between the treatments. In addition, even if the p value was below 0.05, the information above did not specify which treatment had superior results, only that there was a difference. Answer C:It is true that there is not enough information to determine which treatment was more effective at reducing spasticity. However, because the p value was above 0.05, neither treatment is actually more effective. There is no statistically significant difference. Bottom Line:The level of significance, or alpha level, is the criterion that is set to distinguish between results that are real versus those that are due to chance.

A patient is being seen in an outpatient physical therapy clinic for mid-portion Achilles tendinopathy. The patient performs standing bilateral calf raises on the edge of a step for Achilles strengthening. The physical therapist wants to progress the patient's exercise to make it more difficult. Which exercise is the BEST intervention to progress this patient's program? a) seated unilateral calf raise b) seated unilateral calf raise with 10 lb weight c) unilateral calf raise on a step d) bilateral calf raise to unilateral lowering on level surface

Correct answer: D Returning to a level surface instead of a step reduces the range of motion of the exercise and at the same time adding the unilateral lowering progresses the level of difficulty of the exercise and focuses on an eccentric strengthening and control.When progressing weight-bearing calf raises, a typical progression in difficulty goes in the order of easiest to most difficult: calf raise, calf raise on a step, bilateral calf raise to unilateral lowering on level surface and then off a step, unilateral on the floor, and lastly unilateral on a step. Answer A:Although it is unilateral, a seated calf raise is much easier than a weight-bearing calf raise, so this does not progress the exercise. Answer B:Although it is unilateral, a seated calf raise is much easier than a weight-bearing calf raise, even with a 10 lb weight. It is not close to body weight. Answer C:Unilateral calf raise on a step progresses the patient too quickly. It is unilateral for the concentric and the eccentric phase and use of the step makes the ankle move through a greater range of motion. This will be an eventual progression but is not the best choice for now. Bottom Line:When progressing weight-bearing calf raises, a typical progression in difficulty goes in the order of easiest to most difficult: calf raise, calf raise on a step, bilateral calf raise to unilateral lowering on level surface and then off a step, unilateral on the floor, and lastly unilateral on a step.

A skilled nursing facility is trying to determine the quality of their therapy outcomes for all patients admitted post total knee replacement (TKR). The researcher decides to conduct the study on every third patient admitted to the facility with a total knee repair until they have achieved a sample size of 50 patients. What type of sampling method is this researcher using? a) random b) cluster c) convenience d) systematic

Correct answer: D Systematic sampling means potential participants are "counted off;" for example, every third patient is used in the study. This type of sampling is not as truly random as a random sampling method but is stronger than some of the other sampling methods often used with patient populations. Answer A:Random sampling is genuinely random, and every patient who enters the facility with a TKR would be equally as likely to participate in the study. To achieve this type of sampling would mean that drawings were randomly held that included all potential patients. In this case, the sample size was 50, so potentially 200 names could be put into a drawing and 50 pulled out, and the patients who came in on those numbers, for example, patients 10,12,34,75, etc., would be chosen to participate in the study. Answer B:Cluster sampling is often geographical and in this case could be all the patients who are put into a certain wing of the facility or all those whose home address is in a certain location. Answer C:Convenience sampling uses the first available participants, so in this example would be the first 50 patients with TKR to come into the facility after the research has started. Bottom Line:Different types of sampling offer increased rigor to the study. Knowing the difference between samples allows the reader of the study to determine if the results are applicable to certain patient populations and how broadly the results can be generalized.

A 50-year-old female comes to the physical therapy examination with a 6-month history of headaches and neck pain. She reports the need to turn her entire body to see behind her when driving because when she turns only her neck to the right she becomes dizzy and lightheaded, and has tingling down her right arm. Imaging reveals decreased vertebral height and osteophyte formation at C1-C2. Which of the following screening tests would best assist in the determination of a risk for vasculogenic origin of her symptoms? a) Adson's vascular test b) Spurling's test c) Cervical flexion rotation test d) Vertebral artery test

Correct answer: D Testing of the vertebral artery should be considered if the observation and history reveal any of the signs and symptoms that have been linked, directly or indirectly, to vertebral artery insufficiency, such as dizziness with rotation and extension, imaging that reveals structural changes, and postural changes in movement as is the case in this example. The patient reported dizziness, lightheadedness, and upper extremity tingling with right rotation as she is trying to see behind her, functionally changing her rotation to see peripherally. Answer A:The Adson's vascular test is a test for thoracic outlet syndrome with compression of the subclavian artery by the cervical rib or tightened anterior and middle scalene musculature. This test would not address the concern for vertebrobasilar artery insufficiency. Answer B:Spurling neck compression test is useful to evaluate nerve root irritability. The patient's cervical complex is placed in extension and the head is rotated toward the affected shoulder. An axial load is then placed on the spine by applying downward pressure through the patient's head. Reproduction of radicular symptoms to the ipsilateral side to which the head is rotated is a positive test and indicates an irritation of the nerve root.Because the client had a history of dizziness and lightheadedness, it would be necessary to assess vertebrobasilar insufficiency before end range testing or manipulation to rule out vertebrobasilar insufficiency. Answer C:The cervical flexion rotation test is a validated test to localize hypomobility at the C1-C2 segments. This manual provocation test is used for neck mobility and is not an assessment for vertebrobasilar insufficiency. Bottom Line:Dizziness provoked by specific cervical motions, particularly extension or rotation, may also indicate a vertebral artery compromise. Occlusion of the vertebral or basilar arteries can occur because of cervical rotation or cervical rotation coupled with cervical extension. Common signs and symptoms of such occlusion include dizziness, lightheadedness, syncope, and gait disturbances.The therapist must remember that every cervical patient, especially the ones with a history of a hyperextension mechanism and complaints of dizziness provoked by head movements and falling without loss of consciousness, is at potential risk for serious head and neck injuries, including compromise of the vertebral artery.

A patient presents with a 1 month history of severe right sided neck pain with numbness and paresthesias in the right upper extremity. Upon MRI, there was moderate to severe foraminal stenosis at C5-C6. Upon manual muscle testing (MMT), which of the following motions is MOST likely to be weak? a) shoulder flexion b) shoulder abduction c) elbow extension d) elbow flexion

Correct answer: D The C6 nerve root would be compressed between C5 and C6. Upon myotome assessment, elbow flexion would be the weakest as biceps and brachioradialis are innervated by C6. Answer A:Shoulder flexion is performed by many muscles, including deltoid (C5), brachialis (C5-C6), and pectoralis major (C7, C8, T1). A C6 compression may weaken shoulder flexion slightly, but this is not the best answer. Answer B:Shoulder abduction is primarily a function of C5. Answer C:Elbow extension is performed by the tricep muscle, which is innervated by C7. This would not be affected by a compression at C6. Bottom Line:Myotome assessment is often included in musculoskeletal assessment. Upper extremity myotomes are as follows: C5 shoulder abduction, C6 elbow flexion, C7 elbow extension, C8 ulnar deviation, and T1 digit abduction/adduction.

A patient sustained a knee injury during a soccer game. The clinical examination reveals a positive Lachman test. Which LIGAMENT is implicated given this examination finding? A. Posterior cruciate ligament B. Medial collateral ligament C. Lateral collateral ligament D. Anterior cruciate ligament

Correct answer: D The Lachman test is performed with the patient supine and the knee placed in 20-30 degrees of flexion with slight external rotation. The femur is stabilized, and the examiner places one hand on the posterior aspect of the tibia. An anterior directed force is applied on the tibia, and the quantity of movement and end-feel are compared bilaterally. A positive test as indicated by excessive anterior translation and ligamentous laxity implicates an injury to the anterior cruciate ligament. Answer A:The posterior cruciate ligament is a static restraint to posterior translation of the tibia. This ligament injury is not as common as injury of the anterior cruciate ligament. Clinical tests that implicate the posterior cruciate ligament include the posterior sag sign (Godfrey), posterior drawer test, and quadriceps active test. Answer B:The medial collateral ligament is a static restraint to valgus forces at the knee, and injury to this ligament is common in sport. A positive valgus stress testperformed at 0 and 30 degrees implicates this ligament injury. Answer C:The lateral collateral ligament is a static restraint to varus forces at the knee. Injury to this ligament is not as common as injury to the medial collateral and anteriorcruciate ligaments. A varus stress test would be positive with this ligament injury. Bottom Line:Anterior cruciate ligament injuries are common. It is estimated that 80,000 to 250,000 of these injuries occur annually in the United States. The incidence of this injury is more common in sports that require multidirectional activity. Of note, 70% of these injuries are noncontact, and female athletes have a higher risk. The Lachman test is the gold standard for identifying this ligament injury.

A physical therapist is completing an initial examination with a 21-year-old patient who had an ASIA A C6 spinal cord injury 2 weeks ago. Upon inspection of the patient's skin, the therapist noted that there was an area on the patient's right ischial tuberosity that was purple, soft to touch, non-blanchable, and slightly warmer than the surrounding tissue. Which of the following MOST accurately describes the area? A. Stage 1 pressure ulcer B. Stage 2 pressure ulcer C. Stage 3 pressure ulcer D. Suspected deep tissue injury

Correct answer: D The National Pressure Ulcer Advisory Panel and the Agency for Healthcare Research and Quality recommend the use of a standardized staging system for the description of pressure ulcers. A suspected deep tissue injury (DTI) is a purple or maroon local area of discolored skin that is due to damage of underlying soft tissue. The DTI may have a softer feel or a difference in temperature in comparison to the surrounding tissue. In patients with a spinal cord injury, a DTI is often caused by shear or pressure that occurs on the ischial tuberosities as a result of inadequate weight shifting or positioning. When a DTI is treated, the underlying tissue damage may be revealed. Answer A:A stage 1 pressure ulcer describes an area of intact skin with non-blanchable redness that is usually found over a bony prominence. Answer B:A stage 2 pressure ulcer is a partial-thickness loss of skin that appears as a shallow open ulcer with a red-pink wound bed but without slough tissue. Answer C:A stage 3 pressure ulcer is a full-thickness tissue loss that exposes subcutaneous fat. However, bone, tendon, and muscle are not exposed in a stage 3 pressure ulcer.

A physical therapist is completing an initial examination in the home setting with a 75-year-old patient with a history of dementia who reports difficulty with instrumental activities of daily living, such as transportation and meal preparation. Which of the following outcome measures is MOST appropriate for the physical therapist to use to establish a baseline for instrumental activities of daily living? A. Barthel Index B. Fugl-Meyer Assessment C. Functional Independence Measure D. Outcome and Assessment Information Set

Correct answer: D The Outcome and Assessment Information Set (OASIS) was developed to measure patient outcomes in the home and includes a section of 13 items that measure instrumental activities of daily living. The OASIS is collected by all providers of home care services that are funded by Medicare.Physical therapists must be able to choose the appropriate outcome measure based on the clinical situation to accurately and efficiently examine their patients. Answer A:The Barthel Index is a general measure of functional ability and activities of daily living (ADLs) that measures the degree of assistance required on 10 items, such as feeding, ambulation, stair climbing, and toileting, but does not measure instrumental activities of daily living. Answer B:The Fugl-Meyer Assessment is an assessment of motor function, sensation, and balance that was specifically designed for patients who have had a stroke. It does not include measurement of instrumental activities of daily living. Answer C:The Functional Independence Measurement is an outcome measure of physical, psychological, and social function but does not include measurement of instrumental activities of daily living. Bottom Line:The Outcome and Assessment Information Set (OASIS) was developed to measure patient outcomes in the home and includes a section of 13 items that measure instrumental activities of daily living.

Which of the following outcome measures is MOST appropriate for assessing quality of life for patients with chronic obstructive pulmonary disease (COPD)? a) Modified Borg rate of perceived exertion (RPE) scale b) Short form 36 (SF-36) questionnaire c) Six-minute walk test (6MWT) d) St. George's respiratory questionnaire (SGRQ)

Correct answer: D The St. George's Respiratory Questionnaire (SGRQ) is a self-completed questionnaire that measures quality of life in individuals with respiratory disease. It consists of items addressing respiratory symptoms, activity, and effects. Its use has been validated in patients with COPD. Answer A:Although the Modified Borg RPE scale is often used for assessing workload and intensity for patients with COPD, it does not directly measure quality of life. This answer is therefore incorrect. Answer B:The short form 36 (SF-36) questionnaire is a self-administered quality of life outcome measure. It is widely used as a measurement of general health status across different populations. Although it can be an acceptable choice for measuring quality of life in individuals with COPD, it is not disease-specific like the St. George's respiratory questionnaire (SGRQ). Therefore, it is not the most appropriate answer. Answer C:The 6-minute walk test (6MWT) is often used to measure exercise capacity in patients with COPD. It has good correlation with many quality-of-life measures, similar to the St. George's respiratory questionnaire (SGRQ). However, it does not directly measure quality of life. Therefore, this answer is incorrect. Bottom Line:Many quality-of-life outcome measures exist for general populations as well disease-specific populations. When choosing an outcome measure, the therapist must consider the domain that tool covers, as well as what it actually measures. The St. George's respiratory questionnaire, for instance, is specifically used for respiratory diseases and measures quality of life. Using a measure such as this produces more meaningful results than with a more generalized tool, such as the SF-36 questionnaire, which also measures quality of life but is not specific for respiratory disease.

A patient complained of intense pain on the face after undergoing a dental procedure on her right wisdom tooth. It was found that the patient also had difficulty with jaw movements, specifically with mouth opening and with moving her jaw toward the left. Results of the assessment pointed to a possible injury to the trigeminal nerve. What muscle is MOST likely affected by the injury? a) temporalis b) masseter c) stylohyoid d) lateral pterygoid

Correct answer: D The correct answer is lateral pterygoid. The trigeminal nerve (cranial nerve V) has three branches: (a) ophthalmic, (b) maxillary, and (c) mandibular. The motor component of the trigeminal nerve innervates the muscles of mastication. The lateral pterygoid, which is innervated by the mandibular branch of the trigeminal nerve, is responsible for mouth opening and deviation of the jaw to the opposite side. The trigeminal nerve also is responsible for sensation in the face. Answer A:The temporalis muscle is the most powerful muscle of the temporomandibular joint. It elevates the mandible, causing mouth closing, and also retracts the mandible. Answer B:The masseter is responsible for mandible elevation (mouth closing) and mandibular protrusion. Answer C:The stylohyoid is responsible for elevation of the hyoid bone. Bottom Line:The trigeminal nerve (cranial nerve V) innervates the muscles of mastication and is responsible for sensation in the face. The lateral pterygoid, which is innervated by the mandibular branch of the trigeminal nerve, is responsible for mouth opening and deviation of the jaw to the opposite side. The trigeminal nerve also is responsible for sensation in the face.

Which of the following techniques should a physical therapist perform to measure swelling at the ankle status post grade three ankle sprain? a) circumferential measurement 2 inches above the malleoli, then another 2 inches below the malleoli b) circumferential measurement 2 inches above the malleoli, then at the level of malleoli, then another 2 inches below the malleoli c) figure of 8 tape measure connecting the malleoli, first MTP joint, and fifth MTP joint d) figure of 8 tape measure connecting the malleoli, navicular, and base of the fifth metatarsal

Correct answer: D The figure of 8 method applied to these landmarks is most accurate to measure edema at a bony body part where the foot is at a right angle to the leg. This answer describes the correct landmarks. Answer A:This option does not capture the swelling at the central portion of the ankle, and in addition, there is no real way to measure 2 inches below the malleoli. The foot is at a right angle to the leg, and 2 inches down would be at the calcaneus, so staying in the same plane would have the tape measure go all the way along the side of the foot forward to the toes. Answer B:This option is not as accurate and does not capture all the swelling. In addition, there is no real way to measure 2 inches below the malleoli. The foot is at a right angle to the leg. This is a good technique for other body parts, however. Answer C:This is not the correct way to perform a figure 8 measurement. The distal landmarks are not correct. Bottom Line:Following an ankle sprain a body structure function limitation is swelling. To measure swelling, the figure of 8 method is valid and reliable and is performed by connecting the malleoli, navicular, and base of the fifth metatarsal.

In a diabetic patient with poorly controlled hyperglycemia, all of the following can occur in the body EXCEPT which of these? a) kidneys excrete increased amounts of glucose b) kidneys excrete increased amounts of water c) liver increases glucose production d) liver produces more insulin

Correct answer: D The liver does not produce insulin. It is produced in the pancreas. In addition, in patients with diabetes and hyperglycemia, they may not be able to produce more insulin, or the insulin they do produce is ineffective because there is insulin resistance. The body tries to remove the glucose through the urine. Answer A: Since insulin is not doing the job of moving glucose out of the blood and into the cells, the kidneys attempt to excrete it in the urine. That is why patients with diabetes will test positive for glucose in their urine. Answer B: The excess sugar in the urine sets up an osmotic pressure gradient where water is not reabsorbed by the renal tubules and is instead excreted in theurine. This causes a patient to urinate frequently, called polyuria. Answer C: The liver usually absorbs glucose after a meal. If the glucose is not mobilized to the liver, such as when there is insufficient insulin or insulin resistance, the glucose remains in the blood. The liver, sensing it did not absorb new glucose, perceives this as a glucose deficit and produces more glucose and sends it into the circulation. This is meant to help, but it only exacerbates the hyperglycemia. Bottom Line: The liver does not produce insulin. When the body has too much glucose in the blood that is not sent to the cells, many other compensatory processes occur. The excess sugar is excreted through the urine, which also sets up an osmotic pressure gradient, causing water to not be absorbed in the renal tubules and also to be excreted through the urine. This causes patients with hyperglycemia to urinate frequently, called polyuria.

WHICH of the following is a concern when planning interventions for a patient with a stroke affecting the left occipital lobe? a) difficulty understanding instructions b) difficulty verbalizing c) abnormal visual spatial processing d) homonymous hemianopia

Correct answer: D The occipital lobe is located at the posterior portion of the brain, is home to the primary visual cortex, and is responsible for taking the information from the optic nerve and processing it so we can see. Damage can result in partial to complete blindness. Damage to the left lobe, for example, will lead to deficits in the right field of vision of both eyes, an outcome called homonymous hemianopia. Answer A: Wernicke's aphasia or receptive aphasia occurs with damage to the parietal lobe and affects the ability to understand spoken or written language. When treating a patient with receptive aphasia, the therapist must simplify and demonstrate instructions. Answer B:Broca's aphasia occurs with damage to the frontal lobe and affects the ability to speak. Patients cannot express themselves well, and this must be considered during patient treatments. Answer C:The parietal lobe is responsible for visual spatial processing, and with damage to that lobe, patients have vision but have difficulty processing visual information such as length, depth, and the size of objects; object identification; and facial recognition.

A physical therapist is treating a patient with a right transtibial amputation who received a new prosthetic device. After wearing the prosthetic for 1 hour, slight redness was found on the patellar tendon, soft tissue of the posterior residual limb, shaft of the fibula, and tibial crest. Which of these areas should the physical therapist be MOST concerned with monitoring? a) shaft of the fibula b) soft tissue of the posterior residual limb c) patellar tendon d) tibial crest

Correct answer: D The patellar tendon-bearing (PTB) socket is the most common design for transtibial prosthetics. A PTB socket is designed to allow pressure in certain areas of the socket while relieving other areas in order to maintain a good prosthetic fit while protecting the skin of the residual lumb. A PTB has an area of relief for the tibial crest because excessive pressure on this area will cause discomfort and eventual skin breakdown. Answer A:A PTB socket is designed to distribute pressure to the shaft of the fibula to allow for a close prosthetic fit. Answer B:The soft tissue of the posterior limb is a pressure-tolerant area of PTB socket, which is used to distribute pressure through the residual limb. Answer C:The patellar tendon is the main weight-bearing area of a PTB socket and it is tolerant to pressure.

A physical therapist is treating a patient who had a left middle cerebral artery stroke. The patient can produce many words with grammatically correct sentences but is unable to communicate anything to the therapist because the speech content is meaningless. The patient is also unable to understand anything the physical therapist is saying. Which type of aphasia describes this clinical condition? a) anomic b) conduction c) expressive d) receptive

Correct answer: D The patient described in this scenario is experiencing symptoms of aphasia that are consistent with the diagnosis of a left middle cerebral artery stroke. The left hemisphere of the brain is responsible for most of language function in the majority of people. The inability to understand verbal commands along with disorganized and meaningless speech is consistent with a lesion to Wernicke's area that causes receptive aphasia. Receptive aphasia is also known as Wernicke's aphasia. Answer A:A patient with anomic aphasia has poor word-finding abilities but intact language comprehension and language expression. Answer B:A patient with conduction aphasia would have the inability to repeat words that are spoken to them but have intact language comprehension and expression. Answer C:A patient with expressive aphasia would have deficits in language expression and very limited speech output, often with incomplete words consisting of single phonemes and other forms of nonsensical speech. Expressive aphasia is also called Broca's aphasia due to the area of the brain that causes these deficits. Bottom Line: Receptive aphasia causes deficits in language comprehension and language expression that make communication difficult during physical therapy. Physical therapists must be able to predict language and cognitive deficits based on the patient's diagnosis in order to adapt their examination and interventions to the patient's abilities.

A patient describes numbness and tingling in the bottom of the right foot of insidious onset. The patient reports that the symptoms are mild at the start of the day but worsen toward the end of the day when standing for long periods of time. Postural examination reveals normal posture except for excessive pronation of the right ankle. Palpation of the medial side of the foot distal to the medial malleolus causes an increase in paresthesia along the plantar surface of the foot. Based on this information, which of the following structures is MOST likely affected? a) deltoid ligament b) L5/S1 disc c) plantar fascia d) posterior tibial nerve

Correct answer: D The patient described in this scenario is experiencing symptoms of paresthesia and numbness, consistent with a neurological impairment. Tarsal tunnel syndrome is a compressive neuropathy of the posterior tibial nerve as it passes under the flexor retinaculum along the medial side of the ankle. Patients report numbness and tingling along the bottom of the foot. Tapping over the nerve in that area will result in a positive Tinel's sign and an increase in the parasthesia. Contributing factors may include excessive pronation, prolonged standing, ankle inflammation, and trauma, among others, which all can cause a compression of the nerve within the tarsal tunnel. Answer A:The deltoid ligament is a very strong connective tissue that spans the medial side of the ankle. An injury to the deltoid ligament would not occur with an insidious onset. There would have to be some sort of trauma. In addition, the symptoms of a sprain to this ligament would not cause neurological symptoms. A sprain could cause swelling, which could cause tarsal tunnel syndrome; however, the structure affected would be the posterior tibial nerve in that case. Answer B:The L5/S1 disc, if herniated or bulging, could potentially cause a radiculopathy along the L5/S1 nerve root. The symptoms would start more proximally and would not be isolated to only the plantar surface of the foot. Answer C:The plantar fascia is a strong connective tissue located along the plantar surface of the foot spanning the longitudinal arch. Plantar fasciitis occurs with irritation of the plantar fascia, especially at the insertion site on the calcaneus. Symptoms include pain along the bottom of the foot. Pain is worse at the start of the day, when the arch is loaded and elongated by the weight of the body after being in a non-weight bearing, shortened position through the night. For this patient, the symptoms were less evident in the morning and worsened with prolonged standing, which is not consistent with plantar fasciitis. Plantar fasciitis can also be distinguished from tarsal tunnel syndrome by the absence of neurological symptoms such as numbness and paresthesia. Plantar fasciitis would also not worsen with palpation distal to the medial malleolus, because the plantar fascia is not located in that area. Bottom Line:Tarsal tunnel syndrome is a compressive neuropathy of the posterior tibial nerve as it passes under the flexor retinaculum along the medial side of the ankle. Contributing factors may include excessive pronation, prolonged standing, ankle inflammation, and trauma among others, which can all cause a compression of the nerve within the tarsal tunnel. TrueLearn Insight :It is important to utilize critical thinking skills to properly evaluate your findings. In this scenario, the relevant facts that led to a diagnosis of tarsal tunnel syndrome also led to ruling out of the other three choices. Eliminate as many answer choices as possible to improve your ability to choose the correct answer.

A 50-year-old female reports sudden-onset, severe facial pain x 4 days and is seeking the opinion of the physical therapist to rule out temporomandibular jaw (TMJ) dysfunction. There is no trauma or significant past medical history. The patient states she has feelings of stabbing, lightning-type pain that shoots across the right side of her cheek and jaw and lasts about 1 minute any time she touches her face. She cannot even put on makeup or brush her teeth without triggering the pain. Based on this information, what is MOST likely causing this patient's symptoms? A. Facial neuralgia B. Glossopharyngeal neuralgia C. Postherpetic neuralgia D. Trigeminal neuralgia

Correct answer: D The patient is describing symptoms of trigeminal neuralgia. Trigeminal neuralgia is inflammation of the 5th cranial nerve causing symptoms such as intense electric shock-like pain in the face, frequently caused by a blood vessel pressing against the nerve. Any of the three branches can be affected: opthalmic branch, maxillary branch, or mandibular branch. Symptoms are usually provoked by touching the affected area and are usually unilateral. Answer A:The facial nerve, cranial nerve 7, controls the muscles of facial expression and the taste function on the anterior two-thirds of the tongue. Dysfunction of the facial nerve would present as Bell's palsy with facial droop on that side. Answer B:Glossopharyngeal neuralgia is caused by irritation to the 9th cranial nerve. It is rare and more common in men. Symptoms include severe pain in the tongue, throat, ear, and tonsils. It is usually described as lightning pain similar to trigeminal neuralgia and lasts about 1 minute as well. Answer C:Postherpetic neuralgia occurs after a bout of shingles or a herpes zoster attack. There is damage to the peripheral nerve tissue causing intense dermatomal pain in the area of the attack. In this case however, the patient had no significant medical history. Bottom Line:Trigeminal neuralgia is a chronic pain condition affecting the trigeminal nerve. It causes excruciating pain along the area of the face innervated by the trigeminal nerve. The trigger is usually touching the affected area and includes ADLs such as touching the face, eating, drinking, washing the face, putting on makeup, shaving, and smiling.

A patient's upper extremity manual muscle test results are 5/5 for all motions except elbow extension, which is 1/5; elbow flexion, 4-/5; supination, 4/5; wrist extension, 1/5; and finger extension, 1/5. Based on these findings, which of the following is the MOST likely explanation for the weakness?

Correct answer: D The patient is exhibiting weakness in all the muscles of the upper extremity that are innervated by the radial nerve. The radial nerve originates from the posterior cord of the brachial plexus and goes distally along the upper extremity, becoming the posterior interosseous nerve after the level of the forearm and continuing into the hand. It has motor branches that innervate the triceps, brachioradialis, supinator, extensor carpi radialis longus and brevis, extensor carpi ulnaris, and the finger extensors.If the radial nerve is injured at the level of the axilla, the motions of elbow extension, wrist extension, and finger extension would be very weak or even absent. Supination and elbow flexion might be weakened but not fully absent because the biceps brachii and brachialis would still assist with elbow flexion, and the bicep brachii would assist with supination.An injury at the level of a nerve root differs in its distribution from an injury of a peripheral nerve. In this scenario, the weakness in the elbow, wrist, and finger extension resulted from a radial nerve injury and not from a more proximal nerve root. If the nerve root was affected at C6, C7, or C8, additional weakness would have been seen in some flexor muscles as well. Answer A:The C6/C7 myotome includes the biceps brachii and wrist flexors. The patient in this scenario does not have weakness in elbow flexion, supination, or wrist flexion. Answer B:The C7/C8 myotome includes the wrist flexors and finger flexors; the patient in this scenario does not have weakness in those muscles. Patients with a radial nerve injury and wrist drop have a weak grip when the wrist rests in that flexed position due to active insufficiency of the finger flexors. However, if the wrist is placed in neutral or slight extension and a manual muscle test (MMT) is performed, finger flexor weakness would not be observed with an injury to the radial nerve. Answer C:If there was an injury to the median nerve, the wrist and finger flexors would be weak, and in this scenario they are unaffected. In addition, the elbow and wrist extensors are weak, and the median nerve does not have a role in their function. Bottom Line:Manual muscle testing of a radial nerve injury shows very weak (or even absent) elbow extension, wrist extension, and finger extension.

A physical therapist is treating a patient with a T1 American Spinal Injury Association A spinal cord injury who is in an inpatient rehabilitation facility. The patient requires the assistance of nursing for a bowel program and for intermittent catheterization. The patient takes gabapentin and baclofen 2 times per day. At the start of the physical therapy session, the patient reports having a severe headache and blurry vision. The physical therapist checks the patient's blood pressure, which is 180/90 mmHg. Which of the following questions should the physical therapist immediately ask? A. When was your last bowel movement? B. When was the last time you took your baclofen? C. When was the last time you took your gabapentin? D. When was the last time you were catheterized?

Correct answer: D The patient is experiencing an episode of autonomic dysreflexia, which is found in patients with a spinal cord injury that occurs at T6 and above. A painful stimulus below the level of injury causes a dangerous rise in blood pressure. The most common cause of autonomic dysreflexia is a build-up of bladder pressure due to the need for intermittent catheterization or a blockage in an indwelling catheter. Answer A:A bowel impaction is a common cause of autonomic dysreflexia, but bladder issues are more likely to trigger symptoms. Answer B:Baclofen is an anti-spasticity medication, but autonomic dysreflexia is not caused by an increase in spasticity. Answer C:Gabapentin is a medication used to treat neuropathic pain in patients with spinal cord injury, but autonomic dysreflexia is not caused by an increase in neuropathic pain.

A physical therapist completes an overhead lift transfer from the bed to a tilt-in-space wheelchair for a patient with a C5 American Spinal Injury Association A spinal cord injury. The patient reports his head "feels funny." The physical therapist checks the patient's blood pressure, which is 80/50 mmHg. Which of the following actions should the physical therapist immediately complete? A. check the catheter to see if it is kinked B. apply compression stockings C. contact the nurse to check for orders for adrenergic agonist medications D. tilt the wheelchair fully back

Correct answer: D The patient is experiencing an episode of orthostatic hypotension, which is commonly found in patients with a spinal cord injury. When a person is experiencing orthostatic hypotension, the physical therapist needs to increase the patient's blood pressure as quickly as possible so that the patient does not lose consciousness. By positioning the patient in a full tilt, the patient's legs would be above their heart, which would increase peripheral resistance and quickly raise the patient's blood pressure. Answer A:This patient has hypotension. If it were autonomic dysreflexia due to urinary retention, one would see hypertension. Answer B:Compression stockings are used to increase peripheral resistance in order to increase blood pressure in a patient who is hypotensive. However, compression stockings would not immediately raise the patient's blood pressure so it is not the best first response that the physical therapist should complete in this scenario. Answer C:Adrenergic agonist medications are used to increase peripheral resistance in order to increase blood pressure in a patient who is hypotensive. However, medications would not immediately raise the patient's blood pressure, and it would take time for the nurse to confirm the orders and give the medication, so it is not the best first response that the physical therapist should complete in this scenario. Bottom Line:Orthostatic hypotension is commonly found in patients with a spinal cord injury. When a person is experiencing orthostatic hypotension, the physical therapist needs to increase the patient's blood pressure as quickly as possible so that the patient does not lose consciousness. By positioning the patient in a full tilt, the patient's legs would be above their heart, which would increase peripheral resistance and quickly raise the patient's blood pressure.

A physical therapist is treating a 75-year-old patient with a history of polio who recently has reported increased difficulty with walking due to bilateral knee buckling, increased bilateral knee pain, and increased overall fatigue. The patient was previously walking independently without an assistive device but now is limited to walking 2 or 3 blocks prior to needing to rest. Which of the following interventions is the LEAST appropriate in this scenario? a) cardovascular endurance training b) energy conservation techniques c) evaluation for an assistive device d) strength training of the knee extensors

Correct answer: D The patient who is described in the scenario is demonstrating symptoms of post-polio syndrome, which occurs in many people with a history of polio. Post-polio syndrome results from the overextended motor units that have long compensated for loss of motor neurons eventually being unable to keep up with demand and deteriorating. Thus, it is not wise to attempt to strengthen and place more demand. Overuse weakness could lead to decreased independence with functional activities. Answer A:Cardiovascular endurance training is a commonly used intervention for patients with post-polio syndrome to increase activity tolerance. Answer B:Patient education regarding energy conservation techniques is a commonly used intervention for patients with post-polio syndrome to compensate for fatigue and decreased activity tolerance. Answer C:Evaluation for an assistive device is a commonly used intervention for patients with post-polio syndrome to compensate for fatigue and decreased activity tolerance.

During gait analysis a patient's ankle is in neutral at initial contact. The patient demonstrates reduced anterior translation of the tibia over the foot in midstance and an early heel rise. The patient reports a feeling of tightness anteriorly along the ankle. Which of the following interventions is the BEST option to address this patient's functional deficit and gait deviations? a) anterior foot stretch by sitting back on heels b) stretch hanging the heel off a step c) anterior talar mobilization d) posterior talar mobilization

Correct answer: D The patient's gait deviations indicate a lack of dorsiflexion range of motion. Normally the tibia translates anteriorly over the talus into 5 to 10 degrees of closed chain dorsiflexion before the heel rises up. If there is insufficient dorsiflexion, that cannot happen. Of the options provided, posterior mobilization of the talus is the best intervention to increase ankle dorsiflexion and address the gait deviations. In addition, the patient is not reporting a feeling of tightness in the calf musculature, rather a tightness anteriorly, which would be more likely to relate to a capsular restriction as the reason for decreased dorsiflexion. Answer A:Stretching the foot into more plantarflexion would not address the issue of a lack of dorsiflexion ROM. Answer B:The patient is lacking dorsiflexion and is reporting tightness in the anterior ankle, which is indicative of some sort of joint/capsular restriction most efficiently addressed by mobilization. If the issue was tightness of the gastrocsoleus, the patient would report a tightness in the calf. Answer C:Mobilizing the talus in an anterior direction would increase ankle plantarflexion. Bottom Line:During gait, closed chain ankle dorsiflexion begins after foot flat/loading response and ends at heel off. The range of motion needed is 5 to 10 degrees of dorsiflexion. If insufficient, there will be early heel rise. To increase dorsiflexion, the talus is mobilized in a posterior direction, or the tibia is mobilized in an anterior direction.

A patient presents with new onset of dizziness that is worse when rolling over to her right side in order to get out of bed. The dizziness lasts about 30 seconds, and then she is able to continue with her normal activities with only a slight feeling of imbalance throughout the day. What is the MOST important test to include during the examination? a) Epley maneuver b) Rhomberg test c) Clinical test for sensory integration and balance (CTSIB) d) Dix Hallpike

Correct answer: D The patient's symptoms are consistent with posterior canal benign positional paroxysmal vertigo (BPPV). The Dix Hallpike test is the most common positional test used to examine BPPV. The patient is put into a long sitting position with the head rotated 45 degrees toward the test side. The patient is then moved into a supine position with the head extended 30 degrees beyond the horizontal. The therapist looks for vertigo and nystagmus, which is caused by putting the semicircular canals (SCC) into a gravity dependent position. Answer A:The Epley maneuver is a treatment for BPPV, but not a test. Once the diagnosis of BPPV is made with the Dix Hallpike, the therapist can move forward with the Epley maneuver if appropriate. Answer B:The Rhomberg test is one of the oldest sensory tests for postural control. The patient in this scenario does report imbalance, so the Rhomberg test would be appropriate to perform. However, based on the history given, the symptoms are most likely caused by BPPV. Answer C:The CTSIB is an excellent test to examine postural stability and sensory input from vision, somatosensory, and vestibular systems. During a thorough vestibular examination, this test or a similar test should be performed. However, based on the history, the Dix Hallpike is the MOST important test to perform to diagnose the BPPV. Bottom Line:BPPV is a common condition in which the otoconia have been displaced into the semicircular canals (SCC) of the inner ear. The presence of the otoconia in the endolymph makes the SCC sensitive to changes in head position. The most common complaint of these patients is dizziness and vertigo when rolling over in bed. Another classic sign of BPPV is the duration of the symptoms (usually less than 1 minute). The Dix Hallpike is the most common positional test to examine BPPV. If found positive, the patient can then be treated with a variety of painless maneuvers to remove the otoconia from the SCC.

A physical therapist is covering for a coworker at the hospital. The patient to be treated has pneumonia and is in need of chest physical therapy. The physical therapist's daily note from yesterday's session states that, "The patient was positioned prone with a pillow under the abdomen prior to performing chest percussion." What lobe is the physical therapist MOST likely targeting by placing the patient in this position? a) posterior apical segment b) lingular segment c) posterior basal aspect of the lower lobe d) superior aspect of the lower lobe

Correct answer: D The prone position with a pillow under the abdomen targets the superior aspect of the right and left lower lobes. Answer A:To target the posterior apical segment, one would position the patient in sitting with the hip and knee flexed to 90 degrees.Answer B:The lingular segment is located only in the left lobe. This region is in the medial portion of the lung, and to perform postural drainage the patient is placed in a sidelying position with the legs elevated 12 inches above horizontal. Answer C:To access the basal segment, the patient's lower body would need to be elevated in a Trendelenburg-like position to properly facilitate drainage. The patient would be positioned in prone; however, the lower extremities would need to be elevated 18-20 inches above horizontal. Bottom Line:Postural drainage uses positioning to help drain the lungs. The prone position with a pillow under the abdomen targets the superior aspect of the right and left lower lobes. To access the basal segment of the lower lobes, the patient's lower body would need to be elevated in a Trendelenburg-like position to properly facilitate drainage. For the apical segment, the patient would need to be in an upright position to allow for access. The question does not specify L or R lobe; knowing that the lingular segment exists only in the L lobe allows you to be able to rule out that answer choice

A patient presents to the clinic with a report of numbness/tingling along the medial knee and medial malleolus. The patellar tendon reflex was diminished on the affected side. Which key muscle action should the physical therapist examine during myotome testing? a) ankle plantarflexion b) ankle eversion c) knee flexion d) knee extension

Correct answer: D The quadriceps are innervated by the L3 nerve root primarily and should be tested based on the dermatomal sensation changes (L3, L4) and reflex changes (L3). Therefore knee extension tests the key muscle group that should be assessed during myotomal testing. Answer A:This answer is incorrect as this muscle action is performed by muscles innervated by S1. Answer B:This answer is incorrect as this muscle action is performed by muscles innervated by L5. Answer C:This answer is incorrect as this muscle action is performed by muscles innervated by S1-S2.

During gait analysis, the physical therapist observes that during the swing phase, especially mid-swing, as the patient dorsiflexes to achieve foot clearance, the ankle also everts. The therapist also notices that all of the toes extend as well while the ankle dorsiflexes. Which of the following muscles is MOST likely to be impaired? a) extensor digitorum longus b) extensor hallucis longus c) fibularis longus d) tibialis anterior

Correct answer: D The question asks for the muscle that is most likely to be weak or impaired. In this case, the lack of inversion points to involvement of a muscle that performs inversion, which would be primarily tibialis anterior. In addition, dorsiflexion is achieved, but only with toe extension and eversion, which is further evidence that the tibialis anterior is not active. A good understanding of anatomy, especially the different actions of muscles in each joint, is required in order to be able to identify impairments in gait and functional movement.During the swing phase of gait, the tibialis anterior is active in lifting the foot into dorsiflexion (DF) for foot clearance. The anterior tibialis is also an invertor, and so with DF we see inversion. With isolated weakness of the tibialis anterior, other muscles that are secondary dorsiflexors may be used to compensate. In this case, the extensor digitorum longus (EDL), which crosses the talocrural joint anteriorly and has the ability to dorsiflex the ankle, is substituting for the weak anterior tibialis. The EDL line of pull allows the tibialis anterior to dorsiflex and evert, not invert. This would explain the observation of eversion as well as toe extension during foot clearance. Answer A:The extensor digitorum longus, an ankle dorsiflexor, toe extensor, and ankle evertor, would actually be active in this case. It is actively being used to compensate for the lack of tibialis anterior, which is a dorsiflexor and an invertor. The observation of eversion as well as toe extension during foot clearance confirms the role of this muscle being used as a compensation. Answer B:The extensor hallucis longus primarily extends the great toe but also contributes to dorsiflexion of the ankle joint and inversion of the foot. However, this is not the best answer because weakness of this muscle typically would not produce major gait deviations. A noticeable lack of inversion is likely due to involvement of the tibialis anterior. Answer C:The fibularis longus is an evertor and plantarflexor. It is generally not active during the swing phase of gait, except for some contribution in eversion. Therefore, this muscle is not impaired in this case. Bottom Line:During the swing phase of gait, the tibialis anterior is active in lifting the foot into dorsiflexion (DF) for foot clearance. The anterior tibialis is also an invertor, and so with DF we see inversion. With isolated weakness of the tibialis anterior, other muscles that are secondary dorsiflexors may be used to compensate.

A patient is prone and asked to flex each knee. The therapist notices that each time the right knee flexes, the right hip flexes a few degrees as well and the lumbar spine goes into slight lordosis. When the left knee flexes, there is no additional motion noted. Which of the following interventions is MOST appropriate for the right lower extremity? a) strengthen the hamstrings b) strengthen the gluteus maximus c) stretch the iliopsoas d) stretch the rectus femoris

Correct answer: D The rectus femoris attaches from the tibial tubercle to the anterior inferior iliac spine (AIIS); thus it crosses the hip and the knee. When flexing the knee in prone, the rectus femoris is being elongated. If it does not have sufficient length, the hip will flex to allow the muscle some additional flexibility to elongate over the knee joint. By flexing the hip, the muscle is placed on slack over the anterior hip and can then elongate further over the anterior knee to allow further knee flexion ROM. The AIIS insertion will also move in the direction that the muscle is pulling and allow some anterior pelvic tilt and lumbar lordosis. This can also be tested passively and is called Ely's test. Answers A & B:The reason for the motion is a muscle length issue and not a strength issue. Answer C:The iliopsoas does not cross the knee, and so a change in position of the knee would not alter the length of this muscle and would not cause a compensatory motion at the hip. Bottom Line:When the rectus femoris is shortened and asked to elongate over both of the joints, it crosses with hip extension and knee flexion. A compensation may be to flex the hip or anteriorly tilt the pelvis in order to give some slack to one end of the muscle. This can be seen in a positive Ely's test.

A physical therapist is completing an initial examination in the outpatient setting of an 85-year-old man who has a history of hypertension, type 2 diabetes, and cardiac arrhythmia. The patient reports he recently has started falling at home. While taking the patient's medical history, the physical therapist learns that the patient is taking 8 different medications on a daily basis that were ordered by 3 different physicians. Which of the following is the term to describe this scenario with respect to the patient's medications? A. drug adherence B. pharmacodynamics C. pharmacokinetics D. polypharmacy

Correct answer: D The scenario describes polypharmacy,which is typically defined as the taking of 5 or more medications. Polypharmacy is commonly seen in elderly patients with complex medical histories and is associated with an increased risk of falls, cognitive decline, and dependence in activities of daily living. Answer A:Drug adherence describes how well a patient is able to follow his or her prescribed drug prescriptions. Answer B:Pharmacodynamics describes the effects that a drug has on the body and includes both the beneficial and negative effects of a drug. Answer C:Pharmacokinetics refers to the effects of the body on a drug as it enters the body, is absorbed, metabolized, and excreted.

A physical therapist is conducting an initial examination for a patient with an incomplete spinal cord injury. The physical therapist uses a painful stimulus to the right big toe during sensory testing. Which spinal tract is being tested? A. dorsal column medial lemniscus B. gracile fasciculus C. spinocerebellar D. spinothalamic

Correct answer: D The spinothalamic tract relays pain and temperature information in the spinal cord.Depending on which tracts of the spinal cord are affected, patients will demonstrate different patterns of sensory deficits, which will require the physical therapist to adjust the treatment plan and patient education accordingly. Answer A:The dorsal column medial lemniscus relays light touch and conscious proprioception in the spinal cord. Answer B:The gracile fasciculus is the part of the dorsal column medial lemniscus that relays light touch and proprioception of the lower leg in the spinal cord. Answer C:The spinocerebellar tract relays unconscious proprioception in the spinal cord. Bottom Line:The spinothalamic tract relays pain and temperature information in the spinal cord.

Which of the following special tests is BEST to identify a tear of the subscapularis? A. Cross body adduction test where the patient flexes the shoulder to 90 degrees and the physical therapist passively moves the arm into horizontal adduction B. Full can test where the patient's arm is placed at 90 degrees of scaption (between flexion and abduction) and external rotation with the thumb pointing up, and the physical therapist applies resistance in a downward direction C. Hawkins-Kennedy test where the patient's arm is passively flexed to 90 degrees, the elbow is flexed to 90 degrees, and the physical therapist moves the arm into full internal rotation D. Lift-off test where the patient places his/her hand on the lower back and tries to lift the hand away from the back in a posterior direction against resistance from the physical therapist

Correct answer: D The subscapularis is an internal rotator of the shoulder. The lift-off test starts with the patient's arm in full internal rotation with the hand resting against the low back. The patient then tries to lift the hand off of the lower back against a resisting force.This test has a high sensitivity and specificity for detecting a tear of the subscapularis, which is an internal rotator and part of the rotator cuff. Answer A:The cross-body adduction test places the shoulder in full horizontal adduction, and pain in that position can be indicative of acromioclavicular joint injury or subacromial impingement. Answer B:The full can test places the patient's arm at 90 degrees of scaption (between flexion and abduction planes) and external rotation. The physical therapist then applies manual resistance. The test is sensitive for tears of the supraspinatus and is a variation of the empty can test where the patient's arm is placed in internal rotation. Answer C:The Hawkins-Kennedy test is utilized to determine if there is a subacromial impingement, and positions the rotator cuff tendons under the acromion. The test is positive if the test position reproduces the pain. Bottom Line: The lift-off test places the UE in full shoulder IR and has the patient lift the arm off the back against resistance.This test detects a tear of the subscapularis, which is the only rotator cuff muscle that performs internal rotation.

A patient presents to the clinic with hypothenar atrophy of insidious onset which he noticed is worsening over the past months. His medical history is noncontributory. He works out at the gym 2 days a week and rides his bicycle 5 days a week but does not recall any injuries, just some hand discomfort. He is also having difficulty typing at the computer. What nerve damage does the physical therapist most likely suspect led to the muscle atrophy? a) median b) posterior interosseous nerve c) radial d) ulnar

Correct answer: D The ulnar nerve supplies sensation and motor function to the hypothenar eminence. It can be compressed where it enters the hand in Guyon's canal if one leans on the hand for a long periods of time, such as when bicycling. It is also called handlebar palsy. Answer A:The median nerve supplies the muscles of the thenar eminence, not hypothenar. Answer B:This is incorrect because the posterior interosseous nerve is the motor-only branch of the radial nerve and therefore would not affect the hypothenar eminence. Answer C:This is incorrect as the radial nerve does not have any intrinsic hand muscle innervation and therefore cannot affect the thenar or hypothenar eminence. Bottom Line:The ulnar nerve supplies the ulnar side of the hand. The deep branch innervates the hypothenar muscles (abductor digiti minimi, opponens digiti minimi, flexor digiti minimi), all of the interosseous muscles, the 3rd and 4th lumbricals, the adductor pollicis, and the medial head (deep) of the flexor pollicis brevis (FPB). The superficial branch provides sensation to the palmar surface of the medial one and a half fingers.

Which of the following scenarios would prompt a physical therapist to take a patient off of the treadmill during outpatient cardiac rehabilitation? a) a patient who complains of angina at a 2/10 on a 10 point scale, with a BP of 125/90 b) a patient who complains of fatigue with an RPE of 7/10 and a BP of 130/85 c) a patient who complains of dyspnea on exertion at 4/10 on the dyspnea exertion scale, resolving after 30 seconds of rest d) a patient with no complaints, an RPE of 5/10, and a BP of 220/140

Correct answer: D Though this patient has no complaints of fatigue or angina, the blood pressure values greatly exceed normal limits, and the exercise should be stopped. Answer A:Low-scale angina is not a reason to stop the exercise but should be monitored closely. The blood pressure is slightly high, but the patient is exercising, and that is a normal response. Answer B:Complaints of fatigue with exertion are perfectly normal. The patient is exercising and feels they are working at a moderate level, and BP is responding appropriately. Answer C:Exerting oneself during exercise can increase demands on the cardiopulmonary system. Dyspnea on exertion that resolves with rest is acceptable and not a reason to stop the exercise. Bottom Line:Extremely elevated systolic or diastolic BP that may or may not be associated with symptoms is one of the criteria for stopping an exercise or stress test.

Which of the following interventions would be MOST effective in improving the symptoms of an individual presenting with chronic heel pain from plantar fasciitis? a) Achilles tendon stretching with knee extended in standing as soon as possible after taking first steps in the morning b) Achilles tendon stretching with knee flexed in standing as soon as possible after taking first steps in the morning c) Ankle pumps AROM exercises prior to taking first step in the morning d) Stretching of all toes into extension in sitting before taking first step in the morning and before standing after prolonged sitting

Correct answer: D Tissue-specific stretching has been shown to be effective in the treatment of heel pain caused by chronic plantar fasciitis compared to achilles tendon stretching alone. Stretching the toes into extension specifically targets tightness in the plantar fascia because the fascia starts at the calcaneus and ends on the base of the metatarsals after crossing the metatarsophalangeal (MTP) joints. MTP extension would stretch the fascia. All of the other answer choices describe acceptable treatment interventions as part of a comprehensive treatment plan, but stretching of the plantar fascia with toe extension is the best answer because it is the superior treatment among the choices based on anatomy, biomechanics, and best available evidence. Answer A:Achilles tendon stretching with the knee extended specifically targets the gastrocnemius muscle. Tightness in this muscle is common in individuals with chronic heel pain from plantar fasciitis. Therefore, stretching of the gastrocnemius muscle could be part of the treatment for plantar fasciitis. In fact, it is often added as an adjunct intervention. However, the question is asking for the single MOST effective treatment option. Therefore this is not the best treatment option out of all the answer choices. Answer B:Achilles tendon stretching with the knee flexed specifically targets the soleus muscle. As with many disorders of the foot and ankle, tightness in this muscle can be found in individuals with chronic heel pain from plantar fasciitis. Stretching of the Achilles tendon via soleus stretching can be part of the treatment of plantar fasciits. However, it is not likely to have a direct effect on the plantar fascia, which is the tissue of interest that is affected. Therefore, it is not the best answer. Answer C:Active range of motion in the ankle joint is effective in promoting peripheral circulation and overall comfort. It can be part of a warm-up routine prior to activity that would directly affect the plantar fascia. However, this activity on its own is unlikely to have an effect on the plantar fascia extensibility. Bottom Line:The plantar fascia is a thick band of fibrous tissues extending from the calcaneus to the base of the metatarsals. It is an important structure that spans the medial longitudinal arch. Because it attaches from the calcaneus to the base of the metatarsals, toe extension will tense the fascia. In plantar fasciitis, this band of tissue becomes compromised, causing pain and discomfort usually at the attachment on the calcaneus as it is being tensed and pulled away. Pain is worse in the morning when first weight-bearing and asking for the arch to elongate after being in a shortened position all night. A thorough understanding of anatomy, biomechanics, and application of best available evidence is helpful in identifying the most effective treatment strategies.

Patient is 1 day status post right hip repair for intertrochanteric fracture. Per MD orders, patient is toe-touch weight-bearing (TTWB) to the right lower extremity. What instruction is MOST appropriate to give the patient during ambulation in regard to the right lower extremity? a) "Only your toes on the right leg can touch the floor and bear part of your body weight" b) "Put only about 25% weight through the right leg" c) "Just put a little bit of weight up to pain tolerance" d) "Your toes can touch the floor to maintain balance, but don't use them to weight-bear"

Correct answer: D Toe-touch weight bearing (TTWB) or touch-down weight bearing (TDWB) means only the toes of the affected extremity contact the floor to improve balance, not to support body weight. Answer A:The right lower extremity should not bear weight. The toes can touch only for balance purposes. Answer B:Putting 25% weight through the leg is a specific range for patient weight bearing (PWB). Answer C:Putting weight only to pain tolerance is appropriate to tell a patient who is weight-bearing as tolerated. Bottom Line:Toe-touch weight-bearing is solely to help the patient be able to balance and manage the extremity while walking, but the idea is not to bear weight through the limb. Partial weight-bearing is allowing anywhere from approximately 25% to 50% of body weight. Weight-bearing as tolerated status means leaving the amount of weight-bearing up to the patient. It would not be contraindicated to put full weight on the extremity, but the patient should use pain as their guide and only do what they can tolerate.

As part of the medical chart review prior to performing an initial evaluation on a patient who was admitted to the hospital with a non-ST elevation myocardial infarction (NSTEMI) 1 day ago, the physical therapist looks at the recent lab values for this patient. Which of the following would be the MOST important to review? a) blood lipids b) creatinine c) hemoglobin d) troponin level

Correct answer: D Troponins are cardiac enzymes that are released following cardiac muscle damage, and are indicative of a myocardial infarct (MI). It is important to be aware of the troponin levels for several reasons. First, troponin levels rise in the first 4-6 hours after cardiac injury. These levels peak after about 18-24 hours and remain elevated for 10 days. Subsequent increases after troponins have peaked may indicate further cardiac muscle damage or reinfarction. Answer A: Elevated blood lipid level is a significant risk factor in coronary artery disease. However, for a cardiac patient in the acute care setting, this information may not be useful in clinical decision-making, since abnormal levels do not make mobility and activity contraindicated. In addition, some research has shown that blood lipid levels may not be accurate signs around the time of acute myocardial injury. Answer B: Creatinine is a byproduct of muscle metabolism found in the bloodstream. It is primarily excreted by the kidneys. Therefore, serum creatinine is a measure of kidney function. Levels that are abnormally high may indicate kidney injury. In the setting of a cardiac event, end organ functions such as that of the kidneys is important to monitor. However, it is not the most important lab value to review initially. Therefore, it is not the correct answer for this question. Answer C: Hemoglobin binds to oxygen and transports it to the rest of the body. It is important to note hemoglobin level in order to monitor for signs of anemia. However, in the setting of a cardiac event, it is not the most important lab value to review. Therefore, it is not the correct answer for this question.

During observational gait analysis of a patient with a transfemoral amputation with a prosthesis, the physical therapist notes deviations in the sagittal plane. Throughout the swing phase of the prosthetic limb, the patient is noted to have increased plantarflexion on the sound limb. What is a likely EXPLANATION for this deviation, and what should the physical therapist examine for a cause? A. The patient is compensating for a prosthetic limb that is too short; the physical therapist should look at the symmetry of both sides B. The patient is compensating for a prosthetic limb that is too tight; the physical therapist should examine socket fit C. The patient is holding the knee straight on the prosthetic side to prevent buckling; the physical therapist should examine quadriceps strength on the affected lower extremity (LE) D. The patient is compensating for a prosthetic limb that is too loose; the physical therapist should look at the fit of the socket

Correct answer: D Vaulting is a common gait deviation that can be present during the swing phase of the prosthetic limb in both transtibial (below knee) or transfemoral (above knee) amputees. The cause is typically a loose fit in the socket or simply a prosthetic limb that may be too long. During swing, in an attempt to clear the excessively long or sliding prosthesis, the patient may lift the body/vault up by rising onto the toes quickly on the sound side to gain extra clearance. Answer A:This answer is incorrect because the patient is likely presenting with the opposite problem. A prosthesis that is too short would more likely result in deviations seen during the stance phase on the prosthetic side. Although examining the symmetry of both sides is correct, it is important to note that vaulting is usually associated with a prosthesis that is too long or too loose, making it slide down and become too long. Answer B:A tight fit in the socket would not create a reason to vault up on the sound limb. Answer C:The patient is compensating to clear the prosthetic side during swing. If the knee joint of the transfemoral prosthesis is locked in extension, that may create an issue where the leg is more difficult to clear during swing; however, the patient is a transfemoral amputee and does not control the knee with quadriceps strength. Bottom Line:Vaulting in a patient with limb loss may occur on the sound limb in order to clear the prosthetic limb during swing if the prosthetic leg is too long or too loose. It is important to learn the most common gait deviations for these patients as well as their clinical significance in order to better understand and appreciate their movement impairments.

A physical therapist is completing an initial examination with a patient with neck pain. The patient has a history of schizophrenia and recently switched medications to a first-generation antipsychotic medication. Which of the following adverse effects of the medication would the physical therapist be monitoring for in this scenario? a) constipation b) hallucinations c) hypertension d) tardive dyskinesia

Correct answer: D Tardive dyskinesia is a common adverse effect of antipsychotic drugs and is characterized by involuntary rhythmic movements of the mouth, tongue, jaw, trunk, or extremities. Tardive dyskinesia is caused by disuse supersensitivity of the dopamine system since antipsychotic drugs often block dopamine receptors. Answer A:Constipation is not an adverse effect associated with antipsychotic medications. Answer B:Hallucinations are not an adverse effect associated with antipsychotic medications. However, hallucinations are a common symptom for patients who are diagnosed with schizophrenia. Answer C:Hypertension is not an adverse effect associated with antipsychotic medications. However, hypotension is commonly found patients who are taking antipsychotic medications.

A 30-year-old patient was referred to your clinic for physical therapy. The patient was recently involved in a work-related accident where he injured his left upper arm. Medical diagnosis was a radial nerve injury. During manual muscle testing, which muscle will MOST likely be weak? a) pronator teres b) flexor digitorum profundus c) flexor carpi ulnaris d) supinator

Correct answer: D The correct answer is supinator. The radial nerve emerges directly from the posterior cord of the brachial plexus at the lower border of the pectoralis minor. As it descends the arm, it winds around the posterior aspect of the humerus in the musculospiral groove and continues to the radial aspect of the elbow. In the arm it innervates the triceps, anconeus, and upper portion of the extensor and supinator group in the forearm. Answer A:Pronator teres is innervated by the median nerve and will not be affected in an injury to the radial nerve. Answer B:The flexor digitorum profundus is innervated by the ulnar nerve and will not be affected in an injury to the radial nerve. Answer C:The flexor carpi ulnaris is innervated by the ulnar nerve and will not be affected in an injury to the radial nerve. Bottom Line:It is important to remember the nerves innervating the muscles of the upper extremity. The radial nerve innervates the triceps and anconeus in the arm, and the upper portion of the extensor and supinator group in the forearm. Injury to the radial nerve may occur with shoulder dislocations and midhumeral fractures.

A patient has a knee flexion contracture after being casted for 8 weeks. The patient is missing 13 degrees of extension and can flex the knee to a maximum of 108 degrees. Which of the following is the proper method to document that range of motion? a) (13-108) b) (-13-108) c) (13-0-108) d) (0-13-108)

Goniometric measurements are recorded as the position of the joint at the start and then again at the end of the motion. If there is a contracture, one can document the contracture itself. For example, "There is a 13 degree flexion contracture of the knee." However, when recording the ROM, the first number does not get a negative sign in front of it. The first value is where the range starts, and the second is where it ends. If that same knee with the contracture can move to 120 degrees of flexion, it would be recorded as 13-120.The patient is starting with the knee in 13 degrees of flexion since he currently cannot get the leg any straighter. Then he moves through the range to a position of 108 degrees of flexion. Answer B: When recording of the ROM, the first number does not get a negative sign in front of it. The first value is the start of the range, and the second is the end of the range. Answer C: This documentation makes it appear that the knee began in 13 degrees of extension (or hyperextension), moved to zero, and continued flexing to 108. It does not accurately capture the 13-degree flexion contracture. Answer D: This makes it look like the knee began at zero when in fact it cannot attain zero at all.

Which of the following lists of equipment is required to perform the Berg balance scale with a patient accurately? a) stopwatch, armchair, measured walkway, foam pad b) foam pad, 6-inch step, stopwatch, armchair c) chairs with and without arms, stopwatch, ruler, 6 inch step d) measured walkway, ruler, chair without arms, foam pad

correct answer: C Answer A:The Berg balance scale does not include a measured walkway because it does not test gait, and it does not require a foam pad because all standing activities are tested on a stable surface in the Berg balance test. Answer B:The Berg balance scale does not require a foam pad because all standing is done on a stable surface. The sit-stand tests need a chair without arms, although the transfer testing requires a chair with and without arms. Answer D:The Berg balance scale does not test gait, so no measured walkway is required. The Berg balance scale does all standing tests on a stable surface, so it does not require a foam pad. Bottom Line:The Berg balance scale requires minimal equipment to perform accurately. There must be two types of chairs, one with and one without arms for the transfer testing, and then the one without arms is used for the sit-stand. A stopwatch, 6" step and ruler are also needed.


Conjuntos de estudio relacionados

Micro - Ch 5 - Price Elasticity of Demand and Supply

View Set

Copyright Law: 12 Do's and Dont's

View Set

CompTIA Security Plus Architecture and Design Part 7

View Set

Exam 3 Study Guide: 14, 24, 25, 26, 27, 28, 29

View Set

Entrepreneurship Final Exam #1 (Ch. 1,3,4,7, 8, 17)

View Set